Adult health II oncology

Réussis tes devoirs et examens dès maintenant avec Quizwiz!

A nurse in the outpatient clinic is caring for a patient who has a magnesium level of 1.3 mg/dL. Which assessment would be most important for the nurse to make? a. Daily alcohol intake b. Intake of dietary protein c. Multivitamin/mineral use d. Use of over-the-counter (OTC) laxatives

Daily alcohol intake

The nurse obtains information about a hospitalized patient who is receiving chemotherapy for colorectal cancer. Which information about the patient alerts the nurse to discuss a possible change in therapy with the health care provider? a. Poor oral intake b. Frequent loose stools c. Complaints of nausea and vomiting d. Increase in carcinoembryonic antigen (CEA)

ANS: D An increase in CEA indicates that the chemotherapy is not effective for the patient's cancer and may need to be modified. The other patient findings are common adverse effects of chemotherapy. The nurse may need to address these, but they would not necessarily indicate a need for a change in therapy.

The nurse assesses a patient who is receiving interleukin-2. Which finding should the nurse report immediately to the health care provider? a. Generalized muscle aches b. Complaints of nausea and anorexia c. Oral temperature of 100.6° F (38.1° C) d. Crackles heard at the lower scapular border

ANS: D Capillary leak syndrome and acute pulmonary edema are possible toxic effects of interleukin-2. The patient may need oxygen and the nurse should rapidly notify the health care provider. The other findings are common side effects of interleukin-2.

An older adult patient who has colorectal cancer is receiving IV fluids at 175 mL/hour in conjunction with the prescribed chemotherapy. Which finding by the nurse is most important to report to the health care provider? a. Patient complains of severe fatigue. b. Patient needs to void every hour during the day. c. Patient takes only 50% of meals and refuses snacks. d. Patient has audible crackles to the midline posterior chest.

ANS: D Rapid fluid infusions may cause heart failure, especially in older patients. The other findings are common in patients who have cancer and/or are receiving chemotherapy

The nurse will anticipate teaching a patient with a possible seizure disorder about which test? a. Cerebral angiography b. Evoked potential studies c. Electromyography (EMG) d. Electroencephalography (EEG)

ANS: D Seizure disorders are usually assessed using EEG testing. Evoked potential is used for diagnosing problems with the visual or auditory systems. Cerebral angiography is used to diagnose vascular problems. EMG is used to evaluate electrical innervation to skeletal muscle. DIF: Cognitive Level: Understand (comprehension) REF: 1351 TOP: Nursing Process: Planning MSC: NCLEX: Physiological Integrity

The nurse is reviewing a chest x-ray for a client diagnosed with a mucous gland adenoma. Where does the nurse expect to find the tumor? 1 In the bronchi 2 In visceral pleura 3 In the fibrous tissue 4 In the capillaries, spreading to the lungs

Answer: 1

What diagnostic information does a nurse relate to a patient with stage IV lung cancer? 1 Presence of distant metastasis 2 Presence of a tumor that is 5 cm in size 3 Tumor spreading to the regional lymph nodes 4 Tumor spreading to the pleura and pericardium

Answer: 1

While assessing a patient with lung cancer, the nurse suspects that the patient has brain metastasis. Which finding in the patient supports the nurse's inference? 1 Unsteady gait 2 Muscle wasting 3 Pleural effusions 4 Edema of face and neck

Answer: 1

Which ethnic group has the highest rate of lung cancer for men? 1 Whites 2 Hispanics 3 African Americans 4 Asian/Pacific Islanders

Answer: 3

A nurse is assessing a patient in whom lung cancer is suspected. Which symptom is the patient most likely to report first? 1 Chest pain 2 Diaphoresis 3 Persistent cough 4 Low-grade fever

Answer:3

A patient with lung cancer is being scheduled for chemotherapy. About what medication will the nurse educate the patient that is used for treatment? 1 Erlotinib 2 Crizotinib 3 Carboplatin 4 Bevacizumab

Answer:3

A patient with acute respiratory distress syndrome (ARDS) and acute kidney injury has the following medications ordered. Which medication should the nurse discuss with the health care provider before giving? a. Pantoprazole (Protonix) 40 mg IV b. Gentamicin (Garamycin) 60 mg IV c. Sucralfate (Carafate) 1 g per nasogastric tube d. Methylprednisolone (Solu-Medrol) 60 mg IV

Gentamicin (Garamycin) 60 mg IV

IV potassium chloride (KCl) 60 mEq is prescribed for treatment of a patient with severe hypokalemia. Which action should the nurse take? a. Administer the KCl as a rapid IV bolus. b. Infuse the KCl at a rate of 10 mEq/hour. c. Only give the KCl through a central venous line. d. Discontinue cardiac monitoring during the infusion.

Infuse the KCl at a rate of 10 mEq/hour.

Infiltrative emergencies: Carotid artery rupture

Invasion of artery wall By tumor Erosion following surgery or radiation Bleeding can manifest as Minor oozing Spurting of blood in case of a blowout In the case of a blowout, pressure should be applied. IV fluids and blood administered to stabilize for surgery

A diabetic patient's arterial blood gas (ABG) results are pH 7.28; PaCO2 34 mm Hg; PaO2 85 mm Hg; HCO3- 18 mEq/L. The nurse would expect which finding?

Kussmaul respirations Kussmaul (deep and rapid) respirations are a compensatory mechanism for metabolic acidosis.

When the nurse educator is evaluating the skills of a new registered nurse (RN) caring for patients experiencing shock, which action by the new RN indicates a need for more education? a. Placing the pulse oximeter on the ear for a patient with septic shock b. Keeping the head of the bed flat for a patient with hypovolemic shock c. Increasing the nitroprusside (Nipride) infusion rate for a patient with a high SVR d. Maintaining the room temperature at 66° to 68° F for a patient with neurogenic shock

Maintaining the room temperature at 66° to 68° F for a patient with neurogenic shock Patients with neurogenic shock may have poikilothermia. The room temperature should be kept warm to avoid hypothermia. The other actions by the new RN are appropriate.

Complications of Cancer Nutritional Problems

Malnutrition Fat and muscle depletion Supplements when 5% weight loss is noted Patient has potential for protein and calorie malnutrition Altered taste sensation Physiologic basis of altered taste is unknown. Nursing care: Monitor albumin and prealbumin levels. Enteral or parenteral nutrition may be required. Teach patient to experiment with spices and seasonings to mask alterations.

The nurse teaches a patient about the transmission of pulmonary tuberculosis (TB). Which statement, if made by the patient, indicates that teaching was effective?

My husband will be sleeping in the guest bedroom." Teach the patient how to minimize exposure to close contacts and household members

A patient admitted with acute respiratory failure has a nursing diagnosis of ineffective airway clearance related to thick, secretions. Which action is a priority for the nurse to include in the plan of care? a. Encourage use of the incentive spirometer. b. Offer the patient fluids at frequent intervals. c. Teach the patient the importance of ambulation. d. Titrate oxygen level to keep O2 saturation >93%.

Offer the patient fluids at frequent intervals.

Which menu choice indicates that the patient understands the nurse's teaching about best dietary choices for iron-deficiency anemia? a. Omelet and whole wheat toast b. Cantaloupe and cottage cheese c. Strawberry and banana fruit plate d. Cornmeal muffin and orange juice

Omelet and whole wheat toast Eggs and whole grain breads are high in iron. The other choices are appropriate for other nutritional deficiencies but are not the best choice for a patient with iron-deficiency anemia.

After several days of antibiotic therapy, an older hospitalized patient develops watery diarrhea. Which action should the nurse take first? a. Notify the health care provider. b. Obtain a stool specimen for analysis. c. Teach the patient about handwashing. d. Place the patient on contact precautions.

Place the patient on contact precautions. The patient's history and new onset diarrhea suggest a C. difficile infection, which requires implementation of contact precautions to prevent spread of the infection to other patients. The other actions are also appropriate but can be accomplished after contact precautions are implemented.

An experienced nurse instructs a new nurse about how to care for a patient with dyspnea caused by a pulmonary fungal infection. Which action by the new nurse indicates a need for further teaching? a. Listening to the patient's lung sounds several times during the shift b. Placing the patient on droplet precautions and in a private hospital room c. Increasing the oxygen flow rate to keep the oxygen saturation above 90% d. Monitoring patient serology results to identify the specific infecting organism

Placing the patient on droplet precautions and in a private hospital room

Thoracentesis Complication

Pneumothorax-5-10% Pain-usually transient Re-expansion pulmonary edema-rare Hypotension-rare "Dry Tap"-only b/c was not successful Vasovagal episode-rare

A female patient is awaiting surgery for acute peritonitis. Which action will the nurse include in the plan of care? a. Position patient with the knees flexed. b. Avoid use of opioids or sedative drugs. c. Offer frequent small sips of clear liquids. d. Assist patient to breathe deeply and cough.

Position patient with the knees flexed. There is less peritoneal irritation with the knees flexed, which will help decrease pain. Opioids and sedatives are typically given to control pain and anxiety. Preoperative patients with peritonitis are given IV fluids for hydration. Deep breathing and coughing will increase the patient's discomfort.

A patient with a pleural effusion is scheduled for a thoracentesis. Which action should the nurse take to prepare the patient for the procedure?

Position the patient sitting upright on the edge of the bed and leaning forward. When the patient is sitting up, fluid accumulates in the pleural space at the lung bases and can more easily be located and removed.

The optimal teaching plan for an outpatient with stomach cancer receiving radiation therapy should include information about A. cancer support groups, alopecia, and stomatitis b. avitaminosis, ostomy care, and community resources c. prosthetic devices, skin conductance, and grief counseling d. wound and skin care, nutrition, and drugs, and community resources

d.

A patient with bacterial pneumonia has rhonchi and thick sputum. What is the nurse's most appropriate action to promote airway clearance?

d. Assist the patient to splint the chest when coughing. Coughing is less painful and more likely to be effective when the patient splints the chest during coughing.

The nurse is caring for a mechanically ventilated patient with a cuffed tracheostomy tube. Which action by the nurse would best determine if the cuff has been properly inflated?

d. Use a manometer to ensure cuff pressure is at an appropriate level. Measurement of cuff pressure using a manometer to ensure that cuff pressure is 20 mm Hg or lower will avoid compression of the tracheal wall and capillaries

A patient who is receiving methotrexate for severe rheumatoid arthritis develops a megaloblastic anemia. The nurse will anticipate teaching the patient about increasing oral intake of a. iron. b. folic acid. c. cobalamin (vitamin B12). d. ascorbic acid (vitamin C).

folic acid Methotrexate use can lead to folic acid deficiency. Supplementation with oral folic acid supplements is the usual treatment. The other nutrients would not correct folic acid deficiency, although they would be used to treat other types of anemia.

A 52-year-old patient has a new diagnosis of pernicious anemia. The nurse determines that the patient understands the teaching about the disorder when the patient states, "I a. need to start eating more red meat and liver." b. will stop having a glass of wine with dinner." c. could choose nasal spray rather than injections of vitamin B12." d. will need to take a proton pump inhibitor like omeprazole (Prilosec)."

could choose nasal spray rather than injections of vitamin B12." Because pernicious anemia prevents the absorption of vitamin B12, this patient requires injections or intranasal administration of cobalamin. Alcohol use does not cause cobalamin deficiency. Proton pump inhibitors decrease the absorption of vitamin B12. Eating more foods rich in vitamin B12 is not helpful because the lack of intrinsic factor prevents absorption of the vitamin.

The following male patients recently arrived in the emergency department. Which one should the nurse assess first? a. 19-year-old who is complaining of severe scrotal pain b. 60-year-old with a nontender ulceration of the glans penis c. 22-year-old who has purulent urethral drainage and back pain d. 64-year-old who has dysuria after brachytherapy for prostate cancer

19-year-old who is complaining of severe scrotal pain The patient's age and symptoms suggest possible testicular torsion, which will require rapid treatment in order to prevent testicular necrosis. The other patients also require assessment by the nurse, but their history and symptoms indicate nonemergent problems (acute prostatitis, cancer of the penis, and radiation-associated urinary tract irritation)

The nurse receives change-of-shift report on the oncology unit. Which patient should the nurse assess first? a. 35-year-old patient who has wet desquamation associated with abdominal radiation b. 42-year-old patient who is sobbing after receiving a new diagnosis of ovarian cancer c. 24-year-old patient who received neck radiation and has blood oozing from the neck d. 56-year-old patient who developed a new pericardial friction rub after chest radiation

24-year-old patient who received neck radiation and has blood oozing from the neck

The nurse monitors a patient after chest tube placement for a hemopneumothorax. The nurse is most concerned if which assessment finding is observed? a. A large air leak in the water-seal chamber b. 400 mL of blood in the collection chamber c. Complaint of pain with each deep inspiration d. Subcutaneous emphysema at the insertion site

400 mL of blood in the collection chamber

The nurse monitors a patient after chest tube placement for a hemopneumothorax. The nurse is most concerned if which assessment finding is observed?

400 mL of blood in the collection chamber The large amount of blood may indicate that the patient is in danger of developing hypovolemic shock

The nurse notes new onset confusion in an older patient who is normally alert and oriented. In which order should the nurse take the following actions? (Put a comma and a space between each answer choice [A, B, C, D].) a. Obtain the oxygen saturation. b. Check the patient's pulse rate. c. Document the change in status. d. Notify the health care provider.

A B C D

After the nurse has received change-of-shift report, which patient should the nurse assess first?

A patient with possible lung cancer who has just returned after bronchoscopy Because the cough and gag are decreased after bronchoscopy, this patient should be assessed for airway patency.

Which are common causes of cancer? Select all that apply. 1 Smoking 2 Chewing tobacco 3 Genetic mutation 4 Bronchial asthma 5 Coal dust inhalation 6 High levels of pollution

Answer: 1,5,6

The nurse should include which food choice when providing dietary teaching for a patient scheduled to receive external beam radiation for abdominal cancer? a. Fresh fruit salad b. Roasted chicken c. Whole wheat toast d. Cream of potato soup

ANS: B To minimize the diarrhea that is commonly associated with bowel radiation, the patient should avoid foods high in roughage, such as fruits and whole grains. Lactose intolerance may develop secondary to radiation, so dairy products should also be avoided.

Which assessments will the nurse make to monitor a patient's cerebellar function (select all that apply)? a. Assess for graphesthesia. b. Observe arm swing with gait. c. Perform the finger-to-nose test. d. Check ability to push against resistance. e. Determine ability to sense heat and cold.

ANS: B, C The cerebellum is responsible for coordination and is assessed by looking at the patient's gait and the finger-to-nose test. The other assessments will be used for other parts of the neurologic assessment. DIF: Cognitive Level: Analyze (analysis) REF: 1348 TOP: Nursing Process: Assessment MSC: NCLEX: Health Promotion and Maintenance

Syndrome of inappropriate ADH

Abnormal or sustained production Cancer cells are able to manufacture, store, and release ADH. Some chemotherapeutic agents stimulate release. Nursing Care: Treatment of fluid restriction or IV of 3% NaCl in severe cases

emphyema

Accumulation of pus in a body cavity, especially the pleural space; pyothorax

A patient with acute shortness of breath is admitted to the hospital. Which action should the nurse take during the initial assessment of the patient?

Briefly ask specific questions about this episode of respiratory distress When a patient has severe respiratory distress, only information pertinent to the current episode is obtained, and a more thorough assessment is deferred until later.

Bronchoscopic lung biopsy

Bronchoscopic lung biopsy may provide a specific histopathologic diagnosis in select diffuse parenchymal lung diseases

When assessing the respiratory system of an older patient, which finding indicates that the nurse should take immediate action?

Bilateral crackles at lung bases Crackles in the lower half of the lungs indicate that the patient may have an acute problem such as heart failure.

External-beam radiation is planned for a patient with cervical cancer. What instructions should the nurse give to the patient to prevent complications from the effects of the radiation? a. Test all stools for the presence of blood. b. Maintain a high-residue, high-fiber diet. c. Clean the perianal area carefully after every bowel movement. d. Inspect the mouth and throat daily for the appearance of thrush.

Clean the perianal area carefully after every bowel movement.

The nurse obtains the following assessment data on an older patient who has influenza. Which information will be most important for the nurse to communicate to the health care provider?

Diffuse crackles in the lungs The crackles indicate that the patient may be developing pneumonia, a common complication of influenza, which would require aggressive treatment.

The nurse completes a shift assessment on a patient admitted in the early phase of heart failure. When auscultating the patient's lungs, which finding would the nurse most likely hear?

Discontinuous, high-pitched sounds of short duration heard on inspiration Fine crackles are likely to be heard in the early phase of heart failure. Fine crackles are discontinuous, high-pitched sounds of short duration heard on inspiration.

A patient who has ovarian cancer is crying and tells the nurse, "My husband rarely visits. He just doesn't care." The husband indicates to the nurse that he never knows what to say to help his wife. Which nursing diagnosis is most appropriate for the nurse to add to the plan of care? a. Compromised family coping related to disruption in lifestyle b. Impaired home maintenance related to perceived role changes c. Risk for caregiver role strain related to burdens of caregiving responsibilities d. Dysfunctional family processes related to effect of illness on family members

Dysfunctional family processes related to effect of illness on family members

Fiberoptic Bronchoscopy Endoscopic Bronchial Ultrasound

EBUS versus EUS-endobronchial ultrasound via bronchoscopy and endoscopic ultrasound via upper GI endoscopy Uses ultrasound probe to identify lymph nodes or masses and guide TBNA

The nurse is caring for a patient who has a calcium level of 12.1 mg/dL. Which nursing action should the nurse include on the care plan? a. Maintain the patient on bed rest. b. Auscultate lung sounds every 4 hours. c. Monitor for Trousseau's and Chvostek's signs. d. Encourage fluid intake up to 4000 mL every day.

Encourage fluid intake up to 4000 mL every day.

A patient who is taking rifampin (Rifadin) for tuberculosis calls the clinic and reports having orange discolored urine and tears. Which is the best response by the nurse?

Explain that orange discolored urine and tears are normal while taking this medication. Orange-colored body secretions are a side effect of rifampin.

A 51-year-old woman with Crohn's disease who is taking infliximab (Remicade) calls the nurse in the outpatient clinic about new symptoms. Which symptom is most important to communicate to the health care provider? a. Fever b. Nausea c. Joint pain d. Headache

Fever Since infliximab suppresses the immune response, rapid treatment of infection is essential. The other patient complaints are common side effects of the medication, but they do not indicate any potentially life-threatening complications.

A patient arrives in the ear, nose, and throat clinic complaining of a piece of tissue being "stuck up my nose" and with foul-smelling nasal drainage from the right nare. Which action should the nurse take first?

Have the patient occlude the left nare and blow the nose Because the highest priority action is to remove the foreign object from the nare, the nurse's first action should be to assist the patient to remove the object

The nurse is caring for a patient receiving intravesical bladder chemotherapy. The nurse should monitor for which adverse effect? a. Nausea b. Alopecia c. Mucositis d. Hematuria

Hematuria

A nurse obtains a health history from a patient who has a 35 pack-year smoking history. The patient complains of hoarseness and tightness in the throat and difficulty swallowing. Which question is most important for the nurse to ask?

How much alcohol do you drink in an average week?" Prolonged alcohol use and smoking are associated with the development of laryngeal cancer, which the patient's symptoms and history suggest.

Interleukin-2 (IL-2) is used as adjuvant therapy for a patient with metastatic renal cell carcinoma. Which information should the nurse include when explaining the purpose of this therapy to the patient? a. IL-2 enhances the immunologic response to tumor cells. b. IL-2 stimulates malignant cells in the resting phase to enter mitosis. c. IL-2 prevents the bone marrow depression caused by chemotherapy. d. IL-2 protects normal cells from the harmful effects of chemotherapy.

IL-2 enhances the immunologic response to tumor cells.

A patient with pneumonia has a fever of 101.4° F (38.6° C), a nonproductive cough, and an oxygen saturation of 88%. The patient complains of weakness, fatigue, and needs assistance to get out of bed. Which nursing diagnosis should the nurse assign as the highest priority? a. Hyperthermia related to infectious illness b. Impaired transfer ability related to weakness c. Ineffective airway clearance related to thick secretions d. Impaired gas exchange related to respiratory congestion

Impaired gas exchange related to respiratory congestion

A patient in the emergency department has just been diagnosed with peritonitis caused by a ruptured diverticulum. Which prescribed intervention will the nurse implement first? a. Insert a urinary catheter to drainage. b. Infuse metronidazole (Flagyl) 500 mg IV. c. Send the patient for a computerized tomography scan. d. Place a nasogastric (NG) tube to intermittent low suction.

Infuse metronidazole (Flagyl) 500 mg IV. Because peritonitis can be fatal if treatment is delayed, the initial action should be to start antibiotic therapy (after any ordered cultures are obtained). The other actions can be done after antibiotic therapy is initiated.

The home health nurse cares for a patient who has been receiving interferon therapy for treatment of cancer. Which statement by the patient indicates a need for further assessment? a. "I have frequent muscle aches and pains." b. "I rarely have the energy to get out of bed." c. "I experience chills after I inject the interferon." d. "I take acetaminophen (Tylenol) every 4 hours."

"I rarely have the energy to get out of bed."

The nurse at the clinic is interviewing a 64-year-old woman who is 5 feet, 3 inches tall and weighs 125 pounds (57 kg). The patient has not seen a health care provider for 20 years. She walks 5 miles most days and has a glass of wine 2 or 3 times a week. Which topics will the nurse plan to include in patient teaching about cancer screening and decreasing cancer risk (select all that apply)? a. Pap testing b. Tobacco use c. Sunscreen use d. Mammography e. Colorectal screening

-Pap testing -Sunscreen use -Mammography -Colorectal screening

A 34-year-old female patient with a new ileostomy asks how much drainage to expect. The nurse explains that after the bowel adjusts to the ileostomy, the usual drainage will be about _____ cups. a. 2 b. 3 c. 4 d. 5

2 After the proximal small bowel adapts to reabsorb more fluid, the average amount of ileostomy drainage is about 500 mL daily. One cup is about 240 mL.

20. Following chemotherapy, a patient is being closely monitored for tumor lysis syndrome. Which laboratory value requires particular attention? 1. Platelet count 2. Electrolyte levels 3. Hemoglobin level 4. Hematocrit

20. Ans: 2 Tumor lysis syndrome can result in severe electrolyte imbalances and potential kidney failure. The other laboratory values are important to monitor to identify general chemotherapy side effects but are less pertinent to tumor lysis syndrome. Focus: Prioritization

A patient is to receive an infusion of 250 mL of platelets over 2 hours through tubing that is labeled: 1 mL equals 10 drops. How many drops per minute will the nurse infuse?

21

A 198-lb patient is to receive a dobutamine infusion at 5 mcg/kg/minute. The label on the infusion bag states: dobutamine 250 mg in 250 mL normal saline. When setting the infusion pump, the nurse will set the infusion rate at how many mL per hour?

27 In order to administer the dobutamine at the prescribed rate of 5 mcg/kg/minute from a concentration of 250 mg in 250 mL, the nurse will need to infuse 27 mL/hour.

3. Which patient is at greatest risk for pancreatic cancer? 1. An elderly African-American man who smokes 2. A young white obese woman with gallbladder disease 3. A young African-American man with type 1 diabetes 4. An elderly white woman who has pancreatitis

3. Ans: 1 Pancreatic cancer is more common in African-Americans, males, and smokers. Other associated factors include alcohol use, diabetes, obesity, history of pancreatitis, exposure to organic chemicals, consumption of a high-fat diet, and previous abdominal irradiation. Focus: Prioritization

A lobectomy is scheduled for a patient with stage I non-small cell lung cancer. The patient tells the nurse, "I would rather have chemotherapy than surgery." Which response by the nurse is most appropriate?

Tell me what you know about the various treatments available." More assessment of the patient's concerns about surgery is indicated. An open-ended response will elicit the most information from the patient

After change-of-shift report, which patient should the nurse assess first? a. 40-year-old male with celiac disease who has frequent frothy diarrhea b. 30-year-old female with a femoral hernia who has abdominal pain and vomiting c. 30-year-old male with ulcerative colitis who has severe perianal skin breakdown d. 40-year-old female with a colostomy bag that is pulling away from the adhesive wafer

30-year-old female with a femoral hernia who has abdominal pain and vomiting Pain and vomiting with a femoral hernia suggest possible strangulation, which will necessitate emergency surgery. The other patients have less urgent problems.

A patient with cancer has a nursing diagnosis of imbalanced nutrition: less than body requirements related to altered taste sensation. Which nursing action is most appropriate? a. Add strained baby meats to foods such as casseroles. b. Teach the patient about foods that are high in nutrition. c. Avoid giving the patient foods that are strongly disliked. d. Add extra spice to enhance the flavor of foods that are served.

Avoid giving the patient foods that are strongly disliked.

4. Patients receiving chemotherapy are at risk for thrombocytopenia related to chemotherapy or disease processes. Which actions are needed for patients who must be placed on bleeding precautions? (Select all that apply.) 1. Provide mouthwash with alcohol for oral rinsing. 2. Use paper tape on fragile skin. 3. Provide a soft toothbrush or oral sponge. 4. Gently insert rectal suppositories. 5. Avoid aspirin or aspirin-containing products. 6. Avoid overinflation of blood pressure cuffs. 7. Pad sharp corners of furniture.

4. Ans: 2, 3, 5, 6, 7 Mouthwash should not include alcohol, because it has a drying action that leaves mucous membranes more vulnerable. Insertion of suppositories, probes, or tampons into the rectal or vaginal cavity is not recommended. All other options are appropriate. Focus: Prioritization, knowledge

After change-of-shift report, which patient should the nurse assess first?

64-year-old with lung cancer and tracheal deviation after subclavian catheter insertion The patient's history and symptoms suggest possible tension pneumothorax, a medical emergency.

Fiberoptic Bronchoscopy

A flexible tube with special lighting fibers used for bronchoscopy under local and general anesthesia.

A patient with lung cancer is scheduled for surgery. What should the nurse assess to determine the patient's ability to withstand surgery? 1 Neurologic function 2 Integumentary function 3 Musculoskeletal function 4 Cardiopulmonary function

Answer: 4

The nurse is providing a community program about lung cancer. What risk factor should the nurse be sure to include when discussing this topic? 1 Age of smoking onset 2 Use of filtered cigarettes 3 Prolonged exposure to nickel 4 Number of cigarettes smoked per day

Answer: 1

A patient is receiving radiation therapy for treatment of lung cancer. About what side effects should the nurse educate the patient that might occur during treatment? Select all that apply. 1 Fatigue 2 Nausea 3 Anorexia 4 Vomiting 5 Weight loss

Answer:234

A patient who is being treated for stage IV lung cancer tells the nurse about new-onset back pain. Which action should the nurse take first? a. Give the patient the prescribed PRN opioid. b. Assess for sensation and strength in the legs. c. Notify the health care provider about the symptoms. d. Teach the patient how to use relaxation to reduce pain.

Assess for sensation and strength in the legs.

Which action by the unlicensed assistive personnel (UAP) who are assisting with the care of patients with male reproductive problems indicates that the nurse should provide more teaching? a. The UAP apply a cold pack to the scrotum for a patient with mumps orchitis. b. The UAP help a patient who has had a prostatectomy to put on antiembolism hose. c. The UAP leave the foreskin pulled back after cleaning the glans of a patient who has a retention catheter. d. The UAP encourage a high oral fluid intake for patient who had transurethral resection of the prostate yesterday.

The UAP leave the foreskin pulled back after cleaning the glans of a patient who has a retention catheter. Paraphimosis can be caused by failing to replace the foreskin back over the glans after cleaning. The other actions by UAP are appropriate.

Interleukin-2 (IL-2) is used as adjuvant therapy for a patient with metastatic renal cell carcinoma. Which information should the nurse include when explaining the purpose of this therapy to the patient? a. IL-2 enhances the immunologic response to tumor cells. b. IL-2 stimulates malignant cells in the resting phase to enter mitosis. c. IL-2 prevents the bone marrow depression caused by chemotherapy. d. IL-2 protects normal cells from the harmful effects of chemotherapy.

ANS: A IL-2 enhances the ability of the patient's own immune response to suppress tumor cells. IL-2 does not protect normal cells from damage caused by chemotherapy, stimulate malignant cells to enter mitosis, or prevent bone marrow depression.

The nurse assesses a patient with non-Hodgkin's lymphoma who is receiving an infusion of rituximab (Rituxan). Which assessment finding would require the most rapid action by the nurse? a. Shortness of breath b. Temperature 100.2° F (37.9° C) c. Shivering and complaint of chills d. Generalized muscle aches and pains

ANS: A Rituximab (Rituxan) is a monoclonal antibody. Shortness of breath should be investigated rapidly because anaphylaxis is a possible reaction to monoclonal antibody administration. The nurse will need to rapidly take actions such as stopping the infusion, assessing the patient further, and notifying the health care provider. The other findings will also require action by the nurse, but are not indicative of life-threatening complications

A 46-year-old patient tells the nurse about using acetaminophen (Tylenol) several times every day for recurrent bilateral headaches. Which action will the nurse plan to take first? a. Discuss the need to stop taking the acetaminophen. b. Suggest the use of biofeedback for headache control. c. Describe the use of botulism toxin (Botox) for headaches. d. Teach the patient about magnetic resonance imaging (MRI).

ANS: A The headache description suggests that the patient is experiencing medication overuse headache. The initial action will be withdrawal of the medication. The other actions may be needed if the headaches persist. DIF: Cognitive Level: Apply (application) REF: 1418 OBJ: Special Questions: Prioritization TOP: Nursing Process: Planning MSC: NCLEX: Physiological Integrity

A patient reports feeling numbness and tingling of the left arm before experiencing a tonic-clonic seizure. The nurse determines that this history is consistent with what type of seizure? a. Focal b. Atonic c. Absence d. Myoclonic

ANS: A The initial symptoms of a focal seizure involve clinical manifestations that are localized to a particular part of the body or brain. Symptoms of an absence seizure are staring and a brief loss of consciousness. In an atonic seizure, the patient loses muscle tone and (typically) falls to the ground. Myoclonic seizures are characterized by a sudden jerk of the body or extremities. DIF: Cognitive Level: Understand (comprehension) REF: 1421 TOP: Nursing Process: Assessment MSC: NCLEX: Physiological Integrity

When admitting an acutely confused 20-year-old patient with a head injury, which action should the nurse take? a. Ask family members about the patient's health history. b. Ask leading questions to assist in obtaining health data. c. Wait until the patient is better oriented to ask questions. d. Obtain only the physiologic neurologic assessment data.

ANS: A When admitting a patient who is likely to be a poor historian, the nurse should obtain health history information from others who have knowledge about the patient's health. Waiting until the patient is oriented or obtaining only physiologic data will result in incomplete assessment data, which could adversely affect decision making about treatment. Asking leading questions may result in inaccurate or incomplete information. DIF: Cognitive Level: Apply (application) REF: 1343 TOP: Nursing Process: Assessment MSC: NCLEX: Physiological Integrity

A patient develops neutropenia after receiving chemotherapy. Which information about ways to prevent infection will the nurse include in the teaching plan (select all that apply)? a. Cook food thoroughly before eating. b. Choose low fiber, low residue foods. c. Avoid public transportation such as buses. d. Use rectal suppositories if needed for constipation. e. Talk to the oncologist before having any dental work done.

ANS: A, C, E Eating only cooked food and avoiding public transportation will decrease infection risk. A high-fiber diet is recommended for neutropenic patients to decrease constipation. Because bacteria may enter the circulation during dental work or oral surgery, the patient may need to postpone dental work or take antibiotics.

Which information about a 76-year-old patient is most important for the admitting nurse to report to the patient's health care provider? a. Triceps reflex response graded at 1/5 b. Unintended weight loss of 20 pounds c. 10 mm Hg orthostatic drop in systolic blood pressure d. Patient complaint of chronic difficulty in falling asleep

ANS: B Although changes in appetite are normal with aging, a 20-pound weight loss requires further investigation. Orthostatic drops in blood pressure, changes in sleep patterns, and slowing of reflexes are normal changes in aging. DIF: Cognitive Level: Apply (application) REF: 1343-1344 OBJ: Special Questions: Prioritization TOP: Nursing Process: Assessment MSC: NCLEX: Health Promotion and Maintenance

Propranolol (Inderal), a β-adrenergic blocker that inhibits sympathetic nervous system activity, is prescribed for a patient who has extreme anxiety about public speaking. The nurse monitors the patient for a. dry mouth. b. bradycardia. c. constipation. d. urinary retention.

ANS: B Inhibition of the fight or flight response leads to a decreased heart rate. Dry mouth, constipation, and urinary retention are associated with peripheral nervous system blockade. DIF: Cognitive Level: Understand (comprehension) REF: eTable 56-2 TOP: Nursing Process: Evaluation MSC: NCLEX: Physiological Integrity

The health care provider is considering the use of sumatriptan (Imitrex) for a 54-year-old male patient with migraine headaches. Which information obtained by the nurse is most important to report to the health care provider? a. The patient drinks 1 to 2 cups of coffee daily. b. The patient had a recent acute myocardial infarction. c. The patient has had migraine headaches for 30 years. d. The patient has taken topiramate (Topamax) for 2 months.

ANS: B The triptans cause coronary artery vasoconstriction and should be avoided in patients with coronary artery disease. The other information will be reported to the health care provider, but none of it indicates that sumatriptan would be an inappropriate treatment. DIF: Cognitive Level: Apply (application) REF: 1416 OBJ: Special Questions: Prioritization TOP: Nursing Process: Assessment MSC: NCLEX: Physiological Integrity

Which equipment will the nurse obtain to assess vibration sense in a diabetic patient who has peripheral nerve dysfunction? a. Sharp pin b. Tuning fork c. Reflex hammer d. Calibrated compass

ANS: B Vibration sense is testing by touching the patient with a vibrating tuning fork. The other equipment is needed for testing of pain sensation, reflexes, and two-point discrimination. DIF: Cognitive Level: Understand (comprehension) REF: 1348 TOP: Nursing Process: Assessment MSC: NCLEX: Physiological Integrity

A 26-year-old woman is being evaluated for vomiting and abdominal pain. Which question from the nurse will be most useful in determining the cause of the patient's symptoms? a. "What type of foods do you eat?" b. "Is it possible that you are pregnant?" c. "Can you tell me more about the pain?" d. "What is your usual elimination pattern?"

"Can you tell me more about the pain?" A complete description of the pain provides clues about the cause of the problem. Although the nurse should ask whether the patient is pregnant to determine whether the patient might have an ectopic pregnancy and before any radiology studies are done, this information is not the most useful in determining the cause of the pain. The usual diet and elimination patterns are less helpful in determining the reason for the patient's symptoms.

Which laboratory test will the nurse use to determine whether filgrastim (Neupogen) is effective for a patient with acute lymphocytic leukemia who is receiving chemotherapy? a. Platelet count b. Reticulocyte count c. Total lymphocyte count d. Absolute neutrophil count

Absolute neutrophil count Filgrastim increases the neutrophil count and function in neutropenic patients. Although total lymphocyte, platelet, and reticulocyte counts also are important to monitor in this patient, the absolute neutrophil count is used to evaluate the effects of filgrastim.

When caring for a patient with renal failure on a low phosphate diet, the nurse will inform unlicensed assistive personnel (UAP) to remove which food from the patient's food tray? a. Grape juice b. Milk carton c. Mixed green salad d. Fried chicken breast

Milk carton

A patient with newly diagnosed lung cancer tells the nurse, "I don't think I'm going to live to see my next birthday." Which response by the nurse is best? a. "Would you like to talk to the hospital chaplain about your feelings?" b. "Can you tell me what it is that makes you think you will die so soon?" c. "Are you afraid that the treatment for your cancer will not be effective?" d. "Do you think that taking an antidepressant medication would be helpful?"

"Can you tell me what it is that makes you think you will die so soon?"

A patient has a parenteral nutrition infusion of 25% dextrose. A student nurse asks the nurse why a peripherally inserted central catheter was inserted. Which response by the nurse is most appropriate? a. "There is a decreased risk for infection when 25% dextrose is infused through a central line." b. "The prescribed infusion can be given much more rapidly when the patient has a central line." c. "The 25% dextrose is hypertonic and will be more rapidly diluted when given through a central line." d. "The required blood glucose monitoring is more accurate when samples are obtained from a central line."

"The 25% dextrose is hypertonic and will be more rapidly diluted when given through a central line."

A 52-year-old woman is brought to the ED by her husband for altered mental status for 1 day. The patient has hypertension and diabetes but has not been taking her medications for the last 5 days since she lost her insurance and could not afford her prescriptions. Her BP is 168/91 mm Hg, HR is 125 beats per minute, temperature is 99.8°F, and RR is 18 breaths per minute. Her fingerstick glucose is 900 mg/dL. There is glucose in her urine, but no ketones. Which of the following is the most appropriate next step in management? a. Administer IV fluids and insulin b. Obtain head CT scan c. Obtain ECG d. Obtain chest radiograph and urine culture e. Administer broad coverage antibiotics

*A* Profound hyperglycemia, absence of ketonuria, and diabetes medication noncompliance should raise your suspicion for nonketotic hyperosmolar crisis (NKHC) in this patient. This condition is a syndrome of hyperglycemia without ketoacidosis as small amounts of insulin protect against adipose tissue metabolism. This syndrome is more common in type 2 diabetics. Causes of NKHC are similar to those of DKA and include diabetes medication noncompliance, infection, stroke, and myocardial infarction (MI). Patients are profoundly dehydrated because of osmotic diuresis. The mainstay of NKHC therapy consists of replacing fluid losses. Electrolyte deficiencies should be replaced and insulin administered. Fluid deficit in NKHC is significant and needs to be slowly corrected, as rapid correction may lead to cerebral edema. Insulin requirements in NKHC are usually less than in DKA.

A 64-year-old man presents to the ED complaining of knee pain since yesterday. He denies trauma or similar presentation in the past. On examination, you note an erythematous, tender, and swollen knee. Radiographs do not reveal a fracture, but do show calcium deposits. Arthrocentesis demonstrates 20,000/μL white blood cells (WBCs) with a predominance of neutrophils, a negative Gram stain, and rhomboid shaped crystals that are positively birefringent under polarized light. Which of the following is the most likely diagnosis? a. Pseudogout b. Gout c. Septic joint d. Rheumatoid arthritis e. Osteoarthritis

*A* Pseudogout is the most common cause of acute monoarticular arthritis in the elderly. It affects woman and men equally, primarily after their sixth decade of life. It is caused by the deposition of calcium pyrophosphate crystals. The knee is the most commonly involved joint, followed by the wrist and ankle. The synovial fluid reveals rhomboid-shaped crystals that are weakly positive birefringent under polarized light. Treatment is generally supportive with NSAIDs.

A 22-year-old soccer player presents to the ED complaining of right knee pain and swelling. He states that earlier in the day he was in a soccer match and was running for the ball, but stopped abruptly and tried to run in a new direction. Immediately thereafter, he felt intense pain in his knee with instant swelling. Which of the following is the most commonly injured major ligament of the knee? a. Anterior cruciate ligament (ACL) b. Posterior cruciate ligament (PCL) c. Medial collateral ligament (MCL) d. Lateral collateral ligament (LCL) e. Patella ligament

*A* The cruciate ligaments are two internal bands extending from the tibia to the femur, one anteriorly and the other posteriorly. They control anteroposterior and rotary stability of the knee and prevent hyperextension. The ACL is the most frequently injured ligament in the knee. It has a rich blood supply which accounts for the high incidence of hemarthrosis when the ligament is injured. A history that includes a pop or snap at the time of injury (eg, during a sudden turn in direction while playing sports) suggests a rupture of the ACL until proven otherwise, particularly when associated with the rapid development of a knee effusion.

A 43-year-old warehouse worker is helping to stock large refrigerator boxes onto the forklift. One of the boxes starts to fall off the shelf and he tries to catch it before it falls. In doing so, his hand is hyperextended at the wrist and causes immediate pain. In the ED, you note limitation of the normal motion of the wrist with palpable fullness on the volar aspect. The patient also complains of tingling in the distribution of the median nerve. Which of the following injuries is most frequently associated with the development of acute carpal tunnel syndrome? a. Lunate dislocation b. Perilunate dislocation c. Scapholunate dislocation d. Capitate dislocation e. Scaphoid fracture

*A* The median nerve runs through the carpal tunnel between the flexor carpi radialis and the palmaris longus. It provides sensation to the palmar aspect of the radial three and one-half fingers as well as the dorsal aspect of the tips of the index and middle fingers and the radial half of the ring finger. Lunate dislocations are usually caused by hyperextension injuries. The lunate is usually displaced volarly or dorsally. The median nerve may be compressed in the carpal tunnel by the lunate, and the patient may display signs of acute carpal tunnel syndrome.

A 51-year-old woman presents to the ED with heavy vaginal bleeding. She reports that she has bleeding for 12 consecutive days. She also reports missing a period 3 months ago, with an ensuing menses that she states was quite heavy. She denies having any hot flashes, vaginal dryness, night sweats, or changes in weight. The patient further denies any abdominal or back pain, syncope, palpitations but states that recently she often feels lightheaded and can't perform her normal activities of daily living. Her initial vital signs include HR of 110 beats per minute, BP of 140/88 mm Hg, RR of 18 breaths per minute with oxygen saturation of 98% on room air. Upon physical examination, she appears pale with a nontender abdomen and warm skin. Her pelvic examination reveals blood clots in the vaginal vault with a closed os, an enlarged uterus, and no adnexal tenderness. An ultrasound is performed which does not show any abnormalities. Which of the following diagnostic tests is most appropriate for this patient? a. Endometrial biopsy b. Hormonal therapy trial c. Laparoscopic examination d. Dilation and curettage e. Hysterectomy and salpingectomy

*A* The menstrual cycle consists of a follicular phase and a luteal phase. The follicular, or proliferative, phase begins on the first day of menses and continues until ovulation whereupon the luteal, or secretory, phase begins. During the follicular phase, endometrial glands form under the influences of estrogen, primarily estradiol. In the luteal phase, estrogen is still present but progesterone takes over and is mainly responsible for endometrial secretion and prepares the lining of the uterus for implantation. The luteal phase is characterized by an elevated body temperature and stromal edema. When implantation does not occur, menses ensues as a result of falling hormonal levels, which cause coiling and constrict the endometrial arteries. An elevated β-hCG maintains the corpus luteum. Dysfunctional uterine bleeding (DUB) is any abnormality in the regular bleeding pattern of the menstrual cycle and anovulation is the usual cause. This is the ovaries' failure to secrete an ovum, which thereby prevents luteinization from occurring. Without progesterone, the endometrium proliferates. This can lead to endometrial hyperplasia, which increases the risk for carcinoma. Other causes of symptoms of DUB include uterine fibroids, uterine polyps, genital tract trauma, exogenous estrogens, endocrine axis dysfunction, and bleeding disorders. Menopause occurs on average at the age of 51.5 years. Given that this woman is close to this age, this could also be perimenopausal symptoms. Hormone levels may be checked to see if they are falling. However, endometrial carcinoma may present in a similar way and needs to be ruled out with an endometrial biopsy.

A 67-year-old woman with a history of hypertension and congestive heart failure presents with "burning" epigastric pain that began 2 hours after eating a meal. She states that she has had similar pain over the past several weeks, and has been taking antacids and a medication that her primary-care physician had prescribed with moderate relief. The pain has occurred with increasing frequency and now awakens her from sleep. She states she came to the ED today because the pain was not relieved with her usual medications. She denies nausea, vomiting, diarrhea, or fever. She also denies hematemesis, black stool, or bright red blood per rectum. On physical examination, she is tender at the epigastrium, with an otherwise normal abdominal, pulmonary, and heart examination. Stool guaiac tests positive for occult blood. Which of the following is the most common serious complication of peptic ulcer disease? a. GI hemorrhage b. GI perforation c. GI penetration d. Gastric outlet obstruction e. Pernicious anemia

*A* The most serious complications of PUD include hemorrhage, perforation, penetration, and gastric outlet obstruction. Hemorrhage, which occurs in 15 of patients, is the most common complication.

A 35-year-old woman with systemic lupus erythematosus (SLE) is brought to the ED by her brother after he found her febrile and confused. Physical examination reveals fever, tachycardia, a waxing and waning mental status, petechiae over her oral mucosa, pallor, and mildly heme-positive stool. Her urinalysis is positive for blood, red cell casts, and proteinuria. Laboratory results reveal a blood urea nitrogen (BUN) of 40 mg/dL, and a creatinine of 2 mg/dL. Her bilirubin is elevated (unconjugated > conjugated) and her international normalized ratio (INR) is 0.98. Her complete blood count reveals WBC 12,000/μL, hematocrit 29%, and platelet count 17,000/μL with schistocytes on the peripheral smear. Which of the following is the most appropriate next step in management? a. Admit to the intensive care unit (ICU) for plasmapheresis and close monitoring for acute bleeds. b. Admit to the ICU for platelet transfusion and monitoring for acute bleeds. c. Begin corticosteroids, transfuse platelets, and call surgery for immediate splenectomy. d. Admit to the ICU for dialysis and close monitoring for acute bleeds. e. Perform a noncontrast head computed tomography (CT), followed by a lumbar puncture (LP) for analysis of cerebrospinal fluid.

*A* This patient has thrombotic thrombocytopenic purpura (TTP), caused by increased platelet destruction. In TTP, platelet-fibrin thrombi deposit in vessel and cause injury to RBCs and platelets, resulting in microangiopathy hemolytic anemia and thrombocytopenia. Patients tend to be females who are 10 to 45 years of age. Risk factors include pregnancy, autoimmune disorders (eg, systemic lupus erythematosus [SLE]), infection, allogenic bone marrow transplantation, malignancy, and certain medications. The pentad can be remembered with the mnemonic FAT RN: fever, hemolytic anemia, thrombocytopenia, renal failure, and neurologic change (waxing and waning mental status). Treatment includes daily plasmapheresis until platelet count normalizes. RBCs may be transfused in patients ymptomatic from anemia (eg, tachycardia, hypoxia, orthostatic hypotension). All patients with TTP should be admitted to an ICU for close monitoring of acute bleeds.

A 19-year-old man was struck by a motor vehicle while crossing the street. In the ED, he is awake, alert, and oriented, but complaining of severe right-leg pain. His temperature is 98.9°F, BP is 85/50 mm Hg, HR is 125 beats per minute, and RR is 24 breaths per minute. You confirm that his airway is patent, breath sounds are equal bilaterally, and his abdomen is soft and nontender. His right leg is shorter than his left leg, slightly angulated, and swollen in his anterior thigh area. There is no open wound. Which of the following is the most likely diagnosis? a. Hypovolemic shock b. Neurogenic shock c. Cardiogenic shock d. Anaphylactic shock e. Septic shock

*A* This patient is in hypovolemic shock secondary to blood loss from a femoral fracture. Hypovolemic shock occurs when there is inadequate volume in the circulatory system, resulting in poor oxygen delivery to the tissues. Hemorrhage, GI losses, burns, and environmental exposures can all be responsible for hypovolemic shock. In trauma, hemorrhage is the most common cause of hypovolemic shock. This patient fractured his femur, disrupting the nearby vascular supply, resulting in significant blood collection in the soft tissue.

A 58-year-old man is brought to the ED for a syncopal episode at dinner. His wife states that he was well until she found him suddenly slumping in the chair and losing consciousness for a minute. The patient recalls having some chest discomfort and shortness of breath prior to the episode. His rhythm strip, shows prolonged PR interval and random dropped beats Which of the following best describes these findings? a. Mobitz type I b. Mobitz type II c. First-degree atrioventricular (AV) block d. Atrial flutter with premature ventricular contractions (PVCs) e. Sinus bradycardia

*A.* The rhythm strip shows seconddegree AV block type II or Mobitz type II. Mobitz II presents with a prolonged PR interval (PR > 0.2 seconds) and random dropped beats (ie, P wave without QRS complex). The PR intervals are always the same duration. The block is below the level of the AV node, generally the His-Purkinje system. This heart block reflects serious cardiac pathology and may be seen with an anterior wall MI, which is the case with this patient

A 49-year-old man presents to the ED with pain, erythema, and swelling to his left forearm after a chemical spill sustained at work. He irrigated the area with water and applied a cold packet before arriving but states that the burning sensation in his arm is now worse. The patient works in a glass factory and reports using a rust-removing agent. What tissue-saving treatment must be administered emergently? a. Limb tourniquet b. Calcium gluconate c. Alkalinization of urine d. Silver sulfadiazine ointment e. Surgical debridement

*B* Chemical burns are a common occupational hazard and can be caused by a variety of solvents containing acidic or alkali mixtures. Initially, it is important to remove the patient from the offending chemical and irrigate with copious amounts of water to dilute the agent. It is important to assess the affected area, size, and depth of the burn, as some patients will need transfer to a burn center. This patient was chemically burned by a rust-removing agent, which commonly contains hydrofluoric acid (HF). Other solvents include high-octane gas and germicides. Pain out of proportion to sustained injury is usually seen in these cases. As HF is a relatively weak acid, its extreme electronegativity however makes it very dangerous. Fluoride avidly binds to available cations, such as calcium and magnesium, thereby causing cell death. Profound hypocalcemia has been demonstrated in HF exposure. ECG monitoring and administration of exogenous cations, such as topical calcium gluconate gel, can act as chelating agents to the fluoride ions. IV or intradermal calcium gluconate may also be used. Calcium chloride should be avoided given its ability to cause tissue necrosis upon extravasation.

Which information will the nurse include when teaching a patient how to avoid chronic constipation (select all that apply)? a. Many over-the-counter (OTC) medications can cause constipation. b. Stimulant and saline laxatives can be used regularly. c. Bulk-forming laxatives are an excellent source of fiber. d. Walking or cycling frequently will help bowel motility. e. A good time for a bowel movement may be after breakfast.

-Many over-the-counter (OTC) medications can cause constipation. -Bulk-forming laxatives are an excellent source of fiber. -Walking or cycling frequently will help bowel motility. -A good time for a bowel movement may be after breakfast.

What would be the indication for doing a rigid bronchoscopy

1. Pediatric bronchoscopy 2.Massive hemoptysis 3. Removal of large foreign body 4. Biopsy of large airway lesions or proximal lesions at risk of bleeding

Complications of Cancer Infection

1. Primary cause of death in cancer patients 2. Some sites of infection include: Lungs Genitourinary tract system Mouth, rectum Peritoneal cavity, blood 3. Occurs due to: Ulceration Compression of vital organs by the tumor Neutropenia caused by disease or treatment

Therapeutic Bronchoscopy

1. Retained secretions, mucus plugs, clots 2.Foreign body extraction 3.Laser therapy- reducing size of tumors 4.Brachytherapy-radioactive pellets 5.Stent placement-benign tracheal bronchial stenosis 6.Dilatation 7.Treatment of hemoptysis- Saline or EPi injection or tamponade

14. You are monitoring your patient who is at risk for spinal cord compression related to tumor growth. Which patient statement is most likely to suggest an early manifestation? 1. "Last night my back really hurt, and I had trouble sleeping." 2. "My leg has been giving out when I try to stand." 3. "My bowels are just not moving like they usually do." 4. "When I try to pass urine, I have difficulty starting the stream."

14. Ans: 1 Back pain is an early sign of spinal cord compression occurring in 95% of patients. The other symptoms are later signs. Focus: Prioritization

After change-of-shift report, which patient should the nurse assess first? a. 72-year-old with cor pulmonale who has 4+ bilateral edema in his legs and feet b. 28-year-old with a history of a lung transplant and a temperature of 101° F (38.3° C) c. 40-year-old with a pleural effusion who is complaining of severe stabbing chest pain d. 64-year-old with lung cancer and tracheal deviation after subclavian catheter insertion

64-year-old with lung cancer and tracheal deviation after subclavian catheter insertion

Which patient should the nurse assign as the roommate for a patient who has aplastic anemia? a. A patient with chronic heart failure b. A patient who has viral pneumonia c. A patient who has right leg cellulitis d. A patient with multiple abdominal drains

A patient with chronic heart failure Patients with aplastic anemia are at risk for infection because of the low white blood cell production associated with this type of anemia, so the nurse should avoid assigning a roommate with any possible infectious process.

After receiving change-of-shift report on a medical unit, which patient should the nurse assess first? a. A patient with cystic fibrosis who has thick, green-colored sputum b. A patient with pneumonia who has crackles bilaterally in the lung bases c. A patient with emphysema who has an oxygen saturation of 90% to 92% d. A patient with septicemia who has intercostal and suprasternal retractions

A patient with septicemia who has intercostal and suprasternal retractions

A patient who was admitted the previous day with pneumonia complains of a sharp pain of 7 (based on 0 to 10 scale) "whenever I take a deep breath." Which action will the nurse take next? a. Auscultate breath sounds. b. Administer the PRN morphine. c. Have the patient cough forcefully. d. Notify the patient's health care provider.

Auscultate breath sounds.

When caring for a patient who is hospitalized with active tuberculosis (TB), the nurse observes a student nurse who is assigned to take care of a patient. Which action, if performed by the student nurse, would require an intervention by the nurse? a. The patient is offered a tissue from the box at the bedside. b. A surgical face mask is applied before visiting the patient. c. A snack is brought to the patient from the unit refrigerator. d. Hand washing is performed before entering the patient's room.

A surgical face mask is applied before visiting the patient.

In planning care for a patient with metastatic liver cancer, the nurse should include interventions that A. focus primarily on symptomatic and comfort measures b. reassure the patient that chemotherapy offers a good prognosis c. promote the patients confidence that surgical excision of the tumor will be successful d. provide information necessary for the patient to make decisions regarding liver transplantation

A.

When planning a community education program related to prevention of the cancer with the highest death rates in both women and men, the nurse will stress the importance of: a. Smoking cessation b. Screening with colonoscopy c. Regular examination of reproductive organs d. Use of sunscreen and protection from ultraviolet light

A. Rationale: The highest number of cancer deaths by site has been reported for cancer of the lung and bronchus. Smoking is the leading cause of lung and bronchial cancer.

What is the most common symptom of lung cancer? 1 Fatigue 2 Anorexia 3 Hoarseness 4 Persistent cough

Answer 4

After 2 months of tuberculosis (TB) treatment with isoniazid (INH), rifampin (Rifadin), pyrazinamide (PZA), and ethambutol, a patient continues to have positive sputum smears for acid-fast bacilli (AFB). Which action should the nurse take next? a. Teach about treatment for drug-resistant TB treatment. b. Ask the patient whether medications have been taken as directed. c. Schedule the patient for directly observed therapy three times weekly. d. Discuss with the health care provider the need for the patient to use an injectable antibiotic.

Ask the patient whether medications have been taken as directed.

Which instruction will the nurse plan to include in discharge teaching for the patient admitted with a sickle cell crisis? a. Take a daily multivitamin with iron. b. Limit fluids to 2 to 3 quarts per day. c. Avoid exposure to crowds when possible. d. Drink only two caffeinated beverages daily.

Avoid exposure to crowds when possible. Exposure to crowds increases the patient's risk for infection, the most common cause of sickle cell crisis. There is no restriction on caffeine use. Iron supplementation is generally not recommended. A high-fluid intake is recommended.

The nurse is caring for a 78-year-old patient who was hospitalized 2 days earlier with community-acquired pneumonia. Which assessment information is most important to communicate to the health care provider? a. Scattered crackles bilaterally in the posterior lung bases. b. Persistent cough that is productive of blood-tinged sputum. c. Temperature of 101.5° F (38.6° C) after 2 days of IV antibiotic therapy. d Decreased oxygen saturation to 90% with 100% O2 by non-rebreather mask.

Decreased oxygen saturation to 90% with 100% O2 by non-rebreather mask.

Which activity in the care of a 48-year-old female patient with a new colostomy could the nurse delegate to unlicensed assistive personnel (UAP)? a. Document the appearance of the stoma. b. Place a pouching system over the ostomy. c. Drain and measure the output from the ostomy. d. Check the skin around the stoma for breakdown.

Drain and measure the output from the ostomy. Draining and measuring the output from the ostomy is included in UAP education and scope of practice. The other actions should be implemented by LPNs or RNs.

A patient with a possible pulmonary embolism complains of chest pain and difficulty breathing. The nurse finds a heart rate of 142 beats/minute, blood pressure of 100/60 mmHg, and respirations of 42 breaths/minute. Which action should the nurse take first? a. Administer anticoagulant drug therapy. b. Notify the patient's health care provider. c. Prepare patient for a spiral computed tomography (CT). d. Elevate the head of the bed to a semi-Fowler's position.

Elevate the head of the bed to a semi-Fowler's position.

A postoperative patient who had surgery for a perforated gastric ulcer has been receiving nasogastric suction for 3 days. The patient now has a serum sodium level of 127 mEq/L (127 mmol/L). Which prescribed therapy should the nurse question? a. Infuse 5% dextrose in water at 125 mL/hr. b. Administer IV morphine sulfate 4 mg every 2 hours PRN. c. Give IV metoclopramide (Reglan) 10 mg every 6 hours PRN for nausea. d. Administer 3% saline if serum sodium decreases to less than 128 mEq/L.

Infuse 5% dextrose in water at 125 mL/hr.

Which action is appropriate for the nurse to delegate to unlicensed assistive personnel (UAP)?

Label specimens obtained during percutaneous lung biopsy. Labeling of specimens is within the scope of practice of UAP.

A patient with acute respiratory distress syndrome (ARDS) who is intubated and receiving mechanical ventilation develops a right pneumothorax. Which action will the nurse anticipate taking next? a. Increase the tidal volume and respiratory rate. b. Increase the fraction of inspired oxygen (FIO2). c. Perform endotracheal suctioning more frequently. d. Lower the positive end-expiratory pressure (PEEP).

Lower the positive end-expiratory pressure (PEEP)

An older patient receiving iso-osmolar continuous tube feedings develops restlessness, agitation, and weakness. Which laboratory result should the nurse report to the health care provider immediately? a. K+ 3.4 mEq/L (3.4 mmol/L) b. Ca+2 7.8 mg/dL (1.95 mmol/L) c. Na+ 154 mEq/L (154 mmol/L) d. PO4-3 4.8 mg/dL (1.55 mmol/L)

Na+ 154 mEq/L (154 mmol/L)

Which menu choice by the patient with diverticulosis is best for preventing diverticulitis? a. Navy bean soup and vegetable salad b. Whole grain pasta with tomato sauce c. Baked potato with low-fat sour cream d. Roast beef sandwich on whole wheat bread

Navy bean soup and vegetable salad A diet high in fiber and low in fats and red meat is recommended to prevent diverticulitis. Although all of the choices have some fiber, the bean soup and salad will be the highest in fiber and the lowest in fat.

A patient who has just been admitted with community-acquired pneumococcal pneumonia has a temperature of 101.6° F with a frequent cough and is complaining of severe pleuritic chest pain. Which prescribed medication should the nurse give first? a. Codeine b. Guaifenesin (Robitussin) c. Acetaminophen (Tylenol) d. Piperacillin/tazobactam (Zosyn)

Piperacillin/tazobactam (Zosyn)

A patient with a pleural effusion is scheduled for a thoracentesis. Which action should the nurse take to prepare the patient for the procedure? a. Start a peripheral IV line to administer the necessary sedative drugs. b. Position the patient sitting upright on the edge of the bed and leaning forward. c. Obtain a large collection device to hold 2 to 3 liters of pleural fluid at one time. d. Remove the water pitcher and remind the patient not to eat or drink anything for 6 hours.

Position the patient sitting upright on the edge of the bed and leaning forward.

Which collaborative problem will the nurse include in a care plan for a patient admitted to the hospital with idiopathic aplastic anemia? a. Potential complication: seizures b. Potential complication: infection c. Potential complication: neurogenic shock d. Potential complication: pulmonary edema

Potential complication: infection Because the patient with aplastic anemia has pancytopenia, the patient is at risk for infection and bleeding. There is no increased risk for seizures, neurogenic shock, or pulmonary edema.

A 58-year-old man with blunt abdominal trauma from a motor vehicle crash undergoes peritoneal lavage. If the lavage returns brown fecal drainage, which action will the nurse plan to take next? a. Auscultate the bowel sounds. b. Prepare the patient for surgery. c. Check the patient's oral temperature. d. Obtain information about the accident.

Prepare the patient for surgery. Return of brown drainage and fecal material suggests perforation of the bowel and the need for immediate surgery. Auscultation of bowel sounds, checking the temperature, and obtaining information about the accident are appropriate actions, but the priority is to prepare to send the patient for emergency surgery.

An occupational health nurse works at a manufacturing plant where there is potential exposure to inhaled dust. Which action, if recommended by the nurse, will be most helpful in reducing the incidence of lung disease?

Require the use of protective equipment Prevention of lung disease requires the use of appropriate protective equipment such as masks

The nurse is caring for a patient with left-sided lung cancer. Which finding would be most important for the nurse to report to the health care provider? a. Hematocrit 32% b. Pain with deep inspiration c. Serum sodium 126 mEq/L d. Decreased breath sounds on left side

Serum sodium 126 mEq/L

The nurse observes a student who is listening to a patient's lungs who is having no problems with breathing. Which action by the student indicates a need to review respiratory assessment skills?

The student places the stethoscope over the posterior chest and listens during inspiration Listening only during inspiration indicates the student needs a review of respiratory assessment skills. At each placement of the stethoscope, listen to at least

A patient is admitted with active tuberculosis (TB). The nurse should question a health care provider's order to discontinue airborne precautions unless which assessment finding is documented?

Three sputum smears for acid-fast bacilli are negative. Negative sputum smears indicate that Mycobacterium tuberculosis is not present in the sputum, and the patient cannot transmit the bacteria by the airborne route

A patient with a history of a transfusion-related acute lung injury (TRALI) is to receive a transfusion of packed red blood cells (PRBCs). Which action by the nurse will decrease the risk for TRALI for this patient? a. Infuse the PRBCs slowly over 4 hours. b. Transfuse only leukocyte-reduced PRBCs. c. Administer the scheduled diuretic before the transfusion. d. Give the PRN dose of antihistamine before the transfusion.

Transfuse only leukocyte-reduced PRBCs. TRALI is caused by a reaction between the donor and the patient leukocytes that causes pulmonary inflammation and capillary leaking. The other actions may help prevent respiratory problems caused by circulatory overload or by allergic reactions, but they will not prevent TRALI.

Obstructive emergencies: Spinal cord compression

Tumor in epidural space of spinal cord; signs include Intense, localized, persistent back pain Motor weakness Sensory paresthesia and loss Change in bladder or bowel function

Employee health test results reveal a tuberculosis (TB) skin test of 16-mm induration and a negative chest x-ray for a staff nurse working on the pulmonary unit. The nurse has no symptoms of TB. Which information should the occupational health nurse plan to teach the staff nurse? a. Standard four-drug therapy for TB b. Need for annual repeat TB skin testing c. Use and side effects of isoniazid (INH) d. Bacille Calmette-Guérin (BCG) vaccine

Use and side effects of isoniazid (INH)

A nurse is assessing a patient who is receiving a nitroprusside (Nipride) infusion to treat cardiogenic shock. Which finding indicates that the medication is effective? a. No new heart murmurs b. Decreased troponin level c. Warm, pink, and dry skin d. Blood pressure 92/40 mm Hg

Warm, pink, and dry skin Warm, pink, and dry skin indicates that perfusion to tissues is improved. Since nitroprusside is a vasodilator, the blood pressure may be low even if the medication is effective. Absence of a heart murmur and a decrease in troponin level are not indicators of improvement in shock.

The nurse reviews the laboratory results of a patient who is receiving chemotherapy. Which laboratory result is most important to report to the health care provider? a. Hematocrit of 30% b. Platelets of 95,000/µL c. Hemoglobin of 10 g/L d. White blood cell (WBC) count of 2700/µL

White blood cell (WBC) count of 2700/µL

The most normal functioning method of speech restoration in the patient with a total laryngectomy is a. a voice prosthesis b. esophageal speech c. an electrolarynx held at the neck an electrolarynx placed in the mouth

a. Rationale: The voice prosthesis provides the most normal voice reproduction but requires surgical insertion of the device.

The nurse admitting a patient with acute diverticulitis explains that the initial plan of care is to a. administer IV fluids. b. give stool softeners and enemas. c. order a diet high in fiber and fluids. d. prepare the patient for colonoscopy.

administer IV fluids. A patient with acute diverticulitis will be NPO and given parenteral fluids. A diet high in fiber and fluids will be implemented before discharge. Bulk-forming laxatives, rather than stool softeners, are usually given, and these will be implemented later in the hospitalization. The patient with acute diverticulitis will not have enemas or a colonoscopy because of the risk for perforation and peritonitis.

For what diagnostic test that is used to identify the location and extent of masses in the chest will the nurse prepare the patient with lung cancer? 1 Chest x-ray 2 Mediastinoscopy 3 Computed tomography 4 Video-assisted thoracoscopic surgery

answer: 3

The nurse explains to a patient undergoing brachytherapy of the cervix that she A. must undergo simulation to locate the treatment area b requires the use of radioactive precautions during nursing care c. may experience desquamation of the skin on the abdomen and upper legs d. requires shielding of the ovaries during treatment to prevent ovarian damage

b

A 28-year-old man with von Willebrand disease is admitted to the hospital for minor knee surgery. The nurse will review the coagulation survey to check the a. platelet count. b. bleeding time. c. thrombin time. d. prothrombin time.

bleeding time. The bleeding time is affected by von Willebrand disease. Platelet count, prothrombin time, and thrombin time are normal in von Willebrand disease.

When assessing a patient who has just arrived after an automobile accident, the emergency department nurse notes tachycardia and absent breath sounds over the right lung. For which intervention will the nurse prepare the patient?

c. Insertion of a chest tube with a chest drainage system The patient's history and absent breath sounds suggest a right-sided pneumothorax or hemothorax, which will require treatment with a chest tube and drainage.

The nurse caring for a patient with type A hemophilia being admitted to the hospital with severe pain and swelling in the right knee will a. immobilize the joint. b. apply heat to the knee. c. assist the patient with light weight bearing. d. perform passive range of motion to the knee.

immobilize the joint. The initial action should be total rest of the knee to minimize bleeding. Ice packs are used to decrease bleeding. Range of motion (ROM) and weight-bearing exercise are contraindicated initially, but after the bleeding stops, ROM and physical therapy are started.

A 74-year-old patient preparing to undergo a colon resection for cancer of the colon asks about the elevated carcinoembryonic antigen (CEA) test result. The nurse explains that the test is used to a. identify any metastasis of the cancer. b. monitor the tumor status after surgery. c. confirm the diagnosis of a specific type of cancer. d. determine the need for postoperative chemotherapy.

monitor the tumor status after surgery. CEA is used to monitor for cancer recurrence after surgery. CEA levels do not help to determine whether there is metastasis of the cancer. Confirmation of the diagnosis is made on the basis of biopsy. Chemotherapy use is based on factors other than CEA.

The nurse will plan to provide teaching for a 67-year-old patient who has been diagnosed with orchitis about a. pain management. b. emergency surgical repair. c. application of heat to the scrotum. d. aspiration of fluid from the scrotal sac.

pain management. Orchitis is very painful, and effective pain management will be needed. Heat, aspiration, and surgery are not used to treat orchitis.

A 57-year-old patient is incontinent of urine following a radical retropubic prostatectomy. The nurse will plan to teach the patient a. to restrict oral fluid intake. b. pelvic floor muscle exercises. c. to perform intermittent self-catheterization. d. the use of belladonna and opium suppositories.

pelvic floor muscle exercises. Pelvic floor muscle training (Kegel) exercises are recommended to strengthen the pelvic floor muscles and improve urinary control. Belladonna and opium suppositories are used to reduce bladder spasms after surgery. Intermittent self-catheterization may be taught before surgery if the patient has urinary retention, but it will not be useful in reducing incontinence after surgery. The patient should have a daily oral intake of 2 to 3 L.

A 76-year-old patient who has been diagnosed with stage 2 prostate cancer chooses the option of active surveillance. The nurse will plan to a. vaccinate the patient with sipuleucel-T ( Provenge). b. provide the patient with information about cryotherapy. c. teach the patient about placement of intraurethral stents. d. schedule the patient for annual prostate-specific antigen testing.

schedule the patient for annual prostate-specific antigen testing. Patients who opt for active surveillance need to have annual digital rectal exams and prostate-specific antigen testing. Vaccination with sipuleucel-T, cryotherapy, and stent placement are options for patients who choose to have active treatment for prostate cancer.

Thoracentesis procedure

step 1 confirm presence of fluid with Xray or CT *Usually done with patient sitting upright and leaning slightly forward *Typically should avoid tapping below the 9th interspace *Dept of the fluid should be at least 10mm. If doubt exists obtain a decubitus view to confirm-a fluid level at least 1cm wide should be present (amount of fluid determines the procedure and risk) Pleural fluid with less than a 1cm level usually don't require tap and may be difficult technically (increased risk)

Re-expansion pulmonary edema

what is a potential complication of thoracentesis (especially when more than 1L is taken off), What can the rapid re-expansion of a collapsed lung lead to?

The nurse is caring for a patient who has been diagnosed with stage I cancer of the colon. When assessing the need for psychologic support, which question by the nurse will provide the most information? a. "How long ago were you diagnosed with this cancer?" b. "Do you have any concerns about body image changes?" c. "Can you tell me what has been helpful to you in the past when coping with stressful events?" d. "Are you familiar with the stages of emotional adjustment to a diagnosis like cancer of the colon?"

"Can you tell me what has been helpful to you in the past when coping with stressful events?"

A 22-year-old man tells the nurse at the health clinic that he has recently had some problems with erectile dysfunction. Which question should the nurse ask first to assess for possible etiologic factors? a. "Do you experience an unusual amount of stress?" b. "Do you use any recreational drugs or drink alcohol?" c. "Do you have chronic cardiovascular or peripheral vascular disease?" d. "Do you have a history of an erection that lasted for 6 hours or more?"

"Do you use any recreational drugs or drink alcohol?" A common etiologic factor for erectile dysfunction (ED) in younger men is use of recreational drugs or alcohol. Stress, priapism, and cardiovascular illness also contribute to ED, but they are not common etiologic factors in younger men.

When obtaining the pertinent health history for a man who is being evaluated for infertility, which question is most important for the nurse to ask? a. "Are you circumcised?" b. "Have you had surgery for phimosis?" c. "Do you use medications to improve muscle mass?" d. "Is there a history of prostate cancer in your family?"

"Do you use medications to improve muscle mass?" Testosterone or testosterone-like medications may adversely affect sperm count. The other information will be obtained in the health history but does not affect the patient's fertility.

A patient with urinary obstruction from benign prostatic hyperplasia (BPH) tells the nurse, "My symptoms are much worse this week." Which response by the nurse is most appropriate? a. "Have you been taking any over-the-counter (OTC) medications recently?" b. "I will talk to the doctor about ordering a prostate specific antigen (PSA) test." c. "Have you talked to the doctor about surgery such as transurethral resection of the prostate (TURP)?" d. "The prostate gland changes in size from day to day, and this may be making your symptoms worse."

"Have you been taking any over-the-counter (OTC) medications recently?" Because the patient's increase in symptoms has occurred abruptly, the nurse should ask about OTC medications that might cause contraction of the smooth muscle in the prostate and worsen obstruction. The prostate gland does not vary in size from day to day. A TURP may be needed, but more assessment about possible reasons for the sudden symptom change is a more appropriate first response by the nurse. PSA testing is done to differentiate BPH from prostatic cancer.

The nurse is admitting a 67-year-old patient with new-onset steatorrhea. Which question is most important for the nurse to ask? a. "How much milk do you usually drink?" b. "Have you noticed a recent weight loss?" c. "What time of day do your bowels move?" d. "Do you eat meat or other animal products?"

"Have you noticed a recent weight loss?" Although all of the questions provide useful information, it is most important to determine if the patient has an imbalance in nutrition because of the steatorrhea.

The nurse is performing tuberculosis (TB) skin tests in a clinic that has many patients who have immigrated to the United States. Which question is most important for the nurse to ask before the skin test? a. "Is there any family history of TB?" b. "How long have you lived in the United States?" c. "Do you take any over-the-counter (OTC) medications?" d. "Have you received the bacille Calmette-Guérin (BCG) vaccine for TB?"

"Have you received the bacille Calmette-Guérin (BCG) vaccine for TB?"

Which question from the nurse would help determine if a patient's abdominal pain might indicate irritable bowel syndrome? a. "Have you been passing a lot of gas?" b. "What foods affect your bowel patterns?" c. "Do you have any abdominal distention?" d. "How long have you had abdominal pain?"

"How long have you had abdominal pain?" One criterion for the diagnosis of irritable bowel syndrome (IBS) is the presence of abdominal discomfort or pain for at least 3 months. Abdominal distention, flatulence, and food intolerance are also associated with IBS, but are not diagnostic criteria.

A patient undergoing external radiation has developed a dry desquamation of the skin in the treatment area. The nurse teaches the patient about management of the skin reaction. Which statement, if made by the patient, indicates the teaching was effective? a. "I can buy some aloe vera gel to use on the area." b. "I will expose the treatment area to a sun lamp daily." c. "I can use ice packs to relieve itching in the treatment area." d. "I will scrub the area with warm water to remove the scales."

"I can buy some aloe vera gel to use on the area."

The nurse discusses management of upper respiratory infections (URI) with a patient who has acute sinusitis. Which statement by the patient indicates that additional teaching is needed?

"I can use my nasal decongestant spray until the congestion is all gone." The nurse should clarify that nasal decongestant sprays should be used for no more than 3 days to prevent rebound vasodilation and congestion

The nurse completes discharge instructions for a patient with a total laryngectomy. Which statement by the patient indicates that additional instruction is needed?

"I must keep the stoma covered with an occlusive dressing at all times."

A 70-year-old patient who has had a transurethral resection of the prostate (TURP) for benign prostatic hyperplasia (BPH) is being discharged from the hospital today, The nurse determines that additional instruction is needed when the patient says which of the following? a. "I should call the doctor if I have incontinence at home." b. "I will avoid driving until I get approval from my doctor." c. "I will increase fiber and fluids in my diet to prevent constipation." d. "I should continue to schedule yearly appointments for prostate exams."

"I should call the doctor if I have incontinence at home." Because incontinence is common for several weeks after a TURP, the patient does not need to call the health care provider if this occurs. The other patient statements indicate that the patient has a good understanding of post-TURP instructions.

The nurse teaches a patient about pulmonary function testing (PFT). Which statement, if made by the patient, indicates teaching was effective?

"I should inhale deeply and blow out as hard as I can during the test." For PFT, the patient should inhale deeply and exhale as long, hard, and fast as possible. The other actions are not needed with PFT

Which patient statement to the nurse indicates a need for additional instruction about taking oral ferrous sulfate? a. "I will call my health care provider if my stools turn black." b. "I will take a stool softener if I feel constipated occasionally." c. "I should take the iron with orange juice about an hour before eating." d. "I should increase my fluid and fiber intake while I am taking iron tablets."

"I will call my health care provider if my stools turn black." It is normal for the stools to appear black when a patient is taking iron, and the patient should not call the doctor about this. The other patient statements are correct.

The nurse completes discharge teaching for a patient who has had a lung transplant. The nurse evaluates that the teaching has been effective if the patient makes which statement? a. "I will make an appointment to see the doctor every year." b. "I will stop taking the prednisone if I experience a dry cough." c. "I will not worry if I feel a little short of breath with exercise." d. "I will call the health care provider right away if I develop a fever."

"I will call the health care provider right away if I develop a fever."

The nurse provides discharge instructions to a patient who was hospitalized for pneumonia. Which statement, if made by the patient, indicates a good understanding of the instructions? a. "I will call the doctor if I still feel tired after a week." b. "I will continue to do the deep breathing and coughing exercises at home." c. "I will schedule two appointments for the pneumonia and influenza vaccines." d. "I'll cancel my chest x-ray appointment if I'm feeling better in a couple weeks."

"I will continue to do the deep breathing and coughing exercises at home."

Spironolactone (Aldactone), an aldosterone antagonist, is prescribed for a patient. Which statement by the patient indicates that the teaching about this medication has been effective? a. "I will try to drink at least 8 glasses of water every day." b. "I will use a salt substitute to decrease my sodium intake." c. "I will increase my intake of potassium-containing foods." d. "I will drink apple juice instead of orange juice for breakfast."

"I will drink apple juice instead of orange juice for breakfast."

The nurse teaches a patient about discharge instructions after a rhinoplasty. Which statement, if made by the patient, indicates that the teaching was successful?

"I will keep my head elevated for 48 hours to minimize swelling and pain." Maintaining the head in an elevated position will decrease the amount of nasal swelling.

The nurse teaches a postmenopausal patient with stage III breast cancer about the expected outcomes of cancer treatment. Which patient statement indicates that the teaching has been effective? a. "After cancer has not recurred for 5 years, it is considered cured." b. "The cancer will be cured if the entire tumor is surgically removed." c. "Cancer is never considered cured, but the tumor can be controlled with surgery, chemotherapy, and radiation." d. "I will need to have follow-up examinations for many years after I have treatment before I can be considered cured."

"I will need to have follow-up examinations for many years after I have treatment before I can be considered cured."

Which patient statement indicates that the nurse's teaching about sulfasalazine (Azulfidine) for ulcerative colitis has been effective? a. "The medication will be tapered if I need surgery." b. "I will need to use a sunscreen when I am outdoors." c. "I will need to avoid contact with people who are sick." d. "The medication will prevent infections that cause the diarrhea."

"I will need to use a sunscreen when I am outdoors." Sulfasalazine may cause photosensitivity in some patients. It is not used to treat infections. Sulfasalazine does not reduce immune function. Unlike corticosteroids, tapering of sulfasalazine is not needed.

The nurse provides discharge teaching for a patient who has two fractured ribs from an automobile accident. Which statement, if made by the patient, would indicate that teaching has been effective? a. "I am going to buy a rib binder to wear during the day." b. "I can take shallow breaths to prevent my chest from hurting." c. "I should plan on taking the pain pills only at bedtime so I can sleep." d. "I will use the incentive spirometer every hour or two during the day."

"I will use the incentive spirometer every hour or two during the day."

The home health nurse cares for an alert and oriented older adult patient with a history of dehydration. Which instructions should the nurse give to this patient related to fluid intake? a. "Increase fluids if your mouth feels dry. b. "More fluids are needed if you feel thirsty." c. "Drink more fluids in the late evening hours." d. "If you feel lethargic or confused, you need more to drink."

"Increase fluids if your mouth feels dry.

A patient who is scheduled for a right breast biopsy asks the nurse the difference between a benign tumor and a malignant tumor. Which answer by the nurse is correct? a. "Benign tumors do not cause damage to other tissues." b. "Benign tumors are likely to recur in the same location." c. "Malignant tumors may spread to other tissues or organs." d. "Malignant cells reproduce more rapidly than normal cells."

"Malignant tumors may spread to other tissues or organs."

The nurse teaches a patient about the transmission of pulmonary tuberculosis (TB). Which statement, if made by the patient, indicates that teaching was effective? a. "I will avoid being outdoors whenever possible." b. "My husband will be sleeping in the guest bedroom." c. "I will take the bus instead of driving to visit my friends." d. "I will keep the windows closed at home to contain the germs."

"My husband will be sleeping in the guest bedroom."

Which statement by the nurse when explaining the purpose of positive end-expiratory pressure (PEEP) to the family members of a patient with ARDS is accurate? a. "PEEP will push more air into the lungs during inhalation." b. "PEEP prevents the lung air sacs from collapsing during exhalation." c. "PEEP will prevent lung damage while the patient is on the ventilator." d. "PEEP allows the breathing machine to deliver 100% oxygen to the lungs."

"PEEP prevents the lung air sacs from collapsing during exhalation."

Which statement by a patient indicates good understanding of the nurse's teaching about prevention of sickle cell crisis? a. "Home oxygen therapy is frequently used to decrease sickling." b. "There are no effective medications that can help prevent sickling." c. "Routine continuous dosage narcotics are prescribed to prevent a crisis." d. "Risk for a crisis is decreased by having an annual influenza vaccination."

"Risk for a crisis is decreased by having an annual influenza vaccination." Because infection is the most common cause of a sickle cell crisis, influenza, Haemophilus influenzae, pneumococcal pneumonia, and hepatitis immunizations should be administered. Although continuous dose opioids and oxygen may be administered during a crisis, patients do not receive these therapies to prevent crisis. Hydroxyurea (Hydrea) is a medication used to decrease the number of sickle cell crises.

A lobectomy is scheduled for a patient with stage I non-small cell lung cancer. The patient tells the nurse, "I would rather have chemotherapy than surgery." Which response by the nurse is most appropriate? a. "Are you afraid that the surgery will be very painful?" b. "Did you have bad experiences with previous surgeries?" c. "Surgery is the treatment of choice for stage I lung cancer." d. "Tell me what you know about the various treatments available."

"Tell me what you know about the various treatments available."

The nurse teaches a patient who is scheduled for a prostate needle biopsy about the procedure. Which statement, if made by the patient, indicates that teaching was effective? a. "The biopsy will remove the cancer in my prostate gland." b. "The biopsy will determine how much longer I have to live." c. "The biopsy will help decide the treatment for my enlarged prostate." d. "The biopsy will indicate whether the cancer has spread to other organs."

"The biopsy will help decide the treatment for my enlarged prostate."

A patient who is diagnosed with cervical cancer that is classified as Tis, N0, M0 asks the nurse what the letters and numbers mean. Which response by the nurse is most appropriate? a. "The cancer involves only the cervix." b. "The cancer cells look almost like normal cells." c. "Further testing is needed to determine the spread of the cancer." d. "It is difficult to determine the original site of the cervical cancer."

"The cancer involves only the cervix."

A 68-year-old woman with acute myelogenous leukemia (AML) asks the nurse whether the planned chemotherapy will be worth undergoing. Which response by the nurse is appropriate? a. "If you do not want to have chemotherapy, other treatment options include stem cell transplantation." b. "The side effects of chemotherapy are difficult, but AML frequently goes into remission with chemotherapy." c. "The decision about treatment is one that you and the doctor need to make rather than asking what I would do." d. "You don't need to make a decision about treatment right now because leukemias in adults tend to progress quite slowly."

"The side effects of chemotherapy are difficult, but AML frequently goes into remission with chemotherapy." This response uses therapeutic communication by addressing the patient's question and giving accurate information. The other responses either give inaccurate information or fail to address the patient's question, which will discourage the patient from asking the nurse for information.

A 32-year-old woman presents to the ED with a persistent fever of 101°F over the last 3 days. The patient states that she used to work as a convenience store clerk but was fired 2 weeks ago. Since then, she has been using drugs intravenously daily. Cardiac examination reveals a heart murmur. Her abdomen is soft and nontender with an enlarged spleen. Chest radiograph reveals multiple patchy infiltrates in both lung fields. Laboratory results reveal white blood cells (WBC) 14,000/μL with 91% neutrophils, hematocrit 33%, and platelets 250/μL. An ECG reveals sinus rhythm with first-degree heart block. Which of the following is the most appropriate next step in management? a. Obtain four sets of blood cultures, order a transthoracic echocardiogram (TTE) and start antibiotic treatment. b. Order a monospot test and recommend that the patient refrain from vigorous activities for 1 month. c. Administer a nonsteroidal anti-inflammatory drug (NSAID) and inform the patient she has pericarditis. d. Administer isoniazid (INH) and report the patient to the Department of Health. e. Order a Lyme antibody and begin antibiotic therapy.

*A* - to R/O endocarditis which has high incidence in IVDA

A 48-year-old man presents to the ED with ethanol intoxication. His BP is 150/70 mm Hg, HR is 95 beats per minute, temperature is 97.9°F, RR is 14 breaths per minute, and oxygen saturation is 93% on room air. The patient is somnolent and snoring loudly with occasional gasps for air. On examination, the patient's gag reflex is intact, his lungs are clear to auscultation, heart is without murmurs, and abdomen is soft and nontender. He is rousable to stimulation. A head CT is negative for intracranial injury. His ethanol level is 270 mg/dL. Which of the following actions is most appropriate to assist the patient with respirations? a. Nasal airway b. Oral airway c. Bag-valve-mask ventilation d. Laryngeal mask airway e. Tracheoesophageal airway

*A* A nasal airway is made of a pliable material that allows it to be placed into the nostril of a somnolent patient with an intact gag reflex. The nasal airway is an excellent device that can be placed in a patient who may have decreased pharyngeal muscle tone and an obstructing soft palate and tongue. It allows air to bypass such obstructions. The patient in the vignette is intoxicated and appears to have episodes of transient obstruction.

A 55-year-old man is brought to the ED by his family. They state that he has been vomiting large amounts of bright red blood. The patient is an alcoholic with cirrhotic liver disease and a history of portal hypertension and esophageal varices. His vitals on arrival are HR 110 beats per minute, BP 80/55 mm Hg, RR 22 breaths per minute, and yemperature 99°F. The patient appears pale and is in moderate distress. Which of the following is an inappropriate option in the initial management of a hypotensive patient with a history of known esophageal varices presenting with hematemesis? a. Sengstaken-Blakemore tube placement b. Two large-bore IV lines and volume repletion with crystalloid solutions c. Nasogastric (NG) lavage d. IV octreotide e. Gastrointestinal (GI) consult

*A* Acute GI bleeding develops in less than one-third of patients with portal hypertension and varices. With upper GI bleeds, the initial step is assessment of the hemodynamic status. Hypotension with or without tachycardia identifies a high-risk patient with severe acute bleeding. This patient requires immediate treatment. If initial resuscitative efforts fail or if a patient remains hypotensive, more aggressive measures may be required, including consideration of Sengstaken-Blakemore tube placement to physically tamponade the bleeding source, but this is not part of the initial management and has been associated with adverse reactions.

A 7-year-old girl with sickle-cell disease and a previous history of admissions for acute painful crises presents with a 1-day history of fever and cough. She is tachypneic on presentation with a temperature of 102°F. Auscultation of the chest reveals rales on the right. A chest radiograph confirms the diagnosis of pneumonia. After initial treatment with antibiotics and intravenous (IV) fluids, patients with this condition are most at risk for developing what complication? a. Acute chest syndrome b. Sepsis as a result of the relative immunodeficiency of patients with sickle-cell disease c. Empyema d. Stroke e. Congestive heart failure because of the anemia coupled with infection

*A* Acute chest syndrome is characterized by pneumonia with pulmonary infarction, hypoxia, and diffuse pulmonary edema. This is one of the most serious and life-threatening complications of sickle-cell disease. One must be very cautious with IV fluid hydration because over aggressive fluid administration can lead to pulmonary edema in patients who have pneumonia or who have previously suffered from acute chest syndrome. Administration of IV antibiotics, transfusions for anemia, and oxygen are the mainstays of therapy.

A 35-year-old man presents to the ED complaining of a headache over the previous 4 weeks. He was assaulted with a bat 4 weeks ago and was admitted to the hospital for observation in the setting of a small traumatic subdural hematoma. Repeat noncontrast CT scan of the head 2 weeks ago was normal with resolution of the hematoma. He states he has headaches several times each day. They last from 5 minutes to several hours. They are sometimes band-like, other times they are localized to the site where he was struck. They can be pulsating or constant and are associated with sensitivity to sound. A head CT scan today is normal. Which of the following is the most likely diagnosis? a. Postconcussive syndrome b. Posttraumatic hydrocephalus c. Subdural hygroma d. Cluster headache e. Posttraumatic stress disorder

*A* After head trauma, 30% to 90% of patients complain of headache during their convalescence. Postconcussive headaches are notable for their variability in frequency, location, and associated symptoms. They are often exacerbated by physical activity or changes in position and may be clinically difficult to distinguish from other headache syndromes. In patients with preexisting migraines, increased frequency of their normal migraine syndrome is often noted. Most patients have resolution of their headaches after 4 weeks. In 20% of patients, their postconcussive headache persists for longer than a year. Headache may be one feature of a larger postconcussive syndrome including nervous system instability. This may include fragmentation of sleep, emotional lability, inability to tolerate crowds, restlessness, inability to concentrate, and anxiety.

A 40-year-old woman presents to the ED complaining of fever and 1 day of increasingly severe pain in her RUQ. She denies nausea or vomiting and has no history of fatty food intolerance. The patient returned from a trip to Mexico 6 months ago. About 2 weeks ago she experienced intermittent diarrhea with blood-streaked mucus. Her BP is 130/80 mm Hg, HR is 107 beats per minute, temperature is 102°F, and RR is 17 breaths per minute. Physical examination reveals decreased breath sounds over the right lung base. Abdominal examination shows tenderness to percussion over the RUQ and normal active bowel sounds. There is no Murphy sign. Her WBC is 20,500/μL. Chest radiograph reveals a small right-pleural effusion. Which of the following is the most likely diagnosis? a. Amebic abscess b. Cholecystitis c. Cryptosporidium d. Enterobiasis e. Pyogenic abscess

*A* Amebic abscesses are common in countries with tropical and subtropical climates and areas with poor sanitation. Entamoeba histolytica causes an intestinal infection, and the liver is seeded via the portal system. The clinical presentation includes abdominal tenderness in the RUQ, leukocytosis, and fever. Diagnosis is supported by identifying a pathogenic protozoan in the stool. Management consists of supportive care and administering metronidazole. If medical therapy is unsuccessful, percutaneous catheter drainage is required.

An undomiciled 49-year-old man presents to the ED with altered mental status. His BP is 149/75 mm Hg, HR is 93 beats per minute, temperature is 97.5°F, RR is 18 breaths per minute, and O2 saturation is 99% on room air. Physical examination reveals an unkempt man with the odor of "alcohol" on his breath. His head is atraumatic and pupils are 4 mm, equal, and reactive. The neck is supple. Cardiovascular, pulmonary, and abdominal examinations are unremarkable. There is no extremity edema and his pulses are 2+ and symmetric. Neurologically, he withdraws all four extremities to deep stimuli. ECG is sinus rhythm. Laboratory results reveal: Sodium 141 mEq/L, Arterial blood pH 7.26, Potassium 3.5 mEq/L, Lactate 1.7 mEq/L, Chloride 101 mEq/L, Ethanol level undetectable, Bicarbonate 14mEq/L, Measured serum osmolarity 352 mOsm/L, BUN 15 mg/dL, Calculated serum osmolarity 292 mOsm/kg, Creatinine 0.7 mg/dL, Glucose 89 mg/dL, Urinalysis: no blood, ketones, or protein. Which of the following statements below best describes the laboratory findings? a. Anion gap metabolic acidosis and osmol gap b. Anion gap metabolic acidosis without osmol gap c. Nonanion gap metabolic acidosis and osmol gap d. Nonanion gap metabolic acidosis without osmol gap e. Metabolic alkalosis with secondary acidosis

*A* An anion gap is the difference between unmeasured anions (eg, proteins, organic acids) and unmeasured cations (eg, potassium, calcium, magnesium). The anion gap can be calculated from the formula: Anion gap = [Na+] − [HCO3 + Cl−]

30-year-old man presents to the ED complaining of sudden onset of abdominal bloating and back pain lasting for 2 days. The pain woke him up from sleep 2 nights ago. It radiates from his back to his abdomen and down toward his scrotum. He is in severe pain and is vomiting. His temperature is 101.2°F and HR is 107 beats per minute. A CT scan reveals a 9-mm obstructing stone of the left ureter with hydronephrosis. Urinalysis is positive for 2+ blood, 2+ leukocytes, 2+ nitrites, 40 to 50 WBCs, and many bacteria. You administer pain medicine, antiemetics, and antibiotics. Which of the following is the most appropriate next step in management? a. Admit for IV antibiotics and possible surgical removal of stone. b. Observe in ED for another 6 hours to see if stone passes. c. Discharge with antibiotics and pain medicine. d. Discharge patient with instructions to consume large amounts of water. e. Discharge patient with antibiotics, pain medicine, and instructions to drink large amounts of water and cranberry juice.

*A* An obstructing stone with an overlying infection is an impending urologic emergency. Bacteria in an obstructed collecting system can cause abscess formation, renal destruction, and severe systemic toxicity. The patient requires admission for IV antibiotics and removal and drainage of the stone. In addition to obstruction with infection, other indications for admission include persistent pain, persistent nausea and vomiting, urinary extravasation, and hypercalcemic crisis.

A 37-year-old woman with a history of migraines presents to the ED complaining of crampy lower abdominal pain for 3 days. Workup reveals an intrauterine pregnancy and early prenatal care is arranged with obstetrics as an outpatient. You are concerned because her headaches are controlled with a significant number of medications. She uses medications for both abortive therapy and for prophylaxis. Which of the following classes of medications do you advise she discontinue while pregnant? a. Anticonvulsants b. β-Blockers c. Triptans d. Acetaminophen e. Antiemetics

*A* Anticonvulsant medications are sometimes used as prophylaxis for migraines. Phenytoin, valproic acid, phenobarbital, and topiramate are among the antiepileptics commonly used to prevent migraines. Phenytoin causes fetal hydantoin syndrome, a constellation of birth defects, including growth retardation, cleft palates, hand deformities, and structural cardiac defects. Valproic acid has similar effects with the addition of neural tube defects. Phenobarbital, declining in its use for migraine prevention, causes cardiac defects, facial clefts, and urinary tract abnormalities. Early data on topiramate reveals teratogenicity in animal models.

An 81-year-old woman presents to the ED after tripping over the sidewalk curb and landing on her chin causing a hyperextension of her neck. She was placed in a cervical collar by paramedics. On examination, she has no sensorimotor function of her upper extremities. She cannot wiggle her toes, has 1/5 motor function of her quadriceps, and only patchy lower extremity sensation. Rectal examination reveals decreased rectal tone. Which of the following is the most likely diagnosis? a. Central cord syndrome b. Anterior cord syndrome c. Brown-Séquard syndrome d. Transverse myelitis e. Exacerbation of Parkinson disease

*A* Central cord syndrome is often seen in patients with degenerative arthritis of the cervical vertebrae, whose necks are subjected to forced hyperextension. Typically, it is seen in a forward fall onto the face in an elderly person. This causes the ligamentum flavum to buckle into the spinal cord, resulting in a contusion to the central portion of the cord. This injury affects the central gray matter and the most central portions of the pyramidal and spinothalamic tracts. Patients often have greater neurologic deficits in the upper extremities, compared to the lower extremities, since nerve fibers that innervate distal structures are located in the periphery of the spinal cord. In addition, patients with central cord syndrome usually have decreased rectal sphincter tone and patchy, unpredictable sensory deficits.

A 21-year-old woman presents to the ED with a superficial 2-cm midforehead laceration sustained from a fall. You irrigate and close the wound. The patient is an aspiring actress and concerned about her long-term cosmetic outcome. She requests detailed wound-care instructions. Which of the following statements would be an appropriate wound care instruction to give? a. Avoid direct sun exposure while the wound is healing. b. The wound should not be washed until sutures are removed. c. Sutures should be removed in 7 to 10 days. d. Appearance of the scar at suture removal will be the final cosmetic outcome. e. Good suturing technique and meticulous wound care guarantees no scar formation.

*A* Direct sun exposure during healing can lead to permanent hyperpigmentation. Therefore, avoiding direct sun exposure and using sun block for up to 12 months can improve cosmetic outcome.

A 17-year-old boy is found unconscious in a swimming pool. He is brought into the ED by paramedics already intubated. In the ED, the patient is unresponsive with spontaneous abdominal breathing at a rate of 16 breaths per minute, BP of 80/50 mm Hg, and HR of 49 beats per minute. In addition to hypoxemia, what condition must be considered earliest in the management of this patient? a. Cervical spine injury b. Electrolyte imbalance c. Metabolic acidosis d. Severe atelectasis e. Toxic ingestion

*A* Diving injuries must always be suspected in near-drowning patients. This patient presents with abdominal breathing and spontaneous respirations. This pattern provides an important clue to a cervical spine injury. The diaphragm is innervated by the phrenic nerve, which originates from the spinal cord at the C3-C4 level, whereas the intercostal muscles of the rib cage are supplied by nerves that originate in the thoracic spine. Therefore, abdominal breathing in the absence of thoracic breathing indicates an injury below C4. His bradycardia in the presence of hypotension is suspicious for neurogenic hypotension, which is caused by loss of vasomotor tone and lack of reflex tachycardia from the disruption of autonomic ganglia. However, this is a diagnosis of exclusion and should only be made once all other forms of shock are ruled out. It is important to maintain c-spine immobilization to prevent further progression of an injury.

A 51-year-old man is brought to the emergency department (ED) by emergency medical services (EMS) with a blood pressure (BP) of 90/60 mm Hg, heart rate (HR) of 110 beats per minute, respiratory rate (RR) of 18 breaths per minute, and oxygen saturation of 97% on room air. The patient tells you that he has a history of bleeding ulcers. On examination, his abdomen is tender in the epigastric area. He is guaiac positive, with black stool. He has a bout of hematemesis and you notice that his BP is now 80/50 mm Hg, HR is 114 beats per minute, as he is slowly starting to drift off. Which of the following is the most appropriate next step in therapy? a. Assess airway, establish two large-bore intravenous (IV) lines, cross-match for two units of blood, administer 1 to 2 L of normal saline, and schedule an emergent endoscopy. b, Assess airway, establish two large-bore IVs, cross-match for two units of blood, and administer a proton pump inhibitor. c. Place two large-bore IVs, cross-match for two units of blood, administer 1 to 2 L of normal saline, and schedule an emergent endoscopy. d. Intubate the patient, establish two large-bore IVs, cross-match for two units of blood, administer 1 to 2 L of normal saline, and schedule an emergent endoscopy. e. Intubate the patient, establish two large-bore IVs, cross-match for two units of blood, and administer a proton pump inhibitor.

*A* Emergency medicine always starts with an assessment of the patient's airway. For patients suspected of having a significant GI bleed, two large-bore IV lines need to be established rapidly. Treating an undifferentiated upper GI bleed is like treating a gunshot wound to the abdomen—you should expect the worst. Immediate volume resuscitation should begin with 1 to 2 L of normal saline. If there is no improvement in the BP of a hypotensive patient, then blood should be administered. Sending a cross-match early is advisable since it can take up to an hour to retrieve. Use type O, Rh-negative (if female) or type O, Rh-positive (if male) if type-specific blood is not ready. It is crucial to remember that the initial hematocrit of the patient is a poor indicator of the severity of acute bleeding because it takes 24 to 72 hours to equilibrate.

A 20-year-old man was found on the ground next to his car after it hit a tree on the side of the road. Bystanders state that the man got out of his car after the collision but collapsed within a few minutes. Paramedics subsequently found the man unconscious on the side of the road. In the ED, his BP is 175/90 mm Hg, HR is 65 beats per minute, temperature is 99.2°F, RR is 12 breaths per minute, and oxygen saturation is 97% on room air. Physical examination reveals a right-sided fixed and dilated pupil. A head CT shows convex shaped collection of blood. Which of the following is the most likely diagnosis? a. Epidural hematoma b. Subdural hematoma c. Subarachnoid hemorrhage (SAH) d. Intracerebral hematoma e. Cerebral contusion

*A* Epidural hematomas are the result of blood collecting in the potential space between the skull and the dura mater. Most epidural hematomas result from blunt trauma to the temporal or temporoparietal area with an associated skull fracture and middle meningeal artery disruption. The classic history of an epidural hematoma is a lucent period following immediate loss of consciousness after significant blunt head trauma. However, this clinical pattern occurs in a minority of cases. Most patients either never lose consciousness or never regain consciousness after the injury. On CT scan, epidural hematomas appear lenticular or biconvex (football shaped), typically in the temporal region.

A 3-year-old girl is brought to the ED with acute onset of respiratory distress. She recently emigrated from Africa. Her initial vitals include HR of 115 beats per minute, BP of 110/60 mm Hg, and RR of 28 breaths per minute with oxygen saturation of 88% on room air. She is also febrile to 103.5°F. She is ill- and anxious-appearing, sitting forward in her mother's lap, and drooling. Her mother tells you that she had a sore throat that began 2 days prior and that she was going to see her pediatrician this week for her initial vaccinations. Given this patient's history and presentation, which of the following should be of particular concern? a. Epiglottitis b. Retropharyngeal abscess c. Epstein-Barr virus d. Ludwig's angina e. Peritonsillar abscess

*A* Epiglottitis is a life-threatening inflammatory condition of the epiglottis, aryepiglottic, and paraglottic folds. The etiology is usually infectious, with Haemophilus influenzae type b as the classic and most common etiology prior to the introduction of the H influenzae type b vaccination. Most cases now appear in adults and nonimmunized children and are seen most commonly as secondary infections following viral illnesses (the most notorious was primary varicella prior to the widespread use of that vaccine). Signs and symptoms include a prodromal period of 1 to 2 days with high fever, dysphagia, secretion pooling, and dyspnea. Patients usually sit in an erect or "tripod" position, leaning forward with neck extended to give maximum airway opening, to improve their symptoms. Radiographs of the neck may show the classic thumbprint sign of an enlarged, inflamed epiglottis, although a CT scan of the neck may delineate the condition further. However, these are typically difficult to obtain because of the fact that patients are not stable enough to leave the ED and need continuous monitoring in case of airway compromise; so no radiographic studies are typically performed. The classic approach to a patient with suspected epiglottitis is to leave them in their caretaker's arms, try to avoid agitating them, and call immediately for ear, nose, throat (ENT) and anesthesia assistance in the ED. Direct laryngoscopy is contraindicated because it may induce laryngospasm. These patients require direct visualization, often only in the OR, with the appropriate services prepared for the need to place a surgical airway emergently. IV antibiotics and steroids are indicated to help treat the infection and decrease swelling.

A 67-year-old woman presents to the ED with a painful facial rash that has been worsening over the past 2 days. On physical examination, she has a deeply erythematous, shiny area of warm and tender skin on her left face with a sharply-demarcated and indurated border. There is minimal edema. Vitals are HR 88 beats per minute, BP 125/70 mm Hg, RR 16 breaths per minute, and temperature 101°F. Which of the following is the most appropriate next step in management? a. IV antibiotics and hospital admission b. Oral cephalosporin and outpatient follow-up c. IV acyclovir and Tzanck smear d. Systemic steroids and laboratory testing for rheumatoid factor e. Dermatology consult and biopsy of the rash

*A* Erysipelas is an acute superficial cellulitis of the dermis, lymphatics, and subcutaneous tissue. It ischaracterized by a sharply demarcated border surrounding skin that is raised, deeply erythematous, indurated, and painful, and is associated with nephrotic syndrome, postoperative wounds, and small breaks in the skin. Erysipelas is more superficial than cellulitis and is more likely to occur in the young and in the elderly, but the distinction between the two is often subtle and therapeutically irrelevant. Treatment of erysipelas and facial cellulitis requires hospital admission and parenteral antibiotics. Treatment is aimed at the predominant organism, group A Streptococcus, but Staphylococcus and other Streptococcus species are also found. An immediate ophthalmologic consult should be obtained if there is any orbital or periorbital involvement.

A 43-year-old man presents to the ED complaining of right hand pain. The individual states that 3 days ago he accidentally punctured his index finger with a thumb tac. Subsequently, the finger became red and began to swell, and he had pain with movement of the finger. His tetanus status is up to date. You suspect the patient's diagnosis is tenosynovitis. Which of the following statements regarding Kanavel four cardinal signs of flexor tenosynovitis is correct? a. The infected finger is held in slight flexion at rest b. Asymmetric swelling of the finger c. Tenderness along the extensor tendon sheath d. Painless with passive range of motion e. Purplish hue at the distal tip

*A* Flexor tenosynovitis typically results from a puncture wound. The causative agents are usually Staphylococcus aureus or Streptococcus. The diagnosis is based on the presence of Kanavel four cardinal signs of flexor tendon synovitis. These include the finger held in slight flexion at rest, (b) symmetric swelling or sausage digit of the finger, (c) tenderness along the flexor tendon sheath, and (d) pain with passive range of motion of the finger. Answer choice (e) is not one of Kanavel signs.

A 19-year-old man is brought to the ED by EMS after he was found lying on the floor at a dance club. EMS states that the patient seemed unconscious at the dance club but as soon as they transferred him onto the gurney he became combative. Upon arrival in the ED, his BP is 120/65 mm Hg, HR is 75 beats per minute, temperature is 98.9°F, RR is 12 breaths per minute, and oxygen saturation is 98% on room air. On physical examination, his pupils are midsized, equal, and reactive to light. His skin is warm and dry. Lung, cardiac, and abdominal examination are unremarkable. As you walk away from the bedside, you hear the monitor alarm signaling zero respirations and the oxygen saturation starts to drop. You perform a sternal rub and the patient sits up in bed and starts yelling at you. As you leave him for the second time, you hear the monitor alarm again signal zero respirations. You administer naloxone, but there is no change in his condition. Which of the following is most likely the substance ingested by this patient? a. γ-Hydroxybutyrate (GHB) b. Diazepam c. Cocaine d. Phencyclidine (PCP) e. Heroin

*A* GHB is a natural neurotransmitter that induces sleep. GHB has been sold as a muscle builder (release of growth hormone), a diet aid, and a sleep aid. Patients with GHB overdose generally have a decreased level of consciousness. In contrast to other sedative/ hypnotic overdoses, the level of consciousness tends to fluctuate quickly between agitation and depression. A distinctive feature of GHB intoxication is respiratory depression with apnea, interrupted by periods of agitation and combativeness, especially following attempts at intubation.

A 33-year-old man, who was drinking heavily at a bar, presents to the ED after getting into a fight. A bystander tells paramedics that the patient was punched and kicked multiple times and sustained multiple blows to his head with a stool. In the ED, his BP is 150/75 mm Hg, HR is 90 beats per minute, RR is 13 breaths per minute, and oxygen saturation is 100% on non-rebreather. On examination, he opens his eyes to pain and his pupils are equal and reactive. There is a laceration on the right side of his head. He withdraws his arm to pain but otherwise doesn't move. You ask him questions, but he just moans. Which of the following is the most appropriate next step in management? a. Prepare for intubation b. Suture repair of head laceration c. Administer mannitol d. Bilateral burr holes e. Neurosurgical intervention

*A* Head injury severity is assessed on the mechanism of injury and on the initial neurologic examination. Although the GCS is currently used in multiple settings, it was initially developed for the clinical evaluation of hemodynamically stable, adequately oxygenated trauma patients with isolated head trauma. A score of 14 to 15 is associated with minor head injury, 9 to 13 indicates moderate, and 8 or less is associated with severe head injury. His GCS score is 8 (2 points for eye opening to pain, 2 points for mumbling speech, 4 points for withdrawing from pain). He is classified with a severe head injury. The overall mortality of severe head injury is almost 40%. It is recommended to intubate patients with a GCS score of 8 or less for airway protection. These patients are at risk for increased ICP and herniation, which can lead to rapid respiratory decline. All patients with severe traumatic brain injury require an emergent CT scan and should be admitted to the intensive care unit in a hospital with neurosurgical capabilities.

A 68-year-old man presents to the ED complaining of a daily headache for almost a month. He describes the headache as being dull, difficult to localize, most intense in the morning, and abating in the early afternoon. He also noticed progressive weakness of his right upper and lower extremity. Which of the following headache syndromes are the signs and symptoms most consistent with? a. Headache caused by a mass lesion b. Cluster headache c. Tension-type headache d. Headache from intracranial hypertension e. Waking or morning migraine

*A* Headaches caused by a mass lesion are classically described as worse in the morning, associated with nausea and vomiting, and worse with position. Rarely do patients present with focal neurologic symptoms. When they do, imaging is a necessary adjunct prior to leaving the ED. If a mass lesion is part of the differential diagnosis, LP should be deferred until neuroimaging has been performed because of the risk of herniation.

A 29-year-old woman presents to the ED complaining of double vision for 3 days. She states that she has been feeling very tired lately, particularly at the end of the day, when even her eyelids feel heavy. She feels better in the morning and after lunch when she is able to rest for an hour. Her BP is 132/75 mm Hg, HR is 70 beats per minute, temperature is 98.4°F, and RR is 12 breaths per minute. On examination you find ptosis and proximal muscle weakness. What is the most appropriate diagnostic test to perform? a. Edrophonium test b. Serologic testing for antibodies to acetylcholine receptors c. Head computed tomography (CT) scan d. Electrolyte panel e. Lumbar puncture

*A* High clinical suspicion in this case is for myasthenia gravis, an autoimmune condition in which acetylcholine receptor antibodies block acetylcholine binding and prevent normal neuromuscular conduction. The disease typically affects young women and older men and presents with generalized weakness worsening with repetitive muscle use that is usually relieved with rest. Ptosis and diplopia are usually present. The edrophonium test is used to help diagnose myasthenia gravis. It involves administering edrophonium, a short-acting anticholinesterase, which prevents acetylcholine breakdown. With the increased acetylcholine levels at the neuromuscular junction, the patient experiences a subjective and objective improvement of symptoms by preventing rapid breakdown of acetylcholine at the myoneural junction

You receive notification from EMS that they are bringing in a 17-year old male who was found unconscious by a police officer. The police officer at the scene states that he snuck up on a group of kids that he thought were using drugs. Two of them got away and one just fell to the ground seconds after standing up. Lying on the ground next to the adolescent were plastic bags. The emergency medical technician states that the patient was in ventricular fibrillation. He was shocked in the field and is now in a sinus rhythm. The EMT also administered IV dextrose, thiamine, and naloxone without any change in mental status. Which of the following substances was the patient most likely abusing? a. Butane b. Ethanol c. Heroin d. Cocaine e. PCP

*A* Hydrocarbons (HC) are a diverse group of organic compounds that contain hydrogen and carbon. Some common products containing HCs are household polishes, glues, paint remover, and industrial solvents. Acute HC toxicity usually affects three main target organs: the lungs, CNS, and heart. The lungs are most commonly affected by aspiration of ingested HCs. Pulmonary toxicity is associated with cough, crackles, bronchospasm, pulmonary edema, and pneumonitis on chest radiograph. Certain HCs (eg, toluene, benzene, gasoline, butane, chlorinated HCs) can have sedative/opioid-like effect and cause euphoria, disinhibition, confusion, and obtundation. HCs can also cause sudden cardiac death, particularly after sudden physical activity after intentional inhalation. It is thought that the HCs produce myocardial sensitization of endogenous and exogenous catecholamines, which precipitates ventricular dysrhythmias and myocardial dysfunction. One scenario is the solvent-abusing person. EMS workers often describe an individual who has used inhaled solvents, performed some type of physical activity, and then suddenly collapsed. In the scenario above, the patient inhaling butane was approached by a police officer and tried to run away. This sudden exertion most likely led to a cardiac dysrhythmia. Paraphernalia is often found at the scene, including plastic bags used for "bagging" (pouring HCs in a bag, then deeply inhaling) or a HC-soaked cloth used for "huffing" (inhaling through a saturated cloth). Other paraphernalia include gasoline containers, multiple butane lighters, and spray paint cans.

A 74-year-old woman who is a known diabetic is brought to the ED by emergency medical service (EMS) with altered mental status. The home health aide states that the patient ran out of her medications 4 days ago. Her BP is 130/85 mm Hg, HR is 110 beats per minute, temperature is 99.8°F, and her RR is 18 breaths per minute. On examination, she cannot follow commands but responds to stimuli. Laboratory results reveal white blood cell (WBC) count of 14,000/L, hematocrit 49%, platelets 325/L, sodium 128 mEq/L, potassium 3.0 mEq/L, chloride 95 mEq/L, bicarbonate 22 mEq/L, blood urea nitrogen (BUN) 40 mg/dL, creatinine 1.8 mg/dL, and glucose 850 mg/dL. Urinalysis shows 3+ glucose, 1+ protein, no blood or ketones. After addressing the ABCs, which of the following is the most appropriate next step in management? a. Begin fluid resuscitation with a 2- to 3-L bolus of normal saline then administer 10 U of regular insulin intravenously. b. Begin fluid resuscitation with a 2- to 3-L bolus of normal saline then administer 10 U of regular insulin intravenously, begin phenytoin for seizure prophylaxis. c. Administer 10 U of regular insulin intravenously then begin fluid resuscitation with a 2- to 3-L bolus of normal saline. d. Order a computed tomographic (CT) scan of the brain, if negative for acute stroke, begin fluid resuscitation with a 2- to 3-L bolus of normal saline. e. Arrange for urgent hemodialysis.

*A* Hyperglycemic hyperosmolar nonketotic coma (HHNC) is a syndrome representing marked hyperglycemia (serum glucose > 600 mg/dL), hyperosmolarity (plasma osmolarity > 350 mOsm/L), considerable dehydration (9 L in 70-kg patient), and decreased mental functioning that may progress to coma. HHNC may be the initial presentation of previously unrecognized diabetes in an adult with type 2 diabetes mellitus. Elderly diabetics are at greater risk for this illness. Osmotic diuresis is even more pronounced than in DKA. Rapid correction of hyperosmolar state may lead to cerebral edema. Unlike DKA, acidosis and ketosis are usually absent or minimal. The mainstay to treatment is fluid resuscitation, insulin administration, electrolyte repletion, and searching for an underlying precipitant.

A 20-year-old man presents to the ED with fever and severe right lower quadrant (RLQ) pain for 1 day. Prior to this episode, he reports 2 months of crampy abdominal pain, generalized malaise, a 10-lb weight loss, and occasional bloody diarrhea. On examination, his HR is 115 beats per minute, BP is 125/70 mm Hg, RR is 18 breaths per minute, and temperature is 100.8°F. His only significant past medical history is recurrent perirectal abscesses. On physical examination, the patient appears uncomfortable and has a tender mass in the RLQ, without guarding or rebound. Rectal examination is positive for trace heme-positive stool. An abdominal computed tomography (CT) scan reveals no peri-appendiceal fat stranding. There is inflammation of the distal ileum and several areas of the colon. There are no rectal inflammatory changes. Which of the following is the most likely diagnosis? a. Crohn disease (CD) b. Ulcerative colitis (UC) c. Appendicitis d. Pseudomembranous enterocolitis e. Diverticulitis

*A* IBD is a chronic inflammatory disease of the GI tract. There are two major types: CD and UC. CD can involve any part of the GI tract, from mouth to anus, and is characterized by segmental involvement. The distal ileum is involved in the majority of cases; therefore, acute presentations can mimic appendicitis. CD spares the rectum in 50% of cases. There is a bimodal age distribution, with the first peak occurring in patients 15 to 22 years of age, and a second in patients 55 to 60 years of age. Definitive diagnosis is by upper GI series, air-contrast barium enema, and colonoscopy. Segmental involvement of the colon with rectal sparing is the most characteristic feature. Other findings on colonoscopy include involvement of all bowel wall layers, skip lesions (ie, interspersed normal and diseased bowel), aphthous ulcers, and cobblestone appearance from submucosal thickening interspersed with mucosal ulceration. Extraintestinal manifestations are seen in 25% to 30% of patients with CD.

A 55-year-old woman presents to the ED complaining of 1 day of a left-sided headache that is associated with scalp and ear pain. She describes the pain as gradual in onset, dull, and constant. She describes a week of constitutional symptoms prior to the onset of her headache syndrome including joint pain, tenderness of the muscles of her lower extremities, and fatigue. She is afebrile with no nuchal symptoms, photophobia, or phonophobia. Physical examination reveals a tender scalp and a thickened, painful temporal artery. Which of the following is the most appropriate next step in management? a. Initiate corticosteroid therapy b. LP to rule out subarachnoid hemorrhage (SAH) c. Injection of lidocaine at the base of the occiput d. Initiate antibiotic therapy e. Send an ESR

*A* In a case of suspected TA, initiation of corticosteroid therapy is indicated emergently to prevent irreversible complications. Loss of vision is known to occur and prompt initiation of corticosteroid therapy decreases this possibility. TA, also referred to as giant cell arteritis, is a granulomatous inflammation of the proximal great vessels and its carotid bifurcations. It has an overlapping clinical syndrome with polymyalgia rheumatica.

A 31-year-old man is riding his mountain bicycle down a steep hill when he hits a rock in the path and is thrown off the bicycle. In the ED, the individual complains only of left arm pain. Your primary survey is unremarkable. After administering medication for pain control, you send the patient for a radiograph of his arm. Which of the following injuries is consistent with this patient's radiograph showinga proximal fracture of the ulnar shaft and a radial head dislocation? a. Monteggia fracture b. Galeazzi fracture c. Nightstick fracture d. Colles fracture e. Smith fracture

*A* Monteggia fractures are of the proximal one-third of the ulnar shaft combined with a radial head dislocation. This injury commonly occurs from either a direct blow to the posterior aspect of the ulna or a fall on the outstretched hand with the forearm in forced pronation. This fracture is associated with an injury to the radial nerve. It is important to always look for an associated fracture or dislocation when one is noted in a forearm bone. A Galeazzi fracture (b) is a fracture of the distal radial shaft associated with a distal radioulnar dislocation at the distal radioulnar joint (DRUJ). This fracture is often confused with Monteggia fracture. *A way to remember the difference is to recall that Monteggia ends in an "a" and in this fracture, the ulna (also ends in "a") is fractured.*

A 37-year-old woman is brought into the ED by her friend who states that the patient swallowed approximately 50 capsules of 325-mg acetaminophen 6 hours ago in an attempted suicide. The patient states she feels nauseated and vomits while you take her history. Her BP is 100/75 mm Hg, HR is 97 beats per minute, temperature is 98.9°F, RR is 18 breaths per minute, and oxygen saturation is 99% on room air. Examination is unremarkable except for mild epigastric tenderness. Which of the following is the correct antidote for APAP overdose? a. NAC b. Physostigmine c. Flumazenil d. Naloxone e. Digibind

*A* NAC is the cornerstone of therapy for the potentially lethal acetaminophen overdose. NAC acts as a glutathione precursor to reduce NAPQI (N-acetyl-p-benzoquinoneimine), the toxic metabolite of acetaminophen. It can be administered orally or intravenously. NAC is most effective if administered within 8 hours of the ingestion; however, it may still be of benefit if given more than 24 hours after an acute acetaminophen overdose.

A 26-year-old man presents to the ED in agony with a Gila monster still attached to his arm after being bitten. He reports that he is the animal's main handler, with no prior biting incidents. He reports localized pain but denies weakness, nausea, or feeling lightheaded. The animal bite occurred about 45 minutes ago. After the animal is carefully removed, what should be done next in the care of this patient? a. Check for any remaining embedded teeth and begin wound care. b. Administer antivenin. c. Give tetanus prophylaxis. d. Administer broad-spectrum antibiotics. e. Apply suction device.

*A* Only two venomous lizards are found in the world, both of which whose natural habitat is in the southwestern United States and Mexico. These animals are usually not aggressive, despite the Gila monster's name, and bites are usually a result of direct handling as in this case. Both the Gila monster and the Mexican-beaded lizard are easily identified by their thick bodies, beaded scales with either white and black or pink and black configuration. Envenomation from these bites occurs from the glands along the lower jaw and introduced into the victim through grooved teeth, which the animal uses to continuously chew after it has bitten down. These teeth may become embedded in the victim, thereby distributing more venom.

A 20-year-old man presents with several weeks of painful rectal bleeding. He denies fever, nausea, or vomiting. He is sexually active with women only and usually uses condoms. He denies any history of CD, UC, or malignancy. He states that the pain is most severe during and immediately after defecating. Bleeding is bright red and only enough to stain the toilet paper. Which of the following is the most common etiology of painful rectal bleeding? a. External hemorrhoid b. Anal fissure c. Anorectal tumor d. Internal hemorrhoid e. Venereal proctitis

*A* Pain and bleeding are common complaints associated with anorectal disorders. A good history and a thorough physical examination, including a digital rectal examination and anoscopy should be performed whenever feasible. Anal fissures (ie, fissures in ano) result from linear tears of the anal canal at or just inferior to the dentate line and extend along the anal canal to the anal verge. This area has a rich supply of somatic sensory never fibers. Consequently, anal fissures are exquisitely painful and represent the most common cause of painful rectal bleeding in the first year of life and in adults. They are usually produced by the passage of a large, hard stool but may also occur with severe diarrhea.

Paramedics bring a 44-year-old man to the ED. He was found in the middle of the street after being struck by a car. His systolic BP is 70 mm Hg; a diastolic BP cannot be obtained. The heart rate is 125 beats per minute, and oxygen saturation is 89% on room air. The patient's eyes are closed. You ask the patient his name and he doesn't respond. There is no response when you ask him to move his limbs. You notice that his left foot is severely deformed and there is a large laceration to his right arm. Which of the followingis the most appropriate next step in management? a. Prepare for emergent orotracheal intubation. b. Begin aggressive fluid resuscitation and administer morphine for pain. c. Apply a tourniquet just above his left foot and begin fluid resuscitation. d. Apply pressure to the laceration, splint the left foot, and order a radiograph. e. Administer packed RBCs and bring him to the CT scanner for a pan-scan.

*A* Patients often present to the ED with life-threatening conditions that require rapid and simultaneous evaluation and treatment. The fundamentals of emergency medicine begin with the ABCs. Airway assessment and management have priority over all other aspects of resuscitation in the critically ill or injured patient.

A 47-year-old man with a history of hypertension presents to the ED complaining of continuous left-sided chest pain that began while snorting cocaine 1 hour ago. The patient states he never experienced chest pain in the past when using cocaine. His BP is 170/90 mm Hg, HR is 101 beats per minute, RR is 18 breaths per minute, and oxygen saturation is 98% on room air. The patient states that the only medication he takes is alprazolam to "calm his nerves." Which of the following medications is contraindicated in this patient? a. Metoprolol b. Diltiazem c. Aspirin d. Lorazepam e. Nitroglycerin

*A* Patients with chest pain in the setting of cocaine use should be evaluated for possible myocardial ischemia. Patients suspected of ACS should be managed accordingly with oxygen, nitrates, morphine, aspirin, and benzodiazepines; however, β-adrenergic antagonist therapy is absolutely contraindicated. If β-adrenergic receptors are antagonized, α-adrenergic receptors are left unopposed and available for increased stimulation by cocaine. This may worsen into coronary and peripheral vasoconstriction, hypertension, and possibly ischemia. Therefore, benzodiazepines, which decrease central sympathetic outflow, are the cornerstone in treatment to relieve cocaine-related chest pain.

A 55-year-old woman presents to the ED stating that her nose has been bleeding profusely for the last 3 hours. After 25 minutes of bilateral pressure on her nasal septum, there is still profuse bleeding. You place anterior nasal packing bilaterally but bleeding still persists. The patient is starting to get anxious. Her BP is 110/70 mm Hg, HR is 80 beats per minute, RR is 18 breaths per minute, and oxygen saturation is 98%. Laboratory results reveal a white blood cell (WBC) count of 9000, hematocrit (HCT) 34%, platelets of 225,000, and international normalized ratio (INR) 1.1. Under direct visualization, you note the bleeding originating from the posterior aspect of her septum. Which of the following is the most appropriate management? a. Place posterior nasal packing, start antibiotics, and admit the patient to a monitored hospital bed. b. Place the patient supine and wait for spontaneous resolution of the bleeding. c. Keep pressure on her nasal septum and administer fresh frozen plasma and platelets. d. Place posterior nasal packing, and discharge the patient home with follow-up in 24 hours. e. Apply silver nitrate to the nasal mucosa until the bleeding stops.

*A* Posterior epistaxis is identified when posterior bleeding occurs with a properly placed anterior nasal packing. Posterior packing is mandated using either a commercially available balloon or a standard Foley catheter inserted into the posterior nares and inflated with water. Patients with posterior nasal packs should be admitted to a monitored bed. In addition to cardiac dysrhythmias, myocardial infarctions, cerebrovascular accidents, and aspiration have been reported in these patients. Antibiotics are often started to prevent sinusitis and toxic shock syndrome from obstruction of the nasal packing.

A 42-year-old man with a history of schizophrenia is brought into the ED by a friend who states that the patient has not taken his medication for over 2 weeks and is now behaving bizarrely. His BP is 130/70 mm Hg, HR 89 beats per minute, respiratory rate (RR) 15 breaths per minute, and oxygen saturation 99% on room air. On examination he appears agitated and is shouting, "the aliens are about to get me." He is cooperative enough that you decide to use pharmacologic sedation. Which of the following is the most appropriate choice for sedating this patient? a. Haloperidol and lorazepam b. Etomidate and succinylcholine c. Chlorpromazine and lorazepam d. Ketamine and lorazepam e. Clozapine

*A* Rapid tranquilization is a method of pharmacologic management of acute agitation or psychosis using high-potency neuroleptics and benzodiazepines. The most common regimen used is the combination of haloperidol and lorazepam, which can be administered via parenteral, intramuscular, or oral routes. There is a synergistic effect between the two medications. Moreover, the benzodiazepine may prevent the potential extrapyramidal affects that occasionally occur with neuroleptic use.

A 28-year-old G2P0010 presents to the ED stating that she is pregnant and has vaginal spotting of blood. Pelvic examination reveals blood in the vaginal vault, but no active bleed, and a closed internal os. Transvaginal ultrasound reveals an intrauterine pregnancy consistent with a gestational age of 11 weeks. Her BP is 130/75 mm Hg, HR is 82 beats per minute, temperature is 99.1°F, and RR is 16 breaths per minute. Laboratory results reveal a WBC 10,500/μL, hematocrit 40%, platelets 225/μL. She is blood type B, Rh-negative. Which of the following is the most appropriate intervention prior to discharging the patient from the ED? a. Administer 50 μg of anti-D immune globulin. b. Administer 2 g of magnesium sulfate to prevent eclampsia. c. Administer penicillin G to prevent chorioamnionitis. d. Administer ferrous sulfate to prevent anemia. e. Administer packed red blood cells to increase blood volume.

*A* Rh isoimmunization occurs when a Rh-negative female is exposed to Rh-positive blood during pregnancy or delivery. Initial exposure leads to primary sensitization with production of immunoglobulin M antibodies. A patient with a threatened abortion, who has Rh-negative blood, is at increased risk for Rh isoimmunization and therefore should receive anti-D immune globulin. A 50 μg dose is used during the first trimester and a 300 μg dose after the first trimester.

A 24-year-old competitive ice-skater is practicing for the Olympic trials when she falls during one of her jumps and lands on her outstretched hand. She is brought to the ED complaining of wrist pain. On examination, you note tenderness at the anatomic snuffbox and pain with axial loading of the thumb. You suspect she has a scaphoid fracture. In what part of a fractured scaphoid is the incidence of avascular necrosis highest? a. Proximal scaphoid. b. Waist (middle third) of the scaphoid. c. Distal scaphoid. d. Tubercle of the scaphoid. e. The scaphoid is not at risk for avascular necrosis.

*A* The blood supply to the scaphoid normally penetrates the cortex at the distal aspect of the bone. Therefore, there is no direct blood supply to the proximal portion of the bone, which predisposes this fragment to avascular necrosis and delayed union. The more proximal the fracture is in scaphoid injuries, the greater the likelihood of developing vascular necrosis.

A 25-year-old man is brought to the ED by emergency medical service (EMS) accompanied by his girlfriend who reports that the patient had a seizure 30 minutes ago and is still confused. The girlfriend reports that the patient is a known epileptic who has been doing well on his latest medication regimen. The exact seizure medications are unknown. On arrival to the ED, the patient develops continuous clonic movements of his upper and lower extremities. The patient's vital signs are BP of 162/85 mm Hg, HR of 110 beats per minute, and pulse oximetry of 91% on room air. Capillary glucose level is 95 mg/dL. Which of the following is the most appropriate next step in management? a. Place the patient in a lateral decubitus position. b. Administer lorazepam. c. Administer phenytoin. d. Perform rapid sequence intubation on the patient. e. Look up the patient's medical records and administer his current antiepileptic regimen.

*A* The initial approach to a seizing patient should involve protecting the patient from injury. Seizing patients should be immediately placed in a lateral decubitus position to prevent aspiration of gastric contents. Other initial measures are oxygen administration, pulse oxymetry, glucose level determination, and an IV line.

An 82-year-old right-handed woman is brought to the ED by her daughter stating that her mother has not been able to walk after waking up from a nap 30 minutes ago. The patient has a history of hypertension and diabetes. Her BP is 179/76 mm Hg, HR is 91 beats per minute, temperature is 98.9°F, and RR is 14 breaths per minute. On examination, you elicit neurologic deficits and emergently bring her to the CT scanner. The radiologist tells you there is an abnormality in the left parietal lobe and a likely middle cerebral artery stroke. Which of the following motor deficits are you likely to find in this patient? a. Right sensorimotor deficit in arm greater than leg and aphasia b. Left sensorimotor deficit in arm greater than leg and aphasia c. Right sensorimotor deficit in leg greater than arm, slowed response to questions, and impaired judgment d. Right motor deficit and left facial droop e. Right leg hemiplegia only

*A* The middle cerebral artery is the most common site of intracranial cerebral artery thrombosis. Clinical findings can include contralateral hemiplegia, hemianesthesia, and homonymous hemianopsia. The upper extremity deficit is usually more severe than the lower extremity deficit. Aphasia occurs if the dominant hemisphere is involved. Gaze preference is in the direction of the lesion.

A 32-year-old dental hygienist presents to the ED with a painful lesion at the distal aspect of her right index finger. The individual states that she had a low-grade fever and malaise over the last week and subsequently developed pain and burning of the infected digit. Within the next week, she noted erythema, edema, and the development of small grouped vesicles on an erythematous base as depicted in the image below. Which of the following should be avoided when managing this condition? a. Performing an incision and drainage to facilitate healing and avoid bacterial superinfection. b. Splinting the finger and recommending elevation and analgesics. c. Prescribing an antiviral agent, such as acyclovir. d. Applying a dry dressing over the lesions to prevent transmission. e. Prescribing an antibiotic if there is evidence of bacterial superinfection.

*A* The patient has a herpetic whitlow, a viral infection of the distal finger. It is caused by the herpes simplex virus type I or II. This condition typically occurs in health-care providers with exposure to oral secretions, and in patients with coexistent herpes infections. It generally presents with a prodrome period of fever and malaise. Subsequently, there is localized burning, itching and pain that precede the development of the classic clear herpetic vesicles. Typically, only one finger is involved. The diagnosis is usually made clinically, but if doubt remains, or if the presentation is atypical, it can be confirmed with a Tzanck smear or viral culture. When managing this condition, it is important to note that surgical drainage is contraindicated. It can result in secondary infection and delayed healing.

A 61-year-old woman with metastatic breast cancer presents to the ED with chest pain, cough, and shortness of breath. She states these symptoms began 1 week ago and progressively worsened. She denies fever or chills. On examination, you notice jugular venous distension. Her BP is 105/70 mm Hg and HR is 98 beats per minute. A chest radiograph is shown below. Which of the following ECG finding is associated with this presentation? a. Low-voltage complexes b. High-voltage complexes c. ST-segment depression d. ST-segment elevation e. T-wave inversion

*A* The patient presents with a pericardial effusion probably secondary to her metastatic breast cancer. Pericardial effusion is often asymptomatic but with accumulating fluid can cause chest pain, shortness of breath, cough, and fever. Ultimately, it can lead to cardiac tamponade, which develops in up to 10% of all cancer patients. The ECG classically shows low-voltage complexes and, rarely, electricaln alternans. Treatment of nontraumatic pericardial effusion and tamponade is pericardiocentesis to remove the fluid.

A 29-year-old tall, thin man presents to the ED after feeling short of breath for 2 days. In the ED, he is in no acute distress. His BP is 115/70 mm Hg, HR is 81 beats per minute, RR is 16 breaths per minute, and oxygen saturation is 98% on room air. Cardiac, lung, and abdominal examinations are normal. An ECG reveals sinus rhythm at a rate of 79. A chest radiograph shows a small right-sided (less than 10% of the hemithorax) spontaneous pneumothorax. A repeat chest x-ray 6 hours later reveals a decreased pneumothorax. Which of the following is the most appropriate next step in management? a. Discharge the patient with follow-up in 24 hours b. Perform needle decompression in the second intercostal space, midclavicular line c. Insert a 20F chest tube into right hemithorax d. Observe for another 6 hours e. Admit for pleurodesis

*A* The patient presents with a primary spontaneous pneumothorax (PTX), which occurs in individuals without clinically apparent lung disease. In contrast, secondary spontaneous pneumothorax occurs in individuals with underlying lung disease, especially chronic obstructive pulmonary disease (COPD). For otherwise healthy, young patients with a small primary spontaneous PTX (less than 20% of the hemithorax), observation alone may be appropriate. The intrinsic reabsorption rate is approximately 1% to 2% a day, and accelerated with the administration of 100% oxygen. Many physicians observe these patients for 6 hours and then repeat the chest x-ray. If the repeat chest x-ray shows no increase in the size of the PTX, the patient can be discharged with follow-up in 24 hours. Air travel and underwater diving (changes in atmospheric pressure) must be avoided until the PTX completely resolves.

A 53-year-old man presents to your ED stating he has had an excruciating right-sided headache since leaving the movie theater. He states that the headache is unilateral, severe, and associated with nausea and vomiting. His vision is blurry and notes seeing halos around objects. He denies trauma or a history of headaches in the past. Physical examination reveals right conjunctival injection and a pupil that reacts only marginally. Which examination is likely to yield the correct diagnosis? a. Measurement of intraocular pressure b. Funduscopic examination c. Fluorescein examination d. LP with cell count e. Visual acuity testing

*A* The patient presents with acute angle closure glaucoma which results from obstruction of aqueous outflow of the anterior chamber of the eye with a resulting rise in intraocular pressure. It is the result of a shallow anterior chamber or a chamber distorted by the development of a cataract. Classically, it occurs when a patient leaves a prolonged dimly lit situation. When the iris becomes mid-dilated, it maximally obstructs the trabecular meshwork occluding aqueous humor flow. Intraocular pressures may rise from normal (10-21 mm Hg) to levels as high as 50 to 100. Visual acuity is usually decreased in the affected eye as a result of corneal edema. Treatment is aimed at lowering intraocular pressure with acetazolamide, ophthalmic β-blockers, prostaglandin analogues, and pilocarpine to induce miosis. Ophthalmologic consultation and follow-up is indicated. Patients may present complaining of headache, nausea, and vomiting, but will often endorse that the symptoms began with acute eye pain.

A 10-week-old girl is brought to the ED after 5 hours of abdominal distension and green stained vomiting. The patient is a previously well infant with an uncomplicated antenatal course and normal vaginal delivery. However, she spent an extra day in the neonatal intensive care unit (NICU) when born because of "water on the lungs." On the day of presentation, the patient is unable to hold any fluids down without vomiting. Her vital signs reveal HR of 185 beats per minute, RR of 65 breaths per minute, and temperature of 100.8°F. Abdominal examination reveals a diffusely tender abdomen that is hypertympanic. Which of the following is the definitive study of choice? a. Upper GI series b. Abdominal ultrasound c. Findings on physical examination d. CBC, electrolytes, and urine analysis e. Serum lactate

*A* The patient's clinical presentation is consistent with malrotation with midgut volvulus. Malrotation occurs when there is an inappropriate fixation of the intestines at the ligament of Treitz during fetal life. The incidence is uncertain but it is believed to approximate 1 in 500 live births with a male to female ratio of 2:1. Signs and symptoms of malrotation may be nonspecific and include vomiting, which is characteristically bilious, and abdominal distension. Up to 25% of cases present after 1 year of age. The most catastrophic presentation of malrotation occurs when the abnormally fixed mesentery twists around itself and the superior mesenteric artery leading to bowel infarction, shock, sepsis, and death. All of which can all occur within a few hours. The definitive diagnostic study of choice is an upper GI series. Contrast in the gut will fail to demonstrate the classic "C-loop" of the four parts of the duodenum and instead show the "corkscrew" appearance classic of this condition. Some physicians begin the workup with a plain film of the abdomen looking for air fluid levels, dilated loops, or pneumatosis coli that can be seen in patients with severe distention. The classic plain film finding is that of the "double bubble". However, plain films are less sensitive and specific than an upper GI series.

A 28-year-old man who just finished a 7-day course of antibiotics for pharyngitis presents to the ED with progressive difficulty swallowing. His BP is 130/65 mm Hg, HR is 95 beats per minute, temperature is 100.1°F, RR is 16 breaths per minute, and oxygen saturation is 99%. On examination, the patient is in no acute distress but has a fluctuant mass on the right side of the soft palate with deviation of the uvula. Which of the following is the most appropriate next step in management? a. Needle aspiration, antibiotics, and follow-up in 24 hours b. Pain control, observation for 6 hours c. Admission for incision and drainage in the OR d. Antibiotics and follow-up in 24 hours e. CT scan, antibiotics, and follow-up in 24 hours

*A* The patient's presentation is typical for a peritonsillar abscess. Signs and symptoms include a sore throat, muffled voice, trismus, fluctuant mass, deviation of the uvula, odynophagia, and drooling. Many of these patients have a history of being recently treated for strep throat. The abscess is usually unilateral and in the superior pole of the tonsil. Airway patency must be assessed because of the obstructing potential of an abscess. Treatment includes either needle aspiration or incision and drainage of the abscess, in addition to antibiotic treatment. Some studies demonstrate the safety and cost-effectiveness of needle aspiration over incision and drainage.

A thin 16-year-old girl is brought in the ED after collapsing at home. Her initial vitals include an HR of 110 beats per minute, BP of 80/55 mm Hg, and an RR of 18 breaths per minute with an oxygen saturation of 98% on room air. Upon physical examination, you note a cachectic female in mild distress. Her chest is clear to auscultation; her sunken-in abdomen is soft and nontender. Upon inspecting her extremities, you notice small areas of erythema over the dorsum of her right hand distally. Given this patient's presentation and physical examination, which of the following etiologies must be further explored in this patient? a. Bulimia b. Gastroenteritis c. Malingering d. Factitious disorder e. Suicidality

*A* The physical examination in this patient reveals hand abrasions, indicative of self-purging. Her general appearance suggests either inadequate food intake or excessive calorie burning. Bulimics generally consume an adequate amount of food, albeit low-calorie, but purge their intake with the goal of weight loss. Other eating disorders, such as anorexia nervosa, can take the form of starvation, diuretic or laxative use, or excessive exercise. These patients generally suffer from a false visualization of their body, believing that their physical form weighs more than what it does in reality. Social pressures, history of abuse or violence, and other eating disorders may factor in to this patient's presentation.

The following is a radiograph showing The most common findings are curvilinear calcification of the aortic wall or a paravertebral soft tissue mass. of a 72-year-old man who presented to the ED complaining of gradually worsening back pain that he describes as constant and dull. He denies nausea, vomiting, diarrhea, and hematuria. Which of the following is an important predisposing factor for the development of the condition seen in this individual? a. Atherosclerosis b. Hyperparathyroidism c. Ethanol abuse d. Prostate cancer e. Hernia

*A* The radiograph demonstrates an abdominal aortic aneurysm. On the lateral radiograph, it is the circular structure just anterior to the vertebral column; on the AP view, it is the circular structure overlying the lower vertebrae, most prominently on the patient's left side. Signs of AAA large enough to cause symptoms are seen on plain radiographs approximately 66% to 75% of the time. The most common findings are curvilinear calcification of the aortic wall or a paravertebral soft tissue mass. Rarely, with longstanding aneurysms, erosion of one or more vertebral bodies may be seen. Atherosclerosis, age > 60, smoking, and family history are all important predisposing factors for the development of AAA.

A 25-year-old man presents to the ED complaining of dull periumbilical pain that migrated to his RLQ over the last hour. He states that he has no appetite and vomited twice. His BP is 125/75 mm Hg, HR is 87 beats per minute, temperature is 100.6°F, and RR is 16 breaths per minute. Laboratory results reveal WBC 11,000/μL, hematocrit 48%, platelets 170/μL. On physical examination, the patient complains of pain when you flex his knee with internal rotation at his hip. What is the name of this sign? a. Obturator b. Psoas c. Rovsing d. McBurney e. Murphy

*A* The test is the Obturator sign, in which the patient is supine with the right thigh flexed; passive internal or external rotation of the hip eliciting pain is a positive test for appendicitis. The pain is attributed to an inflamed appendix that is irritated by stretching the obturator internus muscle.

A 55-year-old man presents to the ED 6 hours after ingesting two bottles of his baby aspirin. He complains of nausea, vomiting, dizziness, and tinnitus. His temperature is 100.3°F, BP is 140/80 mm Hg, HR is 105 beats per minute, RR is 31 breaths per minute, and oxygen saturation is 99% on room air. Arterial blood gas on room air reveals a pH of 7.52, PCO2 10 mm Hg, and PO2 129 mm Hg. The blood salicylate level returns at 45 mg/dL. Which of the following is the most appropriate next step in management? a. Administer activated charcoal, begin IV hydration, and administer sodium bicarbonate. b. Administer activated charcoal, begin IV hydration, and intubate the patient for respiratory failure. c. Administer activated charcoal, begin IV hydration, and administer NAC. d. Arrange for immediate hemodialysis. e. Observe the patient overnight to allow the body to metabolize the salicylates.

*A* The treatment of salicylate toxicity has three objectives: (1) prevent further salicylate absorption, (2) correct fluid deficits and acid-base abnormalities, and (3) reduce tissue salicylate concentrations by increasing excretion. Activated charcoal should be administered as soon as possible to reduce salicylate absorption. Dehydration occurs early in salicylate intoxication and should be treated with IV hydration. Urine alkalization should be considered in patients with salicylate levels greater than 35 mg/dL. This is performed by administering IV sodium bicarbonate. Because salicylic acid is a weak acid, it is ionized in an alkaline environment and gets "trapped," limiting the amount that crosses the blood-brain barrier and increasing urinary excretion.

The local sorority house recently installed a sun-tanning station. Two days later three sorority girls present to the ED with bilateral eye pain, tearing, and photophobia. After ophthalmic anesthesia instillation, a complete eye examination is performed. Visual acuity is normal. Extraocular eye movements are intact and pupils are equal, round, and reactive to light. IOP is normal. Slitlamp examination is normal, but fluorescein examination under cobalt blue light illuminates small dots throughout the cornea. What is the most likely diagnosis? a. Ultraviolet keratitis b. Anterior uveitis c. Herpes simplex keratitis d. Allergic conjunctivitis e. Corneal ulcer

*A* These girls forgot to wear eye protection while using the sun-tanning lights and have ultraviolet keratitis. History of exposure to sun-tanning lamps, welding, or the sun suggests the diagnosis and fluorescein staining showing superficial punctate keratitis confirms the diagnosis. Treatment consists of analgesia, cycloplegics to reduce ciliary spasm and pain, erythromycin ointment, and ophthalmology followup in 1 to 2 days. Fortunately, most patients with ultraviolet keratitis make a full recovery with supportive care alone.

A 4-year-old girl is brought to the ED by her mother with a 2-day history of abdominal pain and leg pain. The leg pain worsened today to where the patient did not want to walk. She was carried into the ED by her parents. The patient describes two episodes of nonbloody, nonbilious emesis. She denies fever, respiratory complaints, diarrhea, and trauma. There is no significant past medical history, no recent travel, and no family history of inflammatory bowel disease. On examination, the patient has a temperature of 99.9°F, HR 122 beats per minute, RR 24 breaths per minute, BP 95/60 mm Hg. She is alert and interactive with no significant distress. HEENT (head, eyes, ears, nose, throat) is normal. Lungs are clear to auscultation. Heart is regular with no murmur. Abdomen is soft but diffusely tender with no rebound or guarding. Extremities reveal bilateral swelling and tenderness of her knees. Skin examination reveals nonblanching purple and red lesions (macules and patches) on the flexural surface of her lower extremities and on her buttocks. These lesions are nontender. You advise the parents that the most likely organ system to experience any prolonged problem is which of the following? a. Renal b. Integument c. Gastrointestinal (GI) d. Musculoskeletal e. Neurologic

*A* This is a case of Henoch-Schönlein purpura (HSP), also termed anaphylactoid purpura. HSP is an inflammatory vasculitis whose cause is unknown. It is thought to be postinflammatory. The organism most frequently associated with HSP is Campylobacter jejuni, but this is found in a minority of cases, and many other organisms and factors have been implicated. HSP typically occurs in the spring months and is most common in young children between 2 and 12 years of age, though it can occur at any age. HSP is characterized by its characteristic rash, a purpuric rash that typically occurs on the buttocks and flexural surface of the lower extremities onto the soles. Abdominal pain, joint pain and arthritis, and hematuria with renal involvement are also common. Treatment is mostly supportive with NSAIDs for pain control although steroids are used for patients with severe abdominal pain. There is recent evidence to suggest that steroids may benefit a larger percentage of patients. One feared complication of HSP is intussusception. Physicians must consider this diagnosis in patients with abdominal pain and HSP. HSP often affects the kidney causing hematuria. The majority of patients recover from HSP without any complication, but prolonged renal involvement leading to renal failure and hypertension can occur in approximately 1% of patients.

A 4-year-old uncircumcised boy is brought to the ED by his caretaker for an 8-hour history of swelling and redness of the penis. The caretaker states that she retracted the foreskin over the penis to clean it and could not move it back afterward. The patient's vital signs are within normal limits. On examination, the patient is crying and becomes irritable whenever you try to examine the genital area. The glans is edematous and erythematous. The testicular examination shows bilateral descended testicles with a normal cremasteric reflex. Which of the following is the most appropriate next step in management? a. Manual reduction of the foreskin over the glans b. Dorsal slit incision or circumcision c. Topical lidocaine d. Catheterization to prevent obstruction and urinary retention e. Topical steroids to reduce the swelling

*A* This is a case of paraphimosis. Paraphimosis is a true emergency. It occurs when the foreskin of an uncircumcised male is retracted beyond the glans and not returned to its normal position. Ensuing congestion and edema make it difficult for it to be retracted back to its normal position. Manual reduction is the first-line treatment for paraphimosis. Manual reduction is performed by placing your thumbs over the glans as you attempt to gently pull the engorged foreskin tissue back over the head of the penis. Ice can be applied to the penis to help decrease swelling. In addition, a dorsal penile nerve block can be used for analgesia making it easier to perform a manual reduction.

A 27-year-old man presents to the ED with the laceration shown below after an altercation. There are no dental fractures and his tetanus immunization is up to date. What are the most appropriate next steps in management? a. Infraorbital nerve block and then approximation of the vermilion border b. Infiltration of local anesthesia into the lip and then approximation of the vermilion border c. Infraorbital nerve block and closure of mucosal defects prior to the approximation of the vermilion border d. Infiltration of the lip with local anesthesia, closure of dermis and mucosal defects, and then approximation of the vermilion border e. Closure with a commercially available tissue adhesive and Steri-Strips to avoid distorting tissue architecture

*A* This is a laceration of the face crossing the vermilion border (demarcation of the lip mucosa and facial skin). The goal of repair in this case is approximation of the vermilion border with less than 2 mm of displacement since significant displacement is cosmetically unappealing. A nerve block is required because local anesthesia would distort tissue anatomy thereby limiting appropriate approximation. The first stitch should focus on approximating the border prior to closing the other aspects of the wound.

A 56-year-old man is brought in from the homeless shelter for strange, irrational behavior and unsteady gait for 1 day. A worker at the shelter reports that the patient is a frequent abuser of alcohol. On examination, the patient is alert but oriented to name only and is unable to give full history. He does not appear clinically intoxicated. You note horizontal nystagmus and ataxia. What is the most likely diagnosis? a. Wernicke encephalopathy b. Korsakoff syndrome c. Normal pressure hydrocephalus d. Central vertigo e. Alcohol withdrawal

*A* This patient exhibits the classic triad of Wernicke encephalopathy (WE): confusion, ataxia, and ophthalmoplegia. WE is a result of thiamine deficiency leading to decreased glucose metabolism and neuronal destruction, primarily in the cerebellum, hypothalamus, vestibular system, and memory. It is typically found in chronic alcoholics caused by nutritional deficiency, but can also occur in other malnutrition states, pregnancy, persistent vomiting, or dialysis. WE can mimic acute stroke symptoms and can lead to permanent nystagmus and ataxia. It carries 10% to 20% mortality if untreated.

A 46-year-old woman presents to the ED with left-sided arm and leg weakness for half an hour. She has no medical problems except for chronic neck pain after a motor vehicle collision 5 years ago. On examination she has right eye miosis, partial ptosis, and 3/5 strength in her left upper and lower extremities. On further questioning, the patient states that earlier in the day she saw a chiropractor for her neck pain. After the session she developed severe right-sided neck pain. About an hour later she noticed difficulty using her left side of her body. Which of the following is the most likely diagnosis? a. Internal carotid artery dissection b. Cavernous sinus syndrome c. Multiple sclerosis d. Transverse myelitis e. Spinous process fracture of a cervical vertebra

*A* This patient has an internal carotid artery (ICA) dissection secondary to chiropractic neck manipulation. ICA dissection can occur spontaneously or in minor neck trauma and should be considered in a young patient with acute stroke. ICA dissection should also be suspected in patients with neck pain and Horner syndrome because of the disruption of ipsilateral oculosympathetic fibers. In this scenario, it presents with ipsilateral Horner syndrome and contralateral ischemic motor deficits. Other causes of acute Horner syndrome include tumors (ie, Pancoast tumor), stroke, herpes zoster infection, and trauma.

A 22-year-old presents to the ED for left eye pain. He was in an altercation yesterday and was punched in the left eye. On examination, his left eye is ecchymotic and the eyelids are swollen shut. He has tenderness over the infraorbital rim, but no step-offs. You use an eyelid speculum to examine his eye. His pupils are equal and reactive to light. His visual acuity is normal. On testing extraocular movements, you find he is unable to look upward with his left eye. He also complains of diplopia when looking upward. Funduscopic examination is normal. What is the most likely diagnosis? a. Orbital blowout fracture b. Ruptured globe c. Retinal detachment d. Cranial nerve III palsy e. Traumatic retrobulbar hematoma

*A* This patient has an orbital blowout fracture of the inferior wall causing entrapment of the inferior rectus muscle and restricted eye motility with diplopia. A CT scan with thin cuts through the orbits can confirm the diagnosis. Patients with this injury are generally started on oral antibiotics because of the risk of infection with sinus wall fractures and may follow-up with the institution's appropriate surgical service in 3 to 10 days. These injuries are associated with other eye problems and a careful eye examination must be performed to rule out abrasions, lacerations, foreign bodies, hyphema, iritis, retinal detachment, and lens dislocation.

A 32-year-old gravida 1, para 1 who gave birth by normal vaginal delivery at 38-weeks gestation 2 days ago presents to the ED complaining of bilateral hand swelling and severe headache that started 2 hours ago. Her BP is 187/110 mm Hg, HR is 85 beats per minute, temperature is 97.5°F, and RR is 15 breaths per minute. Urinalysis reveals 3+ protein. As you are examining the patient, she proceeds to have a generalized tonic-clonic seizure. Which of the following is the most appropriate next step in management? a. Administer magnesium sulfate IV. b. Administer labetalol to reduce her BP and morphine sulfate to address her headache. c. Administer sumatriptan and place the patient into a dark quiet room. d. Administer a loading dose of phenytoin, order a head CT scan, and call for a neurology consult. e. Administer diazepam and normal saline IV.

*A* This patient has postpartum eclampsia, which needs to be managed with magnesium sulfate and admission to the obstetrical service. Preeclampsia is defined as new-onset hypertension (> 140/90 mm Hg) and proteinuria (1 g/L in random specimen or > 3 g/L over 24 hours). Some clinicians also use generalized edema as a requirement. Preeclampsia is most common in the third trimester. Eclampsia occurs with the development of seizures or coma in a patient with preeclampsia. A preeclamptic woman may worsen after delivery and develop late postpartum eclampsia, which usually occurs in the first 24 to 48 hours postpartum but may present several weeks after delivery. Management of eclamptic seizures in the ED involves administering magnesium sulfate, which is believed to act as a membrane stabilizer and vasodilator, reducing cerebral ischemia. Although magnesium sulfate is not a direct antihypertensive, the hypertension associated with eclampsia is often controlled adequately by treating the seizure.

A 44-year-old woman returns from a mountain excursion with headache symptoms, nausea, and vomiting that are improving. Her initial vital signs in the ED are HR of 93 beats per minute, BP of 120/60 mm Hg, RR of 18 breaths per minute, and oxygen saturation of 96% on 2-L nasal cannula. Her chest examination is clear to auscultation with no palpable overlying crepitus and there are no signs of peripheral edema. Which of the following medications is indicated in this patient? a. Acetazolamide b. Dexamethasone c. Nifedipine d. Furosemide e. Morphine

*A* This patient is recovering from acute mountain sickness with no signs of CHF or pulmonary edema. The first-line treatment is descent and oxygen supplementation, which is already accomplished in this patient. Other treatment considerations include hyperbaric oxygen therapy, which improves hypoxemia for all altitude illnesses. As for medication regimens, acetazolamide is indicated even in patients with a history of altitude illness as prophylaxis. It works to decrease the formation of bicarbonate by inhibiting carbonic anhydrase. This diuretic action counters the fluid retention in acute mountain illness. It also decreases bicarbonate absorption in the kidney, causing a metabolic acidosis, which stimulates hyperventilation. This compensatory mechanism is turned off when the pH is close to the physiologic range of 7.4. It's this hyperventilation that counters the altitude-induced hypoxemia, thereby relieving symptoms.

A 63-year-old man presents to the ED complaining of headache, vomiting, and "not being able to think straight" for 1 day. The patient states that he has hypertension and diabetes but ran out of his medications in the last week. His BP is 245/138 mm Hg, HR is 90 beats per minute, temperature is 98.7°F, and his RR is 14 breaths per minute. Fingerstick glucose is 178 mg/dL. On examination the patient appears slightly confused and oriented to name and place only. The neurologic examination is significant for papilledema. Which of the following is the most ppropriate next step in management? a. Nitroprusside IV b. Magnesium sulfate IV c. Metoprolol by mouth d. Hydrochlorothiazide by mouth e. Obtain head CT

*A* This patient presents with headache and altered mental status in the setting of severe hypertension leading to the diagnosis of hypertensive encephalopathy. A hypertensive emergency is defined by severe hypertension with evidence of organ dysfunction. The BP needs to be aggressively, but carefully lowered to prevent cerebral bleeding and progression to coma and death. Patients can also present with seizures, focal neurological deficits, visual acuity changes, or coma. Patients with hypertensive encephalopathy are managed with short-acting titratable IV antihypertensive medications such as IV nitroprusside or labetalol. The BP should not be significantly lowered since it can result in brain hypoperfusion and infarction. Typically, the mean arterial pressure (MAP) is lowered by 20% to 25% in the first hour of treatment.

A 21-year-old college student is brought by her roommate to the emergency department (ED). The roommate states that earlier in the day the patient complained of a severe headache, stiff neck, and photophobia. On their way to the ED, the roommate states that the patient was confused. Her vital signs are blood pressure (BP) 110/80 mm Hg, heart rate (HR) 110 beats per minutes, respiration rate (RR) 16 breaths per minute, and temperature 102°F. What is the next step in the management of this patient? a. Start empiric antibiotics, noncontrast head computed tomography (CT) prior to performing lumbar puncture (LP). b. Order a noncontrast head CT and start antibiotics once the results are back. c. Give 1 g of acetaminophen, start fluid hydration, and perform an LP. d. Perform an LP and start antibiotics once the results are back. e. Order a noncontrast head CT, perform an LP, then start antibiotics

*A* This patient presents with symptoms consistent with meningitis. Antibiotics are administered empirically as diagnostic workup proceeds. The best choice in this patient is ceftriaxone, which has good CNS penetration. In order to avoid transtentorial herniation in this patient with a neurologic deficit (confusion), a noncontrast head CT should be performed prior to LP. It is controversial whether or not a head CT needs to be performed prior to all LPs. However, if there is papilledema or a neurologic deficit, then head CT is mandatory.

A farmer in Texas presents to the ED with right leg numbness, localized edema, and tremors. He reports being out in the field when the symptoms began. He denies contact with insecticides and reports being at his baseline of health prior to the event. His initial vitals include HR of 105 beats per minute, BP of 175/90 mm Hg, and RR of 22 breaths per minute with oxygen saturation of 97% on room air. Which of the following is the most appropriate initial treatment of choice in this patient? a. Antivenin b. Tetanus prophylaxis c. Antibiotic prophylaxis d. Sedation e. Atropine

*A* This patient suffered a venomous snakebite. Pit vipers, such as rattlesnakes, copperheads, and water moccasins, are the most prevalent and are present in all states except Alaska, Maine, and Hawaii. Coral snakes are the second most prevalent and are present mainly in the southern states. Red on yellow, kill a fellow; red on black, venom lack is a commonly used phrase to help remember which snakes are dangerous. Venomous snakes can also be recognized by their triangular heads, elliptical pupils, fangs, and the presence of a pit between the eye and nostril— characteristics that may not be noticed by the victim initially. Therefore, there is a snakebite grading system, which requires antivenin with progressive symptoms of edema, coagulopathy, and neurologic manifestations. The amount of antivenin is dependent on the severity of these symptoms. Venom itself has a number of substances including proteolytic enzymes and polypeptides, which promote coagulation, neuromuscular blockage, cell lysis, and death. Anaphylaxis may also occur. Airway, breathing, and circulation (ABCs) must always be initially assessed.

A 30-year-old woman presents to the ED with fever, headache, a "sunburn-like" rash, and confusion. A friend states that the patient has complained of nausea, vomiting, diarrhea, and a sore throat over the past few days. Her last menstrual period began 4 days ago. Vital signs are HR 110 beats per minute, BP 80/45 mm Hg, RR of 18 breaths per minute, and temperature of 103°F. On physical examination, you note an ill-appearing female with a diffuse blanching erythroderma. Her neck is supple without signs of meningeal irritation. You note a fine desquamation of her skin, especially over the hands and feet, and hyperemia of her oropharyngeal, conjunctival, and vaginal mucus membranes. Laboratory results reveal a creatine phosphokinase (CPK) of 5000, WBC 15,000/μL, platelets of 90,000/μL, BUN 40 mg/dL, creatinine 2 mg/dL, and elevated liver enzymes. Which of the followin is the most likely causative organism? a. Staphylococcus aureus b. Rickettsia rickettsii c. Streptococcus pyogenes d. Neisseria meningitidis e. Neisseria gonorrhoeae

*A* This patient suffers from toxic shock syndrome (TSS), a severe, life-threatening syndrome characterized by high fever, diffuse macular erythroderma, profound hypotension, desquamation, and multisystem involvement (including vomiting or diarrhea, severe myalgias, mucus membrane hyperemia, renal or hepatic dysfunction, decreased platelets, and disorientation). TSS can rapidly progress to multisystem dysfunction and shock An exotoxin produced by S aureus is the presumed cause in menstrual-related TSS (MRTSS) and two endotoxins have been implicated in non-menstrualrelated TSS (NMRTSS).. TSS should be considered in any unexplained febrile illness associated with erythroderma, hypotension, and diffuse organ pathology. Patients with MRTSS usually present between the third and fifth day of menses. In severe cases, headache is the most common complaint. The rash is a diffuse, blanching erythroderma, often described as a painless "sunburn" that fades within 3 days and is followed by desquamation, especially of the palms and soles. For severe cases, treatment includes aggressive IV fluid resuscitation, IV oxacillin or cefazolin, and hospital admission in a monitored setting.

A 63-year-old woman accompanied by her husband is brought to the ED by EMS with worsening right arm weakness that started 90 minutes ago at the opera. Her husband states that she has a history of hypertension and a long smoking history. She has no surgical history. The husband states that his wife was fine when going to the opera. The patient's BP is 215/118 mm Hg, HR is 97 beats per minute, temperature is 99.3°F, and RR is 14 breaths per minute. On examination, the patient is anxious, mildly aphasic, has 2/5 strength, and diminished sensation in the right upper extremity. An emergent head CT scan is normal. It has been 2 hours since the onset of symptoms. Which of the following is the most appropriate next step in management? a. Administer labetalol. b. Administer fibrinolytic therapy. c. Administer aspirin prior to fibrinolytic therapy to reduce platelet aggregation. d. Administer phenytoin prior to fibrinolytic therapy as seizure prophylaxis. e. Administer mannitol to reduce intracranial pressure prior to fibrinolytic therapy.

*A* This patient's BP of 215/118 mm Hg needs to be lowered to 185/110 mm Hg to make her a good candidate for thrombolytic therapy. Labetalol is the agent of choice in this case

A 9-year-old girl is brought to the ED by her parents after sustaining a forehead laceration. Her dad reports that the patient hit her forehead against a doorknob. On examination the patient is cooperative but anxious about the upcoming anesthetic injection. She has a 0.5-cm laceration on the midforehead with clean edges. The parent's parents inquire about "skin glue" they heard about on TV. Which of the following statements is true regarding cyanoacrylate tissue adhesives? a. It cannot be applied to mucous membranes and areas with thick hair. b. Tissue adhesives have a higher rate of infection and tissue dehiscence than sutures. c. Tissue adhesives are best suited for forehead lacerations between 5 and 10 cm in length. d. Tissue adhesives are applied inside the wound to pull skin edges together. e. Tissue adhesives must be covered with topical antibiotics and bandages to prevent sloughing.

*A* Tissue adhesives close wounds by forming an adhesive layer that brings the edges of the laceration together. They cannot be used on mucosal membranes and areas with thick hair. For optimal results, it should be applied in three to four layers in a dry, bloodless field.

A 28-year-old woman presents with severe abdominal pain with distention and vaginal bleeding. She reports being pregnant with a due date in approximately 4 weeks. Her pain came on suddenly, shortly before arrival to the ED. Upon further history-taking, the patient reports recent cocaine use. What condition needs to be considered in this patient? a. Uterine rupture b. Normal contractions c. Placenta accreta d. Vasa previa e. Ruptured ovarian cyst

*A* Uterine rupture presents as uterine pain without contraction and vaginal bleeding. It is most prevalent in women who have had a previous cesarean section, recent cocaine or prostaglandin use. Cocaine causes extreme vasoconstriction that compromises blood flow to the uterus and fetus causing friable and necrotic tissue, which is prone to rupture. This is an obstetric emergency that necessitates surgical intervention for stabilization of both mother and child.

A 21-year-old man presents to the ED complaining of abdominal pain, nausea, and vomiting for 1 day and increased weakness for the last 2 to 3 days. He states that he is using the bathroom to urinate frequently and is drinking large amounts of water. He has no previous medical problems and is not taking any medications. His BP is 110/72 mm Hg, HR is 119 beats per minute, temperature is 98.8°F, and RR is 14 breaths per minute. On examination he appears mildly confused, is pale, diaphoretic, and has unusually deep respirations and a fruity odor to his breath. Which of the following is the next best step? a. Check fingerstick glucose. b. Administer antiemetics. c. Administer analgesics. d. Send basic metabolic panel. e. Obtain abdominal radiograph.

*A* You should be suspicious for DKA as the initial presentation of diabetes mellitus in this patient. Glucose levels are elevated, typically greater than 200 mg/dL. DKA is an acute, lifethreatening disorder occurring in patients with insulin insufficiency (more common in type 1 diabetes) and results in hyperglycemia, ketosis, and acidosis from osmotic diuresis leading to dehydration and acidosis. This patient exhibits classic signs and symptoms of DKA, such as tachycardia, GI distress, polyuria, polydipsia, fatigue, and confusion. Very deep breathing (Kussmaul respirations) reflects respiratory compensation for metabolic acidosis. Fruity breath odor in this patient is the result of acetoacetate acid conversion to acetone, which is eliminated during respiration.

A 45-year-old man presents to the ED complaining of recurrent episodes of lightheadedness and nausea overnight. He describes the episodes as a room swaying from side-to-side when he lies on his left side. He reports mild headache now and denies tinnitus, hearing loss, fevers, or vomiting. He has no medical problems and takes no medications except for occasional acetaminophen. His BP is 123/65 mm Hg, HR is 69 beats per minute, temperature is 98.5°F, and RR is 12 breaths per minute. The patient is asymptomatic now and the examination is unremarkable. Which of the following is the most appropriate diagnostic test? a. Dix-Hallpike maneuver b. Caloric stimulation testing c. Orthostatic vital signs d. Head CT scan e. ECG

*A* You should suspect BPV in this patient. BPV is a transient positional vertigo associated with nystagmus. The problem occurs secondary to the creation and movement of canaliths (free-moving densities) in the semicircular canals of the inner ear that is associated with a particular head movement. The Dix-Hallpike maneuver is a diagnostic test designed to reproduce transient vertiginous symptoms and nystagmus of BPV. The maneuver involves having the patient go from sitting to a supine position with eyes open and head rotated to the affected side. The test is positive if the maneuver reproduces vertigo and the patient exhibits latent rotary nystagmus. A negative Dix-Hallpike maneuver does not exclude the condition.

A 56-year-old man presents to the ED complaining of intermittent lightheadedness and nausea throughout the day. He believes it started after eating leftover shrimp salad in the morning. On further questioning, he reports that during the lightheadedness episodes the room is spinning around him and the episodes are triggered by turning his head to the right. He denies hearing loss, tinnitus or other associated symptoms. His BP is 137/85 mm Hg, HR is 67 beats per minute, temperature is 98.5°F, and RR is 14 breaths per minute. The patient reproduces the symptoms by turning his head to the right. Which of the following is the most likely diagnosis? a. Benign positional vertigo (BPV) b. Food poisoning c. Meniere disease d. Labyrinthitis e. TIA

*A*BPV is a transient positional vertigo associated with nystagmus. The problem occurs secondary to the creation and movement of canaliths (free-moving densities) in the semicircular canals of the inner ear with a particular head movement. Neurologic deficits are absent in BPV. Note that horizontal, vertical, or rotary nystagmus can occur in BPV. It is important to pay special attention to a patient with vertical nystagmus because it may be associated with a brainstem or cerebellum lesion. BPV is treated with the Epley maneuver (a series of head and body turns that reposition the canalith), antiemetics, and antihistamines. Key differences between peripheral and central vertigo are seen in the chart.

A 45-year-old woman presents to the ED immediately after landing at the airport from a transatlantic flight. She states that a few moments after landing she felt short of breath and felt pain in her chest when she took a deep breath. Her only medications are oral contraceptive pills and levothyroxine. She is a social drinker and smokes cigarettes occasionally. Her BP is 130/75 mm Hg, HR is 98 beats per minute, temperature is 98.9°F, RR is 20 breaths per minute, and oxygen saturation is 97% on room air. You send her for a duplex ultrasound of her legs, which is positive for deep vein thrombosis. What is the most appropriate management for this patient? a. Place patient on a monitor, provide supplemental oxygen, and administer unfractionated heparin. b. Place patient on a monitor, order a chest computed tomography (CT) scan to confirm a pulmonary embolism (PE), and then administer unfractionated heparin. c. Place patient on a monitor and administer aspirin. d. Instruct the patient to walk around the ED so that she remains mobile and does not exacerbate thrombus formation. e. Place the patient on a monitor, provide supplemental oxygen, and administer warfarin.

*A*The patient has a confirmed venous thrombosis and has symptoms consistent with a pulmonary thromboembolism. Data now show that almost every deep venous thrombosis (DVT) embolizes to some extent. The presence of a PE in this patient can be presumed by a confirmed DVT with pulmonary symptoms. All patients need to be on a monitor and should receive supplemental oxygen despite normal oxygen saturation. Oxygen acts as a pulmonary vasodilator. Heparin is the first-line therapy in this patient and should be administered promptly. Failure to achieve a therapeutic partial thromboplastin time (PTT) value within the first 24 hours leads to a 23% incidence of new embolism

A 22-year-old man presents to the ED complaining of dysuria for 3 days. He states that he has never had this feeling before. He is currently sexually active and uses a condom most of the time. He denies hematuria but notes a yellowish discharge from his urethra. His BP is 120/75 mm Hg, HR is 60 beats per minute, and temperature is 98.9°F. You send a clean catch urinalysis to the laboratory that returns positive for leukocyte esterase and 15 white blood cells per high power field (WBCs/hpf). Which of the following is the most appropriate next step in management? a. Send a urethral swab for culture and administer 125-mg ceftriaxone intramuscularly and 1-g azithromycin orally. b. Send urine for culture and administer SMX/TMP orally. c. Discharge the patient with strict instructions to return if his symptoms worsen. d. Order a CT scan to evaluate for a kidney stone. e. Have him follow-up immediately with a urologist to evaluate for testicular cancer.

*A.* Dysuria in young men is almost always because of urethritis, which is commonly caused by a sexually transmitted disease. Urethritis is classically divided into gonococcal (GU) and nongonococcal (NGU) types. GU is caused by N gonorrhoeae, while the major pathogen in NGU is C trachomatis. Nearly all men with GU have purulent urethral discharge. NGU may be asymptomatic or with a yellow, mucopurulent discharge. It was demonstrated in multiple studies that the two pathogens coexist in men with urethritis up to 50% of the time. Therefore, antibiotics should be directed at eliminating both organisms. A third-generation cephalosporin or ciprofloxacin as a one-time dose is used to treat GU. Common antibiotics used to treat NGU include azithromycin as a single dose and doxycycline or erythromycin for 7 days.

A 40-year-old man with a history of untreated HIV for 8 years comes into the ED complaining of cough, fever, and malaise for 3 days. He is tachypneic and diaphoretic. Chest radiograph reveals bilateral infiltrates. Arterial blood gas (ABG) analysis is significant for a PaO2 of 62 on room air. His chest radiograph is seen below. Which of the following is the most appropriate initial management? a. Corticosteroid treatment prior to antibiotic therapy b. Immediate treatment with IV trimethoprim/sulfamethoxazole (TMP/SMX) c. Administer antibiotics after a rapid sputum Gram stain is obtained d. Nebulizer treatment e. Racemic epinephrine

*A.* In a patient with untreated HIV and bilateral infiltrates on chest x-ray, PCP must be considered, in addition to community-acquired pneumonia. Corticosteroids are shown to be beneficial as adjunctive therapy in patients with moderate to severe PCP. They (1) limit oxygen deterioration, (2) decrease mortality and respiratory failure, and (3) accelerate recovery. Therapy should be initiated to all children and adult patients with a PO2 < 70 mm Hg or an A-a gradient > 35 mm Hg. It is important to initiate steroid therapy prior to starting antibiotics to avoid worsening hypoxia that is secondary to the inflammatory reaction caused by dying organisms.

A 68-year-old woman with recently diagnosed uterine cancer is brought to the ED by her daughter. The patient complains of acute onset right-sided chest pain that is sharp in character and worse with inspiration. Her BP is 135/85 mm Hg, HR 107 beats per minute, RR 20 breaths per minute, and oxygen saturation 97% on room air. Physical examination reveals a swollen and tender right calf. ECG is sinus tachycardia. Which of the following is the most appropriate next step in management? a. Start heparin therapy prior to diagnostic study b. Administer thrombolytics c. Order a ventilation-perfusion scan d. Order a CT angiogram e. Order a D-dimer

*A.* In the absence of a contraindication and a pretest probability that exceeds 50%, empiric heparin therapy should be administered. The patient has a high pretest probability for a PE with the following suggestive features: malignancy, tachycardia, tachypnea, suspected DVT, pleuritic chest pain, and dyspnea. Although heparin has no intrinsic fibrinolytic effect, it has an immediate effect on thrombin inhibition, thus preventing extension of the PE.

A 71-year-old man is playing cards with some friends when he starts to feel a pain in the left side of his chest. His fingers in the left hand become numb and he feels short of breath. His wife calls the ambulance and he is brought to the hospital. In the ED, an ECG is performed. Which of the following best describes the order of ECG changes seen in an MI? a. Hyperacute T wave, ST-segment elevation, Q wave b. Q wave, ST-segment elevation, hyperacute T wave c. Hyperacute T wave, Q wave, ST-segment elevation d. ST-segment elevation, Q wave, hyperacute T wave e. ST-segment elevation, hyperacute T wave, Q wave

*A.* The earliest ECG finding resulting from an AMI is the hyperacute T wave, which may appear minutes after the interruption of blood flow. The hyperacute T wave, which is short-lived, evolves to progressive elevation of ST segments. In general, Q waves represent established myocardial necrosis and usually develop within 8 to 12 hours after a ST-elevation MI, though they may be noted as early as 1 to 2 hours after the onset of complete coronary occlusion.

A 51-year-old man describes 1 week of gradually worsening scrotal pain and dysuria. He is sexually active with his wife. His temperature is 100.1°F, HR 81 beats per minute, BP 140/75 mm Hg, and oxygen saturation is 99% on room air. On physical examination, his scrotal skin is warm and erythematous. A cremasteric reflex is present. The posterior left testicle is swollen and tender to touch. Color Doppler ultrasonography demonstrates increased testicular blood flow. Urinalysis is positive for leukocyte esterase. What is the most likely diagnosis? a. Epididymitis b. Testicular torsion c. UTI d. Testicular tumor e. Varicocele

*A.* The patient has epididymitis. It is often difficult to distinguish epididymitis from testicular torsion and the clinician should always rule out torsion first if the diagnosis is in doubt. It is the most common misdiagnosis for testicular torsion. Epididymitis is generally a disease of adult men. The causative organism in men over 35 years old is E coli, while C trachomatis and Neisseria gonorrhoeae predominate in men less than 35 years old. The older patient may have a history of gonococcal urethritis (GU) tract manipulation or a history of prostatitis. The onset is usually gradual and urinary tract symptoms may precede the pain.

A 55-year-old man presents to the ED with worsening weakness, muscle cramps, and paresthesias. His past medical history is significant for hypertension and diabetes. He smokes one pack of cigarettes per day. On examination, the patient is alert and oriented and diffusely weak. An ECG shows a wide QRS complex, peaked T waves, and no P waves. Which of the following is the most important next step in management? a. Administer calcium gluconate. b. Administer insulin and dextrose. c. Administer aspirin and call the catheterization laboratory. d. Order an emergent head CT scan and get a neurology consult. e. Collect a sample of his urine to test for ketones.

*A.* The patient has life-threatening hyperkalemia. His ECG shows a wide QRS complex, peaked T waves, and no P waves. At any moment the patient's rhythm can go into ventricular fibrillation or asystole. There are many sxs of hyperkalemia that are often difficult to discern from those of the primary condition that precipitated the hyperkalemia. Patients may begin with lethargy and weakness and progress to paralysis and areflexia. If there are no ECG abnormalities in a patient with hyperkalemia, treatment can start with potassium binding resins (eg, Kayexalate). However, this patient requires immediate administration of calcium because he has an unstable cardiac rhythm. Calcium (gluconateor chloride) antagonizes the effects of potassium in the myocardium and briefly stabilizes the cardiac membrane.

While eating dinner, a 55-year-old man suddenly feels a piece of steak "get stuck" in his stomach. In the ED, he complains of dysphagia, is drooling, and occasionally retches. On examination, his BP is 130/80 mm Hg, HR is 75 beats per minute, RR is 15 breaths per minute, and oxygen saturation is 99% on room air. He appears in no respiratory distress. Chest x-ray is negative for air under the diaphragm. Which of the following is the most appropriate next step in management? a. Administer 1-mg glucagon intravenously while arranging for endoscopy. b. Administer a meat tenderizer such as papain to soften the food bolus. c. Administer 10-mL syrup of ipecac to induce vomiting and dislodge the food bolus. d. Perform the Heimlich maneuver until the food dislodges. e. Call surgery consult to prepare for laparotomy.

*A.* The patient most likely has a partial or complete obstruction in his lower esophagus secondary to the steak he ate. This usually occurs near the gastroesophageal junction. Administration of glucagon may cause enough relaxation of the esophageal smooth muscle to allow passage of the bolus in approximately 50% of patients. Its relaxant effect is limited to smooth muscle and therefore can only be used for impactions in the lower esophagus. If glucagon does not work, definitive management is with endoscopy.

A 58-year-old woman is brought to the emergency department (ED) by emergency medical service (EMS) after slipping on a patch of ice while walking to work and hitting her head on the cement pavement. Bystanders acknowledged that the patient was unconscious for approximately 1 minute. On arrival, her vital signs are: blood pressure (BP) 155/75 mm Hg, heart rate (HR) 89 beats per minute, respiratory rate (RR) 18 breaths per minute, and pulse oxygenation 98% on room air. She has a 5-cm laceration to the back of her head that is actively bleeding. You ask the patient what happened but she cannot remember. You inform her that she is in the hospital as a result of a fall. Over the next 10 minutes she asks you repeatedly what happened and where she is. You do not find any focal neurologic deficits. As you bring the patient to the CT scanner she vomits once. CT results show a normal brain scan. Which of the following is the most likely diagnosis? a. Cerebral concussion b. Diffuse axonal injury c. Cerebral contusion d. Posttraumatic epilepsy e. Trauma-induced Alzheimer disease

*A.* The patient sustained a cerebral concussion. This is caused by a head injury leading to a brief loss of neurologic function. These individuals are often amnestic to the event and frequently ask the same questions over and over again (perseverations). Headache with or without vomiting is generally present; however, there are no focal neurologic findings on examination. Loss of consciousness results from impairment of the reticular activating system. Patients show rapid clinical improvement. CT scan is normal.

A 60-year-old man is brought to the ED complaining of generalized crampy abdominal pain that occurs in waves. He has been vomiting intermittently over the last 6 hours. His BP is 150/75 mm Hg, HR is 90 beats per minute, temperature is 99.8°F, and his RR is 16 breaths per minute. On abdominal examination you notice an old midline scar the length of his abdomen that he states was from surgery after a gunshot wound as a teenager. The abdomen is distended with hyperactive bowel sounds and mild tenderness without rebound. An abdominal plain film confirms your diagnosis. Which of the following is the most appropriate next step in management? a. Begin fluid resuscitation, bowel decompression with a nasogastric tube, and request a surgical consult. b. Begin fluid resuscitation, administer broad-spectrum antibiotics, and admit the patient to the medical service. c. Begin fluid resuscitation, give the patient stool softener, and administer a rectal enema. d. Begin fluid resuscitation, administer broad-spectrum antibiotics, and observe the patient for 24 hours. e. Order an abdominal ultrasound, administer antiemetics, and provide pain relief

*A.* The patient's clinical picture is consistent with a SBO. Fluid resuscitation is important because of the inability of the distended bowel to absorb fluid and electrolytes at a normal rate. Compounded with vomiting, fluid loses can lead to hypovolemia and shock. Nasogastric suction provides enteral decompression by removing accumulated gas and fluid proximal to the obstruction. A surgical consult is necessary because definitive treatment may require taking the patient to the OR to relieve the obstruction. An old surgical adage states "Never let the sun set or rise on a bowel obstruction." Broad-spectrum antibiotics (b and d) are appropriate when surgery is planned or when there is suspicion for vascular compromise or bowel perforation.

A 49-year-old woman presents to the ED with difficulty breathing after a morning jog. Her initial vitals include a HR of 60 beats per minute, a BP of 120/55 mm Hg, and an RR of 20 breaths per minute with an oxygen saturation of 94% on room air. Upon physical examination, the patient appears to be in mild distress with audible wheezing. She is able to speak in partial sentences and states that she occasionally uses an inhaler. Given this patient's history and physical examination, which of the following measures should be taken next? a. Peak expiratory flow b. Chest radiograph c. β-Natriuretic peptide level d. Rectal temperature e. ABG

*A.* This patient is suffering from an exercise-induced asthma exacerbation. Various triggers can cause bronchospasm. These include dust, various perfumes, underlying upper respiratory infections, cigarette smoke, menstrual flow, and various medications including aspirin. The medical intervention in this patient should include a β2-agonist nebulized solution, corticosteroids, and oxygen administration. The most useful measure to track the patient's progress with each treatment is a peak expiratory flow, and should be a part of the initial assessment and monitoring. The forced expiratory volume in 1 second from a maximal inspiration (FEV1) may be used in the ED setting. More commonly, peak flow meters are used which measure the peak expiratory flow rate in liters per second starting with fully inflated lungs. Both of these measurements require full patient cooperation whose values should be the average of three consecutive forced expirations. Normalized value ranges are determined by the patient's age, sex, and height.

A 60-year-old male cook reports spilling hot oil over his left thigh while attempting to pull a pan off of the stove. He presents to the ED with erythema extending over his left thigh without any blister formation or circumferential involvement. The patient reports that he was wearing thick pants at the time of the accident which he removed quickly and promptly irrigated the area with water. There are no other signs of injury. What is the burn degree and relative area of involvement in this patient? a. First-degree/9% b. First-degree/4.5% c. Second-degree/2.25% d. Second-degree/9% e. Third-degree/4.5%

*B*

A 48-year-old man is brought to the ED by family members who state that the patient has remained home-bound for weeks, sleeping for many hours, and appears disheveled. The patient states that he is "fine" and denies any medical symptoms. Initial vitals include HR of 77 beats per minute, BP of 118/55 mm Hg, and RR of 12 breaths per minute with oxygen saturation of 97% on room air. The patient is afebrile with an unremarkable physical examination. He denies any chest discomfort, difficulty breathing, constipation, cold intolerance, weakness, weight changes, or pain. The patient reports that he has had difficulty concentrating, a decreased appetite, and excessive sleeping patterns. The family reports that this has happened before, but that his symptoms self-resolved and were not nearly as severe. Given this patient's presentation, which of the following is the most likely etiology of this patient's symptoms? a. Hypothyroidism b. Major depressive episode c. Diabetes mellitus d. Subdural hematoma e. Cushing syndrome

*B* A major depressive episode is characterized by two or more of the following symptoms over a 2-week period: loss of interest in usual activities, depressed or irritable mood, changes in weight or appetite, insomnia or hypersomnia, psychomotor agitation or retardation, loss of energy, difficulty concentrating, recurrent thoughts of death, and suicidal ideation. This particular patient has three of these symptoms, including excessive sleeping patterns, difficulty concentrating, and a decreased appetite. The danger is that the patient's feelings may be so intensely painful that suicide may be seen as the only way to cope. Some patients may also complain of generalized physical pain, without any clear medical diagnosis able to be made. The cardinal symptom of depression is a sad or dysphoric mood.

A 63-year-old man is brought to the ED by EMS complaining of severe abdominal pain that began suddenly 6 hours ago. His BP is 145/75 mm Hg and HR is 105 beats per minute and irregular. On examination, you note mild abdominal distention and diffuse abdominal tenderness without guarding. Stool is heme positive. Laboratory results reveal WBC 12,500/μL, hematocrit 48%, and lactate 4.2 U/L. ECG shows atrial fibrillation at a rate of 110. A CT scan shows thickening of bowel wall Which of the following is the most likely diagnosis? a. Abdominal aortic aneurysm b. Mesenteric ischemia c. Diverticulitis d. SBO e. Crohn disease

*B* Acute mesenteric ischemia is caused by the lack of perfusion to the bowel. It primarily affects patients over the age of 50 years, particularly those with significant cardiovascular or systemic disease. The etiology is either from arterial occlusion (eg, embolization from the heart); venous thrombosis that is associated with a hypercoagulable state; or nonocclusive, because of reduced cardiac output (eg, congestive heart failure [CHF], recent myocardial infraction [MI], hypovolemia). Most patients present with abdominal pain that is initially dull and diffuse. In this early state, patients frequently complain of severe pain, but have minimal tenderness on examination (ie, the characteristic "pain out of proportion to examination"). As infarction develops, abdominal distension and peritoneal signs develop. Sudden onset of pain suggests arterial vascular occlusion by emboli. This may occur with patients who present in atrial fibrillation. Insidious onset suggests venous thrombosis or nonocclusive infarction. Radiographs may reveal dilated loops of bowel, air in the bowel wall ("pneumatosis intestinalis"), and thickening of the bowel wall, as seen in the CT scan. Management involves fluid resuscitation, antibiotics, and surgical intervention.

A 27-year-old man is seen in the ED for a leak around a surgical G-tube that was placed 2 weeks ago and has been used for enteral feeding for 1 week. Inspection reveals the tube is pulled out from the stoma, but is still in the cutaneous tissue. The abdomen is soft and nondistended and there are no signs of skin infection. Which of the following is the most appropriate next step in management? a. Insert a Foley catheter into the tract and aspirate. If gastric contents are aspirated the tube can be used for feeding. b. Insert a Foley catheter into the tract, instill water-soluble contrast, and obtain an abdominal radiograph prior to using for feeding. c. Remove the tube and admit the patient for observation. d. Remove the tube and immediately obtain a CT scan of the abdomen. e. Return to the OR for closure of gastrotomy and placement of a new tube.

*B* Although there are no studies stating how long it takes for a tract to mature, tracts that are 7 to 10 days old probably will remain open long enough to allow replacement. Insertion of a new tube should be performed with water-soluble lubricant. If resistance is met, the attempt should be aborted. After replacing the tube, 20- to 30-mL bolus of water soluble contrast material should be instilled into the tube, and a supine abdominal radiograph should demonstrate rugae of the stomach. If there is any question of improper placement, immediate consultation should be obtained.

A 41-year-old man, the restrained driver in a high-speed motor vehicle collision, is brought to the ED by EMS. The patient is breathing without difficulty with bilateral and equal breaths sounds. He has strong pulses peripherally indicating a BP of at least 90 mm Hg. The HR is 121 beats per minute. His Glasgow Coma Scale (GCS) is 14. A secondary survey reveals chest wall bruising. You suspect a cardiac injury. Which of the following locations most commonly involve cardiac contusions? a. Right atrium b. Right ventricle c. Left atrium d. Left ventricle e. Septum

*B* Blunt cardiac injuries usually result from high-speed vehicular collisions in which the chest wall strikes the steering wheel. Although they are all associated with potentially fatal complications, they should be viewed clinically as a continuous spectrum of myocardial damage: concussion (no permanent cell damage), contusion (permanent cell damage), infarction (cell death), tamponade (bleeding into the pericardium), and rupture (exsanguination). The mechanism of injury in a cardiac contusion involves a high-speed deceleration, which causes the heart to move forward, forcibly striking the sternum. In addition, the direct force of hitting an object (eg, the steering wheel) also can damage the heart. The right ventricle is the most commonly injured because it is the most anterior aspect of the heart and is closest to the sternum.

A 45-year-old man is brought into the ED after a head-on motor vehicle collision. His BP is 85/45 mm Hg and HR is 130 beats per minute. He is speaking coherently. His breath sounds are equal bilaterally. After 2 L of fluid resuscitation, his BP is 80/40 mm Hg. A FAST examination reveals fluid in Morison pouch. Which of the following organs is most likely to be injured in blunt abdominal trauma? a. Liver b. Spleen c. Kidney d. Small bowel e. Bladder

*B* Blunt trauma is the most common mechanism of injury seen in the United States. The forces exerted on the abdomen put all of the organs at risk for injury. Motor vehicle collisions with another vehicle or with pedestrians are the major causes of blunt abdominal trauma. The spleen is the organ most often injured, and in approximately 66% of these cases, it is the only damaged intraperitoneal organ.

A 41-year-old office worker presents to the ED with pain in her right wrist and fingers that is associated with a tingling sensation. The pain occasionally awakens her from sleep and improves when the hand is shaken. She recalls falling on her wrist 2 years ago while she was ice-skating. Her symptoms are reproducible when her wrist is held in flexion for 60 seconds. Which of the following nerves is affected in patients with this syndrome? a. Ulnar b. Median c. Axillary d. Radial e. Musculocutaneous

*B* Carpal tunnel syndrome is a neuropathy of the median nerve that occurs as a result of compression of the nerve within the carpal tunnel of the wrist. Symptoms include paresthesias and pain in the distribution of the median nerve; the volar thumb, second, third, and half of the fourth digit. Symptoms are usually worse after strenuous activities and at night. Pain improves with shaking of the hand or holding it in a dependent position. A sensitive test used to diagnose carpal tunnel is Phalen test. Have the patient hold the affected wrist in hyperflexion for 60 seconds. The test is positive if paresthesias or numbness develops in the median nerve distribution

A 40-year-old veterinary assistant presents to the ED with puncture wounds over her right upper extremity and neck. She reports being bitten multiple times by a cat in her care, also stating that the cat's immunizations were up to date. The injury was sustained 2 days ago with minimal initial symptoms. However, today the patient noticed redness and pain in that area. She denies any fever, chills, nausea, vomiting, or any other constitutional symptoms. Her initial vitals include an oral temperature of 99.7°F, a heart rate (HR) of 90 beats per minute, blood pressure (BP) of 125/75 mm Hg, respiratory rate (RR) of 14 breaths per minute, and an oxygen saturation of 99% on room air. Multiple punctate wounds may be seen over her right lateral neck extending down to her deltoid with surrounding erythema and edema. Which antibiotic coverage for the specific organism involved in this type of injury would be most appropriate? a. A first-generation cephalosporin/Staphylococcus aureus b. Amoxicillin/Clavulanate (Augmentin)/Pasteurella multocida c. Clindamycin/Streptococcus sp. d. Vancomycin/Methicillin resistant S aureus e. Bacitracin ointment/S aureus

*B* Cat bites involve puncture wounds often extending down through skin into tendons and bones owing to the nature of the animals' sharp teeth. The reported incidence of infection from these bites may be as high as 30% to 50%, with many patients presenting only after an infection has incurred. It is important to note that cat bites have a higher infection rate compared to dog bites, given the typical puncture wound that inoculates bacteria down the track deep into tissue and becomes enclosed. Pasteurella multocida is a strong gram-negative, facultative anaerobic rod found in the oral cavity of the majority of healthy cats and may cause severe systemic infection, especially in the immunocompromised. Patients need to be followed closely as these infections may seed deep into joints and tissue requiring debridement.

A 67-year-old woman is brought to the ED after being struck by a cyclist while crossing the street. On arrival to the ED, her eyes remain closed to stimuli, she makes no verbal sounds, and withdraws only to painful stimuli. You assign her a GCS of 6. Her BP is 175/90 mm Hg and HR is 75 beats per minute. As you open her eye lids, you notice that her right pupil is 8 mm and nonreactive and her left is 4 mm and minimally reactive. Which of the following is the most common manifestation of increasing intracranial pressure (ICP) causing brain herniation? a. Change in level of consciousness b. Ipsilateral pupillary dilation c. Contralateral pupillary dilation d. Significantly elevated BP e. Hemiparesis

*B* Cerebral herniation occurs when increased ICP overwhelms the natural compensatory capacities of the central nervous system (CNS). Increased ICP may be the result of post traumatic brain swelling, edema formation, traumatic mass lesion expansion, or any combination of the three. When increasing ICP cannot be controlled, the intracranial contents will shift and herniate through the cranial foramen. Herniation can occur within minutes or up to days after a traumatic brain injury. Once the signs of herniation are present, mortality approaches 100% without rapid reversal or temporizing measures. Uncal herniation is the most common clinically significant form of traumatic herniation and is often associated with traumatic extracranial bleeding. The classic signs and symptoms are caused by compression of the ipsilateral uncus of the temporal lobe. This causes compression of cranial nerve III leading to anisocoria, ptosis, impaired extraocular movements, and a sluggish pupillary light reflex. As herniation progresses, compression of the ipsilateral oculomotor nerve eventually causes ipsilateral pupillary dilation and nonreactivity.

22-year-old man is brought to the ED after sustaining a single gunshot wound (GSW) to his right thigh. On arrival, his HR is 105 beats per minute and BP is 115/75 mm Hg. You note a large hematoma of his medial thigh. The patient complains of numbness in his right foot. On extremity examination, the right foot is pale and you cannot palpate a distal pulse but can locate the dorsalis pedis by Doppler. In addition, the patient cannot move the foot. Which of the following is the most appropriate next step in management? a. Angiography b. Exploration and repair in the OR c. Fasciotomy to treat compartment syndrome d. Wound exploration e. CT scan of the right extremity

*B* Clinical manifestations of penetrating arterial injury of the extremity are generally divided into "hard signs" and "soft signs." Hard signs include pulsatile bleeding, expanding hematoma, palpable thrill or audible bruit, and evidence of distal ischemia (eg, pain, pallor, pulselessness, paralysis, paresthesia, poikilothermia). Soft signs include diminished ankle-brachial indices, asymmetrically absent or weak distal pulse, history of moderate hemorrhage or wound close to a major artery, and a peripheral nerve deficit. Emergent surgery is generally necessary when there are hard signs of vascular injury. Although the management of penetrating extremity injury is evolving, whenever there is evidence of distal ischemia, the patient should be taken to the OR for exploration and repair. When severe ischemia is present, the repair must be completed within 6 to 8 hours to prevent irreversible muscle ischemia and loss of limb function. In the presence of hard signs without evidence of ischemia, some surgeons may prefer to first perform angiography to better define the injury.

A 35-year-old woman presents to the ED complaining of left arm weakness and right facial pain for 1 day. She denies any past medical history, but on the review of systems remembers having pain and decreased vision in her left eye approximately 4 months ago that has since resolved. She attributed it to being stressed and tired and did not see a physician at the time. Her BP is 126/75 mm Hg, HR is 76 beats per minute, temperature is 98.8°F, and RR is 12 breaths per minute. The physical examination is unremarkable except for 4/5 strength in the left upper extremity. Which of the following is the most likely diagnosis? a. Myasthenia gravis b. Multiple sclerosis (MS) c. Vertebrobasilar artery occlusion d. Encephalitis e. Guillain-Barré syndrome

*B* Consider MS as a diagnosis in presentations of various neurologic symptoms that are difficult to explain by a single CNS lesion, particularly those occurring in a female in her third decade of life. MS is a multifocal demyelinating CNS disease that in 30% of cases initially presents with optic neuritis (unilateral eye pain and decreased visual acuity).

A 19-year-old man is brought into the ED by paramedics with a stab wound to the right lower abdomen. The medics applied a pressure dressing and started an IV line en route to the hospital. On arrival, the patient has no complaints and wants to leave. His temperature is 98.4°F, BP is 130/95 mm Hg, HR is 111 beats per minute, RR is 20 breaths per minute, and oxygen saturation is 98% on room air. He is alert and oriented to person, place, and time. His abdomen is soft and nontender, with normal bowel sounds. He has a 2-cm stab wound with visible subcutaneous fat in his right lower quadrant (RLQ). You initiate the focused abdominal sonogram for trauma (FAST) examination. Which type of fluid should you start for his initial resuscitation? a. 7% Sodium chloride b. 0.9% Sodium chloride c. 10% Albumin d. Type and cross-matched blood e. Type-specific blood

*B* Crystalloids, such as normal saline (0.9% sodium chloride) or lactated ringers are the preferred resuscitation fluid in the United States. The patient is currently stable; however, he is tachycardic and has suffered an injury with the potential for significant morbidity. The FAST examination will help illustrate the extent of this patient's injury. In the meantime, the patient should be started on isotonic crystalloid solution. There is currently no convincing evidence that either solution—normal saline or Ringers lactate—is superior to the other.

A 19-year-old woman presents to the ED with abdominal pain, nausea, vomiting, diarrhea, and hematemesis after ingesting an unknown substance in a suicide attempt. Which of the following antidotes are correctly paired? a. Organophosphate—Physostigmine b. Iron overdose—Deferoxamine c. Aspirin overdose—NAC d. Benzodiazepine overdose—Narcan e. Anticholinergic overdose—Fomepizole

*B* Deferoxamine is a specific chelator of ferric iron (Fe3+). It binds with iron to form a water-soluble compound, ferrioxamine, which can be excreted by the kidneys. Deferoxamine has a half-life of 1 hour, so continuous infusion is the preferred method of administration. The patient's clinical presentation is consistent with acute iron poisoning. Initial presentation reflects the corrosive effects of iron on the gut and includes nausea, vomiting, diarrhea, and sometimes GI bleeding. Patients with severe overdose may present with shock or coma.

A 27-year-old woman is brought to the ED by her husband after having a first-time seizure at home. She has no past medical history, and had no complications while delivering her newborn vaginally 1 week prior to presentation. In the ED, her BP is 178/95 mm Hg, HR is 97 beats per minute, temperature is 99.1°F, and RR is 18 breaths per minute. On examination, she has mild edema of her hands and feet. The seizure stopped spontaneously, but the patient is postictal appearing and cannot answer your questions. Which of the following is the most appropriate diagnostic test? a. Complete blood count b. Head CT c. LP d. Urinalysis e. ECG

*B* Eclampsia develops after the 20th week of gestation and is considered a complication of severe preeclampsia. The progression from severe preeclampsia to seizures and coma is thought to be a result of hypertensive encephalopathy, vasogenic edema-associated cortical ischemia, edema, or hemorrhage. Therefore, a head CT should be quickly obtained. Eclampsia is a clinical diagnosis with patients having seizures without evidence of CNS, metabolic, or any other seizure etiology. Most patients have systolic BP higher than 160 mm Hg or diastolic BP higher than 110 mm Hg and proteinuria; however, eclampsia can occur with minimally elevated BP—or elevated, relative to baseline—and without proteinuria. Evidence of end-organ damage prior to development of seizures is common. Symptoms include altered mental status, headache, visual disturbances, and abdominal pain; signs include hemolysis, impaired liver function with elevated liver enzyme levels, low platelets (HELLP), hemoconcentration, proteinuria, oliguria, pulmonary edema, generalized peripheral edema, microangiopathic hemolytic anemia, and fetal growth retardation. Eclampsia is most common during the antepartum period, yet 20% to 25% of cases ccur during the postpartum period. Although postpartum eclampsia may occur as long after birth as 3 weeks postpartum, most cases (98%) occur on the first postpartum day. Eclampsia is not the cause of seizures that occur during the first trimester or well into the postpartum period. These seizures are suggestive of CNS pathology.

A 31-year-old man is brought to the ED by EMS who state that the man was found lying on the floor of his garage. He is rousable in the ED, speaks with slurred speech, and vomits. His BP is 140/85 mm Hg, HR is 94 beats per minute, temperature is 98.8°F, RR is 17 breaths per minute, and oxygen saturation is 99% on room air. You place an IV line, draw blood, and start a liter of normal saline running through the line. Laboratory results reveal serum sodium 139 mEq/L, potassium 3.5 mEq/L, chloride 101 mEq/L, bicarbonate 14 mEq/L, BUN 15 mg/dL, creatinine 1 mg/dL, glucose 105 mg/dL, arterial blood pH 7.27, COHb 4%, and lactate 2.8 mEq/L. Urinalysis shows: 1+ protein, trace ketones, WBC 4/hpf, red blood cell (RBC) 2 to 3/hpf, and multiple envelope-shaped and needle-shaped crystals. Which of the following conditions would best explain his metabolic acidosis? a. Carbon monoxide (CO) poisoning b. Ethylene glycol poisoning c. Diabetic ketoacidosis d. Lactic acidosis e. Isopropyl alcohol poisoning

*B* Ethylene glycol is a colorless, odorless, slightly sweet-tasting liquid that is found in antifreeze. Ingestions of antifreeze are either accidental, suicidal, or in substitute of ethanol. Ethylene glycol is metabolized to glycolic acid, which results in a profound anion gap metabolic acidosis (Na) − ([Cl] + [HCO3]); (139 − [101 + 14]) = 24. Glycolic acid is subsequently metabolized to oxalic acid, which combines with calcium to form calcium oxalate crystals, which then precipitate in renal tubules, brain, and other tissues. The finding of crystalluria is considered the hallmark of ethylene glycol ingestion; however, its absence does not rule out the diagnosis. Another useful test in the ED involves examining freshly voided urine for fluorescence with a Wood lamp. Sodium fluorescein is added to antifreeze to aid in the detection of radiator leaks. Ingestion of ethylene glycol is associated with neurologic, cardiopulmonary, and renal abnormalities.

A 26-year-old woman, who was found lying on the floor of her apartment next to an unlabeled empty pill bottle, is brought into the ED. Her HR is 117 beats per minute, BP is 95/65 mm Hg, RR is 14 breaths per minute, and oxygen saturation is 97% on 2-L nasal cannula. On examination, the patient appears obtunded, her pupils are 3 mm and reactive. Her oropharynx is dry and there is no gag reflex to pharyngeal stimulation. Her neck is supple. The heart is tachycardic without murmurs, the lungs are clear to auscultation, and the abdomen is soft. There is normal rectal tone and brown stool that is heme negative. Her skin is cool and moist with no signs of needle tracks. Neurologically, she is unresponsive, but withdraws all extremities to deep palpation. Finger stick blood glucose is 85 mg/dL. Her ECG reveals sinus tachycardia at 119 with a QRS complex of 120 msec and a terminal R wave in lead aVR. Which of the following is the most appropriate next step in management? a. Orotracheal intubation, administer activated charcoal through orogastric tube, and IV Narcan b. Orotracheal intubation, administer activated charcoal through orogastric tube, and IV sodium bicarbonate c. Orotracheal intubation, administer activated charcoal through orogastric tube, and IV NAC d. Orotracheal intubation, administer syrup of ipecac through orogastric tube, and IV sodium bicarbonate e. Induce vomiting prior to intubation to lower the risk of aspiration then administer IV sodium bicarbonate

*B* For young patients with altered mental status, toxic ingestion should be high on the differential. This clinical scenario is most consistent with toxic ingestion of a TCA. Treatment of all toxic ingestions should begin with assessment of airway, breathing, and circulation. As a result this patient's obtunded mental status and loss of gag reflex, orotracheal intubation is indicated for airway protection. Subsequently, activated charcoal can be administered. Because of the anticholinergic effects of TCAs, absorption is prolonged and GI motility is delayed leading to greater toxicity. Therefore, an additional dose of charcoal should be administered several hours later. In an obtunded patient, it is important to first secure an airway prior to administering charcoal to prevent aspiration in the event of vomiting.

A 40-year-old diabetic man presents to the ED with severe perineal pain and fever as high as 103°F. Physical examination demonstrates crepitus over the medial thigh and widespread discoloration with sharp demarcation over the scrotum. The scrotum is warm and markedly edematous. His pain appears out of proportion to the physical examination. Which of the following is the most likely diagnosis? a. Epididymitis b. Fournier gangrene c. Scrotal edema d. Paraphimosis e. Testicular torsion

*B* Fournier gangrene is a polymicrobial necrotizing fasciitis of the perineal subcutaneous tissue and male genitalia that originates from the skin, urethra, or rectum. It usually begins as a simple abscess or benign infection that quickly spreads, especially in an immunocompromised patient, often a diabetic male. If not promptly diagnosed, it can lead to end-artery thrombosis in the subcutaneous tissue and widespread necrosis. Treatment includes aggressive fluid resuscitation; broad-spectrum antibiotics to cover gram-positive, gramnegative, and anaerobic bacteria; wide surgical debridement; and, possibly, hyperbaric oxygen therapy.

A 58-year-old man presents to the emergency department (ED) with reported blister formation over both feet that initially began 2 days ago. He denies any past medical history, medications, or allergies. His social history is significant for alcohol dependence and recently becoming undomiciled. He denies any sick contacts or recent travel. Upon physical examination, these lesions are fluid-filled and his feet are grossly cyanotic and tender to the touch. His foot is shown below. What is the most likely diagnosis in this patient? a. Chilblains b. Frostbite c. Trench foot d. Thermal burn e. Herpes

*B* Frostbite usually occurs when temperatures fall below 0°C (32°F). There are essentially three phases to the freezing injury cascade. Phase 1 (prefreeze) includes initial skin cooling, increased blood viscosity, and microvascular leakage which causes localized edema formation. Phase 2 (freeze-thaw) is when extracellular crystal formation begins thereby causing intracellular shrinkage and essentially collapse of the cellular network. Finally, phase 3 (vascular stasis/progressive ischemia) involves further coagulation, interstitial leakage, and cell death thus resulting in blister formation, cyanosis, and ultimately mummification of the tissue. The bullae formed may also have a hemorrhagic appearance. It is important to note that wind and moisture may increase the freezing rate. Management includes rapid rewarming with water at a temperature of 37°C to 40°C (98.6°F-104°F) with care in preventing refreezing. Friction massage, which furthers tissue loss, should be avoided. Rewarming is a painful procedure that requires parenteral analgesia. Patients may also have a degree of dehydration and benefit from crystalloid administration.

A 40-year-old man who is an employee of the hospital is brought to the ED actively seizing. A coworker states that the patient has a known seizure disorder and currently takes phenytoin for the disorder. He also tells you that the patient has been under stress recently and may not have taken his last few doses of medication. You call for the nurse to place a face mask with 100% oxygen and gain IV access. You then ask for a medication to be drawn up. Which is the most appropriate initial medication you should administer in this actively seizing patient? a. Phenytoin b. Diazepam c. Phenobarbital d. Valproic acid e. Lithium

*B* Generally, the first-line pharmacologic treatment in an actively seizing patient is a parental benzodiazepine, such as diazepam (Valium), lorazepam (Ativan), or midazolam (Versed). Benzodiazepines are effective in terminating ictal activity in 75% to 90% of patients. Diazepam can be administered intravenously, intramuscularly, or down an endotracheal tube. Lorazepam and midazolam can be given intravenously or intramuscularly. All three have similar efficacy in terminating seizures.

An 18-year-old man is brought into the ED and reports that a stray dog at the park bit him. The patient complains of right forearm pain, where he was bitten. Upon physical examination, you note a superficial macerated laceration on the dorsal surface of the distal forearm with no active bleeding. The patient is able to freely flex and extend all joints in the right upper extremity. In addition to localized wound care, antibiotics, and tetanus prophylaxis, what other expeditious measures should be taken in the care of this patient? a. Reporting the incident to local authorities b. Rabies immunization c. Tight suturing of the laceration d. Antihistamines e. Irrigation with povidone-iodine solution

*B* Given that the bite was sustained by an unknown stray dog that may be difficult to locate, rabies immunization should be given promptly. If the dog could be found, quarantining the animal for 10 days for observation may be a valid option and could save the patient from the rigorous postexposure prophylaxis schedule. It is important to note the powerful effects of water and soap irrigation in these cases, as it has been proven as the most effective means of lowering the virulence of the organism and should be initiated quickly. For immunizing against the rabies virus, it is important to give the immunoglobulin in addition to the cell vaccine. As much of the initial dose should be infiltrated around the wound site and then be given intramuscularly. Pregnancy is not a contraindication to giving the vaccine. All patients should be monitored for an antibody response. In the United States, the predominant threat to humans is bats, which most commonly carry the virus. This should be considered in individuals who are cave spelunkers and present with hydrophobia and neurologic symptoms. Small rodents (squirrels, rats, chipmunks) and lagomorphs are herbivores and do not carry the virus.

A 40-year-old man with a known history of ethanol abuse states that 2 hours ago he ingested two bottles of extra-strength Tylenol. The patient has no medical complaints except for some nausea. At 4 hours post ingestion you send blood to the laboratory to measure the serum acetaminophen concentration. The level returns and falls above the treatment line when you plot it on the APAP nomogram. You administer activated charcoal and decide to start IV NAC. Which of the following is a known adverse effect of IV NAC administration? a. Hepatic failure b. Anaphylactoid reaction c. Hypertensive crisis d. Confusion e. Change in urine color

*B* IV NAC has been responsible for anaphylactoid reactions, including rash, bronchospasm, hypotension, and death. These complications are dose and concentration dependent and are prevented by slow administration of dilute NAC. Some other side effects include GI disorders, tachycardia, and chest tightness.

A 34-year-old man is brought to the ED after being shot in the right side of his chest. The patient is awake and speaking. Breath sounds are diminished on the right. There is no bony crepitus or tracheal deviation. His BP is 95/65 mm Hg, HR is 121 beats per minute, and RR is 23 breaths per minute. Supine chest radiograph reveals a hazy appearance over the entire right lung field. You place a 36F chest tube into the right thoracic cavity and note 1200 cc of blood in the chest tube drainage system. Which of the following is an indication for thoracotomy? a. 500 cc of initial chest tube drainage of blood b. 1200 cc of initial chest tube drainage of blood c. Persistent bleeding from the chest tube at a rate of 50 cc/h d. Chest radiograph with greater than 50% lung field white out e. Evidence of a PTX

*B* Indications for thoracotomy include: • Initial chest tube drainage of 1000 to 1500 cc of blood • 200 cc/h of persistent drainage • Patient remains hypotensive despite adequate blood replacement, and other sites of blood loss have been ruled out. • Patient decompensates after initial response to resuscitation. • Increasing hemothorax seen on chest x-ray studies.

A 22-year-old woman presents to the ED with vaginal bleeding that began earlier in the day. She reports that her last menstrual period was 3 weeks ago. She denies any metromenorrhagia in the past. Upon physical examination, her cervical os is closed with clots in the posterior vaginal vault. There is no adnexal or cervical motion tenderness. She has mild abdominal cramps but no localizing pain. Her pregnancy test is negative. Given this patient's clinical presentation, what is the most likely diagnosis? a. Threatened abortion b. Normal menstrual flow c. Ectopic pregnancy d. Dysfunctional uterine bleeding e. PID

*B* It is important to remember that sometimes patients come to the ED with benign or normal conditions in which education and counsel are the treatments of choice. In this case, it is true that her menstrual flow is early but a one-time occurrence does not give the diagnoses of dysfunctional uterine bleeding

A 23-year-old woman who is an elementary school teacher is brought to the ED after syncopizing in her classroom while teaching. Prior to passing out, she describes feeling lightheaded and dizzy and next remembers being in the ambulance. There was no evidence of seizure activity. She hasm no medical problems and does not take any medications. Her father died of a "heart problem" at 32 years of age. She does not smoke or use drugs. BP is 120/70 mm Hg, pulse rate is 71 beats per minute, RR is 14 breaths per minute, and oxygen saturation is 100% on room air. Her physical examination and laboratory results are all normal. A rhythm strip is shows prolonged QT interval. Which of the following is the most likely diagnosis? a. Wolff-Parkinson-White syndrome b. Long QT syndrome c. Lown-Ganong-Levine syndrome d. Complete heart block e. Atrial flutter

*B* Long QT syndrome (LQTS) is a congenital disorder characterized by a prolongation of the QT interval on ECG and a propensity to ventricular tachydysrhythmias, which may lead to syncope, cardiac arrest, or sudden death in otherwise healthy individuals. The QT interval on the ECG, measured from the beginning of the QRS complex to the end of the T wave, represents the duration of activation and recovery of the ventricular myocardium. In general, HR corrected QTc values above 440 msec are considered abnormal. LQTS has been recognized as the Romano-Ward syndrome (ie, familial occurrence with autosomal dominant inheritance, QT prolongation, and ventricular tachydysrhythmias) or the Jervell and Lang-Nielsen syndrome (ie, familial occurrence with autosomal recessive inheritance, congenital deafness, QT prolongation, and ventricular arrhythmias). Patients with LQTS usually are diagnosed after a cardiac event (eg, syncope, cardiac arrest) already has occurred. In some situations, LQTS is diagnosed after sudden death in family members.

A 46-year-old alcoholic man presents to the ED with cough, fever, and rigors for 2 days. He describes his sputum as rust-colored. His vital signs are: temperature 101.1°F, HR 94, BP 125/75 mm Hg, RR 20, and pulse oxygen of 97% on room air. Auscultation reveals crackles in the left-lower lobe. Chest radiograph is significant for a left-lower lobar infiltrate. Which of the following organism is the most common cause of community acquired bacterial pneumonia? a. Haemophilus influenza b. Streptococcus pneumoniae c. Klebsiella pneumonia d. Group A streptococci e. Pseudomonas aeruginosa

*B* Most pneumonia is the result of a single species of bacteria. Streptococcus pneumoniae is the most common cause of community-acquired pneumonia. It is also the most common cause of bacterial pneumonia in HIV-infected patients. Patients may appear ill and exhibit tachypnea. Fever and chills are usually present. Auscultation of the lungs reveals decreased breath sounds over the infiltrate and the radiograph shows a lobar pattern.

A 65-year-old woman is brought into the ED by her family who states that she has been weak, lethargic, and saying "crazy things" over the last 2 days. Her family also states that her medical history is significant only for a disease of her thyroid. Her BP is 120/90 mm Hg, HR is 51 beats per minute, temperature is 94°F rectally, and her RR is 12 breaths per minute. On examination, the patient is overweight, her skin is dry, and you notice periorbital nonpitting edema. On neurologic examination, the patient does not respond to stimulation. Which of the following is the most likely diagnosis? a. Apathetic thyrotoxicosis b. Myxedema coma c. Graves disease d. Acute stroke e. Schizophrenia

*B* Myxedema coma is a syndrome that represents extreme hypothyroidism. It is a life-threatening condition that has a mortality of up to 50%. Signs and symptoms of hypothyroidism are usually present including dry skin, delayed deep tendon reflexes, coarse hair, and generalized nonpitting edema. Myxedema coma, however, is better characterized by profound lethargy or coma and hypothermia. Hypothermia is present in approximately 80% of patients. In addition, patients may present with respiratory depression and sinus bradycardia.

A 17-year-old man presents to the ED after getting hit in the right eye with a tennis ball during a tennis match. On arrival to the ED, you note periorbital swelling and ecchymosis. The patient's visual acuity is 20/20. When you are testing his extraocular muscles, you note that his right eye cannot look superiorly but his left eye can. He also describes pain in his right eye when attempting to look upward. Which of the following is the most likely diagnosis? a. Zygomatic arch fracture b. Orbital floor fracture c. Retrobulbar hematoma d. Ruptured globe e. Mandible fracture with entrapment of the pterygoid

*B* Orbital floor fractures typically occur when a blunt object with a radius of curvature less than 5 cm (eg, often a fist or ball smaller than a softball) strikes the orbit. The blunt force causes an increase in intraorbital pressure causing a fracture along the weakest part of the orbit, usually the inferior or sometimes medial wall. Patients usually complain of pain that is greatest with upward eye movement. They may have impaired ocular motility or diplopia if the inferior rectus muscle becomes entrapped. They may also present with infraorbital hypoesthesia because of compression of the infraorbital nerve. Generally, the patient has normal visual acuity unless there is an associated ocular injury. A classic radiographic finding is the "teardrop sign," which represents herniated orbital fat and muscle in the roof of the maxillary sinus. There may also be an air-fluid level in the maxillary sinus as a result of bleeding into it. Patients usually do well and recover completely in mild to moderate fractures.

A 34-year-old woman presents to the ED after ingesting an unknown quantity of her antidepressant pills. EMS workers found an empty bottle of amitriptyline on her apartment floor. She is awake but appears delirious. Her BP is 130/65 mm Hg, HR is 101 beats per minute, temperature is 99.1°F, RR is 16 breaths per minute, and oxygen saturation is 100% on room air. On examination, her pupils are 7 mm and reactive. Her face is flushed and mucous membranes are dry. Her lungs are clear and heart is without murmurs. The abdomen is soft, nontender, and with decreased bowel sounds. She is moving all four extremities. ECG reveals sinus rhythm at a rate of 99 and QRS just under 100 msec. In a tricyclic antidepressant (TCA) overdose, which of the following is responsible for her mydriasis, dry mucous membranes, and delirium? a. Sodium channel blockade b. Muscarinic receptor blockade c. Inhibition of serotonin and norepinephrine reuptake d. Histamine receptor blockade e. α-Receptor blockade

*B* Overdose of TCA results in toxicity by a number of different mechanisms. The anticholinergic properties of TCAs result in the toxidrome "blind as a bat (mydriasis), red as a beet (flushed skin), hot as a hare (hyperthermia secondary to lack of sweating), dry as a bone (dry mucous membranes), mad as a hatter (mental status changes)." This is secondary to muscarinic receptor blockade. The cardinal signs of TCA overdose include ventricular dysrhythmias, hypotension, and decreased mental status. Sodium bicarbonate is a potentially lifesaving intervention in TCA overdose because an alkaline pH combined with a sodium load increases conductance through cardiac fast sodium channels and prevents ventricular dysrhythmias as evidenced by narrowing of the QRS complex on an ECG.

A 17-year-old girl presents to the ED with vaginal bleeding, nausea, and vomiting. Her vitals include HR of 80 beats per minute, BP of 190/100 mm Hg, and temperature of 99°F. She reports her last menstrual period was 7 weeks ago. Her physical examination is significant for a fundal height at the umbilicus. After obtaining a positive β-hCG, sonographic evaluation reveals the following image showing grape like vesicles. What is the most likely underlying condition for this patient's symptoms? a. Ectopic pregnancy b. Hydatidiform mole c. Ovarian torsion d. Abruptio placentae e. Hyperemesis gravidarum

*B* Patients presenting with a molar pregnancy typically have severe nausea and vomiting, a uterus larger than expected for dates, intermittent vaginal bleeding, or passage of grape-like contents and hypertension. Risk factors include a previous history of molar pregnancy, and very young or advanced maternal ages. Typically, laboratory results reveal anemia on a CBC, β-hCG higher than expected, and an ultrasound that shows intrauterine echogenic material. Treatment includes dilation and curettage with future monitoring and evaluation for the development of choriocarcinoma.

A 24-year-old G2P0010 in her second trimester presents to the ED with vaginal spotting for the past day. She denies any abdominal pain and is otherwise in her usual state of health. Her vital signs are HR of 76 beats per minute, BP of 120/65 mm Hg, respiratory rate (RR) of 16 breaths per minute, and temperature 98.9°F. Which of the following conditions does this patient most likely have? a. Ectopic pregnancy b. Placenta previa c. Abruptio placentae d. Uterine rupture e. Ovarian torsion

*B* Placenta previa is the most probable etiology for this patient's vaginal bleeding given that it is not painful. Bleeding is rarely severe and the physical examination is usually unremarkable except for a gravid uterus and bright-red blood in the vaginal vault. Manual and speculum pelvic examinations should not be done until placenta previa can be ruled out with an ultrasound, which is often diagnostic. Transvaginal ultrasound can be done because of the wide angle of the probe and low probability of penetrating the cervical os. Vaginal examination may then be performed with awareness that surgical intervention may be necessary to control bleeding. It is also important to note the stage of pregnancy given that first-trimester pregnancies more commonly include spontaneous abortions and ectopics that may present with vaginal spotting.

A 54-year-old man with a history of hepatitis C, alcohol abuse, and cirrhotic ascites presents with increasing abdominal girth and abdominal pain. He complains of increasing difficulty breathing, especially when lying down, caused by worsening ascites. On physical examination, the patient is cachectic and appears older than his stated age. He has a diffusely tender abdomen and tense ascites. The liver is palpable 4 cm below the costal margin. Vitals include a BP of 110/65 mm Hg, HR of 110 beats per minute, RR of 22 breaths per minute, and temperature of 102°F. Which of the followingis the most common organism seen in spontaneous bacterial peritonitis? a. Pseudomonas aeruginosa b. Enterococcus c. Streptococcus pneumoniae d. Enterobacteriaceae e. Streptococcus viridans

*B* Spontaneous bacterial peritonitis (SBP) is the most common complication of cirrhotic ascites and should be suspected in all patients with a history of cirrhosis who present with fever, abdominal pain or tenderness, worsening ascites, or encephalopathy. Paracentesis should be performed to confirm the diagnosis. Ascitic fluid should be tested for glucose, total protein, lactate dehydrogenase (LDH), Gram stain, and WBC count. Culture results are often negative; culture results' yield may be improved, however, by placing 10 mL of ascitic fluid in a blood culture bottle. Total WBC count > 1000/μL or neutrophil count > 250/μL is diagnostic for SBP. Gram-negative Enterobacteriaceae (eg, E coli, Klebsiella) account for 63% of all cases. Empiric treatment for suspected SBP includes third-generation cephalosporins, ticarcillin clavulanic acid, piperacillin-tazobactam, or ampicillin-sulbactam.

You are working in the ED on a Sunday afternoon when four people present with acute onset vomiting and crampy abdominal pain. They were all at the same picnic and ate most of the same foods. The vomiting began approximately 4 hours into the picnic. They deny having any diarrhea. You believe they may have "food poisoning" so you place IV lines, administer IV fluids, and observe. Over the next few hours, the patients begin to improve, the vomiting stops and their abdominal pain resolves. Which of the following is the most likely cause of their presentation? a. Scombroid fish poisoning b. Staphylococcal food poisoning c. Clostridium perfringens food poisoning d. Campylobacter e. Salmonellosis

*B* Staphylococcal food poisoning is caused by an enterotoxin-forming strain of Staphylococcus organisms in the food before ingestion. Most protein-rich foods support the growth of staphylococci, particularly ham, eggs, custard, mayonnaise, and potato salad. The illness has an abrupt onset, beginning 1 to 6 hours after ingestion of the contaminated food. Cramping and abdominal pain, with violent and frequent retching and vomiting are the predominant symptoms. Diarrhea is variable; it is usually mild, and occasionally absent. Although often aggressive in onset, staphylococcal food poisoning is short lived and usually subsides in 6 to 8 hours, rarely lasting more than 24 hours. The patient is often recovering when first seen by a physician. The short incubation period and multiple cases in people eating the same meal are highly suggestive of this disease.

A 50-year-old man presents to the ED for suture removal. When he was on vacation in Jamaica, 2 days ago, he sustained a 4-cm right calf laceration from a piece of glass. At that time, the laceration was irrigated, radiographed to rule out a foreign body, and repaired. Currently, there is good wound edge approximation and no surrounding erythema or discharge from the wound. What is the next best step in management? a. Have the patient return in 3 days for suture removal. b. Have the patient return in 8 days for suture removal. c. Have the patient return in 14 days for suture removal. d. Remove the sutures. e. Remove the sutures then replace them to prevent infection.

*B* Suture removal is a balance between cosmetic outcome and adequate wound closure. Increasing suture time improves wound strength at the expense of cosmetic outcome. Most sutures can be removed 7 days after placement. Sutures on the face should be removed in 3 to 5 days to prevent unsightly hatch markings. Sutures in areas of high tension, such as arms, legs, hands, and feet require approximately 10 days. This patient's laceration is large and in a high-tension area; therefore, removing the sutures 10 days after placement is the best management.

A 40-year-old man presents to the ED with a left foot puncture wound from stepping on a rusty nail. His last tetanus vaccine was during childhood, which resulted in a severe allergic reaction and a prolonged hospital stay. Which of the following is true regarding tetanus immunization in this patient? a. Give tetanus toxoid. b. Give tetanus immunoglobulin. c. Give tetanus immunoglobulin and toxoid. d. Give neither tetanus toxoid nor immunoglobulin since he has had an allergic reaction in the past. e. Give tetanus toxoid and immunoglobulin with diphenhydramine.

*B* Tetanus toxoid is contraindicated in patients who have had a severe allergic reaction including respiratory distress or cardiovascular collapse. Tetanus immunoglobulin can be used in these patients. Local reaction at injection site resulting in redness, pain and swelling is not considered a contraindication.

A 74-year-old woman is brought to the ED by EMS for altered mental status. Her BP is 138/72 mm Hg, HR is 91 beats per minute, RR is 17 breaths per minute, and temperature is 100.9°F. A head CT is normal. LP results revealed the following: WBC 1020/mL with 90% polymorphonuclear cells, Glucose 21 mg/dL, Protein 225 g/L. Which of the following is the most likely diagnosis? a. TB meningitis b. Bacterial meningitis c. Viral meningitis d. Fungal meningitis e. Encephalitis

*B* The CSF analysis in bacterial meningitis typically shows an elevated WBC count with predominant polymorphonuclear leukocytes. Protein is elevated and glucose is low. A Gram stain may show bacteria. The most specific marker for the diagnosis is a positive culture. Tests that evaluate the presence of antigen in the CSF (eg, latex agglutination studies) are particularly useful in the diagnosis of partially treated bacterial meningitis.

A 43-year-old homeless man presents to the ED with fever and nuchal rigidity. His mental status is depressed but his neurologic examination is otherwise nonfocal. Noncontrast head CT is normal. You obtain an LP for diagnostic purposes and initiate empiric antibiotic treatment for bacterial meningitis. The result of the CSF analysis is complete after 1 hour. The protein and glucose are within normal range but the WBC count consists of 220 mononuclear cells. The Gram stain is negative. The patient was recently purified protein derivative (PPD) negative and had a normal chest x-ray. In addition to the treatment already initiated, what is the next most appropriate step in this patient's management? a. Empiric treatment with isoniazid b. Empiric treatment with antiviral medications for herpes virus c. Empiric treatment with antifungals d. Antibiotic coverage for Bartonella sp. e. Addition of steroids to the antibiotic regimen

*B* The CSF analysis in this patient is consistent with a viral or atypical cause of meningitis. Although patients may have a mononuclear predominance and still have bacterial meningitis, viral causes should be considered. CSF should be sent for PCR analysis and empiric treatment initiated for herpes encephalitis. The mortality of meningoencephalitis caused by herpes simplex virus (HSV) is exceptionally high if untreated.

A 75-year-old man presents to the ED with a depressed level of consciousness. His wife is at the bedside and states he was stacking heavy boxes when he complained of a sudden intense headache. He subsequently sat down on the couch and progressively lost consciousness. She states that he had a headache the previous week that was also sudden but not as intense. He had gone to visit his primary-care physician who sent him to have a CT scan of the brain, which was normal. Over the course of the past week, he complained of intermittent pulsating headaches for which he took sumatriptan. In the ED, you intubate the patient and obtain the noncontrast head CT shwoing blood in the subarachnoid space. The scan is most consistent with which diagnosis? a. Meningoencephalitis b. SAH c. Normal pressure hydrocephalus d. Epidural hematoma e. Subdural hematoma

*B* The CT depicts subarachnoid arachnoid blood. This patient may have had a sentinel bleed, a small SAH, the previous week. Noncontrast CT misses a small percentage of SAH and therefore, in cases of high suspicion, an LP must be obtained to exclude the diagnosis.

A 63-year-old man who lives in a homeless shelter presents to the ED complaining of headache with photophobia for 6 hours. Upon arrival to the ED, the triage nurse places him in an isolation room. The triage note states that the patient was alert and conversant during the nursing interview. You enter the isolation room and attempt to speak to the patient but he is lethargic and combative. You note that his temperature is 103°F. He is unwilling to move his neck and winces when you attempt to check his pupillary reflexes with a penlight. The nurse informs you that laboratory analyses are delayed this evening because of staffing issues. Which of the following is the most appropriate next step in management? a. Diagnostic LP b. Initiation of IV antibiotic therapy c. A loading dose of IV corticosteroids d. Aggressive antipyretic therapy e. Sedation of the patient and noncontrast head CT

*B* The clinical presentation is consistent with meningitis. The patient's mental status has declined between initial nursing assessment and the physician's interview. Delay of antibiotic therapy in order to first confirm the diagnosis with CSF analysis may lead to increased mortality.

A 73-year-old man is seen in the ED for abdominal pain, nausea, and vomiting. His symptoms have progressively worsened over the past 2 to 3 days. The pain is diffuse and comes in waves. He denies fever or chills, but has a history of constipation. He reports no flatus for 24 hours. Physical examination is notable for diffuse tenderness and voluntary guarding. There is no rebound tenderness. An abdominal radiograph shows distended loops of small bowel and the upright film shows multiple air-fluid levels in a stepladder appearance. Which of the following is the most likely diagnosis? a. Constipation b. SBO c. Cholelithiasis d. Large bowel obstruction e. Inflammatory bowel disease

*B* The clinical scenario and radiograph are consistent with SBO. Patients usually present with diffuse, crampy abdominal pain that is often episodic in nature. Typically, the patient reports no recent bowel movements or flatus passage. The most common causes of SBO are adhesions and hernias. All patients with suspected SBO should have flat and upright abdominal radiographs performed. Flat plate abdominal films can show distended loops of small bowel and the upright film can show multiple air-fluid levels in a stepladder appearance.

A 23-year-old woman is brought to the ED for vision loss in her left eye that began shortly after waking up in the morning. She states that she is very depressed since her father was diagnosed with terminal cancer. She was supposed to visit her father today in the hospital but is now in your ED because of her vision loss. Your physical examination is unremarkable. An evaluation by the ophthalmologist is also normal. A head computed tomography (CT) scan is normal. Which of the following is the most likely diagnosis? a. Somatization disorder b. Conversion disorder c. Hypochondriasis d. Retinal detachment e. Anxiety disorder

*B* The diagnosis of conversion disorder is made by fulfilling the following five criteria: • A symptom is expressed as a change or loss of physical function. • Recent psychologic stressor or conflict. • The patient unconsciously produces the symptom. • The symptom cannot be explained by any known organic etiology. • The symptom is not limited to pain or sexual dysfunction. Conversions disorders generally involve neurologic or orthopedic manifestations. The disorder usually presents as a single symptom with a sudden onset related to a severe stress. In this case, the stress is the diagnosis of terminal cancer in the patient's father. Classic symptoms of conversion disorder include paralysis, aphonia, seizures, coordination disturbances, blindness, tunnel vision, and numbness. The diagnosis cannot be made until all possible organic etiologies are ruled out. Treatment involves identifying the stressor and addressing the issue.

A 65-year-old man with a history of occasional painless rectal bleeding presents with 2 to 3 days of constant, dull RLQ pain. He also complains of fever, nausea, and decreased appetite. He had a colonoscopy 2 years ago that was significant for sigmoid and cecal diverticula, but was otherwise normal. On physical examination he has RLQ tenderness with rebound and guarding. His vitals include HR of 95 beats per minute, BP of 130/85 mm Hg, and temperature of 101.3°F. The abdominal CT demonstrates the presence of sigmoid and cecal diverticula, inflammation of pericolic fat, thickening of the bowel wall, and a fluid-filled appendix. Which of the following is the most appropriate next step in management? a. Discharge the patient with broad-spectrum oral antibiotics and surgical follow-up. b. Begin IV hydration and broad-spectrum antibiotics, keep the patient npo (nothing by mouth), and admit the patient to the hospital. c. Begin IV antibiotics and call a surgical consult for an emergent operative procedure. d. Arrange for an emergent barium enema to confirm the diagnosis. e. Begin sulfasalazine 3 to 4 g/d along with IV steroid therapy.

*B* The diagnosis of diverticulitis is made by abdominal CT, which may demonstrate inflammation of pericolic fat, bowel wall thickening, the presence of diverticula, or peridiverticular abscess. The treatment of diverticulitis includes IV hydration, bowel rest, and broad-spectrum antibiotics to cover both aerobic and anaerobic bacteria. These typically include a combination of metronidazole and ciprofloxacin or levofloxacin. Well-appearing patients can be treated as outpatients with oral antibiotics and close follow-up, but patients with fever, signs of localized peritonitis or obstruction, and those who have failed outpatient therapy must be admitted to the hospital.

A 65-year-old man with a history of diabetes, hypertension, coronary artery disease, and atrial fibrillation presents with loss of vision in his left eye since he awoke 6 hours ago. The patient denies fever, eye pain, or eye discharge. On physical examination of the left eye, vision is limited to counting fingers. His pupil is 3 mm and reactive. Extraocular movements are intact. Slit-lamp examination is also normal. The dilated funduscopic examination is shows macular cherry-red spot with a pale retina and less pronounced arteries. Which of the following is the most likely diagnosis? a. Retinal detachment b. Central retinal artery occlusion c. Central retinal vein occlusion d. Vitreous hemorrhage e. Acute angle closure glaucoma

*B* The differential diagnosis for acute painless loss of vision includes retinal detachment, central retinal artery and vein occlusions, vitreous hemorrhage, and transient ischemic attack. An ophthalmologist should be called immediately when entertaining these diagnoses because a thorough funduscopic examination and prompt treatment is essential. On funduscopic examination, the patient has a macular cherry-red spot with a pale retina and less pronounced arteries. This is diagnostic of central retinal artery occlusion. Occlusion of the central retinal blood supply is commonly caused by emboli, thrombi, vasculitis, or trauma. Treatment aims to dislodge the clot from the main artery to one of its branches and includes digital massage, vasodilation, and lowering IOP.

A 23-year-old woman presents to the ED with RLQ pain for the last 1 to 2 days. The pain is associated with nausea, vomiting, diarrhea, anorexia, and a fever of 100.9°F. She also reports dysuria. The patient returned 1 month ago from a trip to Mexico. She is sexually active with one partner but does not use contraception. She denies vaginal bleeding or discharge. Her last menstrual period was approximately 1 month ago. She has a history of pyelonephritis. Based on the principles of emergency medicine, what are the three priority considerations in the diagnosis of this patient? a. Perihepatitis, gastroenteritis, cystitis b. Ectopic pregnancy, appendicitis, pyelonephritis c. Pelvic inflammatory disease (PID), gastroenteritis, cystitis d. Ectopic pregnancy, PID, menstrual cramps e. Gastroenteritis, amebic dysentery, menstrual cramps

*B* The emergency physician (EP) approaches a problem by considering the most serious disease consistent with the patient complaint and working to exclude it. Thinking of the worst first is a reversal from the sequence of patient management in many other specialties. Once the EP rules out the life-threatening processes, more benign processes may be considered. This principle is particularly important when placed in the context of the patient population seen in the ED. Most of the patients are new to the EP; many are intoxicated or are brought to the ED by others. This leads to an array of fragmented histories, masked physical findings, and high emotional levels. In this setting, it is even more important for the EP to maintain a high level of suspicion for serious diseases.

A 41-year-old man who was a restrained front seat passenger in a highspeed motor vehicle collision is brought to the ED by the paramedics. His vital signs are BP 90/50 mm Hg, HR 125 beats per minute, and RR 20 breaths per minute. On examination, he is alert and answers your questions. His breath sounds are equal bilaterally and chest wall is without contusion or bony crepitus. The abdomen is soft and nondistended. His pelvis is unstable. A FAST examination and DPL are negative for intraperitoneal fluid. Initial radiographs reveal a normal chest film and an open book pelvic fracture. Despite infusion of 2 L of lactated ringer solution, his BP is now 80/40 mm Hg. Which of the following is the most appropriate next step in management? a. Immediate exploratory laparotomy b. Pelvic angiography with embolization of the pelvic vessels c. CT scan d. Pericardiocentesis e. Retrograde urethrogram

*B* The evaluation of patients with pelvic fractures begins with the primary trauma survey (ABCs). Fractured pelvic bones bleed briskly and can lacerate surrounding soft tissues and disrupt their extensive arterial and venous networks. Hemorrhage is a common cause of death in patients with pelvic injuries. Bleeding in the retroperitoneum can accommodate up to 4 L of blood. Most pelvic bleeding is from the fractures and low-pressure sacral venous plexus. In the ED, a stabilizer can be applied to the pelvis to help control the hemorrhage. Once an abdominal source of bleeding is ruled out as a source of hypotension, the patient should undergo pelvic angiography with embolization of bleeding vessels. The patient should also be resuscitated with packed RBCs until the bleeding is controlled.

A 49-year-old man is brought to the ED by EMS stating that he vomited approximately three cups of blood over the last 2 hours. He also complains of epigastric pain. While examining the patient, he has another episode of hematemesis. You decide to place a NG tube. You insert the tube, confirm its placement, and attach it to suction. You retrieve 200 mL of coffee-ground blood. What is the most common etiology of an upper GI bleed? a. Varices b. Peptic ulcer c. Gastric erosions d. Mallory-Weiss tear e. Esophagitis

*B* The most common cause of upper GI bleeds in adults is peptic ulcer disease accounting for approximately 45% of the cases. Hematemesis is the presentation in approximately 50% of patients with upper GI bleeds. The appearance of coffee-grounds in the stomach is caused by the conversion of hemoglobin to hematin or other pigments by hydrochloric acid in the stomach.

A 22-year-old college football player presents to the ED with knee pain. He states that during the football game today, he received a direct blow to the lateral aspect of his knee. Radiograph of the knee is negative for fracture, but shows an effusion. On examination, you confirm the hemarthrosis. As you examine his knee ligaments, you note joint instability with valgus stress in 30 degrees of flexion. The rest of your examination is unremarkable. This patient's injury is most consistent with? a. Rupture of the LCL b. Rupture of the MCL c. Rupture of the ACL d. Rupture of the PCL e. Tear of the medial meniscus

*B* The most common mechanism of injury resulting in ligamentous injury is a valgus stress with an external rotary component on the flexed knee. This is common in skiing and football injuries. The MCL is commonly injured. On stability testing of the knee, patients with MCL tears usually show instability with a valgus stress with the knee in 30 degrees of flexion. Ligamentous tears are also associated with hemarthrosis.

An 82-year-old nursing home patient presents to the ED in septic shock. Her BP is 75/40 mm Hg, HR is 117 beats per minute, temperature is 96.5°F, RR is 29 breaths per minute, and oxygen saturation is 87% on room air. As you perform laryngoscopy to intubate the patient, you easily visualize the vocal cords and subsequently pass the orotracheal tube through the vocal cords. You place the colorimetric end-tidal carbon dioxide device over the tube and get appropriate color change. There are equal, bilateral breath sounds on auscultation and you observe chest wall motion with ventilation. Which of the following is the most reliable method for verifying proper ET tube placement? a. Chest radiograph b. Visualization of the ET tube passing through the vocal cords c. Observation of chest wall motion with ventilation d. Hearing equal, bilateral breath sounds on auscultation e. End-tidal carbon dioxide color change

*B* The most serious complication of ET intubation is unrecognized esophageal intubation with resultant hypoxic brain injury. Esophageal placement is not always obvious. The best assurance that the tube is placed into the trachea is to see it pass through the vocal cords.

A 40-year-old woman is brought to the ED by the paramedics complaining of bilateral foot weakness and numbness that started a few hours ago and is progressively worsening. She denies similar episodes in the past. On the review of systems, she describes having abdominal cramps with nausea, vomiting, and diarrhea 2 weeks ago that resolved after 2 to 3 days. Her BP is 124/67 mm Hg, HR is 68 beats per minute, temperature is 98.8°F, and RR is 12 breaths per minute. On examination, you elicit 2/5 strength, decreased sensation, and loss of deep tendon reflexes in the lower extremities below the hips. Which of the following is the most likely diagnosis? a. Hypokalemic periodic paralysis b. Guillain-Barré syndrome c. Peripheral vascular disease d. Tetanus e. Brain abscess

*B* The patient has a progressive ascending peripheral neuropathy, also known as Guillain-Barré syndrome. Patients can usually remember a preceding viral illness, usually gastroenteritis. Deep tendon reflexes are typically absent.

An unconscious 51-year-old woman is brought to the ED by EMS. A friend states that the patient was complaining of feeling weak. She vomited and subsequently "blacked out" in the ambulance. The friend states that the patient has no medical problems and takes no medications. She also states that the patient smokes cigarettes and uses cocaine, and that they were snorting cocaine together prior to her blacking out. The patient's BP is 195/80 mm Hg, HR is 50 beats per minute, temperature is 98.6°F, and RR is 7 breaths per minute. What is the eponym associated with her vital signs? a. Cushing syndrome b. Cushing reflex c. Cullen sign d. Charcot triad e. Chvostek sign

*B* The patient has a triad of hypertension, bradycardia, and respiratory depression, which is called Cushing reflex. This is observed in one-third of patients with a potentially lethal increase in ICP. Increased ICP may result from traumatic brain injury or, as in this patient's case, from hemorrhagic stroke and subsequent brain edema. Tobacco and cocaine use are known risk factors for hemorrhagic stroke. Increasing ICP can result in cerebral herniation, which has a mortality rate close to 100%. For any chance of survival, it must be rapidly controlled by intubation, elevation of the head of the bed, hyperventilation, mannitol, and definitive neurosurgical intervention.

An 18-year-old girl is brought to the ED from a party for agitation and attacking her boyfriend with a knife. Her boyfriend admits that she had several liquor shots and used intranasal cocaine at the party prior to becoming agitated, paranoid, and attacking him. Her BP is 145/80 mm Hg, HR is 126 beats per minute, temperature is 100.8°F, and RR is 20 breaths per minute. The patient is agitated, screaming, and resisting examination. What is the next best step in the management of this patient? a. IV β-blocker b. IV benzodiazepine c. Acetaminophen d. Lithium e. Drug abuse specialist consult

*B* The patient has cocaine induced autonomic and CNS hyperactivity causing agitation, paranoia, and a hyperadrenergic state. More severe CNS manifestations of cocaine poisoning include hyperthermia, intracranial hemorrhage, seizures, spinal cord infarctions, and acute dystonic reactions. The CNS effects of cocaine are managed with benzodiazepines, which decrease sympathetic tone and prevent hyperthermia and seizures.

A 59-year-old man is brought into the ED accompanied by his son who states that his father is acting irritable and occasionally confused. The son states that his father has a history of hepatitis from a transfusion he received many years ago. Over the past 5 years, his liver function slowly deteriorated. His vital signs include BP of 145/80 mm Hg, a HR of 78 beats per minute, RR of 16 breaths per minute, oxygen saturation of 98%, and temperature of 98°F. Laboratory results are all within normal limits, except for an ammonia level that is significantly elevated. Which of the following is the best therapy? a. Vancomycin and gentamycin b. Lactulose and neomycin c. Ampicillin and gentamycin d. Levofloxacin e. Ciprofloxacin

*B* The patient has hepatic encephalopathy, which is a clinical state of disordered cerebral function occurring secondary to acute or chronic liver disease. Laboratory tests may be normal in patients, but the serum ammonia level is usually elevated. Lactulose and neomycin represent the main therapeutic agents. Lactulose is a poorly absorbed sugar metabolized by colonic bacteria that traps ammonia and helps excrete it in the stool. Neomycin is a poorly absorbed aminoglycoside that is believed to act by reducing colonic bacteria that are responsible for producing ammonia.

A 60-year-old woman presents to the ED complaining of pain in her right eye and burning sensation over half of her forehead and scalp. On physical examination, you notice a patch of grouped vesicles on an erythematous base located in a dermatomal distribution on her scalp and forehead. There are also a few vesicles located at the tip of the patient's nose. Her visual acuity is 20/20 bilaterally, heart is without murmurs, lungs are clear, abdomen is soft, and there are no gross findings on neurologic examination. Which of the following is the most concerning complication of this patient's clinical presentation? a. Central nervous system (CNS) involvement leading to meningitis b. Ophthalmic involvement leading to anterior uveitis or corneal scarring c. Cardiac involvement leading to endocarditis d. Permanent scarring of her face e. Nasopalatine involvement leading to epistaxis

*B* The patient has herpes zoster, or shingles, an infection caused by the varicella-zoster virus. The patient's rash most likely involves the ophthalmic division of the trigeminal nerve. In addition, the vesicles found on the tip of the patient's nose correlate strongly with viral involvement of the eye. Hutchinson sign is used to denote vesicles on the tip of a patient's nose in the setting of herpes zoster. When there is ocular involvement the infection is called herpes zoster ophthalmicus. Ocular complications occur in 20% to 70% of the cases involving the ophthalmic division of the trigeminal nerve. Severity varies from mild conjunctivitis to panophthalmitis. In addition, the patient is at risk for anterior uveitis, secondary glaucoma, and corneal scarring.

A 57-year-old man with a past medical history of hypertension and migraines presents to the ED complaining of a headache that started 2 days ago. He states the headache began suddenly with peak intensity while he was defecating. The pain is continuous particularly in the occipital region and is associated with mild nuchal rigidity and mild photophobia. He denies having a recent fever. A noncontrast head CT is obtained and is normal. Which of the following is the most appropriate next step in management? a. Administer metoclopramide for nausea relief and ketorolac for pain control b. LP c. Empiric treatment for meningitis with IV antibiotics d. IV mannitol to lower intracranial pressure (ICP) e. Angiography to evaluate for an aneurysm

*B* The patient presents with a clinical history that is consistent with a SAH. Brain CT without contrast is the procedure of choice for diagnosing SAH and should be done in any individual with a new onset of a severe or persistent headache. It has a sensitivity of 95% for detecting SAH. If the CT is negative, an LP should be performed because some patients with SAH have a normal CT scan. A yellow supernatant liquid (xanthochromia), obtained by centrifuging a bloody CSF sample, can help distinguish SAH from a traumatic tap. If the diagnosis is still in question, an angiography may be required

An anxious college student presents to the ED at 2 AM stating a bat woke him up. He heard something flying around his bedroom and when he turned the lights on, he saw a bat fly into his closet. He is unsure if he was bit by the bat but noticed bat droppings in his bed. Which is the most appropriate management for the patient? a. Reassure him that there is nothing to do and discharge him. b. Immunoprophylax with human rabies immune globulin and human diploid cell vaccine. c. Administer ciprofloxacin 500 mg. d. Admit him to the hospital for 24 hours of observation. e. Immunoprophylax with human diploid cell vaccine.

*B* The patient should receive full immunoprophylaxis for the rabies virus. The Centre for Disease Control (CDC) recommends postexposure prophylaxis when a bat is found indoors in the same room as a person who might be unaware that a bite or direct contact had occurred and rabies cannot be ruled out by testing the bat. Full prophylaxis in the United States includes passive immunization with human rabies immune globulin and active immunization with human diploid cell vaccine. Immune globulin is administered in and around a bite wound if visualized. Human diploid cell vaccine is administered at a distant site from the immunoglobulin, usually in the deltoid, to avoid cross reactivity. Human diploid cell vaccine is subsequently administered on days 3, 7, 14, and 28.

A 51-year-old woman presents to the ED after 5 consecutive days of crushing substernal chest pressure that woke her up from sleep in the morning. The pain resolves spontaneously after 20 to 30 minutes. She is an avid rock climber and jogs 5 miles daily. She has never smoked cigarettes and has no family history of coronary disease. In the ED, she experiences another episode of chest pain. An ECG reveals ST-segment elevations and cardiac biomarkers are negative. The pain is relieved with sublingual nitroglycerin. She is admitted to the hospital and diagnostic testing reveals minimal coronary atherosclerotic disease. Which of the following is the most appropriate medication to treat this patient's condition? a. Aspirin b. Calcium channel blocker (CCB) c. β-Blocker d. H2-Blocker e. Antidepressant

*B* The patient's clinical presentation is consistent with Prinzmetal or variant angina. This condition is caused by focal coronary artery vasospasm. It occurs at rest and exhibits a circadian pattern, with most episodes occurring in the early hours of the morning. The pain commonly is severe. Distinguishing unstable angina related to coronary atherosclerosis from variant angina may be difficult and requires special investigations, including coronary angiography. Patients may also exhibit ST elevations on their ECGs. Nitrates and CCBs are the mainstays of medical therapy for variant angina. Nitroglycerin effectively treats episodes of angina and myocardial ischemia within minutes of administration, and the long-acting nitrate preparations reduce the frequency of recurrent events. CCBs effectively prevent coronary vasospasm and variant angina, and they should be administered in lieu of β-blockers.

78-year-old man presents to triage of your ED complaining of gradual onset of headache over the course of the day. The headache is present almost everyday and wonders if it is related to the unusually cold temperatures this winter. He describes the headache as bounding and constant. You notice that his face is very ruddy in appearance. He is afebrile but looks rather lethargic and seems somewhat short of breath. He is afebrile and saturating 100% on pulse oximetry. One of your coworkers informs you that the patient's wife is in another part of the emergency room with a similar presentation. Two more ambulances arrive, one with a patient complaining of a similar headache and another with a patient who is obtunded. All live in the same building. What is the next most appropriate step in the management of these patients? a. Place the patient in respiratory isolation for presumed Neisseria meningitidis infection. b. Draw a blood gas and send it for cooximetry. c. Start antibiotic therapy and perform an LP. d. Treatment for migraine with triptans or IV antiemetics. e. Transfer of all three patients to the nearest hyperbaric facility.

*B* The patients are experiencing symptoms of carbon monoxide (CO) poisoning. CO is colorless and odorless. Patients often present with mild nonspecific symptoms including headache, malaise, and fatigue. Severe toxicity manifests as neurologic and cardiac toxicity. Severe cases may manifest as disseminated intravascular coagulation, circulatory shock, multiorgan failure, ischemic cardiac disease, renal failure, or noncardiogenic pulmonary edema. Although there is decreased blood oxygen content, patients will not exhibit cyanosis as there is not enough deoxyhemoglobin present to cause it. Common sources of CO include fossil fuel—burning engines, fumes from coal- or gas-burning stoves, and smoke from accidental fires. CO intoxication is more prevalent during the winter when potentially faulty heating systems are in use or when patients attempt to supplement their home heat using their oven. Initial therapy is aimed at increasing arterial oxygen content by providing supplemental oxygen. Mild intoxication can be managed with supplementary oxygen alone. Elevated carboxyhemoglobin levels require treatment with hyperbaric oxygen.

A 22-year-old college volleyball player presents to the ED complaining of left shoulder pain that began while attempting a serve during a volleyball match. She states that this has happened to her before. On examination, the left shoulder looks "squared-off." She complains of severe pain when she tries to adduct or internally rotate the shoulder. A radiograph is shows anterior dislocation of the left shoulder. What is the most common fracture associated with the patient's diagnosis? a. Bankart fracture b. Hill-Sachs deformity c. Clavicular fracture d. Coronoid fracture e. Scapular fracture

*B* The radiograph confirms an anterior dislocation of the left shoulder. Patient's typically present in severe pain with the dislocated arm held in slight abduction and external rotation by the opposite extremity. The patient leans away from the injured side and cannot adduct or internally rotate the shoulder without severe pain. Associated fractures may occur in up to 50% of anterior dislocations. The most common of these is a compression fracture of the humeral head, known as a Hill-Sachs deformity.

A 24-year-old man presents to the ED complaining of right wrist pain that began after he slipped and fell and landed on his outstretched hand. You examine the hand and wrist, and note no abnormalities except for snuffbox tenderness. A radiograph does not reveal a fracture. Which of the following is the most appropriate next step in management? a. Place an ACE wrap around the hand and wrist until the pain resolves. b. Immobilize the wrist in a thumb spica splint and have the patient follow-up with an orthopedist for repeat radiographs in 10 to 14 days. c. Continue to ice the wrist for 24 to 48 hours. d. Order a CT scan to evaluate for an occult fracture. e. Place the wrist and arm in a cast for 6 weeks.

*B* The scaphoid is the most commonly fractured carpal bone. It is typically seen in patients in their twenties to thirties after a FOOSH. Classically, physical examination reveals tenderness in the anatomic snuffbox, the space between the extensor pollicis longus and the extensor pollicis brevis. On radiography, however, up to 15% of scaphoid fractures are not detected. As the necrotic bone at the fracture site is resorbed, the fracture line often becomes apparent on radiographs at 10 to 14 days after injury. Therefore, patients with snuffbox tenderness and an initial radiograph should be splinted in a thumb spica splint and asked to return for repeat radiographs in 10 to 14 days.

A 71-year-old man is found lying on the ground one story below the balcony of his apartment. Paramedics bring the patient into the ED. He is cool to touch with a core body temperature of 96°F. His HR is 119 beats per minute and BP is 90/70 mm Hg. His eyes are closed but they open when you call his name. His limbs move to stimuli and he answers your questions but is confused. On examination, you note clear fluid dripping from his left ear canal and an area of ecchymosis around the mastoid bone. Which of the following is the most likely diagnosis? a. LeFort fracture b. Basilar skull fracture c. Otitis interna d. Otitis externa e. Tripod fracture

*B* The skull base comprises the floors of the anterior, middle, and posterior cranial fossae. Fractures in this region typically do not have localized symptoms. However, indirect signs of injury may include visible evidence of bleeding from the fracture into surrounding soft tissue. Ecchymosis around the mastoid bone is often described as Battle sign and periorbital ecchymosis is often described as "raccoon eyes." The most common basilar skull fracture involves the petrous portion of the temporal bone, the external auditory canal, and the tympanic membrane. It is commonly associated with a torn dura leading to cerebrospinal fluid (CSF) otorrhea or rhinorrhea. Other signs and symptoms of a basilar skull fracture include hemotympanum (eg, blood in the tympanic cavity of the middle ear), vertigo, decreased hearing or deafness, and seventh nerve palsy. Periorbital and mastoid ecchymosis develop gradually over hours after an injury and are often absent in the ED. If clear or pink fluid is seen from the nose or ear and a CSF leak is suspected, the fluid can be placed on filter paper and a "halo" or double ring may appear. This is a simple but nonsensitive test to confirm a CSF leak. Evidence of open communication, such as a CSF leak, mandates neurosurgical consultation and admission.

A 47-year-old man is brought to the ED by EMS after being found wandering in the street mumbling. His BP is 150/75 mm Hg, HR is 110 beats per minute, temperature is 100.5°F, RR is 16 breaths per minute, oxygen saturation is 99% on room air, and fingerstick glucose is 98 mg/dL. On examination, the patient is confused with mumbling speech. His pupils are dilated and face is flushed. His mucous membranes and skin are dry. Which of the following toxic syndromes is this patient exhibiting? a. Sympathomimetic syndrome b. Anticholinergic syndrome c. Cholinergic syndrome d. Opioid syndrome e. Ethanol syndrome

*B* The term toxidrome refers to a constellation of physical findings that can provide important clues in a toxic ingestion. This is particularly useful in patients that cannot provide an adequate history. The anticholinergic syndrome typically presents with delirium, mumbling speech, tachycardia, elevated temperature, flushed face, dry mucous membranes and skin, dilated pupils, and hypoactive bowel sounds. The anticholinergic syndrome can be remembered by the phrase "blind as a bat (mydriasis), red as a beet (flushed skin), hot as a hare (hyperthermia secondary to lack of sweating), dry as a bone (dry mucous membranes), and mad as a hatter (mental status changes)."

A 22-year-old man is brought to the ED 20 minutes after a head-on motor vehicle collision in which he was the unrestrained driver. On arrival, he is alert and coherent but appears short of breath. His HR is 117 beats per minute, BP is 80/60 mm Hg, and oxygen saturation is 97% on a nonrebreather. Examination reveals bruising over the central portion of his chest. His neck veins are not distended. Breath sounds are present on the left, but absent on the right. Following administration of 2 L of lactated ringer solution, his systolic BP remains at 80 mm Hg. Which of the following is the most appropriate next step in management? a. Sedate, paralyze, and intubate b. Perform a needle thoracostomy c. Perform a DPL d. Perform a FAST examination e. Perform a pericardiocentesis

*B* The treatment of a tension PTX involves immediate reduction in the intrapleural pressure on the affected side of the thoracic cavity. The simplest and quickest way to establish this is by inserting a 14-gauge catheter into the thoracic cavity in the second intercostal space in the midclavicular line. After this procedure, a chest tube should be inserted as definitive management. Needle thoracostomy is necessary when a patient's vital signs are unstable; otherwise, direct insertion of a chest tube is adequate for suspicion of a hemo- or pneumothorax. A tension PTX is a life-threatening emergency caused by air entering the pleural space that is not able to escape secondary to the creation of a one way valve. This increased pressure causes the ipsilateral lung to collapse, shifting the mediastinum away from the injured lung, compromising vena caval blood return to the heart. The severely altered preload results in reduced stroke volume, increased cardiac output, and hypotension.

A 4-year-old girl is brought to the ED after a falling from a tree. She hit her head on the ground and has significant temporal swelling on the left side. In transit to the hospital by her parents, the patient had multiple episodes of emesis. On arrival to the ED, the patient is confused and agitated and then becomes acutely unresponsive and apneic. You make the decision to endotracheally intubate the patient. Which of the following is the most appropriate endotracheal tube (ETT) to use in this intubation? a. 4.0 uncuffed ETT b. 4.0 cuffed ETT c. 5.0 cuffed ETT d. 5.5 uncuffed ETT e. 4.5 uncuffed ETT

*B* There were recent changes in the Pediatric Advanced Life Support (PALS) recommendations regarding ETTs. Previously, uncuffed ETTs were recommended for all patients under the age of 8 years. This was based on the anatomical consideration that the narrowest part of the airway in children is the subglottic area whereas in adults it is the glottis (larynx) itself. Though this is true, it has become clear through further research that cuffed tubes are preferable for providing a better seal without an increased risk for endothelial damage from overinflation of the balloon and will provide better ventilation in patients with poor lung compliance. With this complication in mind, it is important not to blow the balloon up to a pressure greater than 20 cm H2O. There are two acceptable methods for choosing the appropriate ETT size. One relies on the length-based resuscitation tape that should be available and utilized in the resuscitation of all small children. The formula that has been endorsed by the American Heart Association for calculation of the appropriate tube size for a cuffed tube is ETT size (mm of internal diameter of tube) = (age in years/4) + 3. This calculation in the above patient would yield a size of 4.0 mm for a cuffed tube.

A 57-year-old man presents to the ED with generalized weakness and pain, abdominal discomfort, and nausea for 2 days. On the review of systems he also admits to recent polydipsia, polyuria, and a 10-lb weight loss. His medical history includes hypertension for which he takes no medications. He has a 20-pack-year smoking history. The vital signs are remarkable for mild tachycardia and hypertension. Laboratory results reveal a serum sodium 131 mEq/L, potassium 3.5 mEq/L, chloride 101 mEq/L, bicarbonate 22 mEq/L, blood urea nitrogen (BUN) 15 mg/dL, creatinine 1.1 mg/dL, glucose 125 mg/dL, and serum calcium level of 12.6 mEq/L. Which of the following is the most appropriate next step in management? a. Administer calcitonin b. Start 0.9% normal saline intravenous (IV) bolus c. Administer furosemide d. Obtain chest radiograph e. Obtain ECG

*B* This common presentationm of hypercalcemia is initially managed with aggressive isotonic saline IV hydration to restore volume status. Hypercalcemia impairs renal concentrating ability, and patients typically present with polyuria, polydipsia, and dehydration and may develop kidney stones. Increased calcium levels also cause generalized weakness, bone pain, neurologic symptoms (ataxia, altered mental status), GI dysfunction (abdominal pain, nausea, vomiting, anorexia), and ECG abnormalities (shortened QT interval). A handy mnemonic for symptoms of hypercalcemia is "bones, stones, groans, and psychiatric overtones."

A 29-year-old man is 10 ft up on a ladder, painting his barn ceiling when he loses his balance and falls off the ladder, landing on both of his feet and then falling to the ground. He is brought to the ED by emergency medical services (EMS). On examination there are no obvious deformities, but you note swelling, tenderness, and ecchymosis of the right hindfoot. He has full range of motion at the ankle, knee, and hip. There is mild tenderness over his lumbar spine. Hs neurovascular status is normal. Radiographs confirm the diagnosis. Which of the following statements regarding this type of injury is true? a. This injury is the second most commonly fractured tarsal bone. b. 10% are associated with compression fractures of the dorsolumbar spine. c. 90% are bilateral. d. The mechanism of injury is usually secondary to rotational force at the subtalar joint. e. These fractures require operative repair to avoid gait defects

*B* This individual sustained a calcaneal fracture of the right foot. The calcaneus is the most commonly fractured tarsal bone (a); the talus is the second most commonly injured tarsal bone. A calcaneal fracture is usually caused by a compression injury (d), such as a fall from a height with the patient landing on his or her feet. The examination reveals swelling, tenderness, and ecchymosis of the hindfoot and the inability to bear weight on the fracture. 10% are bilateral (c) and 10% are associated with compression fractures of the dorsolumbar spine. Therefore, it is important to examine the patient's entire spine. Treatment varies depending on the extent of injury. In general, nondisplaced or minor extra-articular fractures only require supportive care with immobilization in a posterior splint and follow-up with an orthopedic surgeon

A 7-year-old boy presents to the ED 1 hour after slipping and landing on his right outstretched hand. He was evaluated and splinted by EMTs and brought to the ED. On examination, you note a deformity of his right wrist. There are no neurovascular deficits. A radiograph of the wrist is shown below. Which of the following states is true regarding physeal fractures in children? a. Salter-Harris type IV is defined as a crush injury of the growth plate. b. The most common type of fracture is the Salter-Harris type II. c. This patient's radiographic findings are consistent with Salter-Harris type V. d. Fractures through the physis, metaphysis, and epiphysis are classified as Salter-Harris type III. e. The worse prognosis is seen with Salter-Harris type I fractures.

*B* This is a Salter-Harris type II fracture that involves the physis and metaphysis. The physis or growth plate is a common site of fractures in pediatric patients. Approximately 20% of fractures involve this area of active bone growth with a peak incidence in early adolescence. The most widely used system of classification for these fractures is the Salter-Harris classification. In this system, there are five classifications of injuries. In general, the higher the Salter-Harris classification, the worse the prognosis, and the higher the chance for growth arrest. Salter-Harris I occurs when the fracture involves the growth plate. There may be no radiographic evidence of these fractures initially therefore you must have a high suspicion. This injury should be suspected in any patient who is tender around the physis of a bone even in the absence of obvious radiographic fracture. Treatment for this injury includes early immobilization and referral. Prognosis is usually good. Salter-Harris II is the most common type, accounting for approximately 75% of fractures involving the growth plate. The fracture occurs through the growth plate and into the metaphysis. Salter-Harris III occurs when a fracture breaks through the growth plate and into the epiphysis. Salter-Harris IV involves the metaphysis, physis, and epiphysis. Salter-Harris V is a crush injury to the growth plate and has a high potential for growth arrest.

A 13-year-old African American boy is brought to the ED by his mother for a complaint of right knee pain over the course of 1 to 2 weeks. The only notable trauma that the patient can recall was jumping on a trampoline 1 to 2 weeks ago with friends that resulted in right lower extremity pain. On the morning of presentation, the patient complained of increased pain when ambulating and was noted to be limping. He denies fever. No other trauma and no recent illness were noted by the family. On examination, the patient is afebrile with normal vital signs. He has no previous medical problems and is noted to be overweight but is in otherwise good health. The lower extremity examination reveals no swelling or erythema over any of the joints. His knee has no focal tenderness or pain with range of motion but the hip is noted to be painful with internal and external rotation. He has a normal neurosensory examination of the distal extremity. A radiograph is performed. Which of the following is the most likely diagnosis? a. Legg-Calve-Perthes disease b. Slipped capital-femoral epiphysis c. Septic arthritis d. Osgood Schlatter disease e. Transient synovitis of the hip

*B* This is a case of a patient with a slipped capital femoral epiphysis (SCFE). SCFE is a condition of the femur where the femoral head slips or shears off from the neck of the femur through the physis. It is more common in boys than in girls, typically in early or mid-adolescence and is more common in African American children than in Caucasian children. Its cause is unknown. SCFE can present either with a complaint of tightness that progresses to more significant pain leading to a limp or more dramatically with extreme pain and inability to bear weight. It is not uncommon for a child to present with a complaint of knee pain that is referred to the hip. A careful examination will often better localize the origin of the pain to the hip. It is usually unilateral though it can be bilateral in upward of a quarter of the cases. The diagnosis is made on radiograph. It is classically described as the "ice cream falling off of the cone" appearance. Treatment is surgical.

A 45-year-old man is brought to the ED by his coworkers after collapsing to the floor while at work. A coworker states that the patient mistakenly took several tablets of his oral diabetic medications a few hours ago. The patient is unresponsive and diaphoretic. His BP is 142/78 mm Hg, HR is 115 beats per minute, temperature is 98.9°F, and RR is 12 breaths per minute. A bedside glucose reads 42 mg/dL. Which of the following is the most appropriate management of this patient? a. Administer IV dextrose, obtain a repeat fingerstick glucose every hour and if normal after 6 hours discharge the patient home. b. Administer IV dextrose and continue monitoring his blood sugar for at least 24 hours. c. Administer IV fluids and insulin. d. Administer IV fluids. e. Administer activated charcoal and IV fluids.

*B* This is a case of hypoglycemia as a result of oral hypoglycemic medication overdose. Diabetic oral agents that cause hypoglycemia work by increasing pancreatic insulin secretion. This group includes sulfonylureas (glyburide, glipizide) and nonsulfonylurea secretagogues (repaglinide, nateglinide). Other common causes of hypoglycemia are insulin overdose, alcohol abuse (inhibition of gluconeogenesis) and sepsis. The presentation of a hypoglycemic patient generally involves signs and symptoms of CNS dysfunction owing to the release of counterregulatory hormones secondary to the unavailability of glucose. Symptoms include anxiety, diaphoresis, palpitations, and confusion. Don't be fooled by improved blood glucose levels after dextrose administration in overdose with oral hypoglycemic agents. Hypoglycemia can last more than 24 hours because of long-lasting pancreatic effects and will recur after dextrose infusion. Patients need to be observed in the hospital with frequent bedside glucose checks. They can be placed on a dextrose drip. Octreotide, an inhibitor of insulin release can also be administered.

A 3-week-old girl is brought to the ED by her parents after they noticed blood in her stool. The patient is the full-term product of an uncomplicated pregnancy and delivery without any medical issues up to this point. She has been feeding well (breastfeeding primarily) and active without fever, respiratory problems, or fussiness. She has had several episodes of nonforceful, nonbilious emesis after feeds with multiple wet diapers each day. She normally has several soft, seedy stools. In the last day, the parents noticed streaks of blood in her stool and today she had grossly bloody stool. The patient does not seem to be in any distress or discomfort. On examination, the temperature is 98.9°F, HR 155 beats per minute, and RR 44 breaths per minute. The patient is awake, active and in no apparent distress. Her abdomen is soft and nontender with normal bowel sounds and no masses. Examination of her anus does not reveal a fissure. Which of the following is the most likely diagnosis? a. Acute gastroenteritis b. Milk protein colitis c. Clostridium difficile colitis d. Intestinal malrotation e. Necrotizing enterocolitis

*B* This is a case of milk protein colitis, a common cause of rectal bleeding in this age group. Some infants have sensitivity to cow's milk protein (which is fundamentally different from human milk proteins) that leads to an allergic antibody response causing a true colitis that results in bloody stools. Approximately a third of these patients will have a similar reaction to soy milk proteins. The resulting symptoms are dramatic but generally without other consequence except in cases where the exposure is longstanding and bleeding is ongoing. This can lead to malnourishment, anemia, and poor growth. Milk protein colitis is more frequently seen in bottle fed infants but can be seen in exclusively breast fed infants where the proteins are thought to transfer from the mother's milk into the infant, which results in the same symptoms as if the patient were drinking milk directly. Treatment is elimination of offending agent from the diet, typically with total resolution of symptoms. Most patients will grow out of their sensitivity by 2 years of age and milk products can be reintroduced at that time.

A 9-month-old boy is brought to the ED with a 2-day history of fever, vomiting, and fussiness. The patient has had multiple episodes of emesis that follow intense periods of fussiness after which the patient seems to relax and go to sleep. He has had no fever or diarrhea. In between these episodes, he has slightly decreased energy but otherwise seems well. Oral intake is decreased and urine output has been decreased since the day of presentation. Which of the following statements is true regarding this condition? a. The majority present with vomiting, colicky abdominal pain, and currant jelly stools. b. Air enema is the therapeutic intervention of choice. c. Plain films of the abdomen can usually confirm the diagnosis. d. Surgical intervention is often indicated. e. Most of these have a "lead point" as the underlying pathologic cause.

*B* This is a classic story and presentation for intussusception, where a part of intestine telescopes inside of another and causes ischemia, which can lead to infarction of bowel. This is a true GI emergency. The most common type of intussusception is ileo-colic and the most common reason is idiopathic. Ileo-ileal and colo-colonic intussusceptions are possible though rare in comparison. The most common age group is 3 to 12 months with greater than 80% occurring within the first 2 years of life. Air enema is both diagnostic and therapeutic and has been adopted as the intervention of choice for these patients.

A 23-year-old woman is brought to the ED by police officials who state that they found the patient in the middle of a busy intersection screaming. Upon arrival, you see a disheveled woman who is yelling, "You can't get away with this! I'm the Queen of England!" She does not allow for the triage nurse to obtain her vitals, but you can see a young woman who is of normal habitus without any signs of trauma. Her speech is pressured, she is easily distracted by the commotion of the ED and begins to answer your questions but then continues to describe grandiose ideas about her social status. Given this patient's acute presentation, what is the most likely etiology of her symptoms? a. Hypothyroid disorder b. Manic episode c. Benzodiazepine overdose d. Anticonvulsant overdose e. Barbiturate overdose

*B* This is a typical manic episode of which an elevated mood, grandiosity, flight of ideas, distractibility, and psychomotor agitation are its cardinal features. Other medical conditions, such as hyperthyroidism, antidepressant, or stimulant abuse may cause similar symptoms and must be ruled out with laboratory testing. Certain frontal lobe release syndromes that impair executive functioning must also be investigated as a cause of this patient's symptoms. These patients are usually combative, display impaired judgment and impulsivity, and may need to be chemically or physically restrained.

A 28-year-old woman presents to the ED with heavy menstrual flow for the last 2 days with clots. She reports using about two pads every hour. The patient states that she has occasional metromenorrhagia in the past and has been treated with oral contraceptives. She reports symptoms of feeling lightheaded but denies any syncope, palpitations, chest pain, abdominal pain, or weakness. Her initial vital signs include HR of 96 beats per minute, BP of 135/70 mm Hg, RR of 14 breaths per minute with oxygen saturation of 99% on room air. Upon physical examination, the patient is obese and you note a pronounced hairline. Which of the following conditions is most consistent with this patient's presentation? a. Intrauterine pregnancy b. Polycystic ovaries c. Ectopic pregnancy d. Follicular cyst rupture e. Corpus luteum cyst rupture

*B* This patient exhibits signs of hyperandrogenism and anovulation as evidenced by her history of irregular, heavy periods and being treated with oral contraceptives. Her physical examination is consistent with hirsutism, which is a result of increased serum testosterone. Many of these patients are also obese. For these reasons, polycystic ovarian syndrome is the most probable reason for this patient's symptoms.

You are caring for a 54-year-old woman with a history of schizophrenia and coronary artery disease who presents to the ED for chest pain. Her vital signs are within normal limits and her ECG is normal sinus rhythm with nonspecific ST/T wave changes. Her first troponin is sent to the laboratory and you are planning to admit her to the hospital for a complete acute coronary syndrome (ACS) evaluation. She receives aspirin and nitroglycerin and her chest pain resolves. A few minutes later, the nurse alerts you that the patient has become unconscious. You go to the bedside and find the patient awake and alert. You review the rhythm strip showing Torsades. What is your next step in management? a. Observe patient b. Magnesium sulfate IV c. Lidocaine IV d. Transvenous pacemaker e. Isoproterenol IV

*B* This patient had a run of torsades de pointes, an atypical VT where the QRS axis swings from positive to negative within a single ECG lead. This dysrhythmia is frequently seen in patients with significant heart disease who have a prolonged QT interval. There are many possible causes of prolonged QT; however, common etiologies include drugs (eg, antidysrhythmics, psychotropics), electrolyte abnormalities, and coronary heart disease. This patient was likely on a phenothiazine for her schizophrenia leading to prolonged QT syndrome and an episode of torsades de pointes. Administration of magnesium sulfate has been shown to decrease runs of torsades.

A 62-year-old man with a history of hypertension presents to the ED with severe constant mid-epigastric pain for the past hour. Over the last several months, he has had intermittent pain shortly after eating, but never this severe. He states he now has generalized abdominal pain that began suddenly about 15 minutes ago. He has no history of trauma, has never had surgery, and takes no medications. His vitals include a HR of 115 beats per minute lying supine, increasing to 135 when sitting up, BP of 170/105 mm Hg supine, falling to 145/85 mm Hg when sitting up. He appears pale. His abdomen is rigid and diffusely tender with guarding and rebound. Bowel sounds are absent and stool hemoccult is positive. The white blood cell (WBC) count is 8500/μL, hemoglobin 8.5 mg/dL, hematocrit 27%, and platelets 255/μL. Which of the following is the most likely diagnosis? a. Boerhaave syndrome b. Perforated gastric ulcer c. Abdominal aortic aneurysm (AAA) d. Inflammatory bowel disease (IBD) e. Diverticulosis

*B* This patient had an untreated gastric ulcer that just perforated. The history of epigastric pain related to eating points to a gastric ulcer, whereas pain 2 to 3 hours after eating is more likely caused by a duodenal ulcer. The sudden onset of generalized abdominal pain associated with a rigid abdomen is concerning for a perforated viscus, in this case, a perforated gastric ulcer. This is a surgical emergency. An abdominal and upright chest radiograph can be performed quickly to look for free air, which will be seen under the diaphragm on the chest radiograph. This is useful for the majority of perforations, which are anterior, but may miss posterior perforations because the posterior duodenum is retroperitoneal. The treatment includes IV hydration, antibiotics, and immediate surgical correction.

A 32-year-old woman presents to the ED complaining of sudden onset of left facial weakness that began half an hour ago that was noticed by her coworker. She denies having medical problems or taking medications. On the review of systems she admits to subjective fevers, fatigue, and arthralgias for the last week, which she attributed to the flu. She also reports having a rash on the back of her thigh a month ago, around the time she was hiking in Rhode Island. On examination she has left facial paralysis. Which of the following is the most likely diagnosis? a. Bell's palsy b. Lyme disease c. Ramsay Hunt syndrome d. Brain tumor e. Rocky Mountain spotted fever (RMSF)

*B* This patient has Lyme disease, the most common vector-borne zoonotic infection in the United States. The spirochete, Borrelia burgdorferi, is transmitted to humans by the deer tick Ixodes, and although the risk of infection after a bite is about 3% in highly endemic areas (Northeast and Midwest); it increases to 25% if the tick is attached for longer than 72 hours. This patient presents in the second stage of Lyme disease. The first stage involves the development of erythema migrans, a spreading annular erythematous lesion with central clearing occurring commonly at the tick bite site 2 to 20 days after the bite. The second stage typically occurs within 6 months of the initial infection and is characterized by fever, fatigue, arthralgias, neuropathies (ie, Bell palsy), cardiac abnormalities (ie, myocarditis presenting with conduction delay), and multiple annular lesions. Tertiary stage of Lyme disease occurs year after the infection and consists of chronic arthritis, subacute encephalopathy, and polyneuropathy. Primary and secondary stages of the disease are treated with doxycycline. Tertiary stage is treated with IV ceftriaxone or penicillin

A 24-year-old girl presents to the ED at 4 AM with severe left eye pain that woke her from sleep. She wears soft contact lenses and does not routinely take them out to sleep. She is in severe pain and wearing sunglasses in the examination room. You give her a drop of proparacaine to treat her pain prior to your examination. On examination, her vision is at baseline and she has no afferent pupillary defect. There is some perilimbic conjunctival erythema. On fluorescein examination, a linear area on the left side of the cornea is highlighted when cobalt blue light is applied. No underlying white infiltrate is visualized. No white cells or flare are visualized in the anterior chamber. What is the most appropriate treatment for this condition? a. Immediate ophthalmology consult b. Tobramycin ophthalmic ointment c. Erythromycin ophthalmic ointment d. Eye patch e. Proparacaine ophthalmic drops

*B* This patient has a corneal abrasion from prolonged contact lens use. The abrasion lights up after fluorescein staining and cobalt blue illumination of the cornea. Contact lens wearers with abrasions are at high risk for Pseudomonas infection and should be treated with an antipseudomonal agent (ie, tobramycin ophthalmic ointment) or fluoroquinolone drops. It is critical to distinguish an abrasion from a corneal ulcer. Ulcers are deeper infections of the cornea that develop from corneal epithelial defects (ie, abrasions). Contact lens wearers are also at high risk for corneal ulcers. The hallmark of a corneal ulcer is a shaggy, white infiltrate within the corneal epithelial defect.

Paramedics bring in a 54-year-old man who was found down in his apartment by his wife. He is successfully intubated in the field and paramedics are currently performing cardiopulmonary resuscitation (CPR). He is transferred to an ED gurney and quickly attached to the cardiac monitors. You ask the paramedics to hold CPR and assess the patient and the rhythm strip. The monitor shows sinus bradycardia, but no pulses are palpable. On examination you appreciate bilateral breath sounds with mechanical ventilation, a soft abdomen, no rashes, and a left arm AV graft. In addition to CPR with epinephrine and atropine every 3 to 5 minutes, which intervention should be performed next? a. Administer 1 ampule of sodium bicarbonate b. Administer 1 ampule of calcium gluconate c. Administer 1 ampule of D50 (dextrose) d. Place left-sided chest tube e. Perform pericardiocentesis

*B* This patient has cardiac electrical activity (sinus bradycardia), but no detectable pulses. He is therefore in a state of pulseless electrical activity (PEA) and management should be directed by the AHA PEA algorithm. Patients in PEA should be treated with CPR, epinephrine every 3 to 5 minutes, and atropine every 3 to 5 minutes (if PEA rate is less than 60 per minute), but a search for an underlying etiology with targeted interventions should still be performed.

A 27-day-old boy presents to the ED with a complaint of a 2-day history of nonbilious vomiting. The child has had no fever and no diarrhea. The baby has always been a "spitter," according to mom, but this seems more excessive and "forceful." The patient has had no wet diapers over the course of the previous 12 hours and is fussy in the examination room. There are no other complaints. The mother has just finished feeding the child formula as you walk into the room and you see the child have an episode of projectile vomiting. The examination reveals temperature 99.8°F, HR 180 beats per minute, RR 50 breaths per minute, and pulse oxymetry of 95% on room air. The remainder of the examination is nonfocal and benign except for slightly prolonged capillary refill. You order the appropriate radiographic studies and consult the appropriate services. If you were to check a set of electrolytes in this patient, the most likely result would be? a. Na 137, K 3.7, Cl− 112, HCO3 22, glucose 110 b. Na 137, K 3.1, Cl− 89, HCO3 39, glucose 55 c. Na 145, K 6.2, Cl− 122, HCO3 35, glucose 55 d. Na 145, K 3.1, Cl− 89, HCO3 16, glucose 80 e. Na 122, K 6.2, Cl− 122, HCO3 35, glucose 55

*B* This patient has pyloric stenosis, and will exhibit a hypochloremic, metabolic alkalosis with hypokalemia. Hypoglycemia is common in children this age who experience poor intake of calories and who have very poor glycemic reserves. The low chloride and high bicarbonate are the result of hydrochloric acid loss from stomach with repeated episodes of vomiting leading to an alkalemia. As a result, the patient will physiologically try to balance ions and trade intracellular H+ for extracellular K+, thus lowering the serum potassium.

A 50-year-old man with a history of hypertension presents to the ED with severe left-sided chest pain for 1 hour. The pain radiates down his left arm and he feels nauseated. His temperature is 98.3°F, BP is 160/92 mm Hg, HR is 92 beats per minute, RR is 16 breaths per minute, and oxygen saturation is 98% on room air. The physical examination is normal. His ECG shows ST-segment elevations in leads II, III, and aVF. You administer aspirin, nitroglycerin, and morphine sulfate and wait for his laboratory results. The nurse calls you over 10 minutes later and tells you that the patient's BP dropped to 60/30 mm Hg with a HR of 100 beats per minute. Which of the following most likely explains his hypotension? a. Medication-related adverse reaction b. Cardiogenic shock c. Papillary muscle rupture d. Free wall rupture e. Rupture of the interventricular septum

*B* This patient is in cardiogenic shock from a right ventricular infarction. The ECG shows STsegment elevation in leads II, III, and aVF indicating the patient is having an inferior wall MI. Approximately 30% of inferior MIs involve the RV. A right-sided ECG would confirm the diagnosis. RV infarction results in reduced right ventricular end-systolic pressure, ultimately leading to decreased cardiac output. Patients with RV infarctions are therefore dependent on preload to maintain cardiac output. In this case, nitroglycerin, a powerful preload reducer, pushed this patient into cardiogenic shock. Patients with RV infarction and hypotension should be treated with IV normal saline to support preload. Dobutamine may also be necessary to support the BP. Furthermore, this patient is having an acute ST-segment elevation MI and may be a candidate for percutaneous coronary intervention or fibrinolytics.

A 25-year-old man fell off his surfboard and landed on rocks. He was pulled from the water by lifeguards and brought to the ED in full cervical and spinal immobilization. He is alert and oriented to person, place, and time. He is complaining of weakness in all of his extremities. His temperature is 98.4°F, BP is 85/50 mm Hg, HR is 60 beats per minute, RR is 20 breaths per minute, and oxygen saturation is 98% on room air. On examination, he has no external signs of head injury. His heart is bradycardic without murmurs. The lungs are clear to auscultation and the abdomen is soft and nontender. He has grossly normal peripheral sensation, but no motor strength in all four extremities. Which of the following is the most likely diagnosis? a. Hypovolemic shock b. Neurogenic shock c. Cardiogenic shock d. Anaphylactic shock e. Septic shock

*B* This patient is in neurogenic shock. He suffered an acute cervical spine injury after his fall onto rocks and has hypotension and bradycardia. The pathophysiology behind neurogenic shock is still under investigation but it's thought to be partially caused by disrupted sympathetic outflow tracts and unopposed vagal tone. Note that all other forms of shock attempt to compensate for hypotension with tachycardia. Neurogenic shock lacks sympathetic innervation; therefore, bradycardia results. Given that this is a trauma patient, all other sources for hypotension must be ruled out. He should be treated with cervical spine immobilization and IV fluids. Pressors may be needed if hypotension does not respond to fluids or fluid overload becomes a concern.

A 25-year-old female scuba diver presents to the ED with multiple areas of periarticular joint pain and red, itchy skin. Her initial vitals include an oral temperature of 98°F, BP of 110/65 mm Hg, HR of 88 beats per minute, RR of 14 breaths per minute, and oxygen saturation of 97% on room air. Upon physical examination, the patient has pain upon palpation of bilateral knees and ankles with full range of motion in these joints. Areas of erythema that do not follow a specific dermatomal pattern cover most of the lower extremities, torso, and back with areas of excoriation where patient reports scratching. There are no other lesions. Which of the following is the most likely diagnosis? a. Sexually transmitted disease (STD) b. Decompression sickness c. Descent barotrauma d. Ascent barotrauma e. Nitrogen narcosis

*B* This patient is suffering from decompression sickness, more commonly known as "the bends." This term refers to a spectrum of states whereupon bubbles of nitrogen gas collect in the blood and tissues. To help illustrate, picture a bottle of soda being opened, allowing the bubbles to rapidly come out of the solution to the top. Clinically, the degree of collection is a result of the depth and length of the dive. Other risk factors include inherent fatigue, heavy exertion, dehydration, and flying after a dive. A patent foramen ovale may also prove to be dangerous in causing gas bubbles to embolize to the arterial system. Decompression sickness can progress from its initial musculoskeletal involvement to include the cardiovascular, respiratory, and central nervous system (CNS). Divers should ascend in a slow, gradual manner to avoid to collection of nitrogen gas in these tissues. Transport to the nearest hyperbaric chamber is the treatment of choice. IV fluid hydration and supplemental oxygen may also be warranted.

20-year-old college student presents to the ED with a cutaneous lesion _______. He is brought in by friends who report being out in an open field during stormy weather when the injury occurred. The patient denies any recent travel or sick contacts. He also denies any symptoms except for some generalized confusion. Which aspect of the physical examination of this patient is initially most pertinent to the nature of this type of injury? a. Testing of cranial nerves b. Otoscopic evaluation of tympanic membranes c. Evaluation of gait cadence d. Testing of cerebellar deficits e. Palpating the cervical spine for tenderness

*B* This patient sustained a direct lightning injury, as evidenced by the typical fern-like pattern exhibited. These injuries may inflict fractures, cardiovascular collapse, burns, blunt abdominal injuries, and neurologic damage. Tympanic membrane ruptures are a common associated injury caused by the outflow tract of the lightning strike and it is important to check for blood in the ear canals of these patients. It is important to quickly assess the ABCs of these patients and establish an airway. Immobilization of the cervical spine is often indicated as well as close ECG monitoring. Patients should be admitted for observation after obtaining a complete blood count (CBC), creatinine kinase with MB fraction, basic metabolic panel, and appropriate radiographs of involved areas. Although 50 to 300 people die because of lightning strikes each year in the United States, most injuries sustained are not lethal.

A 23-year-old woman presents to the ED complaining of dizziness and weakness for 2 days. She complains that she does not have energy to perform her duties at work and even gets short of breath going up the stairs to her third-floor apartment. She denies shortness of breath at rest, fevers, nausea, vomiting, diarrhea, chest pain, headache, recent travel, or other associated symptoms. She does not have any medical problems and takes no medications. On further questioning, she reports that she is on day 9 of her menstrual period, which has been heavy. Her periods are regular and last about 10 days. Her BP is 122/75 mm Hg, HR is 108 beats per minute, temperature is 98.7°F, and RR is 12 breaths per minute. Physical examination is unremarkable except for pale conjunctiva and mild tachycardia. Which of the following is the most appropriate initial diagnostic test? a. Basic metabolic panel b. Complete blood count c. Obtain ECG d. Obtain chest x-ray e. Obtain chest CT with contrast

*B* This woman presents with iron deficiency anemia secondary to menorrhagia. A history of chronic heavy menses and pale conjunctiva on examination should make you suspicious of this common disorder. About 20% of women and 3% of men are iron deficient. Complete blood count provides hemoglobin and hematocrit levels to diagnose anemia. Typically, the mean corpuscular volume (MCV) is low in iron-deficiency anemia. In addition, the patient usually exhibits low serum iron, low serum ferritin, and a high total iron-binding capacity.

A 30-year-old woman presents to the ED with a left hand laceration. She reports cutting herself while attempting to slice a bagel. She has a superficial 2-cm laceration over the left thenar eminence. No deeper structures are involved. What is the best way to clean the wound? a. Punch holes in a normal saline bag or bottle and irrigate the wound. b. Use a high-pressure syringe and irrigate with water. c. Wound cleaning is unnecessary since wound was sustained from a clean knife. d. Soak her hand in normal saline for 30 minutes. e. Apply povidone-iodine to the wound.

*B* To achieve low bacterial counts through irrigation, a pressure of 5 to 8 psi is recommended. This pressure can be generated with an irrigation syringe or a 35-mL syringe and a 19-gauge needle. Water (sterile or tap) is as efficacious as sterile normal saline.

A 54-year-old undomiciled woman presents to the ED with severe cough, general malaise, and subjective fevers for the last week. She also describes coughing up "chicken livers" during this time and reports that her symptoms are getting progressively worse. Her initial vitals include a HR of 100 beats per minute, a BP of 145/66 mm Hg, temperature of 99.9°F and an RR of 16 breaths per minute with an oxygen saturation of 95% on room air. She states that she has a history of alcohol abuse, but denies taking any medications or illicit drugs. A chest radiograph shows a lobar pneumonia. Given this patient's clinical presentation, which of the following is this patient at most risk for contracting? a. Streptococcus pneumoniae b. Klebsiella pneumoniae c. Mycoplasma pneumoniae d. Legionella pneumophila e. Haemophilus influenzae

*B* Undomiciled, alcoholic patients are at particular risk for contracting K pneumoniae. Classically, it presents with a productive cough with currant jelly sputum, fever, general malaise, and an overall toxic appearance. A dense lobar infiltrate with a bulging fissure appearance on a chest radiograph is often described.

You are a passenger aboard an airplane and a 78-year-old woman is complaining of chest pain and difficulty breathing. You are the only medical professional available and volunteer to help. Fortunately, the aircraft is well-equipped with basic medical equipment, as well as with advanced cardiac life support (ACLS) medications and a cardiac monitor. On examination, the passenger's BP is 75/40 mm Hg, HR is 180 beats per minute, and RR is 24 breaths per minute. On examination, the patient is in obvious distress, but able to answer basic questions. Her heart is tachycardic, regular, and without murmurs, rubs, or gallops. Physical examination is remarkable for a bounding carotid pulse. You attach the cardiac monitor and see a regular rhythm at 180 beats per minute with wide QRS complexes and no obvious P waves. After asking the pilot to make an emergency landing what do you do next? a. Amiodarone IV b. Synchronized cardioversion c. Verapamil IV d. Lidocaine IV e. Procainamide IV

*B* Ventricular tachycardia (VT) originates from ectopic ventricular pacemakers and is usually a regular rhythm with rate greater than 100 beats per minute and wide QRS complexes. Treatment of VT is primarily dependent on whether or not the patient is stable. Evidence of acute altered mental status, hypotension, continued chest pain, or other signs of shock are signs of instability. Unstable patients, such as the passenger on this airplane, should receive immediate synchronized cardioversion. It is critical that the cardioverter be placed in the synchronized mode, which permits a search for a large R wave and a corresponding shock around the incidence of such a wave. A shock administered outside of this constraint can induce ventricular fibrillation (VF).

A husband and wife present to the ED with 1 day of subjective fever, vomiting, watery diarrhea, and abdominal cramps. They were at a restaurant a day before for dinner and both ate the seafood special, which consisted of raw shellfish. In the ED, they are both tachycardic with temperatures of 99.8°F and 99.6°F for him and her, respectively. Which of the following is responsible for the majority of acute episodes of diarrhea? a. Parasites b. Viruses c. Enterotoxin-producing bacteria d. Anaerobic bacteria e. Invasive bacteria

*B* Viral diarrheal diseases are responsible for the majority of all acute episodes of diarrhea. Rotavirus, typically a disease of young children and Norwalk virus are the most frequent etiologic agents. In addition, enteric adenovirus is a common cause of gastroenteritis. Though dehydration is a common complication, these illnesses are usually self-limited, requiring only supportive care.

A 59-year-old man presents to the emergency department (ED) complaining of new onset chest pain that radiates to his left arm. He has a history of hypertension, hypercholesterolemia, and a 20-pack-year smoking history. His electrocardiogram (ECG) is remarkable for T-wave inversions in the lateral leads. Which of the following is the most appropriate next step in management? a. Give the patient two nitroglycerin tablets sublingually and observe if his chest pain resolves. b. Place the patient on a cardiac monitor, administer oxygen, and give aspirin. c. Call the cardiac catheterization laboratory for immediate percutaneous intervention (PCI). d. Order a chest x-ray; administer aspirin, clopidogrel, and heparin. e. Start a β-blocker immediately.

*B*. (Rosen, pp 1039-1044.) The patient's presentation is classic for an ACS. He has multiple risk factors with T-wave abnormalities on his ECG. The most appropriate initial management includes placing the patient on a cardiac monitor to detect dysrhythmias, establish intravenous access, provide supplemental oxygen, and administer aspirin. If the patient is having active chest pain in the ED, sublingual nitroglycerin or morphine should be administered until the pain resolves. This decreases wall tension and myocardial oxygen demand. A common mnemonic used is MONA(Morphine, Oxygen, Nitroglycerin, Aspirin) greets chest pain patients at the door. (a) Although nitroglycerin is one of the early agents used in ACS, it is prudent to first rule out a right ventricular infarct, which if present, may lead to hypotension. (c) PCI is warranted if the patient's ECG showed STsegment elevation. (d) The patient will require a chest x-ray and most likely receive clopidogrel and heparin; however this is done only afterbeing on a monitor with oxygen and chewing an aspirin. (e) β-Blockers are usually added for tachycardia, hypertension, and persistent pain and only given once the patient is evaluated for contraindications. Relative contraindications to the use of β-blockers include asthma or chronic obstructive lung disease, CHF, and third-trimester pregnancy.

A 29-year-old man is brought to the ED by EMS for a syncopal episode that occurred during a basketball game. A friend states that the patient just dropped to the ground shortly after scoring a basket on a fast-break. On examination, you note a prominent systolic ejection murmur along the left sternal border and at the apex. An ECG reveals left ventricular hypertrophy, left atrial enlargement, and septal Q waves. You suspect the diagnosis and ask the patient to perform the Valsalva maneuver while you auscultate his heart. Which of the following is most likely to occur to the intensity of the murmur with this maneuver? a. Decrease b. Increase c. Remain unchanged d. Disappear e. The intensity stays the same but the heart skips a beat

*B*The patient has hypertrophic cardiomyopathy, which is characterized by left ventricular hypertrophy without associated ventricular dilation. The hypertrophy is usually asymmetric, involving the septum to a greater extent than the free wall. Patients are at increased risk of dysrhythmias and sudden death. Syncope is usually exertion-related and is caused by a dysrhythmia or a sudden decrease in cardiac output. The murmur associated with hypertrophic cardiomyopathy is a prominent systolic ejection murmur heard along the left sternal boarder and at the apex with radiation to the axilla. The murmur is a result of LV outflow obstruction and mitral regurgitation. It is increased with maneuvers that decrease left ventricular end-diastolic volume, such as the Valsalva maneuver, sudden standing, and exercise.

A 72-year-old man is in the ED for the evaluation of generalized weakness over the previous 24 hours. He has a past medical history of coronary artery disease with a CABG performed 5 years ago, diabetes mellitus, and arthritis. The nurse places the patient on a cardiac monitor and begins to get his vital signs. While the nurse is obtaining the vital signs, he notices the patient suddenly becomes unresponsive. You arrive at the bedside, look at the monitor, and see the following rhythm showing vfib. Which of the following is the most appropriate next step in management? a. Wait 5 minutes to see if he awakens on his own. b. Immediately defibrillate at 200 J (biphasic). c. Perform synchronized cardioversion at 100 J. d. Immediately intubate the patient. e. Insert an IV line and administer amiodarone.

*B*The rhythm is VF. Along with pulseless VT, these are nonperfusing rhythms that are treated identically because it is thought to be caused by the same mechanisms. The earlier a "shock" is administered in cardiac arrest, the more likely the patient will return to spontaneous circulation with a perfusing rhythm. If there is a delay to defibrillation (> 4 minutes), CPR should be administered for 60 to 90 seconds before defibrillation. If after defibrillation (200 J biphasic or 360 J monophasic) the patient's rhythm is still VF or pulseless VT, then assisted ventilation and chest compressions should be started. Intubation should be performed and IV access obtained for the administration of epinephrine or vasopressin. If the rhythm is unchanged after administration of vasopressor therapy, then another attempt at defibrillation at 360 J (or 200 J biphasic) with subsequent administration of an antidysrhythmic (eg, amiodarone) is recommended. Of note, monophasic defibrillation delivers a charge in only one direction. Biphasic defibrillation delivers a charge in one direction for half of the shock and in the electrically opposite direction for the second half. Biphasic defibrillation significantly decreases the energy necessary for successful defibrillation and decreases the risk of myocardial damage.

A 62-year-old woman presents to the ED with general weakness, shortness of breath, and substernal chest pain that radiates to her left shoulder. Her BP is 155/80 mm Hg, HR is 92 beats per minute, and RR is 16 breaths per minute. You suspect that she is having an acute MI. Which of the following therapeutic agents has been shown to independently reduce mortality in the setting of an acute MI? a. Nitroglycerin b. Aspirin c. Unfractionated heparin d. Lidocaine e. Diltiazem

*B.* Aspirin is an antiplatelet agent that should be administered early to all patients suspected of having an ACS, unless there is a contraindication. The ISIS-2 trial provides the strongest evidence that aspirin independently reduces the mortality of patients with acute MI.

A 62-year-old man presents to the ED with gradual dyspnea over the last few weeks. He reports that he is a daily smoker and has not seen a physician in years. Upon physical examination, there are decreased breath sounds on the right as compared to the left. A chest radiograph indicates blunting of the right costophrenic angle with a fluid line. A thoracentesis is performed. Given this patient's history, which of the following most likely describes his effusion? a. Transudative effusion b. Exudative effusion c. Transudative and exudative effusion d. Lactate dehydrogenase < 200 U e. Fluid-to-blood protein ratio < 0.5

*B.* Given this patient's longstanding history of tobacco use and having not seen a doctor for annual examinations, it is likely that the pleural effusion is exudative as a result of an underlying malignancy. Other causes of exudative effusions include the following: infection, connective tissue diseases, neoplasm, pulmonary emboli, uremia, pancreatitis, esophageal rupture, postsurgical, trauma, and drug-induced. Pleural fluid analysis includes LDH, glucose, protein, amylase, cell count, Gram stain, culture, and cytology. Effusions that have a LDH > 200 U, fluid-to-blood LDH ratio > 0.6 and a fluid-to-blood protein ratio > 0.5 are classified as exudative. Levels less than these are classified as transudative.

A 26-year-old woman presents to the ED with an acute onset of dyspnea after falling down a few steps. The patient denies any loss of consciousness and reports feeling short of breath. Her initial chest x-ray appears normal; however she continues to be symptomatic with stable vital signs. Which of the following procedures should be performed next? a. Repeat upright chest x-ray b. Inspiratory and expiratory chest radiographs c. Chest CT scan d. Chest thoracostomy e. Chest thoracotomy

*B.* Inspiratory and expiratory radiographs allow better visualization of the lung pleura and may help better elucidate the presence of a pneumothorax not initially visualized on the chest radiograph.

59-year-old woman presents to the ED complaining of worsening lower abdominal pain over the previous 3 days. She describes feeling constipated recently and some burning when she urinates. Her BP is 135/75 mm Hg, HR is 89 beats per minute, temperature is 101.2°F, and her RR is 18 breaths per minute. Her abdomen is mildly distended, tender in the LLQ, and positive for rebound tenderness. CT scan is consistent with diverticulitis with a 7-cm abscess. Which of the following is the most appropriate management for this condition? a. Reserve the OR for emergent laparotomy. b. Start treatment with ciprofloxacin and metronidazole and plan for CT-guided draining of the abscess. c. Give an IV dose of ciprofloxacin and have the patient follow-up with her primary physician. d. Start treatment with ciprofloxacin and metronidazole and plan for an emergent barium enema. e. Start treatment with ciprofloxacin and metronidazole and prep for an emergent colonoscopy.

*B.* Management for complicated acute diverticulitis involves admission and antibiotic treatment. Treatment is directed against both anaerobic and gram-negative bacteria. Intraabdominal abscess formation secondary to diverticulitis requires prompt surgical consultation and should be drained using CT or ultrasound-guided percutaneous draining. Abscesses less than 5 cm in diameter may be treated with antibiotics alone.

A 54-year-old woman presents to the ED because of a change in behavior at home. For the past 3 years she has end-stage renal disease requiring dialysis. Her daughter states that the patient has been increasingly tired and occasionally confused for the past 3 days and has not been eating her usual diet. On examination, the patient is alert and oriented to person only. The remainder of her examination is normal. An initial 12-lead ECG shows peaked T waves. Which of the following electrolyte abnormalities best explains these findings? a. Hypokalemia b. Hyperkalemia c. Hypocalcemia d. Hypercalcemia e. Hyponatremia`

*B.* Patients with end-stage renal disease, who require dialysis, are prone to electrolyte disturbances. This patient's clinical picture is consistent with hyperkalemia. The ECG can provide valuable clues to the presence of hyperkalemia. As potassium levels rise, peaked T waves are the first characteristic manifestation. Further rises are associated with progressive ECG changes, including loss of P waves and widening of the QRS complex. Eventually the tracing assumes a sinewave pattern, followed by ventricular fibrillation or asystole

An 18-year-old woman presents to the ED complaining of acute onset of RLQ abdominal pain. She also describes the loss of appetite over the last 12 hours, but denies nausea and vomiting. Her BP is 124/77 mm Hg, HR is 110 beats per minute, temperature is 102.1°F, RR is 16 breaths per minute, and oxygen saturation is 100% on room air. Abdominal examination reveals lower abdominal tenderness bilaterally. On pelvic examination you elicit cervical motion tenderness and note cervical exudates. Her WBC is 20,500/μL and β-hCG is negative. Which of the following is the most appropriate next step in management? a. Bring her to the OR for an appendectomy b. Begin antibiotic therapy c. Perform a culdocentesis d. Bring her to the OR for immediate laparoscopy e. Order an abdominal plain film

*B.* Pelvic inflammatory disease (PID) comprises a spectrum of infections of the female upper reproductive tract. Although N gonorrhoeae and C trachomatis are thought to cause the majority of infections, new evidence points to greater rates of polymicrobial infections. Most cases of PID are thought to start with a sexually transmitted disease of the lower genital tract and ascend to the upper tract. Women typically present with lower abdominal pain and may have vaginal discharge, vaginal bleeding, dysuria, and fever. The examination usually reveals lower abdominal tenderness and cervical motion tenderness or adnexal tenderness. Many patients are treated as outpatients with antibiotics. Considerations for admission include those women who are pregnant, failed outpatient therapy, are toxic appearing, have evidence for a tubo-ovarian abscess, or a surgical emergency cannot be ruled out. Long-term outcomes are improved if antibiotics are begun immediately.

Paramedics bring a 45-year-old man to the ED after being involved in a high-speed motor vehicle collision. His BP is 85/50 mm Hg and HR is 131 beats per minute after administering 2 L of normal saline. He is awake but slow in responding to questions. A right upper quadrant (RUQ) ultrasound image shows fluid in the Morison pouch. Which of the following is the most appropriate next step in management? a. Emergent abdominal computed tomographic (CT) scan b. Transfer to the operating room (OR) for laparotomy c. Perform a diagnostic peritoneal lavage (DPL) d. Observe until one more liter of crystalloid fluid is administered e. Serial abdominal examinations

*B.* The image reveals fluid in the space between the right kidney and liver, known as Morison pouch. Free fluid is seen as an anechoic area (black in color). The FAST examination is replacing the DPL in cases of abdominal trauma. It is inexpensive, noninvasive, and confirms the presence of hemoperitoneum in minutes. The minimum amount of intraperitoneal fluid needed for detection by ultrasound is approximately 70 cc. Patients who remain unstable despite volume resuscitation and have intraperitoneal fluid demonstrated by the FAST examination need to be taken to the OR for a laparotomy to identify and treat the source of bleeding.

A 55-year-old man presents to the ED with chest pain and shortness of breath. His BP is 170/80 mm Hg, HR is 89 beats per minute, and oxygen saturation is 90% on room air. Physical examination reveals crackles midway up both lung fields and a new holosystolic murmur that is loudest at the apex and radiates to the left axilla. ECG reveals ST elevations in the inferior leads. Chest radiograph shows pulmonary edema with a normalsized cardiac silhouette. Which of the following is the most likely cause of the cardiac murmur? a. Critical aortic stenosis b. Papillary muscle rupture c. Pericardial effusion d. CHF e. Aortic dissection

*B.* The patient's presentation is consistent with acute mitral valve regurgitation because of a ruptured papillary muscle in the setting of an AMI. Patients usually present with pulmonary edema in the setting of an AMI. Chest x-ray characteristically reveals pulmonary edema with a normal heart size. The characteristic murmur of mitral regurgitation is a holosystolic murmur that is loudest at the apex.

A 73-year-old man who is a 1-pack-per-day smoker and has a medical history of hypertension and peripheral vascular disease presents to the ED complaining of mid-abdominal and right flank pain. He states that he had this same pain 1 week ago and that it got so bad that he passed out. His BP is 125/75 mm Hg, HR is 85 beats per minute, temperature is 98.7°F, and his RR is 17 breaths per minute. Physical examination reveals a bruit over his abdominal aorta and a pulsatile abdominal mass. Which of the following is the most appropriate initial test to evaluate this patient? a. Angiography b. Ultrasound c. MRI d. Plain radiograph e. D-dimer

*B.* The patient's presentation is worrisome for an AAA. If the patient was hemodynamically unstable, he should be brought immediately to the OR for definitive repair. However, in the stable patient, imaging studies can aid in the diagnosis of an AAA. Ultrasound is almost 100% sensitive in detecting AAA, it is noninvasive, and can be performed rapidly at the patient's bedside. If the entire abdominal aorta is visualized and found to be of normal diameter, it is safe to say that the patient's symptoms are not from an AAA. An alternative to ultrasound in the stable patient is a CT scan, which is essentially 100% accurate in determining the presence of an AAA. The CT scan is less subject to technical and interpretation errors than an ultrasound.

A 23-year-old woman presents to the ED complaining of lower abdominal pain and vaginal spotting for 2 days. Her menstrual cycle is irregular. She has a history of ovarian cysts and is sexually active but always uses condoms. Her BP is 115/75 mm Hg, HR is 75 beats per minute, temperature is 98.9°F, and RR is 16 breaths per minute. Which of the following tests should be obtained next? a. Chlamydia antigen test b. β-Human chorionic gonadotropin (β-hCG) c. Transvaginal ultrasound d. Abdominal radiograph e. Observe her abdominal pain, if it resolves discharge her with a diagnosis of menstruation

*B.* β-Human chronic gonadotropin (β-hCG) should be obtained in all women of child-bearing age who present with abdominal pain or vaginal bleeding. Diagnostically, it is one of the most important tests in female patients. A positive pregnancy test in the setting of abdominal pain and vaginal bleeding demands that the physician rule out an ectopic pregnancy. Even though the patient states she always uses condoms during intercourse, there is still a small risk of pregnancy.

A 31-year-old kindergarten teacher presents to the ED complaining of acute onset substernal chest pain that is sharp in nature and radiates to her back. The pain is worse when she is lying down on the stretcher and improves when she sits up. She smokes cigarettes occasionally and was told she has borderline diabetes. She denies any recent surgeries or long travel. Her BP is 145/85 mm Hg, HR is 99 beats per minute, RR is 18 breaths per minute, and temperature is 100.6°F. Examination of her chest reveals clear lungs and a friction rub. Her abdomen is soft and nontender to palpation. Her legs are not swollen. Chest radiography and echocardiography are unremarkable. Her ECG shows diffuse ST elevation w/ PR depression. Which of the following is the most appropriate next step in management? a. Anticoagulate and CT scan to evaluate for a PE b. Prescribe a NSAID and discharge the patient c. Aspirin, heparin, clopidogrel, and admit for ACS d. Administer thrombolytics if the pain persists e. Prescribe antibiotics and discharge the patient

*B.*The classic presentation of pericarditis includes chest pain, a pericardial friction rub, and ECG abnormalities. A prodrome of fever and myalgias may occur. Pericarditis chest pain is usually substernal and varies with respiration. It is classically sharp or pleuritic in character. It is typically relieved by sitting forward and worsened by lying down or swallowing. The physical examination hallmark of acute pericarditis is the pericardial friction rub. The earliest ECG changes are seen in the first few hours to days of illness and include diffuse ST-segment elevation seen in leads I, II, III, aVL, aVF, and V2 to V6. Most patients with acute pericarditis will have concurrent PR-segment depression. The mainstay of treatment includes supportive care with anti-inflammatory medications (eg, NSAIDs).

A 25-year-old man presents to the emergency department (ED) with a right forearm laceration that he sustained from a piece of glass during a bar fight. He complains of pain at the laceration site but denies a foreign body sensation. On examination, the laceration is superficial and 4 cm long. The patient demonstrates intact strength and sensation distally. Which of the following statements regarding imaging this patient is true? a. Imaging can be omitted because he does not have a foreign body sensation. b. Imaging is not indicated because retained glass does not cause an inflammatory reaction. c. Plain film is the next best step in the management of this patient. d. Computed tomographic (CT) scan is the most commonly used modality to rule out a foreign body. e. Plain film cannot rule out bone, teeth, or glass in soft tissue.

*C* 80% to 90% of foreign bodies can be seen on plain films. Glass > 2 mm is visible on plain film.

A 23-year-old woman presents to the emergency department (ED) with irregular menstrual bleeding. She denies any abdominal pain, dizziness, or palpitations. The patient reports that her last menstrual period was 2 weeks ago with normal flow and duration. Which of the following ancillary tests is critical in defining the differential diagnosis for this patient? 'a. Type and screen b. Coagulation panel c. β-Human chorionic gonadotropin (β-hCG) d. Complete blood count (CBC) e. No ancillary tests are needed

*C* A β-hCG should be one of the first ancillary tests considered in a patient presenting with vaginal bleeding regardless of their sexual, contraceptive, and menstrual history. In addition to the initial and timely urine qualitative test, a follow-up serum quantitative β-hCG is warranted if positive. Patients that present with these symptoms and a positive β-hCG need follow-up for repeat testing to check trending levels. Determining if a patient is pregnant may also help distinguish if, for example, the emergent cause of this patient's bleeding is an ectopic pregnancy versus a pathologic cervical lesion.

A 32-year-old man is brought to the ED by paramedics after a diving accident. The lifeguard on duty accompanies the patient and states that he dove head first into the shallow end of the pool and did not resurface. On examination, the patient is speaking but cannot move his arms or legs and cannot feel pain below his clavicle. He is able to feel light touch and position of his four extremities. A cervical spine radiograph does not reveal a fracture. Which of the following is the most likely diagnosis? a. Spinal cord injury without radiographic abnormality (SCIWORA) b. Central cord syndrome c. Anterior cord syndrome d. Cauda equina syndrome e. Brown-Séquard syndrome

*C* Anterior cord syndrome results from cervical flexion injuries (eg, diving in shallow water) that cause cord contusion or protrusion of a bony fragment or herniated intervertebral disk into the spinal canal. It may also occur from vascular pathology, such as laceration or thrombosis of the anterior spinal artery. The syndrome is characterized by different degrees of paralysis and loss of pain and temperature sensation below the level of injury. Its hallmark is the preservation of the posterior columns, maintaining position, touch, and vibratory sensation.

A 33-year-old woman presents to the ED with a painful sprained ankle. She has a past medical history of depression for which she is taking phenelzine, a monoamine oxidase inhibitor. After you place an elastic wrap on her ankle, she asks you to prescribe her some pain medication. Which of the following medications is contraindicated in patients taking a monoamine oxidase inhibitor? a. Ibuprofen b. Acetaminophen c. Meperidine d. Oxycodone e. Hydrocodone

*C* Any patient taking a monoamine oxidase inhibitor (MAOI) is at risk for developing the serotonin syndrome if the individual congests a selective serotonin reuptake inhibitor (SSRI) or another drug that raises CNS serotonin levels. Some medications that can cause this interaction include indirect-acting and mixed-acting sympathomimetics (eg, cocaine, amphetamine), antidepressants (eg, TCA, SSRI), meperidine (eg, Demerol), and dextromethorphan, which is found in many nonprescription antitussives. In contrast, morphine and its derivatives lack serotonin-potentiating effects. Serotonin syndrome is characterized by altered mental status, hyperthermia, neuromuscular dysfunction, and autonomic dysfunction. Symptoms may also include shivering, trismus, akathisia, coma, and seizures.

A 43-year-old undomiciled man is brought to the ED after being found intoxicated on the street. He is currently rousable and expresses a request to be left alone. Initial vitals include a HR of 92 beats per minute, a BP of 125/80 mm Hg, and an RR of 14 breaths per minute with an oxygen saturation of 93% on room air. His rectal temperature is 101.2°F. A chest radiograph shows infiltrates involving the right-lower lobe. Given this clinical presentation, what initial antibiotic coverage is most appropriate for this patient? a. Gram-negative coverage only b. Gram-positive coverage only c. Broad-spectrum with anaerobic coverage d. PCP coverage e. Antifungal therapy

*C* Aspiration pneumonia occurs secondary to the inhalation of either oropharyngeal or gastric contents into the lower airways. Aspiration of gastric juices may cause a pulmonary inflammatory response. This type of mechanism of acquiring pneumonia is commonly seen in those with swallowing difficulties or a relaxed cardiac sphincter because of alcohol. Given these factors, this patient is in a high risk category for aspiration pneumonia. The small degree of angulation of the right mainstem bronchus makes the right lung at higher risk. Most particles easily travel down this route, ending up in the right middle or lower lobe of the lung. Antibiotic coverage should be broad, covering for both gram positive and gram-negative organisms including anaerobes, which are commonly present in the mouth. Given the severity, these patients may go on to develop ARDS, an inflammatory response to infection, and, subsequently, respiratory failure.

A 29-year-old woman presents to the ED complaining of worsening left wrist pain for 1 month. She states that approximately 3 months ago she fell from a ladder and landed on her outstretched hand. She never went to the hospital and just dealt with the pain for a while. On examination, there is no deformity of the wrist. Neurovascular status is normal, but there is tenderness when you palpate the snuffbox. What is the most likely reason for this patient's wrist pain? a. Fracture of the distal ulna b. Acute fracture of the scaphoid c. Avascular necrosis of the scaphoid d. Hematoma of the radial artery e. Fracture of the lunate

*C* Avascular necrosis of the scaphoid is seen in approximately 3% of scaphoid fractures. The typical presentation of a scaphoid fracture is a FOOSH. On examination patients have snuff box tenderness or tenderness with axial loading of the thumb. Blood supply to the scaphoid is provided by a single artery that flows into the distal portion of the bone leaving the proximal portion vulnerable in the setting of a fracture. This increases the likelihood of complications, particularly avascular necrosis, in the setting of a poorly healed fracture.

21-year-old girl presents to the ED complaining of diarrhea, abdominal cramps, fever, anorexia, and weight loss for 3 days. Her BP is 127/75 mm Hg, HR is 91 beats per minute, and temperature is 100.8°F. Her abdomen is soft and nontender without rebound or guarding. WBC is 9200/μL, β-hCG is negative, urinalysis is unremarkable, and stool is guaiac positive. She tells you that she has had this similar presentation four times over the past 2 months. Which of the following extraintestinal manifestations is associated with Crohn disease but not ulcerative colitis? a. Ankylosing spondylitis b. Erythema nodosum c. Nephrolithiasis d. Thromboembolic disease e. Uveitis

*C* Crohn disease is characterized by chronic inflammation extending through all layers of the bowel wall. Onset is generally between the ages of 15 and 40 years. Crohn disease should be suspected in any patient whose symptoms show a picture consistent with chronic inflammatory colitis. Extraintestinal manifestations are see in 25% to 30% of patients. The incidence is similar for Crohn disease and ulcerative colitis. They include aphthous ulcers, erythema nodosum, iritis or episcleritis, arthritis, and gallstones. Nephrolithiasis is seen as a result of hyperoxaluria because of increased oxalate absorption in patients with ileal disease. Because ulcerative colitis affects only the large bowel, this extraintestinal manifestation is seen only in patients with Crohn disease.

As a senior resident in the ED, an intern calls you over to see a patient he treated for a migraine headache. The patient is a 21-year-old woman with a history of poorly controlled migraines. The patient was using a number of migraine medications at home and several were administered intravenously. As you approach the patient, you note her tongue is protruding and her head is tilted to the left. She is grimacing. The intern is concerned that the patient is having an acute stroke and would like to obtain a head CT scan. You advise the intern that the symptoms are likely the result of a medication side effect. Which of the following medications is likely to have caused the patient's symptoms? a. Morphine sulfate b. Acetaminophen c. Metoclopramide d. Caffeine e. Sumatriptan

*C* Dystonic reactions may occur with the use of dopamine blocking agents. Medications classically associated with dystonic reactions are typical antipsychotics (eg, haloperidol) but can also occur with the antiemetics used to treat migraines. They are generally not life-threatening and respond almost immediately to administration of diphenhydramine (Benadryl) given intravenously or intramuscularly or benzodiazepines. Common dystonic reactions include oculogyric crises (eyes deviating in different directions), torticollis, tongue protrusion, facial grimacing, and difficulty speaking.

A 45-year-old woman presents to the ED complaining of 3 days of fever and worsening throat pain and painful odynophagia without cough or coryza. She sits on a chair, leaning forward with her mouth slightly open. She has a cup of saliva and a box of facial tissues at her side. Vitals are HR of 120 beats per minute, BP of 110/70 mm Hg, RR of 22 breaths per minute, oxygen saturation of 99% on room air, and temperature of 102°F. Her voice is hoarse, but she is able to open her mouth fully. Her posterior oropharynx is moderately hyperemic, without exudates or tonsillar enlargement. A soft tissue lateral cervical radiograph shows marked edema of the prevertebral soft tissues and absence of the vallecular space. Which of the following is the most likely diagnosis? a. Retropharyngeal abscess b. Peritonsillar abscess c. Epiglottitis d. Pharyngitis e. Laryngotracheitis

*C* Epiglottitis is a lifethreatening inflammatory condition, usually infectious, of the epiglottis and the aryepiglottic and periglottic folds. Since most children are immunized against Haemophilus influenzae type B (Hib), most cases of epiglottitis are now seen in adults, with an average age of 46 years. Signs and symptoms include a prodromal period of 1 to 2 days consisting of constitutional symptoms, then the patient exhibits high fever, dysphagia, odynophagia, drooling, and dyspnea. Stridor is primarily inspiratory and softer and lower-pitched than croup. The "thumbprint sign" seen on lateral cervical radiograph demonstrates a swollen epiglottis obliterating the vallecula.

A 24-year-old man presents to the ED with diffuse facial pain, pruritus, erythema, and dizziness after reportedly falling on to an anthill. His initial vital signs include HR of 102 beats per minute, BP of 118/75 mm Hg, and RR of 18 breaths per minute with oxygen saturation of 98% on room air. What is the initial appropriate assessment in this clinical scenario? a. Give tetanus prophylaxis b. Begin local wound care c. Assess airway, breathing, and circulation d. Start IV fluid administration e. Give oral antihistamines

*C* Fire ants have proven to be a real threat to humans. Ninety-five percent of clinical cases result from the Solenopsis invicta species, another member of the Hymenoptera, which was imported from Brazil in the 1930s. This ant is found in many of the southern states given that it cannot survive long winters and is slowly replacing the less dangerous species native to North America. It is small and light-reddish to dark brown and its venom is 99% alkaloid, which is unique to the animal kingdom. This causes hemolysis, membrane depolarization, local tissue destruction, and activation of the complement pathway all of which could be especially dangerous in this patient with facial injuries. The sting usually produces a pustule within 24 hours. Local burning, erythema, and pruritus are common. About 10% of cases have progressive, systemic symptoms including nausea, vomiting, dizziness, respiratory distress, and further hypersensitivity. Therefore, assessment of this patient's airway and circulation are imperative. Continuous monitoring is indicated to detect hemodynamic instability.

A 74-year-old man presents to the ED after being involved in a motor vehicle collision. He states he was wearing his seat belt in the driver's seat when a car hit him from behind. He thinks his chest hit the steering wheel and now complains of pain with breathing. His RR is 20 breaths per minute, oxygen saturation is 98% on room air, BP is 145/75 mm Hg, and HR is 90 beats per minute. On examination, you notice paradoxical respirations. Which of the following best describes a flail chest? a. One rib with three fracture sites b. Two adjacent ribs each with two fracture sites c. Three adjacent ribs each with two fracture sites d. One fractured right sided rib and one fractured left-sided rib e. Two fractured right sided ribs and two fractured left-sided ribs

*C* Flail chest results when three or more adjacent ribs are fractured at two points, allowing a freely moving segment of the chest wall to move in a paradoxical motion. It is one of the most commonly overlooked injuries resulting from blunt chest trauma.

A 2-year-old boy is brought to the ED shortly after a choking episode. His parents noted he had been playing with coins on the floor just prior to the episode. There has been no previous history of fever or runny nose in the past few days. The parents tried to feed the patient after the episode but he has been unwilling to take anything orally. On examination, the patient is calm with stable vital signs and a pulse oximetry of 98% on room air. He spits in a cup every couple of minutes, but is otherwise in no apparent distress. His oropharynx is unremarkable and his lungs are without wheezes or rales. His radiograph shows a coin in a horizontal orientation. Which of the following most likely accounts for the patient's symptoms? a. Foreign body aspiration b. Bronchospasm c. Foreign body ingestion d. Epiglottitis e. Allergic reaction

*C* Foreign body ingestion and aspiration are common in this age group with peak occurrence between 6 months and 4 years. In this patient, the position of the coin on the anterior-posterior chest film aids with the diagnosis. When the coin is in the esophagus it is seen head-on in the AP projection. When it is localized in the trachea it is seen in the sagittal plane because the cartilaginous tracheal rings in children are incomplete and remain open posteriorly, causing the coin to sit sagittal or sideways. After assessing the patient's status (respiratory distress, inability to swallow, etc), it is important to assess characteristics of the foreign body ingested. Ingested foreign bodies usually get obstructed in three common locations: thoracic inlet (60%-80%), at the gastroesophageal junction (10%-20%), and at the aortic arch (5%-20%). A good way to remember this is by their level in the spine: C4-C6, T8, and T4, respectively. Once they pass the pylorus, most foreign bodies pass through the remainder of the GI tract. Objects that are larger than 2 cm × 5 cm, sharp objects (needles, tacks), or disc batteries should be considered for removal.

A 30-year-old woman with no prior pregnancies presents to the ED with diffuse pelvic pain and vaginal spotting. The following transvaginal ultrasound is performed and shows _____ What is the minimum β-hCG level needed to obtain this sonographic image? a. 1000 mIU/mL b. 950 mIU/mL c. 1500 mIU/mL d. 4000 mIU/mL e. 6500 mIU/mL

*C* Given that this is a transvaginal ultrasound, the threshold for visualizing a pregnancy is lower than the transabdominal approach (β-hCG > 6500 mIU/mL). A transvaginal ultrasound can typically identify a gestational sac when the β-hCG > 1000 mIU/mL, and the yolk sac when the β-hCG > 2500 mIU/mL.

A 60-year-old woman with a history of diabetes is brought into the ED by EMS workers who state that the patient was found on a bus in a lethargic and diaphoretic. Her fingerstick glucose level at the scene was 35 mg/dL. EMS workers quickly administered dextrose through an IV line. The patient became alert and responsive and remained this way throughout her trip to the ED. However, in the ED you notice that the patient is again diaphoretic and is mumbling her speech. Her fingerstick glucose is now 47 mg/dL. You administer dextrose and she perks right up. Which of the following diabetes medications commonly causes hypoglycemia for which the patient is likely to require hospital admission? a. Regular insulin b. Metformin c. Glyburide d. Rosiglitazone e. Acarbose

*C* Glyburide is a commonly prescribed sulfonylurea. Sulfonylureas are oral agents that stimulate the beta cells of the pancreas to produce insulin. Many of the sulfonylureas have relatively long durations of action. Glyburide can act up to 24 hours after ingestion. Hypoglycemia secondary to sulfonylureas generally requires hospital admission to monitor for recurrent hypoglycemia.

A 67-year-old man presents to the ED for worsening confusion. His wife states that he received his first dose of chemotherapy for lung cancer 2 days ago. Over the last 24 hours, the patient became confused. His BP is 130/70 mm Hg, HR is 87 beats per minute, and temperature is 98.9°F. While in the ED, the patient seizes. You administer an antiepileptic and the seizure immediately stops. You compare his current electrolyte panel to one taken 2 days ago. Sodium (mEq/L) 139 -> 113 Potassium (mEq/L) 4.1 -> 3.9 Chloride (mEq/L) 105 -> 98 Bicarbonate (mEq/L) 23 -> 20 BUN (mg/dL) 13 -> 17 Creatinine (mg/dL) 0.4 -> 0.7 Glucose (mg/dL) 98 -> 92 Which of the following is the most appropriate treatment? a. 0.45% saline b. 0.9% saline c. 3% saline d. 5% dextrose e. 50% dextrose

*C* Hyponatremia is defined as a measured serum sodium less than 135 mEq/L. However, the development of symptoms secondary to hyponatremia is related more to the rate of change in the serum sodium than to the absolute value. Levels less than 120 mEq/L tend to cause symptoms regardless of the rate to reach this value. Symptoms can include confusion, lethargy, nausea, vomiting, anorexia, muscle cramps, and seizures. There are many causes of hyponatremia, including renal or GI losses, third-spacing, endocrine abnormalities, syndrome of inappropriate antidiuretic hormone release (SIADH), cirrhosis, CHF, and nephrotic syndrome. Many medications cause SIADH, in addition to pulmonary and CNS disease. This patient, in particular, just started chemotherapy for lung cancer. The treatment for hyponatremia is guided by the cause of the process. However, if a patient is symptomatic (eg, seizing), hypertonic saline (3%) should be carefully administered to raise the serum sodium to 120 mEq/L. A known complication of hypertonic saline when it is administered too fast and sodium levels rise rapidly is the development of central pontine myelinosis.

A 22-year-old woman with known IIH who is scheduled for a ventriculoperitoneal shunt in 2 weeks presents to the ED complaining of severe headache. She states the headache is similar to the normal headaches associated with her condition except that it is refractor to her regular medications, including triptans, and opiates. Her neurologist increased her dose of acetazolamide, but this also did not help. Her noncontrast head CT is unchanged from previous and she does not have papilledema. Which of the following is likely to provide prompt relief? a. IV corticosteroids b. Infusion of mannitol c. LP with removal of 15 cc of CSF d. IV metoclopramide e. Nonsteroidal anti-inflammatory drugs (NSAIDs)

*C* IIH is an idiopathic elevation of ICP. In the setting of a normal CT, the diagnosis is made by LP with an elevated opening pressure often between 250 and 450 mm H2O. Patients often experience complete relief of their symptoms with LP and return of their ICP to levels < 200 mm H2O.

An 18-year-old man presents to the ED after telling a school counselor that he wanted to harm himself. His physical examination is unremarkable without any signs of trauma. He reports occasional excessive alcohol use. He states that his parents recently separated and that he has been living with either parent on a rotating schedule. Overall, he feels supported by family and friends but continues to feel hopeless despite this. Which of the following factors in this patient is most likely to increase his risk of an actual suicidal attempt? a. Sex b. Age c. Hopelessness d. Alcohol use e. Parent separation

*C* In the psychiatric literature, there exists a "SADPERSONS" scale that enumerates risk factors. Two points are given for factors that are considered higher risk. These include depression or hopelessness, rational thinking loss, organized or serious attempt, and stated future intent. Lower risk factors, given one point, are male sex, age < 19 years or > 45 years, previous attempts or psychiatric care, excessive alcohol or drug use, separation, divorce or widowed status, and no social supports. Firearms in the household, family violence, abuse, and chronic illness also increase risk.

A 25-year-old G1P0 presents to the ED with vaginal bleeding. She recently discovered that she was pregnant and has yet to be medically evaluated. Prior to presentation, the patient was in her usual state of good health with no history of trauma or any other symptoms. She reports using two absorbent pads an hour for the bleeding and does not notice passing fetal tissue. On physical examination, her cervical os is open. Which of the following is the most appropriate diagnosis? a. Threatened abortion b. Complete abortion c. Inevitable abortion d. Incomplete abortion e. Missed abortion

*C* Inevitable abortions are diagnosed in first-trimester bleeding with an open cervical os but no passage of fetal products. Determining whether the internal os is open is often misleading and confused with the normally distended external os. It is defined as open when one can place more than a fingertip within the cervix. In these cases, dilation and curettage with full evacuation of the pregnancy is warranted. Rh immunization may also be needed depending on the status of the patient; therefore, a type and screen should be obtained. First-trimester vaginal bleeding occurs in about 40% of pregnancies with approximately half of them eventually resulting in spontaneous miscarriages.

A 32-year-old woman is brought to the ED by paramedics after being involved in a motor vehicle collision. The patient was the front-seat passenger of the car and was not wearing a seat belt. In the ED, the patient is speaking and complains of abdominal pain. Her breath sounds are equal bilaterally. You note a distended abdomen. A FAST eamination is positive for fluid in the left upper quadrant (LUQ). Her BP is 90/70 mm Hg and HR is 120 beats per minute. You administer 2 L of crystalloid solution. Her repeat BP is 80/60 mm Hg. Which of the following is the most appropriate next step in management? a. Administer a vasoconstrictor, such as epinephrine. b. Administer another 2 L of crystalloid. c. Administer type O, Rh-negative blood. d. Bring patient to the CT scanner for an emergent scan. e. Perform another FAST examination to see if the fluid is increasing.

*C* Initial fluid resuscitation usually begins with crystalloid fluids such as 0.9% normal saline or Ringer lactate. In general, if the patient remains hemodynamically unstable after 40 cc/kg of crystalloid administration (approximately 2-3 L), then a blood transfusion should be started. Fully cross-matched blood is preferable; however, this is generally not available in the early resuscitation period. Therefore, type specific blood (type O, Rh-negative or type O, Rh-positive) is a safe alternative and is usually ready within 5 to 15 minutes. Type O, Rh-negative blood is typically reserved for women in their childbearing years to prevent Rh sensitization. Type O, Rh-positive blood can be given to all men and women beyond their childbearing years.

A 29-year-old G1P0010 presents to the ED with sharp, right-sided flank pain of acute onset associated with nausea. The pain began approximately 1 hour before arrival. She denies any fever, hematuria, vomiting, change in bowel habit, or sick contacts. She cannot recall her last menstrual period at this time. The patient is afebrile and her vitals are within normal limits as you begin your physical examination, which reveals a soft abdomen with mild diffuse tenderness to palpation without rebound. The patient has exquisite right flank pain. An initial urine dip is negative. Given the information you have so far, which of the following is the most probable diagnosis? a. Appendicitis b. PID c. Ectopic pregnancy d. Ureteral stone e. Diverticulitis

*C* It is very important to stress the possibility that any fertile female is pregnant until proven otherwise. Obtaining a urine β-hCG along with an ultrasound could save your patient's life, as other essential tests and consults are being acquired. In this case of a ruptured ectopic pregnancy, the presentation is atypical but it may be said that it is essentially typical of ectopics. These are, by very nature, implantations that can occur anywhere in the abdominal cavity. It is therefore imperative to keep this life-threatening diagnosis at the top of your differential.

A 50-year-old man presents to the ED complaining of fever, sore throat, and neck pain for 24 hours. He states that 1 week ago he had two molars extracted from his mouth. His BP is 145/75 mm Hg, HR is 102 beats per minute, temperature is 101.2°F, and his RR is 16 breaths per minute. On examination you notice that the patient is drooling. There is erythema and swelling of his submandibular area that gives the appearance of a "bull neck." His tongue is swollen and elevated and the floor of his mouth is tender. There is no fluctuant mass in his mouth. Which of the following is the most likely diagnosis? a. Acute mastoiditis b. Peritonsillar abscess c. Ludwig angina (LA) d. Acute necrotizing ulcerative gingivitis (ANUG) e. Streptococcus pharyngitis

*C* Ludwig angina is a potentially fatal disease that can progress to death within hours. It is a progressive cellulitis of the floor of the mouth and neck that begins in the submandibular space. A dental cause, such as an extraction, is present in approximately 90% of cases. The most common symptoms include dysphagia, neck pain, and swelling. Physical findings include bilateral submandibular swelling, tongue swelling, and protrusion. A tense edema and induration of the neck may occur that is described as a "bull neck." Management involves securing an airway and starting IV antibiotic therapy. immediately. There is debate on whether these patients should be managed surgically with incision and drainage or medically with antibiotics.

A 22-year-old woman presents to the ED by ambulance from a dance club. The paramedics report that the patient was agitated in the club and had a generalized seizure. Her BP is 165/100 mm Hg, HR is 119 beats per minute, temperature is 100.9°F, RR is 17 breaths per minute, oxygen saturation is 98% on room air, and fingerstick glucose is 92 mg/dL. On examination, the patient is hyperactive and appears to be hallucinating. Her pupils are dilated to 6 mm bilaterally and reactive. Her neck is supple. Examination of the heart is unremarkable except for tachycardia. Her lungs are clear and abdomen is soft and nontender. The patient moves all four extremities. Laboratory results are as follows: Sodium 109 mEq/L, WBC 12,000/mm3, Potassium 3.5 mEq/L, Hct 49%, Chloride 83 mEq/L, Platelets 350/μL, Bicarbonate 20 mEq/L, BUN 10 mg/dL, Creatinine 1 mg/dL, Glucose 103 mg/dL. Which of the following substances did this patient most likely consume? a. Cocaine b. Heroin c. 3,4-Methylenedioxymethamphetamine (MDMA) d. Ketamine (special K) e. PCP

*C* MDMA is currently one of the most widely abused amphetamines by college students and teenagers. It is commonly known as "ecstasy," "E," "XTC," and "M&M." MDMA is an entactogen, a substance capable of producing euphoria, inner peace, and a desire to socialize. Negative effects include ataxia, restlessness, confusion, poor concentration, and memory problems. MDMA, although classified as an amphetamine, is also a potent stimulus for the release of serotonin. MDMA can also cause significant hyponatremia. The increase in serotonin results in the excessive release of vasopressin (antidiuretic hormone [ADH]). Moreover, large free-water intake (increased thirst) combined with sodium loss from physical exertion (dancing) certainly contributes to the development of hyponatremia.

An 18-year-old man presents to the ED with nausea and vomiting complaining of testicular pain for the past hour that began while playing volleyball. He recalls having similar pain 1 week ago that resolved spontaneously after 10 minutes. He was recently well and reports no fever, diarrhea, urinary frequency, or dysuria. Physical examination reveals vital signs within normal limits. The patient appears in moderate discomfort, holding his scrotum. His abdomen is soft and nontender. His right hemiscrotum is swollen, erythematous, and diffusely tender. It is not possible to palpate the testis separate from the epididymis. The right cremasteric reflex is absent. His left testis has a horizontal lie and is nontender. You suspect testicular torsion. What is the correct way to attempt manual detorsion? a. Elevate the painful testis until there is pain relief. b. Rotate the testes in a lateral to medial direction as if you were closing a book. c. Rotate the testes in a medial to lateral direction as if you were opening a book. d. Rotate the testes in an inferior to superior direction. e. Rotate the testes in a superior to inferior direction.

*C* Manual detorsion is a maneuver used to untwist the spermatic cord to reestablish blood flow to the testis. This procedure should be performed in any patient with suspected torsion while the patient is being prepared for the OR. Most testes torse lateral to medial. If you were to stand at the foot of the patient's bed, you would perform detorsion for either testis just as you would open a book—rotating each testicle in a medial to lateral direction. Successful detorsion results in immediate reduction of pain. In many cases, detorsion is not successful or the testis twists again.

A 31-year-old woman with a history of schizophrenia presents to the ED for altered mental status. A friend states that the patient is on multiple medications for her schizophrenia. Her BP is 150/80 mm Hg, HR 121 beats per minute, RR 20 breaths per minute, and temperature 104.5°F. On examination, the patient is diaphoretic with distinctive "lead-pipe" rigidity of her musculature. You believe the patient has neuroleptic malignant syndrome. After basic stabilizing measures, which of the following medications is most appropriate to administer? a. Haloperidol b. Droperidol c. Dantrolene d. Diphenhydramine e. Acetaminophen

*C* Neuroleptic malignant syndrome (NMS) is a rare, but potentially fatal reaction commonly associated with the use of antipsychotic drugs. The classic triad for its clinical presentation includes altered mental status, hyperthermia, and muscle rigidity. The cornerstone of treatment is supportive care with rapid cooling, fluid and electrolyte repletion, and monitoring. Dantrolene, a nonspecific skeletal muscle relaxant, generally used in the treatment of malignant hyperthermia, is also effective for NMS. In addition, benzodiazepines are useful in the treatment of NMS. The offending agent should be discontinued.

A 72-year-old man presents with right eye pain for 1 day. The patient has a history of diabetes, hypertension, and "some type of eye problem." He does not recall the name of his eye problem or the name of his ophthalmic medication. However, he does remember that the eye drop has a yellow cap. Which class of ophthalmic medication is the patient taking? a. Antibiotic b. β-Blocker c. Mydriatic/cycloplegic agent d. Miotic e. Anesthetic

*C* Ophthalmic medications are color-coded. While this question may seem esoteric, it can be clinically useful to know the colors of eye medications. In this case, knowing that yellow caps are β-blockers suggests that this patient may be currently treated for glaucoma. Medication color knowledge can also help you rapidly locate a specific medication from a large group of eye medications.

A 32-year-old diabetic man steps on a nail through his running shoe. Two weeks later, he presents to the ED with fever and right foot pain. On physical examination, his heel is mildly erythematous and diffusely tender to palpation, with overlying warmth and edema. There is a small amount of purulent drainage through the puncture hole in his heel. A plain radiograph of his foot demonstrates a slight lucency of the calcaneus. He has decreased range of motion, but you are able to passively dorsiflex and plantarflex his ankle without difficulty. His vital signs include a temperature of 101.4°F, HR of 98 beats per minute, BP of 130/75 mm Hg, and RR of 16 breaths per minute. Which of the following is the most common causative organism of this condition? a. Salmonella sp. b. Pseudomonas aeruginosa c. Staphylococcus aureus d. Group B streptococci e. Pasteurella multocida

*C* Osteomyelitis is an infection or inflammation of a bone with an incidence following plantar puncture wounds of 0.1% to 2%. For patients overall, S aureus is the leading cause of osteomyelitis, followed by Streptococcus species. Pain, swelling, fever, redness, and drainage may all occur, but pain is the presenting complaint in most cases. Risk factors include trauma, surgery, soft tissue infections, and being immunocompromised (eg, HIV, diabetes, IV drug user, sickle-cell disease, alcoholism). Definitive diagnosis is made by bone scan which will demonstrate osteomyelitis within 72 hours of symptom onset. Radiographs may be normal early on, but will demonstrate periosteal elevation within 10 days. ESR is often elevated, but a normal or slightly elevated ESR does not rule out the diagnosis. The ESR is most valuable in following response to treatment, as the ESR should fall as the infection resolves. Blood cultures, which are positive in 50% of cases, should be used to guide antibiotic treatment. All patients with puncture wounds should receive tetanus prophylaxis.

A 32-year-old woman presents to the ED with a 1-month history of general malaise, mild cough, and subjective fevers. She states that she is human immunodeficiency virus (HIV) positive and her last CD4 count, 6 months ago, was 220. She is not on antiretroviral therapy or any other medications. Initial vitals include a HR of 88 beats per minute, a BP of 130/60 mm Hg, and an RR of 12 breaths per minute with an oxygen saturation of 91% on room air. Her chest radiograph shows bilateral diffuse interstitial infiltrates. Subsequent laboratory tests are unremarkable except for an elevated lactate dehydrogenase level. Given this patient's history and physical examination, which of the following is the most likely organism responsible for her clinical presentation? a. Coccidioides immitis b. Mycobacterium tuberculosis c. Pneumocystis jiroveci d. Mycoplasma pneumoniae e. Haemophilus influenzae

*C* PCP is a commonly seen opportunistic infection in the HIV/AIDS population. It typically presents with mild subjective symptoms of cough and general malaise. Objectively, patients are hypoxic and have a chest radiograph with a bilateral interstitial process. Risk factors include a CD4 count < 200. Serum lactate dehydrogenase (LDH) is also considerably higher in AIDS patients with PCP. In fact, greater the elevation in LDH, worse the prognosis. Despite the classic PCP radiograph demonstrating bilateral diffuse interstitial infiltrates, beginning in the perihilar region and extending into a "bat-wing" pattern, the chest radiograph may be normal in up to 30% of patients. In addition to Kaposi sarcoma involvement in the lungs, pulmonary infections, such as tuberculosis, cytomegalovirus, and fungal infections, should be considered.

An asymptomatic young adult was brought to the ED by a police officer after his home was raided. The patient swallowed five small packets of an unknown substance before being arrested. His BP is 125/75 mm Hg, HR is 85 beats per minute, temperature is 98.7°F, and RR is 16 breaths per minute. Physical examination is unremarkable. An abdominal radiograph confirms intraluminal small bowel densities. Which of the following is the most appropriate treatment? a. Magnesium citrate b. Gastric lavage c. Activated charcoal and polyethylene glycol d. Syrup of ipecac e. NAC

*C* Patients being arrested who swallow illegal drugs to conceal the evidence are referred to as "body stuffers." They commonly tend to ingest any and all the drugs they possess, potentially resulting in a polypharmaceutic overdose. Body stuffers are usually seen in the ED before symptoms have developed. Activated charcoal should be administered immediately and whole-bowel irrigation may be indicated. Sometimes there is radiographic evidence of the swallowed substances as seen in crack vials or staples on the packaging materials. Whole-bowel irrigation uses a polyethylene glycol electrolyte solution (eg, GoLYTELY), which is not absorbed and flushes drugs or chemicals through the GI tract. This procedure seems to be most useful when radiopaque tablets or chemicals, swallowed packets of street drugs, or sustained-released drugs have been ingested.

A 46-year-old woman presents to the ED with her husband complaining of flu-like symptoms, headache, vomiting, and dyspnea. She states that she never had similar symptoms in the past and that her husband is getting sick with similar symptoms as well but refuses to see a doctor. She reports feeling well yesterday and even helped her husband set up a home generator in their garage. Her BP is 142/85 mm Hg, HR is 97 beats per minute, temperature is 100.6°F, and RR is 20 breaths per minute. On examination the patient is slow to respond to questions. Which of the following is the most appropriate diagnostic test? a. Send blood to check the white blood cell (WBC) count. b. Order a head CT scan. c. Send blood to check the carboxyhemoglobin level. d. Perform an LP. e. No testing is necessary at this point.

*C* Patients with initial flu-like symptoms from the same household who were exposed to combustion products (ie, from a home generator) are at risk for carbon monoxide (CO) poisoning. CO binds to hemoglobin with greater affinity than oxygen and shifts the oxygen-hemoglobin dissociation curve to the left, thus decreasing oxygen release. Clinically, patients with mild CO toxicity present with flu-like symptoms, nausea, and vomiting, which progresses to chest pain, dyspnea, confusion, seizures, dysrhythmias, and coma. CO level can be obtained by a carboxyhemoglobin level from blood. CO poisoning is treated with oxygen and, if severe, with hyperbaric oxygen therapy.

A 24-year-old G3P1112 presents to the ED with vaginal bleeding and clots. Her vital signs include a HR of 110 beats per minute, BP of 130/70 mm Hg, RR of 14 breaths per minute with an oxygen saturation of 99% on room air. She is afebrile and in mild distress. Recent medical history is significant for vaginal delivery 2 days ago with prolonged labor. Pelvic examination is significant for a large, boggy uterus and a normal vaginal wall. What is the most likely diagnosis in this patient? a. Genital tract trauma b. Endometritis c. Uterine atony d. Ectopic pregnancy e. Uterine artery rupture

*C* Postpartum bleeding is classified as early, within 24 to 48 hours of delivery, or late, up to 1 to 2 weeks. Causes of early postpartum bleeding include uterine atony (most common cause), genital tract trauma, retained products of conception, and uterine inversion. Late bleeding episodes may be caused by endometritis or retained products of conception. Uterine atony is common after prolonged labor and oxytocin administration. Physical examination will reveal a soft uterus and blood in the vaginal vault. Treatment consists of bimanual massage and intravenous (IV) oxytocin to stimulate uterine contractions. Ergot alkaloids may be given in refractory cases.

A 32-year-old man is brought to the ED by EMS for confusion. EMS reports that the patient was at a local pharmacy filing his prescriptions when the pharmacist noticed the patient sweating and having difficulty answering questions. In the ED, the patient's BP is 130/68 mm Hg, HR is 120 beats per minute, temperature is 98.9°F, and RR is 12 breaths per minute. The patient is unable to explain what happened. His fingerstick glucose is 410 mg/dL and his urine is positive for ketones. An electrolyte panel reveals Na+ 131 mEq/L, K+ 4 mEq/L, Cl− 91 mEq/L, and Ca2+ 11 mEq/L. Which of the following electrolytes are most important to supplement during the management of his medical condition? a. Sodium, potassium, and calcium b. Sodium c. Potassium d. Calcium e. Sodium and calcium

*C* Potassium is the most important electrolyte to follow in DKA therapy. Renal losses and vomiting in DKA cause profound total body potassium deficit. The measured potassium levels, however, are often falsely normal or elevated because of acidosis and total body fluid deficit. Acidemia causes extracellular shift of potassium in exchange for hydrogen ions. With initiation of DKA therapy, potassium levels will quickly fall to true levels causing significant hypokalemia, if not closely monitored and replaced.

What life-threatening complication is associated with Guillain-Barre syndrome? a. Permanent paralysis b. Thrombocytopenia c. Respiratory failure d. Need for surgery e. Kidney failure

*C* Progressive paralysis in Guillain-Barré syndrome can rapidly ascend to the respiratory system and cause respiratory failure. Patients need to be monitored and provided ventilator support as necessary.

A 45-year-old woman presents to the ED with multiple facial lacerations. She was sitting by her window when the neighbor's kid hit a baseball through her window. The shattered glass caused multiple superficial lacerations. The most notable laceration is 1 cm on the right eyelid. You rule out the presence of a foreign body and give her a tetanus shot. Which of the following statements regarding eyelid trauma is correct? a. A slit-lamp examination is unnecessary since it does not help evaluate the eyelid. b. Tissue adhesives can be used to close eyelid laceration since they are usually small. c. Lacerations causing ptosis require ophthalmology consultation. d. Ptosis following an eyelid laceration is common. e. A lacrimal duct injury should be suspected in a laceration to the lateral canthus.

*C* Ptosis results from a laceration through the levator palpebrae muscle or its tendinous attachment to the tarsal plate. Repair requires reattachment of the muscle. If the emergency physician is unable to recognize and repair the muscular defect, an ophthalmology consult is warranted.

A 44-year-old woman presents to the ED with a deep puncture wound to her left forearm from a dog bite. The dog is appropriately vaccinated. Which of the following statements regarding management of animal bite wounds is true? a. Infection after dog and cat bites is rare. b. Irrigate the wound thoroughly and place deep sutures to prevent infection. c. Delayed primary closure can be used for bite wounds. d. Dog bite infections are most commonly secondary to Pasteurella multocida. e. Clindamycin is the most commonly used antibiotic for dog bites.

*C* Puncture wounds owing to their depth cannot be cleaned adequately. Delayed primary closure in these wounds decreases the risk of abscess formation and wound infection.

A 26-year-old woman presents to the ED with fever, malaise, and an evolving rash in the right axilla that she initially thought was from an insect bite that she received while hiking 1 week earlier. She complains of generalized fatigue, nausea, headache, and joint pain over the past several days. Her vitals are BP of 120/75 mm Hg, HR of 75 beats per minute, RR of 16 breaths per minute, and temperature of 101°F. On physical examination, she is awake and alert, with a nonfocal neurological examination. Her neck is supple, but she is diffusely tender over the shoulder, knee, and hip joints bilaterally without any distinct effusions. Her abdomen is soft and nontender. She has a 9-cm erythematous annular plaque with a central clearing under her right axilla. Which of the following is the next best step? a. Consult dermatology for a biopsy of the rash. b. Perform a LP and begin treatment with IV ceftriaxone. c. Prescribe doxycycline 21 days and arrange follow-up with her primary-care doctor. d. Prescribe hydrocortisone cream for the rash and acetaminophen for the headache and joint pain. e. Perform serologic testing for Borrelia burgdorferi and begin treatment only if positive.

*C* Recommended treatment for early Lyme disease includes doxycycline, amoxicillin (for children younger than 8 years or for pregnant or lactating women), or cefuroxime. Erythromycin and azithromycin can be used, but are less effective. The same drugs can be used for the second stage of disease, but their course of therapy needs to be longer.

An 18-year-old man presents to the ED after getting stabbed in his abdomen. His HR is 140 beats per minute and BP is 90/40 mm Hg. He is yelling that he is in pain. Two large-bore IVs are inserted into his antecubital fossa and fluids are running wide open. After 2 L of fluids, his BP does not improve. Which of the following is the most common organ injured in stab wounds? a. Liver b. Small bowel c. Stomach d. Colon e. Spleen

*C* Since most people are right handed and hold the offending instrument in their right hand, the LUQ is most commonly injured in stab wounds. However, the liver occupies the most space in the abdomen and therefore is the most common organ injured.

Paramedics bring a 17-year-old high school football player to the ED on a backboard and with a cervical collar. During a football game, the patient "speared" another player with his helmet and subsequently experienced severe neck pain. He denies paresthesias and is able to move all of his extremities. A cervical spine CT scan reveals multiple fractures of the first cervical vertebra. Which of the following best describes this fracture? a. Odontoid fracture b. Hangman's fracture c. Jefferson fracture d. Clay-shoveler's fracture e. Teardrop fracture

*C* Spearing—hitting another player with the crown of the helmet—generates an axial loading force that is transmitted through the occipital condyles to the superior articular surfaces of the lateral masses of the first cervical vertebra (C1). This fracture is commonly referred to as a Jefferson fracture. It is considered an unstable fracture and is associated with C2 fractures 40% of the time. On plain radiograph, it is best seen on the open-mouth odontoid view as the lateral masses are shifted laterally. It is associated with diving accidents and in this scenario, "spearing" in a football game, which places an increased axial load to the cervical spine. Proper cervical spine precautions should remain in place throughout his management in the ED.

A 43-year-old man, who currently uses drugs intravenously (IV), presents to the emergency department (ED) with 2 weeks of fever, back pain, and progressive weakness in his arms bilaterally. He reports having a cough with whitish sputum. He denies any history of recent trauma. His blood pressure (BP) is 130/75 mm Hg, heart rate (HR) is 106 beats per minute, temperature is 103°F, and respiratory rate (RR) is 16 breaths per minute. On physical examination, he has tenderness to palpation in the mid-thoracic spine, and decreased strength in the upper extremities bilaterally, with normal range of motion. Laboratory results reveal a white blood cell (WBC) count of 15,500/μL, hematocrit 40%, and platelets 225/μL. Which of the following is the most likely diagnosis? a. Lung abscess b. Ankylosing spondylitis c. Spinal epidural abscess d. Vertebral compression fracture e. Spinal metastatic lesion

*C* Spinal abscesses are most commonly found in immunocompromised patients, IV drug users, and the elderly. Signs and symptoms of epidural abscess usually develop over a week or two and include fever, localized pain, and progressive weakness. An elevated WBC count is also commonly seen. MRI is the most useful diagnostic test. Staphylococcus aureus is the most common causative organism, followed by gram-negative bacilli and tuberculosis bacillus.

A 32-year-old woman presents to the ED after an aggressive outburst at work where her behavior was deemed a threat to others. Her coworkers state that she is normally very dependable, kind, and gracious but that over the course of the week they noticed that she was especially reserved and at times found her conversing with herself. Her initial vitals include HR of 89 beats per minute, RR of 15 breaths per minute, and BP of 130/75 mm Hg with oxygen saturation of 99% on room air. She tells you that she was recently started on a new medication. Which of the following types of medications may be responsible for this patient's behavior? a. β-Blockers b. Oral contraceptives c. Corticosteroids d. Nonsteroidal anti-inflammatory drugs (NSAIDs) e. Calcium-channel blockers

*C* Steroid psychosis is described as a constellation of psychiatric symptoms within the first 5 days of treatment with a corticosteroid. Studies indicate that the amount needed to produce this effect is greater than 40 mg of prednisone, or its equivalent, prescribed daily. Symptoms include emotional lability, anxiety, distractibility, pressured speech, sensory flooding, insomnia, depression, agitation, auditory and visual hallucinations, intermittent memory impairment, mutism, disturbances of body image, and delusions and hypomania. It is important to note that previous history of psychologic difficulties does not predict the development of steroid psychosis. Symptoms can be very severe and should be taken into account when prescribing this medication to patients. Three percent of patients with steroid psychosis commit suicide.

A 19-year-old man is brought into the trauma room by EMS after a head on cycling accident. The patient was not wearing a helmet. Upon presentation his BP is 125/75 mm Hg, HR is 105 beats per minute, RR is 19 breaths per minute, and oxygen saturation is 100% on mask. His eyes are closed, but open to command. He can move his arms and legs on command. When you ask him questions, he is disoriented but able to converse. What is this patient's GCS score? a. 11 b. 12 c. 13 d. 14 e. 15

*C* The GCS score, as seen below, may be used as a tool for classifying head injury and is an objective method for following a patient's neurologic status. The GCS assesses a person's eye, verbal, and motor responsiveness. This patient received a score of 3 for eye opening to verbal command, 4 for disorientation, but conversant, and 6 for obeying verbal commands.

A 29-year-old figure skater lands incorrectly after a jump and injures her right ankle. She stands up on her own but has difficulty bearing weight so she calls EMS. She presents to the ED with swelling and tenderness of her lateral malleolus. You palpate a dorsalis pedis and posterior tibialis pulse. She is able to plantar and dorsiflex without difficulty. You are unsure if the patient requires an ankle radiograph because you think she only sustained a sprain. In addition to pain near the malleoli, which of the following is an indication for a radiograph according to the Ottawa ankle and foot rules? a. Boney tenderness is present at the anterior edge of the distal 12 cm or at the tip of either malleolus. b. The patient is able to bear weight for at least four steps immediately after the injury but not at the time of evaluation. c. Boney tenderness is present at the navicular or at the base of the fifth metatarsal. d. The Achilles reflex is diminished on the side of the injury. e. An effusion is present on clinical examination.

*C* The Ottawa rules are a prospectively validated clinical decision tree for radiograph ordering in adults. By following these rules, emergency physicians can eliminate up to 30% of radiographs that are routinely ordered without missing clinically significant fractures. Radiographs are only required if there is bony pain in the malleolar or midfoot area, and any one of the following: bony tenderness at the navicular bone or at the base of the fifth metatarsal;bony tenderness is present at the posterior edge of the distal 6 cm or at the tip of either malleolus, and the patient is unable to bear weight for at least four steps immediately after the injury and at the time of evaluation

A 31-year-old homebuilder is cutting a piece of wood when the table saw backfires and slices through the base of his left thumb. In the ED his BP is 135/70 mm Hg, HR 87, RR 18, and pulse oxygen is 99%. You stabilize the patient and call the vascular surgeon. In the meantime, what is the best method of preserving the amputated finger? a. Irrigate it with 10% povidone-iodine solution to remove gross contamination, wrap it in sterile gauze moistened with normal saline and place it between two ice packs. b. Irrigate it with normal saline to remove gross contamination, wrap it in sterile gauze moistened with normal saline and then place it on ice. c. Irrigate it with normal saline to remove gross contamination, wrap it in sterile gauze moistened with normal saline, place it in a sterile water-tight container and store this container in ice water. d. Place it in a container of 10% povidone-iodine solution and store this container in ice water. e. Attempt to partially replant the part with 3-0 nylon sutures until the vascular surgeon arrives.

*C* The best way to preserve an amputated part is to rinse it with normal saline to remove gross contamination, wrap it in sterile gauze moistened with saline or lactated ringers, and place it in a sterile, water tight container. Then store this container in ice water.

A 71-year-old woman presents to the ED with 12-hours of emesis and abdominal pain. Her temperature is 101.2°F, BP is 100/79 mm Hg, and HR is 104 beats per minute. Physical examination reveals a tender (2 × 2)-cm bulge with erythema below the inguinal ligament and abdominal distension. An occasional high-pitched bowel sound is heard. After placing an IV line and nasogastric tube, which of the following is the most appropriate course of management? a. Administer broad-spectrum antibiotics and then obtain a CT scan of abdomen b. Administer broad-spectrum antibiotics and attempt reduction c. Administer broad-spectrum antibiotics and prepare the patient for the OR d. Administer broad-spectrum antibiotics and obtain a plain radiograph e. Administer broad-spectrum antibiotics and observe

*C* The clinical scenario is highly suspicious for a strangulated loop of bowel incarcerated in a femoral hernia. If the contents of a hernia can be returned to their natural cavity by manual reduction, the hernia is termed reducible; if they cannot, it is termed irreducible or incarcerated. Incarcerated hernias are subject to inflammatory and edematous changes and are at risk for strangulation, which refers to vascular compromise of the incarcerated contents. When strangulation is not emergently relieved, necrosis and gangrene develop. The treatment for an incarcerated hernia that cannot be manually reduced is surgical fixation. If strangulation is suspected or shock is present, broad spectrum antibiotics and fluid resuscitation are necessary.

A 78-year-old man with a history of atherosclerotic heart disease and congestive heart failure presents with increasing abdominal pain. The pain began suddenly a day ago and has progressively worsened since then. He denies nausea, vomiting, and diarrhea, but states that he had black tarry stool this morning. He denies any history of prior episodes of similar pain. Vitals are BP 120/65 mm Hg, HR 105 beats per minute, temperature 99°F. The patient is at high risk for which of the following conditions? a. Cholecystitis b. Cecal volvulus c. Mesenteric ischemia d. Perforated peptic ulcer e. Small bowel obstruction

*C* The complaint of abdominal pain in the elderly patient should prompt the emergency physician to lower his or her threshold for considering more serious intra-abdominal conditions. Patients with a history of arrhythmias (especially atrial fibrillation), low cardiac output (such as congestive heart failure), or who take particular medications, such as digoxin, are at high risk for the elusive condition of mesenteric ischemia. A history of sudden onset abdominal pain with increasing severity, but with a benign physical examination should prompt consideration of this entity. Mesenteric ischemia is associated with a high mortality rate, and initial diagnosis is often incorrect. The diagnostic study of choice is angiography.

A 58-year-old woman is brought in to the ED after a witnessed syncopal event. Upon arrival, the patient appears confused and agitated. Her vitals include HR of 89 beats per minute, BP of 145/70 mm Hg, RR of 16 breaths per minute, and oxygen saturation of 98% on room air. Within a few minutes, the patient is more alert and oriented. She denies any chest pain, headache, abdominal pain, or weakness preceding the event and is currently asymptomatic. She also states that she has not taken her antiepileptic medications in 2 days. The patient's examination is unremarkable including a nonfocal neurological examination. Given this patient's history and evolving examination, what is the most likely etiology of this patient's syncopal event? a. Cerebrovascular accident b. Transient ischemic attack c. Seizure d. Aortic dissection e. Pulmonary embolus

*C* The emergency medicine physician is often faced with differentiating whether the cause of a patient losing consciousness is a result of syncope or a seizure. The most likely etiology of this patient's symptoms is in the history that she gives. She tells you that she has not taken her antiseizure medications in 2 days. Also, given her evolving mental status and improvement in alertness, this patient most likely presented in a postictal state after she has seized. Without any focal deficit and further improvement in her mental status, one might be comfortable with this diagnosis. Serum testing of her antiepileptic drug levels must be performed to further investigate this suspicion. A CT of the head, ECG, and further investigation is warranted if these levels are normal and do not explain her loss of consciousness.

A 27-year-old woman presents to the ED complaining of headache lasting approximately 1 hour in duration that is unrelieved by aspirin and acetaminophen. She states the headache was not preceded by any visual phenomena, is left sided, pulsatile, and has occurred nearly monthly coinciding with her menstrual period for the past 6 months. She also complains of nausea and sensitivity to sound and light. Which of the following is the most appropriate therapy for this patient at the time of presentation? a. IV morphine sulfate b. Another trial of aspirin and acetaminophen c. Sumatriptan d. Topiramate e. Hydromorphone and acetaminophen

*C* The headache described is a menstrual migraine, a common variant of the migraine headache syndrome. Appropriate abortive therapies (this headache is just starting) are diverse and include IV ergot, triptans, and antiemetics. Sumatriptan (Imitrex) is a triptan which acts by blocking the 5-hydroxytryptamine (5-HT, serotonin) 1D receptor. It also has less associated nausea and vomiting than ergots. It may have a higher incidence of minor side effects (flushing, injection site reaction) and a higher relapse rate than ergots. Contraindications to triptans or ergots include pregnancy, hypertension, coronary artery disease, or use of either class of agent within the last 24 hours.

A 28-year-old male military recruit presents to the ED complaining of headache, fever, and neck stiffness. His temperature is 102.2°F and he refuses to move his neck. He is somewhat lethargic appearing and winces when the lights are turned on in the examining room. He has a nonfocal neurologic examination and you proceed with LP. Which of the following is the most specific finding for suspected bacterial meningitis? a. The presence of phonophobia, photophobia, and neck stiffness b. Fever higher than 102.2°F c. Elevated polymorphonuclear white blood cell (WBC) count on CSF analysis d. Elevated protein on CSF analysis e. Increased glucose on CSF analysis

*C* The normal number of WBCs in the CSF is 5 or fewer with 1 or less polymorphonuclear neutrophils (PMN). Numbers greater than these should be taken as evidence for CNS infection. In cases of acute bacterial meningitis, cell counts of 1000/μL to 20,000/μL WBCs are observed, often with neutrophil predominance. In cases of aseptic (viral) meningitis, cell counts are generally lower with lymphocyte predominance. Initial treatment with antibiotics prior to LP is unlikely to affect the cell count, after 6 hours though, the culture is less likely to return positive. It should be noted that a subset of patients with bacterial meningitis may present with lymphocytic predominance. Therefore, lymphocytic predominance does not rule out bacterial meningitis and antibiotics should be given.

After being fired from his job, a 35-year-old man attempts suicide by drinking from a bottle labeled "insecticide." Three hours later, emergency medical services (EMS) brings him in to the emergency department (ED) and you notice that he is extremely diaphoretic, drooling, and vomiting. He is awake, but confused. His vital signs include a blood pressure (BP) of 170/90 mm Hg, heart rate (HR) of 100 beats per minute, respiratory rate (RR) of 22 breaths per minute, temperature of 98.6°F, and oxygen saturation of 95% on room air. Physical examination demonstrates pinpoint pupils and crackles on lung examination. What is the treatment to reverse this patient's poisoning? a. Naloxone b. N-acetylcysteine (NAC) c. Atropine and pralidoxime (2-PAM) d. Flumazenil e. Physostigmine

*C* The patient drank insecticide, which is primarily composed of organophosphate compounds (eg, Malathion). These compounds inhibit acetylcholinesterase, the enzyme responsible for the breakdown of acetylcholine. The patient is having a "cholinergic crisis." Overstimulation of muscarinic and nicotinic receptors leads to his symptoms, commonly remembered by the mnemonics SLUDGE (salivation, lacrimation, urination, defecation, gastrointestinal [GI] upset, emesis) or DUMBBELS (defecation, urination, miosis, bronchospasm, bronchorrhea, emesis, lacrimation, salivation). The treatment for organophosphate toxicity is atropine and pralidoxime (2-PAM). Atropine is an anticholinergic, therefore it competitively inhibits the excess acetylcholine. Pralidoxime works to regenerate acetylcholinesterase, therefore also limiting the amount of acetylcholine available in neuronal synapses.

A 22-year-old woman presents to the ED complaining of headache. She states that while at home she experienced a headache that was associated with blurry vision in both eyes with a shimmering line in her vision. She subsequently lost her vision and felt uncoordinated, followed by increased pain at the base of her skull. Upon arrival in the ED, her vision returned to normal. A head CT scan and an LP are both negative. She now complains of a persistent, severe, pulsatile headache. She has had two similar episodes in the past year with the headache refractory to over-the-counter medications. Which of the following is likely to relieve her symptoms? a. Diazepam b. High flow oxygen c. Sumatriptan d. Acetaminophen e. LP with removal of 15 cc cerebrospinal fluid (CSF)

*C* The patient is experiencing a migraine variant known as a basilar migraine. Its onset is similar to other migraines in that it can begin with scotomata or aura. The visual symptoms are often bilateral and followed by a brief period of cortical blindness. Symptoms related to the basilar circulation then predominate including incoordination, dysarthria, vertigo, and numbness and tingling in the arms or legs. These symptoms generally last 10 to 30 minutes then resolve. Occasionally, transient coma or quadriplegia can develop but persist for only several hours. The resulting headache is occipital and pulsating. The symptoms may mimic a vertebrobasilar ischemic event. Treatment with first line agents for migraine is recommended. Threshold should be low to seek neurologic consultation unless the diagnosis is certain

A 24-year-old woman presents to the emergency department (ED) complaining of right eye pain and blurry vision since waking up this morning. She states that the pain began after taking out contact lenses that were in her eyes for over 1 week. Her blood pressure (BP) is 120/75 mm Hg, heart rate (HR) is 75 beats per minute, temperature is 99.1°F, and respiratory rate (RR) is 16 breaths per minute. Her right and left eye visual acuity is 20/60 and 20/20, respectively. Her conjunctivae are injected. The slitlamp examination reveals a large area of fluorescein uptake over the visual axis. Which of the following is the most appropriate therapy? a. Call the ophthalmology consult for an emergent corneal transplant. b. Prescribe a systemic analgesic for pain control and advise the patient to not wear her contact lenses for the next week. c. Prescribe ciprofloxacin eye drops, oral analgesia, update tetanus prophylaxis, and arrange for ophthalmology follow-up. d. Prescribe oral amoxicillin, a topical anesthetic, such as tetracaine and have patient follow-up with an ophthalmologist. e. Prescribe ciprofloxacin eye drops and have patient strictly wear an eye patch until her pain resolves.

*C* The patient most likely sustained a corneal abrasion from prolonged contact lens use. Epithelial defects of the cornea are diagnosed by slit-lamp examination by observing fluorescein uptake in the area of the defect. Treatment usually consists of cycloplegia and topical or oral pain medications. Contact lens patients should be treated with topical antibiotics with antipseudomonal coverage. Patients should not wear their contact lens until complete healing of the corneal epithelium. All patients should follow-up with an ophthalmologist.

A 69-year-old woman with a past medical history of hypertension, hypercholesterolemia, diabetes mellitus type 1, and alcohol abuse is brought to the emergency department (ED) by her daughter who states that her mom has been acting funny over the last hour. She states that the patient did not know where she was despite being in her own house. She also did not recognize her family and was speaking incomprehensibly. Her blood pressure (BP) is 150/80 mm Hg, heart rate (HR) is 90 beats per minute, temperature is 98.9°F, and her respiratory rate (RR) is 16 breaths per minute. On physical examination she is diaphoretic, agitated, and tremulous. Electrocardiogram (ECG) is sinus rhythm with normal ST segments and T waves. Which of the following is the most appropriate course of action for this patient? a. Administer a benzodiazepine to treat her ethanol withdrawal. b. Activate the stroke team and bring the patient directly to the computed tomographic (CT) scanner. c. Get a stat fingerstick and administer dextrose if her blood sugar is low. d. Request a psychiatric consult for probable sundowning. e. Administer haloperidol for sedation.

*C* The patient never received a fingerstick glucose at triage. Hypoglycemia can mimic a cerebrovascular accident or seizure. Therefore, it is critical that all patients who present with altered mental status get fingerstick glucose. Glucose level should be considered a vital sign. Hypoglycemia is a common problem in patients with type 1 diabetes. The clinical presentation of hypoglycemia is caused by increased secretion of epinephrine, as well as central nervous system (CNS) dysfunction. Symptoms include diaphoresis, nervousness, tremor, tachycardia, hunger, and neurologic symptoms ranging from confusion and bizarre behavior to seizures and coma.

A 25-year-old man is carried into the ED by two of his friends who state that he is not breathing. The patient has a history of heroin abuse. His vital signs are BP 115/70 mm Hg, HR 99 beats per minute, temperature 98.9°F, RR 3 breaths per minute, and oxygen saturation 87% on room air. You notice fresh needle marks and miotic pupils. You begin bag-valve-mask ventilation and his oxygen saturation increases to 99%. Which of the following is the most appropriate next step in management? a. Continue bag-valve-mask ventilation until he breathes on his own b. Endotracheal intubation of the patient c. Administration of naloxone d. Administration of flumazenil e. Place a nasogastric tube and administer activated charcoal

*C* The patient presents to the ED with central nervous system (CNS) and respiratory depression and miotic pupils. Along with his history of heroin abuse and fresh needle marks, this is most likely a heroin overdose. Opioid toxicity is associated with the toxidrome of CNS depression, respiratory depression, and miosis. Attention is always first directed at airway management in emergency medicine. The first action for this patient is to provide oxygen via bagvalve-mask ventilation. Because his respiratory depression is most likely secondary to opioid overdose, an opioid antagonist should be administered. Naloxone is the antidote most frequently used to reverse opioid toxicity. The goal of naloxone therapy is not necessarily complete arousal; rather, it is to reinstitute adequate spontaneous respiration, while attempting to avoid inducing acute opioid withdrawal.

A 78-year-old woman is transferred from a nursing home with altered mental status and fever. The nursing home reports that the patient was febrile to 102.3°F, disoriented, confused, and incontinent of urine. Her past medical history includes hypertension, a stroke with residual right-sided weakness, and nighttime agitation for which she was started on haloperidol 3 days ago. Her BP is 215/105 mm Hg, HR is 132 beats per minute, temperature is 102.8°F, and RR is 20 breaths per minute. On examination, the patient is oriented to name only, tremulous, diaphoretic and has marked muscular rigidity and three out of five right upper and lower extremity strength. What is the most likely diagnosis? a. Urinary tract infection b. Malignant hyperthermia c. Neuroleptic malignant syndrome d. Recurrent stroke e. Meningoencephalitis

*C* The patient presents with a rare but potentially life-threatening NMS. Antipsychotic drugs (eg, haloperidol) are the most common offending agents in the development of NMS, causing central dopamine depletion. The disorder is typically characterized by hyperthermia, muscle rigidity, altered mental status, and autonomic instability. Since NMS carries a high mortality, it is important to aggressively treat it with muscle relaxers, such as IV benzodiazepines, dantrolene, and dopamine agonists.

A 76-year-old woman is brought into the ED complaining of hip pain after a motor vehicle collision. She states that her knee slammed into the dashboard after her car struck another vehicle. On examination you note her right leg appears shortened, adducted, internally rotated, and flexed. She has no other injuries. The radiograph of her hip confirms your diagnosis of a hip dislocation. Which of the following statements regarding this type of injury is true? a. The most common presentation of this injury is an abducted, externally rotated, and flexed lower extremity. b. This type of injury accounts for 5% to 10% of all hip dislocations. c. The injury is commonly caused by a direct force applied to a flexed knee. d. The most common complication is injury to the femoral artery. e. The femoral nerve is almost always injured in this type of dislocation.

*C* The patient sustained a posterior hip dislocation. The most common mechanism for this to occur is during a motor vehicle collision in which a direct force is applied to the flexed knee (eg, the patient's knee hits the dashboard at high velocity). The limb appears shortened, adducted, internally rotated, and flexed. Because of the strong forces needed to cause such an injury, hip dislocations rarely occur in isolation. Life threatening injuries should always be addressed first, prior to treating hip dislocations. Nonetheless, early reduction is required since the likelihood of avascular necrosis of the femoral head increases in direct proportion to a delay in reduction. Early complications of posterior dislocations include sciatic nerve injury. Late complications include avascular necrosis of the femoral head, as mentioned above.

A 50-year-old man presents to the ED complaining of abrupt onset of epigastric pain that radiates to his back. He describes the pain as constant and associated with nausea and vomiting. The pain improves mildly if he leans forward. He has a 20-pack-year smoking history and has consumed 6 packs of beer daily for more than 5 years. His BP is 150/80 mm Hg, HR is 98 beats per minute, temperature is 100.1°F, and his RR is 18 breaths per minute. He is tender to palpation in the epigastric area. A chest x-ray reveals a small left-sided pleural effusion. His WBC count is 12,000/μL, hematocrit 39%, plasma glucose 225 mg/dL, AST and ALT are within normal limits, alkaline phosphatase 96 U/L, and lipase 520 U/L. His CT scan is seen below. What is the most likely diagnosis? a. Abdominal aortic aneurysm b. Mesenteric ischemia c. Pancreatitis d. Bowel perforation e. Cholecystitis

*C* The patient's clinical picture is consistent with acute pancreatitis, an inflammation and self-destruction of the pancreas by its digestive enzymes. There are many risk factors for pancreatitis, the most common being gallstones and alcohol, which account for more than 80% of the cases. Pancreatitis can be divided into mild and severe defined by the presence of organ failure or local complications, such as necrosis, pseudocyst, or abscess. It should be suspected in all patients with epigastric pain. Elevation in lipase, a pancreatic enzyme, is used to make the diagnosis of pancreatitis. At five times the upper limit of normal, the specificity of lipase approaches 100% for pancreatitis.

A 65-year-old man with a history of chronic hypertension presents to the ED with sudden-onset tearing chest pain that radiates to his jaw. His BP is 205/110 mm Hg, HR is 90 beats per minute, RR is 20 breaths per minute, and oxygen saturation is 97% on room air. He appears apprehensive. On cardiac examination you hear a diastolic murmur at the right sternal border. A chest x-ray reveals a widened mediastinum. Which of the following is the preferred study of choice to diagnose this patient's condition? a. Electrocardiogram (ECG) b. Transthoracic echocardiography (TTE) c. Transesophageal echocardiography (TEE) d. Computed tomography (CT) scan e. Magnetic resonance imaging (MRI)

*C* The patient's clinical picture of chronic hypertension, acute onset tearing chest pain, diastolic murmur of aortic insufficiency, and chest x-ray with a widened mediastinum is consistent with an aortic dissection. The preferred study of choice is a transesophageal echocardiogram (TEE), which is highly sensitive. It can be quickly performed at the bedside and does not require radiation or contrast. (b) A TTE is limited in diagnosing aortic dissections because wave transmission is hindered by the overlying sternum. It can be useful to see pericardial fluid as a result of proximal dissection.

A 35-year-old agitated man presents to the ED in police custody. He denies any past medical history and takes no medication. He admits to using some drugs today. His BP is 195/90 mm Hg, HR is 121 beats per minute, temperature is 100.1°F, RR is 18 breaths per minute, and oxygen saturation is 99% on room air. On examination, he is diaphoretic, and has pupils that are 8 mm in diameter, along with 3+ patella reflexes bilaterally. Electrocardiogram (ECG) reveals sinus tachycardia with a rate of 123. Which of the following toxic syndromes is this patient exhibiting? a. Anticholinergic b. Cholinergic c. Sympathomimetic d. Opioid e. Serotonin

*C* The sympathomimetic syndrome usually is seen after acute abuse of cocaine, amphetamines, or decongestants. Patients are usually hypertensive and tachycardic and exhibit mydriatic pupils. In massive overdoses, cardiovascular collapse can result in shock and wide-complex dysrhythmias. CNS effects include seizures. Sympathomimetic syndrome is sometimes difficult to distinguish from anticholinergic syndrome. The difference is that patients usually present with dry mucous membranes with an anticholinergic overdose, whereas patients are diaphoretic with sympathomimetics.

An 81-year-old woman is brought to the ED by her children who state that the patient is acting more tired than usual, has had fever for the last 2 days, and is more confused. Ordinarily, the patient is high functioning: she is ambulatory, cooks for herself, and walks on a treadmill 30 minutes a day. Her vital signs are BP 85/60 mm Hg, HR 125, RR 20, temperature 101.3°F, and pulse oxygenation 97% on room air. On examination, the patient has dry mucous membranes, but is otherwise unremarkable. She is oriented to person and place, but states that the year is 1925. Her laboratory results show a WBC 14,300/μL, hematocrit 31%, and platelets 350/μL. Her electrolytes are within normal limits. Blood glucose is 92 mg/dL. A chest radiograph does not show any infiltrates. Urinalysis reveals 2+ protein, trace ketones, WBC > 100/hpf, RBC 5 to 10/hpf, nitrite positive, and leukocyte esterase positive. After administering a 500 cc normal saline fluid bolus and broad-spectrum antibiotics through her peripheral IV line, the patient's BP is 82/60 mm Hg. You suspect that the patient is in septic shock. Which of the following is the next most appropriate course of action to manage this patient with early-goal-directed therapy? a. Start vasopressor therapy; repeat BP, if below a systolic of 90 mm Hg, increase the dose b. Check the hematocrit and if it is less than 30% prepare to transfuse packed red blood cells (RBC) c. Place a central venous line into the right internal jugular vein, measure a central venous pressure (CVP), administer normal saline boluses if the CVP is less than 8 mm Hg d. Place a central venous line into the left subclavian vein, measure a CVP, administer normal saline boluses if the CVP is less than 12 mm Hg e. Place a central venous line into the right femoral vein, measure a CVP, administer normal saline boluses if the CVP is less than 8 mm Hg

*C* The treatment for sepsis has evolved considerably over the past 10 years. Early recognition and early aggressive therapy for patients with sepsis have a significant impact on mortality.

A 21-year-old man presents to the ED. He has a known history of type 1 diabetes. He is hypotensive with BP of 95/65 mm Hg, tachycardic at 120 beats per minute, and tachypneic at 30 breaths per minute. Laboratory results reveal a WBC 20,000/μL, hematocrit 45%, platelets 225/μL, sodium 131 mEq/L, potassium 5.3 mEq/L, chloride 95 mEq/L, bicarbonate 5 mEq/L, BUN 20 mg/dL, creatinine 0.9 mg/dL, and glucose 425 mg/dL. Arterial blood gas reveals a pH of 7.2. Urinalysis reveals glucosuria and ketosis. There is a fruity odor to his breath. Which of the following provides the strongest evidence for the diagnosis? a. Hypotension, tachycardia, and tachypnea b. Glucose of 425 mg/dL, ketosis, and leukocytosis c. Glucose of 425 mg/dL, ketosis, pH 7.2, and bicarbonate of 5 mEq/L d. Glucose of 425 mg/dL, hypotension, and fruity odor to breath e. Glucosuria, hypotension, and leukocytosis

*C* The triad of hyperglycemia, ketosis, and acidosis is diagnostic for DKA. All abnormalities in DKA are connected and are based on insulin deficiency. When hyperglycemia surpasses the renal threshold for resorption, glucose is excreted in the urine. This causes an osmotic diuresis which in combination with decreased oral intake and vomiting leads to dehydration and electrolyte abnormalities. Cells, unable to receive glucose from the circulation, switch to starvation mode by increasing proteolysis. The liver starts producing ketoacid subsequently causing acidemia. The acidotic patient increases RR, in an attempt to blow-off excess carbon dioxide, and bicarbonate is used up in the process.

A 9-day-old boy is brought to the ED for fever and fussiness. He was born full-term. However, the delivery was complicated by prolonged rupture of membranes (PROM) and the mother had a fever for which she was treated with antibiotics prior to delivery. The baby did well in the nursery and has been at home and feeding without any difficulties until the day of presentation when he became more fussy, less interested in feeding, and the parents noted a temperature of 101.5°F (measured under patient's arm). On presentation, his temperature is 102.4°F, HR 160 beats per minute, RR 48 breaths per minute, and pulse oxymetry of 97% on room air. His anterior fontanelle is open and flat, conjunctivae are clear, neck is supple without masses, and the heart, lungs, and abdominal examinations are normal. The skin shows tiny (1-3 mm) pustules with a surrounding rim of erythema on the patient's trunk. No vesicular lesions are noted. The most likely organism responsible for this patient's condition is: a. Escherichia coli b. Listeria monocytogenes c. Group B streptococcus d. Staphylococcus aureus e. Herpes simplex virus

*C* This is a case of newborn sepsis. Neonatal sepsis is defined as an invasive bacterial infection occurring during the first 90 days of life. The most common cause of newborn sepsis is group B β-hemolytic Streptococcus (GBBS). Approximately a third of women are carriers of this organism in their vagina and exposure is through the birth canal. There is increased risk with PROM that is thought to result in ascending infection. Increased colony counts results in increased risk for neonatal sepsis and thus the tendency to screen pregnant women and to treat at the time of delivery for either screen positive women or women with fever or PROM. Any infant under 4 weeks with fever greater than 100.3°F (and most would say any infant under 8 weeks) requires a full workup to search for the source of infection. This workup includes a CBC, blood culture, urinalysis and urine culture, and a lumbar puncture for cell counts and culture. Many people recommend sending the CSF for herpes simplex virus (HSV) culture and polymerase chain reaction, particularly if there are any maternal risk factors or suspicious skin lesions, or in any infant who is very ill.

A 5-month-old previously healthy girl presents to the ED with a 4- to 5-day history of constipation and decreased feeding. The patient is a full term product of an uncomplicated antenatal course and delivery. The patient also had recent congestion and cough that resolved with administration of tea prepared by a family friend. The mother denies any recent fever or vomiting. On examination, the temperature is 99.9°F, HR 165 beats per minute, and RR 22 breaths per minute. The BP was not obtained. The patient has a weak cry, is notably flaccid and ill-appearing. You note that patient is drooling. Her pupils are poorly responsive and she is not tracking to light or faces. Which of the following is the most likely cause of this condition? a. Type I spinal muscular atrophy b. Brain tumor c. Infant botulism d. Meningitis e. Organophosphate poisoning

*C* This is a classic case of infant botulism characterized by generalized weakness, pupillary unresponsiveness, and hypoventilation. Patients often present with an initial complaint of constipation. They are noted to have a particularly weak cry. The condition is caused by release of the botulinum toxin from Clostridium botulinum. The toxin prevents release of acetylcholine from the neuromuscular junction. Older children present with complaints of diplopia, dysarthria, and dysphagia. Young infants are particularly susceptible from ingestion of honey, which can harbor C botulinum spores. The pH in infant's stomachs does not destroy the spores. Therefore, honey is not recommended for any infant under 1 year of age.

A 15-month-old girl is brought to the ED by EMS secondary to seizure activity noted at home. The patient is a previously well child. Her immunizations are up to date. The patient is reported to have had a fever for the last 2 to 3 days. She has had no cough, congestion, vomiting, diarrhea, or rash. Her urine output is appropriate. After defervescence she seems to act appropriately and feed well. This afternoon, the patient was being observed in the playroom when her eyes rolled back and she began having generalized tonic-clonic activity that lasted for approximately 2 minutes. When EMS arrived, the patient was in her mother's arms, tired but rousable, and in no apparent distress. On examination, the temperature is 103°F, HR 155 beats per minute, RR 32 breaths per minute, and BP 85/50 mm Hg. She has red tympanic membranes with no fluid and good mobility, oropharynx clear, lungs and heart examinations are normal, and abdomen is soft and nontender. Skin examination is clear with brisk capillary refill. Over the course of your evaluation, the patient becomes increasingly interactive, well appearing, and playful. Which of the following is the most appropriate course of action for this patient? a. Obtain complete blood count (CBC), blood culture, urinalysis, urine culture chest radiograph, and determine treatment on the basis of the results of these tests. b. Obtain blood and urine for culture, administer ceftriaxone, and discharge home. c. Obtain catheterized urine specimen for analysis and treat as outpatient if urinalysis is suspicious for infection. d. Obtain routine blood work and head computed tomographic (CT) scan and call for neurology consultation for first-time seizure. e. Obtain head CT scan and perform lumbar puncture secondary to fever and seizure to rule out meningitis.

*C* This patient had a simple febrile seizure. Approximately 2% to 5% of children will experience a febrile seizure during their lifetimes. Febrile seizures occur between the ages of 3 months to 5 years, are associated with fever, and are categorized as either simple or complex. Simple febrile seizures are generalized and last less than 15 minutes. Complex febrile seizures are prolonged, recur within 24 hours, or are focal. The approach to a patient with a febrile seizure is, for all intents and purposes, identical to that of the same patient who has not had a seizure, so long as the seizure has stopped by the time you are evaluating the patient. If the seizure continues, attend to it as you would any seizure with indicated medications and attention to ABCs. Once the patient is stable, it is important to identify the source of the fever. This female patient, having a temperature of 103°F for 2 days and having no other source of infection mandates that her urine be checked to evaluate for urinary tract infection (UTI).

A 60-year-old man with a history of hypertension and migraine headaches presents to the ED with a headache. He describes left-sided headache and eye pain that is associated with nausea and vomiting. The patient has a long history of migraines, but says his migraines do not usually include eye pain. On examination, his temperature is 97.6°F, HR 84 beats per minute, RR 12 breaths per minute, and BP 134/80 mm Hg. His neurologic examination is normal. His left eye is mid-dilated and nonreactive. His cornea is cloudy. His corrected visual acuity is 20/50 in the left eye and 20/20 in the right eye. What is the most appropriate next step in management? a. Administer hydromorphone b. Head CT scan c. Check IOP d. Check erythrocyte sedimentation rate (ESR) e. Discharge patient

*C* This patient has acute angle-closure glaucoma, when the aqueous humor production in the posterior chamber of the eye is unable to drain through the anterior chamber and the resultant obstruction causes increased IOP. On examination, patients with increased IOP have a mid-dilated, nonreactive pupil with corneal clouding and decreased vision. The diagnosis is clenched by checking the IOP with a tonopen or tonometer. Normal IOP is 10 to 21 mm Hg. Patients with increased IOP should be treated with medications to decrease production of aqueous humor. Checking IOP is a simple, rapid test that should be performed on all patients with headache and orbital pain. If this test is negative, other causes of headache and eye pain should be investigated.

You are examining the pupils of a patient. On inspection, the pupils are 3 mm and equal bilaterally. You shine a flashlight into the right pupil and both pupils constrict to 1 mm. You then shine the flashlight into the left pupil and both pupils slightly dilate. What is this condition called? a. Anisocoria b. Argyll Robertson pupil c. Afferent pupillary defect d. Horner syndrome e. Normal pupil reaction

*C* This patient has an afferent pupillary defect (APD), also known as a Marcus Gunn pupil. In patients with an APD, light shined into the affected pupil causes a small dilation with no constriction. APD is a result of a lesion in the anterior visual pathway of the retina, optic nerve, or optic chiasm preventing reception of the light in the affected eye. Neither pupil constricts since constriction is centrally mediated in the midbrain. APDs are sensitive for disease, but not specific. The differential diagnosis for an APD includes central retinal artery or vein occlusion, optic nerve disorders, such as optic neuritis, tumor or glaucoma, and lesions in the optic chiasm or tract.

A 32-year-old male construction worker reports standing on scaffolding before it suddenly gave way beneath him. In an attempt to catch his fall, he spontaneously grabbed and hung on to an overhead beam for approximately 30 seconds. Two hours later in the ED, the patient presents with right shoulder pain and decreased range of motion. He denies any loss of consciousness or other sustained trauma. Vital signs are within normal limits, except for the patient's pain scale of 10/10. He is holding his right arm with the contralateral hand. On physical examination, the patient's shoulder appears swollen with no skin breakage. The upper extremity is without obvious deformity. The patient has palpable brachial, radial, and ulnar pulses with capillary refill that is less than 2 seconds. He can wiggle his fingers, but cannot internally rotate his shoulder or raise his right arm above his head. Pinprick testing reveals paresthesias along the lateral deltoid of the affected arm. What is the most likely etiology of this patient's pain and paresthesias? a. Acromioclavicular joint sprain b. Posterior shoulder dislocation with axillary nerve impingement c. Anterior shoulder dislocation with axillary nerve impingement d. Anterior shoulder dislocation with median nerve impingement e. Posterior shoulder dislocation with ulnar nerve impingement

*C* This patient has an anterior shoulder dislocation. The glenohumeral joint is the most commonly dislocated joint in the body, mainly because of the lack of bony stability and its wide range of motion. Anterior dislocations account for 95% to 97% of cases and are most commonly seen in younger, athletic males and geriatric females. It usually happens by way of an indirect force that involves an abduction plus extension plus external rotation injury. Directly, it may occur as a result of a posterior blow that forces the humeral head out of the glenoid rim anteriorly. Radiographs obtained must include an axillary view to determine positioning of the humeral head. Patients usually present in severe pain, holding the affected arm with the contralateral hand in slight abduction. The lateral acromial process is prominent giving the shoulder a full or squared-off appearance. Patients typically cannot internally rotate their shoulder. Axillary nerve injuries can occur in up to 54% of anterior dislocations; however, these are neuropraxic in nature and tend to resolve on their own. Following the C5/C6 dermatome distribution, patients have a loss of sensation over the lateral aspect of the deltoid with decreased muscle contraction with abduction. After proper muscle relaxation with conscious sedation or intra-articular injection, closed reduction may be attempted using a variety of methods. After reduction, it is imperative to repeat a neurovascular examination and obtain confirmatory radiographs.

A 6-month-old boy is brought to the ED after being found apneic and cyanotic at home. The patient's mother called 911 and began cardiopulmonary resuscitation (CPR). The patient responded within seconds to minutes. On arrival to the ED, the patient was noted to be awake and responsive, but slightly mottled with mild respiratory distress. Within minutes of arrival patient becomes apneic suddenly, cyanotic, and bradycardic. Which of the following is the most important initial response? a. Administer epinephrine b. Provide oxygen via non-rebreather face mask c. Jaw thrust, chin lift, and bag-valve-mask ventilation d. Endotracheal intubation e. Chest compressions

*C* This patient has become apneic and requires assisted ventilation. The correct approach is to do a jaw thrust and chin lift in order to provide optimal airway positioning and begin bag-valve-mask ventilation. This is the most important skill to learn in pediatric resuscitation. In many cases, providing oxygen appropriately is all that is needed to resolve the bradycardia and circulatory issues. The most common cause of arrest in children is respiratory. This is a classic case of respiratory arrest causing cyanosis and subsequent bradycardia.

A 35-year-old woman presents with a right-sided red eye for 3 days. She denies pain and notes that she has watery discharge from the eye. She has been coughing and congested for the past 5 days. On examination, the patient has a temperature of 98.4°F, HR of 72 beats per minute, BP of 110/70 mm Hg, and RR of 14 breaths per minute. Her visual acuity is 20/20. On inspection, the conjunctiva is erythematous with minimal chemosis and clear discharge. The slit-lamp, fluorescein, and funduscopic examinations are otherwise unremarkable. The patient has a nontender, preauricular lymph node and enlarged tonsils, without exudates. What is the most likely diagnosis? a. Gonococcal conjunctivitis b. Bacterial conjunctivitis c. Viral conjunctivitis d. Allergic conjunctivitis e. Pseudomonal conjunctivitis

*C* This patient has classic viral conjunctivitis that is associated with a viral upper respiratory infection. Patients with viral conjunctivitis typically have reddened conjunctiva and watery discharge. Preauricular adenopathy is also associated with viral etiology. Treatment includes cool compresses and an antihistamine/α-adrenergic combination medication—naphazoline/pheniramine—for symptomatic care.

A 12-year-old girl presents to the ED for left eye pain and swelling for 2 days. The patient has had cough, congestion, and rhinorrhea for the last week that is improving. On examination, her temperature is 100.8°F, HR 115 beats per minute, RR 12 breaths per minute, and BP 110/70 mm Hg. On eye examination, there is purple-red swelling of both upper and lower eyelids with injection of the conjunctiva. Pupils are equal and reactive to light. There is restricted lateral gaze. Visual acuity is 20/70 in the left eye and 20/25 in the right eye. The rest of the physical examination is normal. What is the most appropriate next step in management? a. Administer diphenhydramine b. Administer amoxicillin/clavulanate c. Administer vancomycin IV d. Computed tomographic (CT) scan of orbits and sinuses e. Administer artificial tears

*C* This patient has orbital cellulitis, an infection deep to the orbital septum. The patient had a recent upper respiratory infection and sinusitis which likely resulted in orbital extension of the infection. Staphylococcus aureus and Haemophilus influenzae are common etiologies, and mucormycosis must be considered in diabetics and immunocompromised patients. Distinctive clinical findings of orbital cellulitis are eye pain, fever, impaired eye motility, decreased visual acuity, and proptosis. Patients should be treated with IV antibiotics, such as cefuroxime, a combination of penicillin and nafcillin, or vancomycin and admitted to the hospital. In this case, the diagnosis is clear from the history and physical examination and treatment should be started promptly. Orbital cellulitis must be differentiated from preseptal cellulitis and allergic reactions. Preseptal cellulitis is a superficial cellulitis that does not penetrate the orbital septum. Patients present with swollen, red eyelids, but no vision change, pupil changes, or changes in eye motility.

A 23-year-old avid high altitude skier presents to the ED with intense pain, tearing and bilateral ocular foreign body sensation. He denies any trauma, past medical problems or contact lens use. His physical examination is significant for bilateral decreased visual acuity, injected conjunctiva, and diffuse punctuate corneal lesions with a discrete inferior border. His pupils are equal, round, and reactive to light. Given this patient's history and physical examination, which of the following is the most likely diagnosis? a. Corneal abrasion b. Iritis c. Ultraviolet keratitis d. Corneal foreign body e. Corneal laceration

*C* This patient has ultraviolet keratitis, also known as snow blindness. This is essentially a radiation burn when an individual comes in close contact with an ultraviolet-ray—containing light source. It may be caused by sun lamps, tanning booths, or high-altitude environments. Patients usually present about 6 to 10 hours postexposure complaining of eye pain, blepharospasm, tearing, photophobia, and foreign body sensation. Physical examination reveals an injected eye with decreased visual acuity. Corneal examination reveals punctuate lesions that are clearly demarcated by a protective covering from the ultraviolet rays, such as the inferior conjunctiva. Treatment consists of a short-acting cycloplegic agent for pain management and broad-spectrum ophthalmologic antibiotics. Treatment is guided to assist in the quick, natural healing capacity of the cornea, in which the patient should feel better in a couple of days.

A 24-year-old man is brought into the ED by paramedics after being run over by a car. His systolic BP is 90 mm Hg by palpation, HR is 121 beats per minute, RR is 28 breaths per minute, and oxygen saturation is 100% on non-rebreather. The airway is patent and breath sounds are equal bilaterally. You establish large-bore access and fluids are running wide open. Secondary survey reveals an unstable pelvis upon movement with lateral to medial force. Bedside focused assessment by sonography for trauma (FAST) is negative for intraperitoneal fluid. Which of the following is the most appropriate immediate next step in management? a. Bilateral chest tubes b. Application of external fixator c. Application of pelvic binding apparatus d. Venographic embolization e. Angiographic embolization

*C* This patient is hemodynamically unstable with a pelvic fracture. The retroperitoneum can accommodate up to 4 L of blood after severe pelvic trauma. A few options are useful in managing hemorrhage from an unstable pelvic fracture. However, the initial and simplest modality to use in a patient in shock from a pelvis fracture is placement of a pelvic binding garment. This device can be applied easily and rapidly and is typically effective in tamponading bleeding and stabilizing the pelvis.

A 79-year-old woman with a history of coronary artery disease who underwent a coronary artery bypass graft (CABG) surgery in 2000 is brought to the emergency department (ED) by her family for 2 days of worsening shortness of breath. For the past 2 days, she has not gotten out of bed and is confused. She does not have chest pain, fevers, or cough. Her temperature is 98.1°F, blood pressure (BP) is 85/50 mm Hg, heart rate (HR) is 125 beats per minute, and respiratory rate (RR) is 26 breaths per minute. On examination, she is unable to follow commands and is oriented only to name. The cardiovascular examination reveals tachycardia with no murmurs. Her lungs have rales bilaterally at the bases. The abdomen is soft, nontender, and nondistended. Lower extremities have 2+ edema to the knee bilaterally. Which of the following is the most likely diagnosis? a. Hypovolemic shock b. Neurogenic shock c. Cardiogenic shock d. Anaphylactic shock e. Septic shock

*C* This patient is in cardiogenic shock from decreased cardiac output producing inadequate tissue perfusion. Support for this diagnosis includes an older patient with a history of coronary artery disease, and new mental status changes coupled with signs of volume overload. Common causes of cardiogenic shock include acute MI, pulmonary embolism, COPD exacerbation, and pneumonia. This patient should be stabilized with IV pressors since there is already pulmonary congestion evident on examination. A rapid workup including ECG, chest x-ray (CXR), laboratory tests, echocardiogram, and hemodynamic monitoring should help confirm the etiology and direct specific treatment of the underlying cause.

A 43-year-old insulin-dependent diabetic woman presents with an intensely pruritic rash across her back for the last several days. She reports a recent trip to the Caribbean with her family last week, none of whom has similar symptoms. She denies any insect bites, new detergents, soaps, or clothes and has otherwise been at her baseline of health with no constitutional symptoms. The patient is on insulin glargine for her diabetes and reports a hemoglobin A1C of < 7 mg/dL. Upon physical examination, the patient has small vesicular lesions on an erythematous base with no visible purulence. The rash does not follow a dermatome distribution and is clustered across her back. Which of the following is the most appropriate treatment choice for this patient? a. Calamine lotion b. Acyclovir c. Chlorhexidine lotion d. Benadryl e. Parenteral antibiotics

*C* This patient is suffering from miliaria rubra, more commonly known as prickly heat or heat rash. This is an acute inflammatory disorder of the skin that occurs in tropical climates, also lending the term "lichen tropicus." It occurs because of sweat gland blockage and staphylococcal infection. The acute phase is noted by vesicular lesions that are caused by obstruction of the sweat glands, which subsequently rupture. The rash is confined to clothed areas and may progress to a profunda stage, in which the obstruction delves deeper into the skin and produces larger vesicles that may become infected. These, however, are not pruritic and may resemble a chronic dermatitis. The antibacterial treatment of choice is Chlorhexadine, which is to be used in the acute phase. Salicylic acid may also be used to assist in desquamation, but should not be used on large areas because of possible salicylate intoxication.

Which of the following clinical presentations requires hospitalization? a. A 37-year-old man with paranoid schizophrenia who has been off his medications for a week and is hearing voices with no apparent violence in his thought content. b. A 19-year-old woman who ingested 10 multivitamin pills after an argument with her boyfriend. c. A 39-year-old man with no previous psychiatric history presents with pressured speech, no sleep for 4 days, and is feeling "great." d. A 22-year-old woman who is having visual hallucinations after ingesting D-lysergic acid diethylamide (LSD). e. A 43-year-old homeless man with a history of schizophrenia who was recently discharged from the psychiatric ward being managed on antipsychotic medications.

*C* This patient presents with a first manic episode. It is necessary to admit these patients in order to prevent behavior that is impulsive and dangerous. A full manic syndrome is one of the most striking and distinctive conditions in clinical practice. The main disturbance in mood is one of elation or irritability.

A 47-year-old man is brought to the ED by EMS after being persistently agitated at a business meeting. The patient's coworkers state that he has been working nonstop for a day-and-a-half and that he always seemed like a healthy guy who frequented bars every night. EMS administered 25 g of dextrose and thiamine with no symptom improvement. In the ED, the patient is anxious, confused, tremulous, and diaphoretic. He denies any medical problems, medications, or drug ingestions. His BP is 182/92 mm Hg, HR is 139 beats per minute, temperature is 100.4°F, RR is 18 breaths per minute, and fingerstick glucose is 103 mg/dL. An ECG reveals sinus tachycardia. Which of the following is the next best step? a. Administer acetaminophen b. Administer folate c. Administer diazepam d. Recheck fingerstick glucose e. Administer labetalol

*C* This patient presents with alcohol withdrawal. Signs and symptoms of this condition occur along a continuum ranging from simple shakes to delirium tremens (DTs) following a reduction or cessation of alcohol. Early symptoms usually appear 6 to 8 hours after cessation of drinking and involve tremulousness, anxiety, mild hypertension, and tachycardia. In more severe withdrawal, these symptoms worsen and paranoia, auditory, and visual hallucinations may develop proceeding to DTs with severe autonomic hyperactivity and profound altered mental status. DTs usually occur 3 to 5 days after alcohol cessation and carry 5% to 15% mortality even with supportive care. Additionally, alcohol withdrawal seizures may occur anywhere from 6 to 48 hours after cessation of alcohol. Benzodiazepines are the mainstay of therapy in alcohol withdrawal, as well as in sympathomimetic overdose and sedative-hypnotic withdrawal.

A 36-year-old woman presents to the ED complaining of worsening weakness over the past few weeks. She states that initially she attributed it to being overworked but for the last few days she has been having unusual difficulty getting out of chair or walking the steps to her fourth-floor apartment. She has no prior medical history and takes no medications. Her vital signs are unremarkable. On examination she has 5/5 strength in her bilateral upper and lower extremities distally but 3/5 strength proximally. Sensory examination and reflexes are normal. You also notice a red confluent macular rash on her eyelids. Which of the following is the most likely diagnosis? a. Myasthenia gravis b. Multiple sclerosis c. Dermatomyositis d. Rhabdomyolysis e. Disseminated gonococcal infection

*C* This patient presents with symmetric proximal muscle weakness and characteristic heliotrope rash of dermatomyositis. It is an idiopathic inflammatory myopathy with associated dermatitis. The characteristic of the disease is progressive symmetric proximal muscle weakness with possible dysphagia, symmetric heliotrope rash in the periorbital region or neck, elevated creatinine kinase, and abnormal electromyogram and muscle biopsy. There is also an associated risk of malignancy.

A 36-year-old G4P2103 presents to the ED with vaginal spotting for the past 2 days. She also reports occasional abdominal cramping and low back pain. The patient states that she is 4 months pregnant. She denies any other past medical history or gynecologic problems. Her initial vital signs include HR of 89 beats per minute, BP of 144/70 mm Hg, RR of 15 breaths per minute with oxygen saturation of 98% on room air. Her pelvic examination reveals a closed os. Upon ultrasound examination, an intrauterine pregnancy is visualized; however, a fetal HR is not detected. Given this patient's symptoms and physical examination, which of the following is the most appropriate diagnosis? a. Threatened abortion b. Complete abortion c. Missed abortion d. Inevitable abortion e. Incomplete abortion

*C* This patient presents with vaginal bleeding and a closed os, which narrows the diagnosis to either a threatened, complete, or missed abortion. The ultrasound examination reveals an intrauterine pregnancy that is still present without a fetal HR, which confirms the missed abortion. A dilation and curettage and Rh prophylaxis is warranted to prevent further infection and coagulopathy.

A 64-year-old man presents to the ED complaining of an episode of vertigo he experienced while exercising in the gym today. He states that he has been having similar episodes with exercise for a week. His routine consists of running on a treadmill, lifting weights, and doing leg presses. The vertigo usually occurs mid-routine when he is lifting weights and resolves with cessation of exercise. He also noticed unusual left arm pain during these episodes. He has hypertension for which he takes antihypertensive medication and had a myocardial infarction 6 years ago. He decreased his smoking from one pack per day to five cigarettes per day over the last 6 years. His BP in the right arm is 148/80 mm Hg and in the left arm it is 129/74 mm Hg. Which of the following is the most likely diagnosis? a. Superior vena cava syndrome b. Aortic dissection c. Subclavian steal syndrome d. Angina pectoris e. Vestibular neuronitis

*C* This patient presents with vertebrobasilar insufficiency (vertigo) and claudication (atypical arm pain with exercise), symptoms consistent with subclavian steal syndrome. This phenomenon occurs in patients with subclavian artery occlusion or stenosis proximal to the vertebral artery branch, which causes retrograde blood flow in the vertebral artery with ipsilateral arm exercise. Collateral arteries arising from the subclavian artery distal to the obstruction deliver blood to the arm. During arm exercise these vessels dilate and siphon blood from the head, neck, and shoulder to increase perfusion of ischemic arm muscles. This results in temporary reversal of blood flow in the vertebral artery leading to vertebrobasilar insufficiency and symptoms of vertigo, dizziness, syncope, dysarthria, and diplopia. Arm pain is a result of muscle ischemia.

A 28-year-old presents to the ED with depressive symptoms and feelings of hopelessness. She denies any active suicidal ideation, but has had increasing feelings of guilt over the last few weeks. The patient denies any past medical history except for giving birth 8 weeks ago. Her initial vitals are within normal limits. Given this patient's presentation, which of the following etiologies is most likely? a. Dysthymic disorder b. Bipolar disorder c. Postpartum depression d. Major depressive disorder e. Cyclothymic disorder

*C* This patient recently gave birth and presents with symptoms of hopelessness and guilt in the subsequent weeks, typical of symptoms seen in this postpartum period. Over half of mothers report depressed mood after childbirth, also known as the "baby blues." This may be linked to rapid decreases in estrogen and progesterone immediately after childbirth. About 40% mothers reports that these symptoms last a few days with about half of those having continuing symptoms into weeks. About 2% express suicidal ideation. Postpartum depression is more prevalent in those who are unemployed, do not have help with childcare or have a mood disorder. Treatment varies from cognitive management to antidepressants.

A 26-year-old woman presents to the ED with a history of spilling hot soup over her left arm earlier in the day. She states that she immediately put her hand under cold water and applied ice. Upon physical examination, the involved area covers the dorsum of her hand and extends up to the middle of her forearm with no circumferential or digital involvement. She can freely flex and extend all joints of that upper extremity. In addition, two large fluid-filled bullae are noted in the area of the forearm. What is the burn degree and relative area of involvement in this patient? a. First-degree/9% b. Second-degree/9% c. Second-degree/2.25% d. Third-degree/9% e. Third-degree/4.5%

*C* This patient sustained a second-degree burn involving 2.25% of her body surface area (BSA), according to the rule of nines. The rule of nines aids in estimating the area of involvement as seen below. In the adult, each anterior or posterior surface of the upper extremity and head are equal to 4.5%. Each surface of the lower extremity is equal to 9%. Each surface of the chest and torso is equal to 18%. Perineal areas account for 1% of BSA. Children and infants have increased total BSA relative to their weight, thereby requiring different fluid requirements and area estimation. More percentage is dedicated to the head given their larger head-to-body ratios (7% in children and 9% in infants). The treatment for this patient involves silver sulfadiazine dressings and close follow-up. Prophylactic antibiotics are not warranted in these cases, given the chance of increased future resistance rates. For areas of greater involvement and inhalation burns, fluid resuscitation may

An 18-year-old man presents to the ED with right leg pain and swelling after swimming at a local beach. He reports swimming in the ocean, whereupon he felt a sharp sting in his right leg. Upon physical examination, there is no gross deformity of the right lower extremity and there is a palpable dorsalis pedis and posterior tibial pulse. There is tenderness to palpation over the lateral calf with many punctuate, erythematous lesions as seen below. Which of the following is the most appropriate treatment of choice for this patient? a. Fresh water b. Vegetable oil c. Vinegar d. Toothpaste e. Household window cleaner

*C* This patient sustained an injury from a venomous marine animal. Approximately 50,000 such incidents occur yearly. Marine animals may be divided into two classes: stingers and nematocysts. Stingers include sea urchins, stingrays, catfish, cone shells, and starfish. Radiographs may be useful in delineating the calciferous material deposited in the skin for removal. Nematocysts are much more prevalent and account for the majority of envenomations and most likely account for this patient's distress. This class includes jellyfish, fire corals, Portuguese man-o-war, and anemones. These creatures have spring-loaded venomous glands that discharge upon mechanical or chemical stimulation. The number of nematocysts on a tentacle can number in the thousands. These stinging cells can remain activated after weeks of the animal being beached. The venom contains various peptides and enzymes that may cause progression of symptoms including nausea, muscle cramps, dyspnea, angioedema, and anaphylaxis. The preferred treatment is vinegar, which deactivates the nematocyst. In cases where medical attention cannot be sought in a timely manner, urine has been shown to be just as efficacious. An attempt may be made to shave off nematocysts after proper analgesia. Patient should be given tetanus prophylaxis and antihistamines as needed.

A 31-year-old man presents to the ED with left calf pain. He reports general malaise, nausea, and myalgias since a recent trip to Arkansas. His initial vitals include oral temperature of 99°F, HR of 76 beats per minute, BP of 128/70 mm Hg, RR of 16 breaths per minute, and oxygen saturation of 98% on room air. Upon physical examination, there are diffuse petechiae of the left lower extremity from the anterior distal tibia to the mid-thigh. Closer examination reveals a small necrotic lesion at the level of the lateral mid-calf with surrounding edema as seen below. The patient's calf is tender to the touch and upon dorsiflexion. Which of the following is the most likely cause of this patient's clinical symptoms? a. Deep venous thrombosis (DVT) b. Scorpion sting c. Brown recluse spider bite d. Black widow spider bite e. Thrombocytopenia

*C* This patient's necrotic lesion and travel to the south-central states is classic for a brown recluse spider bite. These injuries may present with systemic symptoms of fever, chills, myalgias, hemolysis, petechiae and eventually seizure, renal failure, and death. The brown recluse spider can be distinguished by its violin-shaped cephalothorax. Its venom contains hemolytic enzymes and substances that cause vasoconstriction. Initial lesions may appear targetoid, as blood supply to the central area is diminished and becomes necrotic. Lesions have been shown to cause significant scarring and infection. Initial treatment should include the ABCs, wound care, analgesia, and tetanus prophylaxis. Patients may be observed in the ED for a period of 6 hours to see if envenomation occurred. Antibiotics may become warranted if infection ensues. Loxosceles reclusa antivenin is manufactured in Brazil, but is not currently available in the United States.

A 64-year-old woman with a history of depression and hypertension was found down by her husband and brought in by the paramedics. Her husband says that she has recently been depressed and expressed thoughts of suicide. She usually takes fluoxetine for depression and atenolol for hypertension. On arrival, the patient is obtunded, but responds to pain and is maintaining her airway. Her temperature is 98.1°F, BP is 70/40 mm Hg, HR is 42 beats per minute, RR is 12 breaths per minute, and oxygen saturation is 94% on room air. On examination, her pupils are 3 mm and reactive bilaterally. Lungs are clear to auscultation. Heart is bradycardic, but regular, with no murmurs, rubs, or gallops. Extremities have no edema. An ECG shows first-degree AV block at 42 beats per minute, but no ST/T wave changes. Blood sugar is 112 mg/dL. What is the most specific treatment for this patient's ingestion? a. Fluid bolus b. Atropine c. Glucagon d. Epinephrine e. Cardiac pacing

*C* Toxic ingestion must always be considered, especially in suicidal patients. The patient regularly takes atenolol for hypertension and may have overdosed on this occasion. β-Blocker toxicity classically causes bradycardia and hypotension. Antidotes for β-blocker toxicity, such as glucagon, should be given to this patient immediately. Glucagon is thought to work through a separate receptor that is not blocked by β-adrenergic antagonists, ultimately enhancing inotropy and chronotropy. Other medications that may be useful are phosphodiesterase inhibitors, which block cAMP breakdown and maintain intracellular calcium levels. High-dose insulin is a promising experimental treatment for β-blocker toxicity. Ultimately, the patient may require the other treatment options, but glucagon should be the first-line therapy.

A 35-year-old woman is eating dinner at a restaurant. Approximately 1 hour after finishing the main course of lamb, red wine, and a fine selection of cheese, the patient experiences a severe occipital headache, diaphoresis, mydriasis, neck stiffness, and palpitations. Which of the following medications is this patient most likely taking? a. Paroxetine b. Alprazolam c. Tranylcypromine d. Citalopram e. Amitriptyline

*C* Tranylcypromine is a monoamine oxidase inhibitor (MAOI) that is used in the treatment of depression. Patients who take this medication should avoid eating or drinking foods that contain tyramine (similar structure to amphetamine). Tyramine is found in many foods such as aged cheese, wine, certain fish, meats, and sauerkraut. The combination of a MAOI and tyramine can lead to a sympathomimetic reaction called a tyramine reaction. It occurs within 15 to 90 minutes of ingestion of tyramine. The hallmark symptoms include headache, hypertension, diaphoresis, mydriasis, neck stiffness, pallor, neuromuscular excitation, palpitations, and chest pain. Most symptoms gradually resolve over 6 hours; however, deaths have been reported secondary to intracranial hemorrhage and myocardial infarction. Patients who take a MAOI should be instructed to avoid all tyramine-containing foods.

A 55-year-old woman presents to the ED after a reported syncopal event. Her initial vitals include an HR of 105 beats per minute, an RR of 16 breaths per minute, a BP of 125/60 mm Hg, and an oxygen saturation of 98% on room air. Her ECG is shows QRS interval prolongation. Which of the following substances is responsible for this patient's ECG findings? a. Benzodiazepine b. Alcohol c. Tricyclic antidepressant d. Insulin e. Valproic acid

*C* Tricyclic antidepressants (TCAs) are responsible for more drug-related deaths than any other prescription medication. This ECG shows QRS interval prolongation, a common finding with this medication. The mechanism of toxicity is multifold and includes blocking the reuptake of dopamine, serotonin, and norepinephrine. It also binds to the γ-aminobutyric acid (GABA) receptor, thereby lowering seizure threshold. Sodium-channel blockade produces the widened QRS interval. There are also anticholinergic and antihistamine effects. Sodium bicarbonate is a first-line intervention for dysrhythmias, acting as alkalizing binder for the acidic TCA. This treatment has been shown to improve conduction and contractility with the goal of preserving a narrow QRS.

A 46-year-old woman presents to the ED with left-sided arm and leg weakness for half an hour. She has no medical problems except for chronic neck pain after a motor vehicle collision 5 years ago. On examination she has right eye miosis, partial ptosis, and 3/5 strength in her left upper and lower extremities. Which of the following is consistent with the patient's ocular findings? a. Oculomotor nerve palsy b. Bell palsy c. Horner syndrome d. Kehr sign e. Nikolsky sign

*C* Unilateral findings of ptosis and miosis as well as anhidrosis are seen in Horner syndrome which results from interrupted sympathetic nerve supply to the eye

An 87-year-old woman with a history of dementia, arthritis, and hypertension presents to the ED for abdominal pain. Her caretaker reports that she is having mid-epigastric pain and had one episode of nonbloody, nonbilious vomiting prior to arrival. The patient is oriented to name only. Temperature is 99.8°F, HR is 110 beats per minute, BP is 80/44 mm Hg, RR is 16 breaths per minute, and oxygen saturation is 96% on room air. On examination, the abdomen is soft, nontender, with no masses, rebound or guarding. Stool is brown and guaiac negative. You place two IV lines and begin fluid resuscitation. You send her blood to the laboratory and order a radiograph of her chest that is shows free air under the diaphragm. Which of the following is the most appropriate next step in management? a. Start IV antibiotics b. Order a CT scan of her abdomen c. Call the surgery service d. Place a central venous line e. Discharge home with Maalox

*C*Abdominal pain in the elderly can be challenging for many reasons, including poor histories and deviation from classic presentations of diseases. However, abdominal pain in patients over 65 must be taken seriously since 25% to 44% require surgical intervention and more than 50% require admission to the hospital. This CXR reveals free air under the right diaphragm, likely from a perforated viscous. This is a surgical emergency and the surgical service should be contacted.

4-year-old boy is brought to the ED by a concerned mother after she noticed lesions under his nose and around his mouth as seen in the image below. The patient is otherwise well with no change in behavior, fever, or vomiting reported. On physical examination, you note a well nourished, well developed male in no acute distress with multiple small round, honeycolored lesions with slightly erythematous centers. What is the predominant organism involved? a. Group B streptococcus b. Staphylococcus aureus c. Streptococcus pyogenes d. Streptococcus pneumoniae e. Salmonella spp.

*C*This is a case of impetigo. It is a superficial skin infection caused most commonly by S pyogenes (group A β-hemolytic Streptococcus). When large pustules are observed (> 1 cm) the term used to describe this condition is bullous impetigo. The typical presentation if impetigo includes honey-crusted lesions on erythematous skin. These tend to be pruritic and easily spread by the patient. Although possible, systemic infection is uncommon so patients typically are well appearing. Treatment includes either oral or topical antibiotic therapy. Options include erythromycin or cephalexin; linezolid may be used in resistant cases of Staphylococcus. Topical therapy with mupirocin is very effective. Hand washing is critical to also limit the spread of infection to other family members. Treatment of the skin lesions does not prevent the development of nephritis.

A 25-year-old G3P1011 presents to the ED with a 6-hour history of worsening lower abdominal pain, mostly in the RLQ. She also noticed some vaginal spotting this morning. She is nauseated, but did not vomit. Her last menstrual period was 2 months ago, but her cycles are irregular. She is sexually active and has a history of pelvic inflammatory disease. Her BP is 120/75 mm Hg, HR is 95 beats per minute, temperature is 99.2°F, and RR is 16 breaths per minute. Her abdomen is tender in the RLQ. Pelvic examination reveals right adnexal tenderness. Her WBC count is slightly elevated and her β-hCG is positive. After establishing IV access, which of the following is the most appropriate next step in management? a. Call the OR to prepare for laparoscopy b. Order an emergent CT scan of the abdomen c. Perform a transvaginal ultrasound d. Order a urinalysis e. Swab her cervix and treat for gonorrhea and Chlamydia

*C.* Any woman with abdominal pain, vaginal bleeding, and a positive pregnancy test needs to be ruled out for an ectopic pregnancy. Her vital signs are stable so that she can undergo a transvaginal ultrasound. This is used to document an intrauterine pregnancy and the health of the fetus. If no intrauterine pregnancy is observed, the suspicion for an ectopic pregnancy increases.

A 72-year-old man presents to the ED with worsening dyspnea. His initial vitals include a HR of 93 beats per minute, BP of 110/50 mm Hg, and RR of 20 breaths per minute with an oxygen saturation of 88% on room air. The patient appears thin and anxious. He is using accessory muscles to breathe. Despite distant breath sounds, you hear end-expiratory rhonchi and a prolonged expiratory phase. An ECG shows peaked P waves in leads II, III, and aVF. Given this patient's history and physical examination, which of the following conditions does this patient most likely have? a. Chronic bronchitis b. Asthma c. Emphysema d. Congestive heart failure (CHF) e. Pneumothorax

*C.* COPD is often referred to as a single disease but is in fact a triad of three distinct disease processes: emphysema, chronic bronchitis, and asthma. This clinical scenario paints a typical picture of emphysema. Individuals appear dyspneic, thin, and anxious. They generally have an increased anterior-posterior (AP) chest diameter and often use their accessory muscles to help with breathing. Lungs sounds are distant and associated with wheezes, rhonchi, and a prolonged expiratory phase. These individuals are often classified as "pink puffers" and use their pursed lips to push air that remains trapped in alveoli owing to the prolonged expiratory phase. Exacerbations should be treated with corticosteroids, anticholinergic inhalers, and intermittent β2-agonists to decrease inflammation, decrease mucous production, and relax smooth muscle in an effort to open up the distal airways. Patients with COPD are at higher risk for developing bacterial bronchitis and pneumonia. COPD is generally caused by smoking but may also result from air pollution, occupational exposure, and genetic factors, such as α1-antitrypsin deficiency. Patients may require supplemental outpatient oxygen to function and perform basic activities of daily living.

An 82-year-old woman becomes acutely short of breath while at rest on the rehabilitation unit. She is brought into the ED with an oxygen saturation of 86% on room air and in acute respiratory distress. Her initial ECG is within normal limits and unchanged from a recent previous examination. Her initial chest x-ray is also negative. Upon chest auscultation, there are equal bilateral breath sounds with some scattered rhonchi. Her nurse tells you that 2 days ago she underwent internal fixation of a right-femur fracture and has been on anticoagulant therapy. Given the history and presentation of this patient, what is the most likely etiology of her symptoms? a. Venous thromboembolism b. Air embolism c. Fat embolism d. Pulmonary hemorrhage e. Rib fracture

*C.* Fat embolism refers to the presence of fat globules within the lung parenchyma and peripheral circulation after a long bone fracture, major trauma, or orthopedic procedure. Symptoms usually appear 1 to 2 days after the event or intramedullary nailing. Unlike thromboemboli, fat emboli may pass through the pulmonary vasculature into the systemic arterial circulation where any organ may be affected. Respiratory distress is a common initial system with subsequent neurologic manifestations given that the cerebral circulation is at particular risk. Treatment is primarily supportive in an intensive care setting.

55-year-old man presents to the ED at 2:00 AM with left-sided chest pain that radiates down his left arm. He takes a β-blocker for hypertension, a proton-pump inhibitor for gastroesophageal reflux disease, and an antilipid agent for high cholesterol. He also took sildenafil the previous night for erectile dysfunction. His BP is 130/70 mm Hg and HR is 77 beats per minute. Which of the following medication is contraindicated in this patient? a. Aspirin b. Unfractionated heparin c. Nitroglycerin d. Metoprolol e. Morphine sulfate

*C.* It is contraindicated to administer nitroglycerin to individuals who have taken sildenafil in the previous 24 hours. The combination of nitroglycerin and sildenafil can lead to hypotension and death.

A 51-year-old man presents to the ED complaining of epigastric pain that radiates to his back. He states that he drinks a 6 pack of beer daily. You suspect he has pancreatitis. His BP is 135/75 mm Hg, HR is 90 beats per minute, temperature is 100.1°F, and his RR is 17 breaths per minute. Laboratory results reveal WBC 13,000/μL, hematocrit 48%, platelets 110/μL, amylase 1150 U/L, lipase 1450 IU, lactate dehydrogenase (LDH) 150 U/L, sodium 135 mEq/L, potassium 3.5 mEq/L, chloride 105 mEq/L, bicarbonate 23 mEq/L, BUN 15 mg/dL, creatinine 1.1 mg/dL, and glucose 125 mg/dL. Which of the following laboratory values are most specific for pancreatitis? a. Elevated amylase b. Hyperglycemia c. Elevated lipase d. Elevated LDH e. Leukocytosis

*C.* Lipase is a pancreatic enzyme that hydrolyzes triglycerides. In the presence of pancreatic inflammation it increases within 4 to 8 hours and peaks at 24 hours. At five times the upper limits of normal, lipase is 60% sensitive and 100% specific for pancreatitis. The diagnosis is usually made with a lipase of two times the normal limit, thereby increasing its sensitivity.

As you evaluate a patient with shortness of breath, you appreciate decreased breath sounds at the left-lung base. You suspect the patient has a small pleural effusion. Which of the following views of the chest is this small pleural effusion most likely to be detected? a. Supine b. Lateral decubitus right-side down c. Lateral decubitus left-side down d. Lateral e. Posterior-anterior (PA)

*C.* Pleural effusions are most easily detected on a lateral decubitus film with the affected side down, which in this case is the left side. Accumulations of 5 to 50 mL of fluid can be detected with this view

Which of the following patients has the *lowest* clinical probability for the diagnosis of pulmonary embolism? a. A 21-year-old woman 2 days after a cesarean delivery b. A 55-year-old woman on estrogen replacement therapy who underwent a total hip replacement procedure 3 days ago c. A 39-year-old man who smokes cigarettes occasionally and underwent an uncomplicated appendectomy 2 months ago d. A 62-year-old man with pancreatic cancer e. A 45-year-old man with factor V Leiden deficiency

*C.* Risk factors for venous thromboembolism were first described by Virchow triad: hypercoagulability, stasis, and endothelial injury. Hypercoagulability can be broadly classified into malignancy-related or nonmalignancy-related. Malignancies of primary adenocarcinoma or brain malignancy are the most likely to cause thrombosis. Some causes of nonmalignancy-related thrombosis are estrogen use, pregnancy, antiphospholipid syndromes, factor V Leiden mutation, and protein C and S deficiencies. Immobility such as paralysis, debilitating diseases, or recent surgery or trauma also place patients at risk.

A 30-year-old man is brought to the ED by emergency medical service (EMS) in respiratory distress. His initial vitals include a HR of 109 beats per minute, a BP of 180/90 mm Hg, and an RR of 20 breaths per minute with an oxygen saturation of 92% on room air. A chest x-ray shows a bilateral diffuse infiltrative process. A subsequent toxicologic screen is positive. Which of the following agents is most likely responsible for this patient's presentation? a. Cannabis b. Opioid c. Crack cocaine d. Methamphetamine e. Ethanol

*C.* The crystallized free base of cocaine is known as "crack cocaine." This form, smoked through a pipe, produces a highly lipid-soluble vapor that allows for rapid transport from the lungs to the brain for a quicker high compared to cocaine that is snorted or injected. As a result of this mechanism, the substance can be concentrated in high amounts in the lung parenchyma causing an infiltrative inflammatory process and pneumonitis referred to as "crack lung." This can subsequently result in respiratory failure.

A 55-year-old man with hypertension and a one-pack-per-day smoking history presents to the ED complaining of three episodes of severe heavy chest pain this morning that radiated to his left shoulder. In the past, he experienced chest discomfort after walking 20 minutes that resolved with rest. The episodes of chest pain this morning occurred while he was reading the newspaper. His BP is 155/80 mm Hg, HR 76 beats per minute, RR 15 breaths per minute. He does not have chest pain in the ED. An ECG reveals sinus rhythm with a rate of 72. A troponin I is negative. Which of the following best describes this patient's diagnosis? a. Variant angina b. Stable angina c. Unstable angina d. Non-ST-elevation MI e. ST-elevation MI

*C.* The patient exhibits unstable angina, which is defined as new-onset angina, angina occurring at rest lasting longer than 20 minutes, or angina deviating from a patient's normal pattern. Unstable angina is considered the harbinger of an acute MI and, therefore, should be evaluated and treated aggressively. Unstable angina is one of the three diagnoses that make up ACS, the other two being stable angina and acute MI (ST or non-ST elevation). Patients may be pain-free and have negative cardiac biomarkers with unstable angina. In general, unstable angina is treated with oxygen, aspirin, clopidogrel, low molecular weight or unfractionated heparin, and further risk stratification in the hospital.

A 22-year-old college student went to the health clinic complaining of a fever over the last 5 days, fatigue, myalgias, and a bout of vomiting and diarrhea. The clinic doctor diagnosed him with acute gastroenteritis and told him to drink more fluids. Three days later, the student presents to the ED complaining of substernal chest pain that is constant. He also feels short of breath. His temperature is 100.9°F, HR is 119 beats per minute, BP is 120/75 mm Hg, and RR is 18 breaths per minute. An ECG is performed revealing sinus tachycardia. A chest radiograph is unremarkable. Laboratory tests are normal except for slightly elevated WBCs. Which of the following is the most common cause of this patient's diagnosis? a. Streptococcus viridans b. Staphylococcus aureus c. Coxsackie B virus d. Atherosclerotic disease e. Cocaine abuse

*C.* The patient has myocarditis. The enteroviruses, especially the coxsackievirus B, predominate as causative agents in the United States. Coxsackie B virus usually causes infection during the summer months. Some other causes of myocarditis include adenovirus, influenza, HIV, Mycoplasma, Trypanosoma cruzi, and steroid abuse. Flu-like complaints, such as fatigue, myalgias, nausea, vomiting, diarrhea, and fever, are usually the earliest symptoms and signs of myocarditis. Tachycardia is common and can be disproportionate to the patient's temperature (ie, HR faster than what is expected). This may be the only clue that something more serious than a simple viral illness exists. Approximately 12% of patients also complain of chest pain. Cardiac enzymes may be elevated and the CBC and erythrocyte sedimentation rate (ESR) are nonspecific.

A 27-year-old woman presents to the ED complaining of an intensely pruritic rash all over her body, abdominal cramping, and chest tightness. She states that 1 hour ago she was at dinner and accidentally ate some shrimp. She has a known anaphylactic allergy to shrimp. Her BP is 115/75 mm Hg, HR is 95 beats per minute, temperature is 98.9°F, RR is 20 breaths per minute, and oxygen saturation is 97% on room air. She appears anxious, and her skin is flushed with urticarial lesions. Auscultation of her lungs reveals scattered wheezes with decreased air entry. Which of the following is the most appropriate next step in management? a. Administer oxygen via non-rebreather, place a large-bore IV, begin IV fluids, and administer methylprednisolone intravenously. b. Administer oxygen via non-rebreather, place a large-bore IV, begin IV fluids, and administer methylprednisolone and diphenhydramine intravenously. c. Administer oxygen via non-rebreather, place a large-bore IV, begin IV fluids, administer methylprednisolone and diphenhydramine intravenously, and give subcutaneous epinephrine. d. Administer oxygen via non-rebreather, place a large-bore IV, begin IV fluids, and start aerosolized albuterol. e. Administer oxygen via non-rebreather, place a large-bore IV, begin IV fluids, and start aerosolized epinephrine.

*C.* The patient is having an anaphylactic reaction to the shrimp she ate. Anaphylaxis refers to a severe systemic allergic reaction with variable features such as respiratory difficulty, cardiovascular collapse, pruritic skin rash, and abdominal cramping. Anaphylaxis is a hypersensitivity reaction caused by an IgE-mediated reaction. Foods are the major cause in cases of anaphylaxis in which a source can be determined. Common foods that cause anaphylaxis include nuts, shellfish, and eggs. In the ED, attention is focused on reversing cardiovascular and respiratory disturbances. Epinephrine is the first drug of choice for patients with anaphylaxis. The route of administration is chosen by the severity of the patient's presentation. In a patient with upper airway obstruction or hypotension, IV epinephrine should be administered. Patients with stable vital signs can receive subcutaneous epinephrine. Epinephrine should be used with caution in the elderly or any patient with coronary artery disease or dysrhythmias. Antihistamines, such as diphenhydramine and ranitidine, should be used in all cases. These drugs block the action of circulating histamines at target tissue receptors. Corticosteroids, such as methylprednisolone, have an onset of action approximately 4 to 6 hours after administration and, therefore are of limited value in the acute setting. However, since giving them early may blunt the biphasic reaction of anaphylaxis and therefore, it is advised to administer to patients in anaphylaxis.

A 78-year-old woman is brought to the ED by EMS complaining of vomiting and abdominal pain that began during the night. EMS reports that her BP is 90/50 mm Hg, HR is 110 beats per minute, temperature is 101.2°F, and RR is 18 breaths per minute. After giving her a 500-mL bolus of NS, her BP is 115/70 mm Hg. During the examination, you notice that her face and chest appear jaundiced. Her lungs are clear to auscultation and you do not appreciate a murmur on cardiac examination. She winces when you palpate her RUQ. An ultrasound reveals dilation of the common bile duct and stones in the gallbladder. What is the most likely diagnosis? a. Cholecystitis b. Acute hepatitis c. Cholangitis d. Pancreatic cancer e. Bowel obstruction

*C.* The patient's clinical picture is consistent with cholangitis, which is caused by an obstruction of the biliary tract leading to bacterial infection. Obstruction is commonly secondary to a stone, but may be because of malignancy or stricture. Cholangitis is a surgical emergency. The classic triad of physical findings described by Charcot is RUQ pain, fever, and jaundice. Sepsis is a common complication. Sonography may demonstrate intrahepatic or ductal dilation. The presence of stones in the gallbladder suggests obstruction as the etiology.

A 31-year-old man from Florida presents to the ED complaining of severe pain that starts in his left flank and radiates to his testicle. The pain lasts for about 1 hour and then improves. He had similar pain last week that resolved spontaneously. He noted some blood in his urine this morning. His BP is 145/75 mm Hg, HR is 90 beats per minute, temperature is 98.9°F, and his RR is 24 breaths per minute. His abdomen is soft and nontender. As you examine the patient, he vomits and has trouble lying still in his stretcher. Which of the following is the most appropriate next step in management? a. Call surgery consult to evaluate the patient for appendicitis. b. Order an abdominal CT. c. Start intravenous (IV) fluids and administer an IV nonsteroidal anti-inflammatory drug (NSAID) and antiemetic. d. Perform an ultrasound to evaluate for an abdominal aortic aneurysm (AAA). e. Perform an ultrasound to evaluate for testicular torsion.

*C.* The patient's history of colicky flank pain that radiates to the groin and hematuria is consistent with a ureteral stone. Adequate analgesia is critical in treating a patient with a ureteral stone. Intravenous ketorolac, an nonsteroidal anti-inflammatory drug (NSAID), is frequently administered as a first-line analgesic, but morphine may be necessary for continued pain. In addition to their analgesia, NSAIDs decrease ureterospasm and renal capsular pressure in the obstructed kidney. Antiemetics, such as metoclopramide, help with the nausea and vomiting.

While playing a match of tennis, a 56-year-old man with a medical history significant only for acid reflux disease starts to feel substernal chest pain that radiates into his left arm and shortness of breath. His pain feels better after drinking antacid, but since it is not completely resolved, his partner calls 911. Upon arrival, EMS administers aspirin and sublingual nitroglycerin. After 20 minutes, the man's symptoms resolve. He is brought to the ED for further evaluation where his ECG shows sinus rhythm without any ischemic abnormalities. You order a chest radiograph and send his blood work to the laboratory for analysis. Which of the following statements regarding the diagnosis of acute MI is most accurate? a. A normal ECG rules out the diagnosis of acute MI. b. One set of negative cardiac enzymes is sufficient to exclude the diagnosis of MI in this patient. c. Troponin may not reach peak levels for at least 12 hours. d. Relief of symptoms by antacids essentially rules out a cardiac cause of his chest pain. e. Epigastric discomfort and indigestion is a rare presentation of ACS.

*C.* The patient's presentation is concerning for a cardiac cause of his chest pain. Chest pain radiating to the left arm that is associated with shortness of breath is a classic presentation of ACS. On arrival to the ED, the patient should be placed on a monitor, receive oxygen by nasal cannula, have an IV placed and blood sent to the laboratory, and an ECG performed. Any abnormalities in his vital signs should also be addressed. Serum cardiac markers are useful in detecting MI. Troponin T and I appear in the serum within 6 hours of symptom onset and remain elevated for 1 to 2 weeks. Troponin I is the most specific cardiac marker available (almost 100%) and peaks between 12 and 18 hours. However, the disposition of patients with suspected acute coronary syndromemshould be based on the clinical examination and not the cardiac enzymes. Initial determination of these markers has a low sensitivity for detecting ischemia and cannot be used to reliably diagnose or exclude the presence of an ACS.

A 57-year-old man complains of chest palpitations and lightheadedness for the past hour. Five years ago he underwent a cardiac catheterization with coronary artery stent placement. He smokes half a pack of cigarettes daily, and drinks a glass of wine at dinner. His HR is 140 beats per minute, BP is 115/70 mm Hg, and oxygen saturation is 99% on room air. An ECG reveals a wide complex tachycardia at a rate of 140 that is regular in rhythm. An ECG from 6 months ago shows a sinus rhythm at a rate of 80. Which of the following is the most appropriate medication to treat this dysrhythmia? a. Digoxin b. Diltiazem c. Amiodarone d. Adenosine e. Bretylium

*C.* This patient has a ventricular tachycardia defined by a QRS complex greater than 120 ms and a rate greater than 100 beats per minute. Ventricular tachycardia is the result of a dysrhythmia originating within or below the termination of the His bundle. Most patients with ventricular tachycardia have underlying heart disease. Treatment begins with assessing whether or not the patient is stable. If the patient shows signs of instability, such as hypotension or altered mental status, then cardioversion should be performed. However, if the patient is stable, medications can be administered to treat the dysrhythmia. Amiodarone, a class III antidysrhythmic that has pharmacologic characteristics of all fourclasses, is considered a first-line agent in treating ventricular dysrhythmias.

A 42-year-old man presents to the ED via ambulance after activating EMS for dyspnea. He is currently on an oxygen face mask and was administered one nebulized treatment of a β2-agonist by the paramedics. His initial vitals include an RR of 16 breaths per minute with an oxygen saturation of 96% on room air. The patient appears to be in mild distress with some intercostal retractions. Upon chest auscultation, there are minimal wheezes localized over bilateral lower lung fields. The patient's symptoms completely resolve after two more nebulizer treatments. Which of the following medications, in addition to a rescue β2-agonist inhaler, should be prescribed for outpatient use? a. Magnesium sulfate b. EpiPen c. Corticosteroids d. Cromolyn sodium e. Ipratropium

*C.*Corticosteroids have been shown to improve asthma symptoms in subsequent days after an exacerbation and prevent acute recurrences in patients who are deemed suitable to be discharged from the ED. An acceptable dosage is 40- to 60-mg prednisone daily for 3 to 10 days after the initial event. Inhaled steroids may also be an alternative to prevent relapses in more intractable cases, and should be used daily with the guidance of the patient's primary-care provider. Spacers are available to ensure adequate delivery of the medications deep into the alveoli.

A 76-year-old man presents to the ED in acute respiratory distress, gasping for breath while on face mask. Paramedics state that he was found on a bench outside of his apartment in respiratory distress. Initial vitals include a HR of 90 beats per minute, a BP of 170/90 mm Hg, and an RR of 33 breaths per minute with an oxygen saturation of 90%. Upon physical examination, the patient is coughing up pink, frothy sputum, has rales two-thirds of the way up both lung fields, and has pitting edema of hislower extremities. A chest radiograph reveals bilateral perihilar infiltrates, an enlarged cardiac silhouette, and a small right-sided pleural effusion. After obtaining IV access and placing the patient on a monitor, which of the following medical interventions is most appropriate? a. Morphine sulfate only b. Nitroglycerin only c. Nitroglycerin and a loop diuretic d. Aspirin e. Antibiotics

*C.*Pulmonary edema can be divided into cardiogenic and noncardiogenic. Cardiogenic varieties are commonly seen in the ED and are usually a result of high hydrostatic pressures. It is seen in patients with MI or ischemia, cardiomyopathies, valvular heart disease, and hypertensive emergencies. Nitroglycerin acts to decrease the preload of the heart by venous dilation. This lowers the work of the heart so that it can function more effectively. A loop diuretic is used to induce diuresis and is also thought to act as a venous dilator. In conjunction with one another, these medications act to improve the overall functional capacity of the heart. If medical interventions are not stabilizing, preparation should be made for endotracheal intubation. Positive airway pressure devices (eg, BiPAP) may also be used as a temporizing measure for oxygen delivery.

A 57-year-old woman presents to the ED with a basin in her hand and actively vomiting. You insert an IV catheter, start IV fluids, and administer an antiemetic agent. The patient feels much better but also complains of severe crampy abdominal pain that comes in waves. You examine her abdomen and note that it is distended and that there is a small midline scar in the lower abdomen. Upon auscultation, you hear high-pitched noises that sound like "tinkles." Palpation elicits pain in all four quadrants but no rebound tenderness. She is guaiac negative. Which of the following is the most common cause of this patient's presentation? a. Travel to Mexico b. Ethanol abuse c. Hysterectomy d. Hernia e. Constipation

*C.*The patient presents with the clinical findings of SBO: vomiting, intermittent crampy abdominal pain, abdominal distention, hyperactive bowel sounds, and general tenderness. The most common cause of SBO in developed countries is postoperative adhesions, responsible for more than 50% of all SBO. There is a particularly high incidence of SBO after gynecologic surgeries, such as a hysterectomy

A 31-year-old woman with a known psychiatric history presents to the ED after ingesting an unknown quantity of pills from her medication vial. In the ED, she complains of nausea, abdominal cramping, and feels unsteady on her feet. On physical examination you observe that she is tremulous, ataxic, and exhibits dystonia. Which of the following substances will not be adsorbed by activated charcoal? a. Digoxin b. Diphenhydramine c. Amitriptyline d. Lithium e. Acetaminophen

*D* A major change has occurred in the approach of GI decontamination over the past decade. Previous recommendations indicated that the stomach should be emptied by either syrup of ipecac or by gastric lavage. Activated charcoal alone has demonstrated similar or superior results and now is the recommended GI decontaminant. The complications of gastric emptying procedures, primarily aspiration, are largely avoided when only activated charcoal is used. Most ingested drugs and chemicals are adsorbed to activated charcoal. The few agents that do not adsorb to charcoal include ions (eg, mineral acids and alkalis, lithium, borates, bromides), hydrocarbons (HCs), metals (eg, iron), and ethanol. The patient in question ingested lithium that is used to treat her bipolar disorder. A lithium level should be drawn and management decided based on those results. Potential treatments include whole-bowel irrigation and dialysis. The substances in the other answer choices are absorbed by activated charcoal.

A 31-year-old woman presents to the emergency department (ED) 20 minutes after the sudden onset of chest pain, palpitations, shortness of breath, and numbness of her mouth, fingers, and toes. She tells you that it feels like she has a lump in her throat. Her medical history includes multiple visits to the ED over a period of 6 months with a similar presentation. Her only medication is an oral contraceptive. She occasionally drinks a glass of wine after a busy day of work when she feels stressed. She tells you that her mom takes medication for anxiety. Her blood pressure (BP) is 125/75 mm Hg, heart rate (HR) 88 beats per minute, and oxygen saturation 99% on room air. An electrocardiogram (ECG) shows a sinus rhythm. Her symptoms resolve while in the ED. Which of the following is the most likely diagnosis? a. Paroxysmal atrial tachycardia b. Hyperthyroidism c. Major depressive disorder d. Panic disorder e. Posttraumatic stress disorder

*D* A person with panic attacks will often present to multiple EDs and be discharged after a workup is normal. A panic attack is a discrete period of intense fear or discomfort in which four or more of the following symptoms develop acutely and peak within 10 minutes: accelerated HR, palpitations, pounding heart, diaphoresis, trembling or shaking, sensation of shortness of breath or a choking feeling, chest pain, nausea, dizziness, lightheadedness, paresthesias, fear of dying, chills, or hot flushes.

165. Paramedics bring a 55-year-old woman to the ED after she was struck by a motor vehicle traveling at 30 mile/h. Her BP is 165/95 mm Hg, HR is 105 beats per minute, and RR is 20 breaths per minute. Upon arrival, she does not open her eyes, is verbal but not making any sense, and withdraws to painful stimuli. You assign her a GCS score of 8. As you prepare to intubate the patient, a colleague notices that her left pupil has become dilated compared to the right. Which of the following has the quickest effect to reduce ICP? a. Cranial decompression b. Dexamethasone c. Furosemide d. Hyperventilation e. Mannitol

*D* A unilateral dilated pupil in the setting of head trauma is an indicator of increased ICP. If ICP is not lowered immediately, the patient has little chance of survival. Hyperventilation to produce an arterial PCO2 of 30 to 35 mm Hg will temporarily reduce ICP by promoting cerebral vasoconstriction and subsequent reduction of cerebral blood flow. The onset of action is within 30 seconds. In most patients, hyperventilation lowers the ICP by 25%. PCO2 should not fall below 25 mm Hg because this may cause profound vasoconstriction and ischemia in normal and injured areas of the brain. Hyperventilation is a temporary maneuver and should only be used for a brief period of time during the acute resuscitation and only in patients demonstrating neurologic deterioration.

A 53-year-old woman is brought to the ED by her husband, who states that she is feeling very weak over the last 2 days, is nauseated, and vomiting at least three times. The husband states that his wife was taking a high dose medication for her joint pain but ran out of her pills last week. Her vital signs are BP of 90/50 mm Hg, HR 87 beats per minute, RR 16 breaths per minute, and temperature 98.1°F. You place her on the monitor, begin IV fluids, and send her blood to the laboratory. Thirty minutes later the metabolic panel results are back and reveal the following: Na+ 126 mEq/L K+ 5 mEq/L Cl− 99 mEq/L HCO3 21 mEq/L BUN 24 mg/dL Creatinine 1.6 mg/dL Glucose 69 mg/dL Ca+ 11 mEq/L What is the most likely diagnosis? a. Myxedema coma b. Thyroid storm c. Hyperaldosteronism d. Adrenal insufficiency e. Diabetic ketoacidosis (DKA)

*D* Adrenal cortical insufficiency is an uncommon, potentially life-threatening condition that if recognized early, is readily treatable. The most common cause of adrenal insufficiency is hypothalamic-pituitary-adrenal axis suppression from longterm exogenous glucocorticoid administration. This patient abruptly stopped her high-dose steroids. The clinical presentation of adrenal insufficiency is vague but typically includes weakness, fatigue, nausea, vomiting, hypotension, and hypoglycemia. Electrolyte abnormalities are common. Hyponatremia and hyperkalemia are present in more than two-thirds of cases. Management includes supportive care with administration of glucocorticoids and electrolyte correction.

A 70-year-old woman with a history of hypertension, congestive heart failure, and atrial fibrillation presents to the ED with several hours of acute onset diffuse abdominal pain. She denies any nausea or vomiting. The pain is constant but she is unable to localize it. She was diagnosed with a renal artery thrombosis several years ago. Vital signs include HR of 95 beats per minute, BP of 110/70 mm Hg, and temperature of 98°F. Her abdomen is soft and mildly tender, despite her reported severe abdominal pain. Her WBC count is 12,000/μL, hematocrit 38%, platelets 250/μL, and lactate 8 mg/dL. The stool is trace heme-positive. You are concerned for acute mesenteric ischemia. What is the best way to diagnose this condition? a. Serum lactate levels b. Abdominal radiograph (supine and upright) c. CT scan d. Angiography e. Barium contrast study

*D* Angiography remains the "gold standard" in the diagnosis of mesenteric ischemia. Unlike any other diagnostic tools, it is capable of both diagnosing and treating the problem. It is capable of identifying all four types of acute mesenteric ischemia: (1) arterial embolus, (2) arterial or (3) venous thrombosis, and, under most circumstances, (4) nonocclusive mesenteric ischemia. Angiography should be obtained without delay when the diagnosis is suspected.

As you arrive for your ED shift, you are called to help with a "coding" patient. The senior resident has just intubated the patient and the nurses have established IV access and attached the cardiac monitor. An emergency medical treatment (EMT) student is performing chest compressions. You ask the EMT student to stop compressions. The monitor shows a flat line with no electrical activity. You are unable to detect any pulses. What is your next step in management? a. Defibrillate at 360 J b. Epinephrine 1-mg IV push c. Atropine 1-mg IV push d. Ask the nurse to run a rhythm strip in an additional lead e. Apply transcutaneous pacers

*D* Asystole is absent heart rhythm or more colloquially, "flat line." A common cause of asystole is a disconnected lead or malfunctioning equipment, so the AHA recommends confirmation of asystole by switching to another lead on the cardiac monitor. Confirmation can also be achieved with a 12-lead ECG if the equipment is readily available.

A 48-year-old man is brought to the ED by paramedics for generalized weakness. His medical history is significant for a CABG last month. He has been unable to get out of bed for the past day because of dizziness when changing position. He denies chest pain, shortness of breath, or syncope. His temperature is 98.9°F, BP is 86/60 mm Hg, HR is 44 beats per minute, RR is 18 breaths per minute, and oxygen saturation is 98% on room air. There is a well-healing midline sternotomy incision. Cardiac examination reveals a III/VI systolic ejection murmur. There are minimal rales at his lung bases. He is immediately attached to the cardiac monitor. His rhythm strip is shows _______. What is your initial treatment? a. Observe on monitor b. Transcutaneous pacing c. Transvenous pacing d. Atropine 0.5-mg IV e. Epinephrine IV drip at 2 μg/min

*D* Atropine is the initial treatment of choice for patients in second-degree, Mobitz I AV block. The majority of patients respond to atropine without further treatment. Mobitz I is commonly seen with acute inferior MI, digoxin toxicity, myocarditis, and after cardiac surgery.

A 19-year-old rookie Navy Seal presents to the ED with a history of syncope upon ascent from a dive. The length and depth of the dive was within decompression regulation. He currently complains of feeling light-headed with a moderate frontal headache. Vital signs include HR of 86 beats per minute, BP of 130/65 mm Hg, and RR of 16 breaths per minute with oxygen saturation of 93% on room air. Upon physical examination, he appears somewhat confused and is oriented only to person and place. He has no focal neurologic deficits. What underlying event is the cause for this patient's symptoms? a. Pulmonary embolism b. Cardiac ischemia c. Transient ischemic attack d. Dysbaric air embolism e. Nitrogen narcosis

*D* Being a rookie diver, this patient ascended to the surface too quickly. As in decompression sickness, nitrogen gas bubbles form and subsequently travel to the arterial system through the pulmonary veins into the cardiac chambers. Arterial gas embolism may also occur in patients with an underlying patent foramen ovale. Symptoms are usually sudden and dramatic. Divers, who may have been thought to drown, actually passed out during ascent because of an underlying embolism. Treatment includes hyperbaric oxygen therapy and the avoidance of air transport.

55-year-old woman with a past medical history of diabetes presents to the ED with fevers, headache, vision complaints, and right-sided weakness. She was treated for otitis media 2 weeks ago with amoxicillin as an outpatient. CT shown in answer. What is the most likely diagnosis? a. Central nervous system (CNS) toxoplasmosis b. Subdural hygroma c. Glioblastoma multiforme d. Brain abscess e. SAH

*D* Brain abscesses are uncommon and their incidence has decreased over the past several years as a result of better antibiotic treatment of the remote infections that cause them. Today, the majority of brain abscesses in developed countries are the result of contiguous spread from otitis media, mastoiditis, paranasal sinusitis, or meningitis. They can also occur after trauma, classically after a basilar skull fracture. Presentation is often nonspecific, with almost half present with headache alone. Focal weakness, fevers, and nausea are other common presenting complaints. Antibiotic choice should be guided by suspected source and ability to penetrate the CNS. In general, MRI is more sensitive at detecting CNS infection than CT. With contrast CT, the walls of the abscess enhance and there is a central necrotic area of lower density. On this CT with IV contrast, there are multiple ring-enhancing lesions with surrounding edema in the left frontal lobe. The large extra-axial collections with enhancing margins represent emphysemas. There is also midline shift secondary to mass effect of the abscesses.

A 29-year-old man is brought to the ED by EMS after being stabbed in the left side of his back. His BP is 120/80 mm Hg, HR is 105 beats per minute, RR is 16 breaths per minute, and oxygen saturation is 98% on room air. On the secondary survey, you note motor weakness of his left lower extremity and the loss of pain sensation in the right lower extremity. Which of the following is the most likely diagnosis? a. Spinal shock b. Central cord syndrome c. Anterior cord syndrome d. Brown-Séquard syndrome e. Cauda equina syndrome

*D* Brown-Séquard syndrome or hemisection of the spinal cord, typically results from penetrating trauma, such as a gunshot or knife wound. Patients with this lesion have ipsilateral motor paralysis and contralateral loss of pain and temperature distal to the level of the injury. This syndrome has the best prognosis for recovery of all of the incomplete spinal cord lesions.

A 61-year-old man presents to the ED with low back pain after slipping on an icy sidewalk yesterday. He states that the pain started on the left side of his lower back and now involves the right and radiates down both legs. He also noticed difficulty urinating since last night. On neurologic examination, he cannot plantar flex his feet. Rectal examination reveals diminished rectal tone. He has a medical history of chronic hypertension and underwent a "vessel surgery" many years earlier. Which of the following is the best diagnosis? a. Abdominal aortic aneurysm b. Disk herniation c. Spinal stenosis d. Cauda equina syndrome e. Osteomyelitis

*D* Cauda equina syndrome is an injury to the lumbar, sacral, and coccygeal nerve roots, causing peripheral nerve injury that can lead to permanent neurologic defects if not recognized and corrected rapidly. Because of the central location of the disk herniation, symptoms are often bilateral and involve leg pain, saddle anesthesia, and impaired bowel and bladder function (retention or incontinence). On examination, patients may exhibit loss of rectal tone and display other motor and sensory losses in the lower extremities. Patients with suspected cauda equina syndrome require an urgent CT scan or MRI.

A 28-year-old mechanic with no past medical history presents to the ED after a small amount of battery acid was splashed in his right eye. He is complaining of extreme pain and tearing from his eye. Which of the following is the most appropriate next step in management? a. Call ophthalmology now. b. Check visual acuity. c. Check the pH of the tears. d. Irrigate with normal saline. e. Apply erythromycin ointment.

*D* Chemical injuries to the eye must be irrigated with a minimum of 1 to 2 L of normal saline as soon as they arrive in the ED. Topical anesthesia and the use of a Morgan Lens (a special device to provide large volume irrigation to the eye) can help facilitate this procedure.

A 22-year-old woman presents to the ED with diffuse pelvic pain and vaginal bleeding. She reports that it is about the same time that she normally has her menses. She also reports some pain with defecation, dyspareunia, and points of dysmenorrhea in the past. The patient states that she has felt this way before, but that the pain has now worsened and is intolerable. Her physical examination reveals a soft abdomen with normal bowel sounds without rebound tenderness. The patient does not guard and there is no costovertebral tenderness. Her pelvic examination is significant for blood in the posterior vaginal vault, a closed os and no palpable masses or cervical motion tenderness. Given this patient's history and physical examination, which of the following is the most likely diagnosis? a. Ureteral colic b. Pregnancy c. Ruptured ectopic pregnancy d. Endometriosis e. Appendicitis

*D* Endometriosis is defined by the presence of endometrial glands and tissue outside the lining of the uterus. This tissue may be present on the ovaries, fallopian tubes, bladder, rectum, appendix, or other GI tissue. There are many different hypotheses as to how this ectopic tissue forms, the most commonly accepted being "retrograde menstruation." Pain most commonly occurs before or at the beginning of menses. Other symptoms that indicate ectopic endometrial implantation and activation include dyspareunia and problems with defecation. Clinical suspicions can only be confirmed with direct visualization through laparoscopy. Treatment includes analgesia for acute episodes and hormonal therapy to suppress the normal menstrual cycle so that purely the endometrial tissue may be sloughed during menses. Surgical intervention is taken for those cases that are truly refractory to these treatments.

A 23-year-old man presents to the ED with left lower-abdominal pain and left testicular pain that started 1 to 2 weeks ago and has gradually worsened. He denies nausea and vomiting. His HR is 98 beats per minute, BP is 125/65 mm Hg, temperature is 100.9°F, and RR is 18 breaths per minute. Physical examination reveals a tender left testicle with a firm nodularity on the posterolateral aspect of the testicle. Pain is relieved slightly with elevation of the testicle and the cremasteric reflex in normal. Which of the following is the next best step? a. Prescribe pain medications and penicillin for coverage of syphilis. b. Recommend bed rest and scrotal elevation with urology follow-up. c. Attempt to untwist the left testicle by rotating it in a clockwise direction and order an immediate scrotal ultrasound. d. Give ceftriaxone 250 mg intramuscularly (IM), plus a 10-day course of oral doxycycline. e. Confirm the diagnosis with transillumination of the testicle, and then consult urology for surgical drainage.

*D* Epididymitis is an inflammation of the epididymis, often caused by age-dependent bacterial infection. Sexually transmitted diseases (STDs), such as gonorrhea and Chlamydia, are most common in patients younger than 35 years, while urinary pathogens, such as Escherichia coli and Klebsiella, are most common in patients older than 40 years. Unlike testicular torsion, the onset of pain in epididymitis is usually gradual and the cremasteric reflex is (usually?) intact. Scrotal elevation may transiently relieve pain (positive Prehn sign). Treatment includes bed rest, scrotal elevation or support when ambulating, avoidance of heavy lifting, and antibiotics for infection.

A 50-year-old man is brought to the ED by ambulance with significant hematemesis. In the ambulance, paramedics placed two large-bore IVs and began infusing normal saline. In the ED, his HR is 127 beats per minute, BP is 79/45 mm Hg, temperature is 97.9°F, RR is 24 breaths per minute, and oxygen saturation is 96%. On physical examination, his abdomen is nontender, but you note spider angiomata, palmar erythema, and gynecomastia. Laboratory results reveal WBC 9000/μL, hematocrit 28%, platelets 40/μL, aspartate transaminase (AST) 675 U/L, alanine transaminase (ALT) 325 U/L, alkaline phosphatase 95 U/L, total bilirubin 14.4 mg/dL, conjugated bilirubin 12.9 mg/dL, sodium 135 mEq/L, potassium 3.5 mEq/L, chloride 110 mEq/L, bicarbonate 26 mEq/L, blood urea nitrogen (BUN) 20 mg/dL, creatinine 1.1 mg/dL, and glucose 150 mg/dL. Which of the following is the most likely diagnosis? a. Perforated gastric ulcer b. Diverticulosis c. Splenic laceration d. Esophageal varices e. Ruptured AAA

*D* Esophageal varices develop in patients with chronic liver disease in response to portal hypertension. Approximately 60% of patients with portal hypertension will develop varices. Of those who develop varices, 25% to 30% will experience hemorrhage. Patients who develop varices from alcohol abuse have an even higher risk of bleeding, especially with ongoing alcohol consumption. This patient has evidence of chronic liver disease with thrombocytopenia and elevated bilirubin and liver enzymes. In alcoholic hepatitis, the AST is greater than the ALT by a factor of 2. Spider angiomata, palmar erythema, and gynecomastia further suggest underlying liver disease.

A 23-year-old man presents to the ED complaining of a wound to his left lower extremity. He states he was pulling branches out of a flooded creek in his backyard when he punctured his left calf. He states that his legs were in the water when the wound occurred. On examination, the puncture wound is 1 cm in diameter, several centimeters deep, and demonstrates surrounding erythema. After irrigating the wound, you discover a splinter within the wound. What is the most appropriate next step in management? a. Radiograph to map the exact location of the foreign body. b. Close the wound with 4-0 nylon. c. Dress the wound with a sterile bandage and treat with a fluoroquinolone. d. Remove the splinter. e. Remove the splinter, close the wound, and treat with fluoroquinolone.

*D* Foreign bodies with potential to cause infection and inflammatory response require immediate removal (examples include thorns, splinters, spines, teeth, soil-covered objects). These materials may cause intense and excessive inflammatory responses. If the foreign body cannot be removed by the ED physician, the appropriate specialist should be consulted.

A 47-year-old man presents to the ED complaining of knee pain after slipping on ice on his way to work. On examination, you note significant swelling to his knee and palpate a large effusion. The patient has limited range of motion as a result of the swelling. You perform a palliative arthrocentesis and note blood and fat globules in the syringe. Which of the following findings is the presence of fat globules most suggestive of? a. ACL tear b. PCL tear c. Medial meniscus tear d. Fracture e. Vascular injury

*D* Frequently, there is a fracture to account for the presence of fat within the joint because the fat has entered from the bone marrow cavity. The presence of fat globules may be seen in the joint aspirate and even on the radiograph of the knee showing a joint effusion with a fat-fluid level.

A mother brings her 7-year-old daughter to the ED with a reported illness of 3 days where the child has been weak and not eating her usual amount. The mother also reports that she noticed her daughter's urine to be of a reddish color and that her stools have been smaller in caliber. She brings a specimen of her daughter's urine with her. She reports that her daughter has been hospitalized multiple times before for similar reasons and dehydration. Upon physical examination, the patient is without distress, quiet, and states that she feels tired. Her chest is clear to auscultation, her abdomen is benign and her neurologic examination nonfocal. A urinalysis performed fails to show any blood or myoglobin in the child's urine. Which of the following should be included in the differential diagnosis of this child? a. Malingering b. Factitious disorder c. Munchausen syndrome d. Munchausen syndrome by proxy e. Conversion disorder

*D* Given that this is an apparent illness or health-related abnormality produced by a parent or caregiver upon another, the by proxy title is given. This is a psychiatric illness defined by the Diagnostic and Statistical Manual of Mental Disorders (Fourth Edition) (DSM-IV) as an apparent illness concocted by a caregiver upon a child who presents for medical aid multiple times without ever being given a true diagnosis. There is a failure by the perpetrator to acknowledge the true etiology and a cessation of symptoms in the child once they are separated from the perpetrator. Simulated illness, without producing direct harm upon the child, is commonly seen. As seen in this example, placing tincture in the child's urine to mimic blood. Sadly, produced illness in which the child is harmed is most commonly seen (50% of cases). The most common presentations include bleeding, seizures, vomiting, diarrhea, fever, and rash. Ninety-eight percent of perpetrators are biologic mothers from all socioeconomic backgrounds. Many have a background in health professions or have features of Munchausen syndrome in themselves. Most of these mothers have had an abusive experience early in life and use the health-care system as a means to satisfy personal nurturing demands. They are said to gain a sense of purpose and meaning when their child is in the hospital, as well as an outlet for pity and comfort. These children may display incidental characteristics that cannot be linked to the presenting complaints. These children may also suffer from learning difficulties or clinical depression caused by many hospitalizations that are incurred upon them.

A 45-year-old woman presents to the ED with 1 day of painful rectal bleeding. Review of systems is negative for weight loss, abdominal pain, nausea, and vomiting. On physical examination, you note an exquisitely tender swelling with engorgement and a bluish discoloration distal to the anal verge. Her vital signs are HR 105 beats per minute, BP 140/70 mm Hg, RR 18 breaths per minute, and temperature 99°F. Which of the following is the next best step in management? a. Recommend warm sitz baths, topical analgesics, stool softeners, a high-fiber diet, and arrange for surgical follow-up. b. Incision and drainage under local anesthesia or procedural sedation followed by packing and surgical follow-up. c. Obtain a complete blood cell (CBC) count, clotting studies, type and cross, and arrange for emergent colonoscopy. d. Excision under local anesthesia followed by sitz baths and analgesics. e. Surgical consult for immediate operative management.

*D* Hemorrhoids are dilated venules of the hemorrhoidal plexuses. They are associated with constipation, straining, increased abdominal pressure, pregnancy, increased portal pressure, and a low-fiber diet. Hemorrhoids can be either internal or external. Those that arise above the dentate line are internal and painless. Those that are below the dentate line are external and painful. Individuals commonly present with thrombosed external hemorrhoids. On examination, there is a tender mass at the anal orifice that is typically bluish-purple in color. If pain is severe and the thrombosis is less than 48 hours then the physician should excise the thrombus under local anesthesia followed by a warm sitz baths. This patient is suffering from an acutely thrombosed external hemorrhoid. If not excised, symptoms will most often resolve within several days when the hemorrhoid ulcerates and leaks the dark accumulated blood. Residual skin tags may persist. Excision provides both immediate and long-term relief and prevents the formation of skin tags

A 23-year-old man presents to the ED after sustaining a bee sting. The patient points to his left arm when asked where the pain is. Upon physical examination, you see a single puncture wound with surrounding erythema and swelling. The patient is in no respiratory distress and is phonating well. Chest auscultation reveals clear breath sounds bilaterally with no wheezing. The oropharynx is patent without any tonsillar or uvular displacement. Which of the following is the most appropriate next step in management? a. Subcutaneous epinephrine 0.01 mL/kg b. IV epinephrine 0.01 mL/kg c. Steroids d. Observation e. Antihistamines

*D* History-taking should include prior bee stings in this child, as each successive sting increases the possibility of anaphylaxis given sensitization. The ABCs must be initially addressed. Stings most commonly present with localized burning, erythema, and edema at the sting site lasting for about 24 hours. The patient is asymptomatic and only warrants observation at this time. Toxicity and anaphylaxis are evident soon after the sting and include vomiting, diarrhea, fever, and neurologic manifestations, such as seizures and altered mental status. This child most likely sustained a sting from a honeybee or bumblebee, given the single puncture wound. Aphids contain retroserrate-barbed stingers, which are removed upon stinging, thereby eviscerating and killing the insect. Vespids, such as wasps, hornets, and yellow jackets, do not contain this mechanism and may sting many times. "Killer bees," as popularized by the film industry, contain the same amount of venom as other species. The difference is that they are more aggressive, not more toxic.

A 25-year-old man is brought into the trauma resuscitation room after his motorcycle is struck by another vehicle. EMS reports that the patient was found 20 ft away from his motorcycle, which was badly damaged. His vital signs include a BP of 90/60 mm Hg, HR of 115 beats per minute, RR of 2 breaths per minute, and pulse oxygenation of 100% on facemask Which of the following is the smallest amount of blood loss that produces a decrease in the systolic BP in adults? a. Loss of 5% of blood volume b. Loss of 10% of blood volume c. Loss of 15% to 30% of blood volume d. Loss of 30% to 40% of blood volume e. Loss of greater than 40% of blood volume

*D* Hypovolemia secondary to hemorrhage is the most common cause of shock in the trauma patient. The earliest signs of hemorrhagic shock are tachycardia and cutaneous vasoconstriction. The amount of blood loss present can be estimated based on the individual's initial clinical presentation. Class III hemorrhage is characterized by 30% to 40% blood loss (1500-2000 mL). This stage exhibits tachypnea, tachycardia (HR > 120), decrease in systolic BP, delayed capillary refill, decreased urine output, and a change in mental status.

A 27-year-old woman with known idiopathic intracranial hypertension (IIH) presents to the ED complaining of a bifrontotemporal headache several times a day for 6 weeks after running out of her medications. She complains of occasional pulsatile tinnitus but no visual disturbances. Funduscopic examination reveals no papilledema and normal venous pulsations. Which of the following factors determines the need for urgent treatment in patients with IIH? a. The presence of papilledema on funduscopic examination b. A history of pulsatile tinnitus c. Presence of an empty sella on CT scan d. Complaint of visual loss or visual disturbances e. A history of concomitant minocycline use

*D* IIH formerly pseudotumor cerebri, requires urgent treatment when there is a history of visual phenomena, particularly transient vision loss. Agents used to lower ICP in this setting include carbonic anhydrase inhibitors (ie, acetazolamide) and loop diuretics (ie, furosemide). Transient visual loss, pain, or blurring occurs frequently and can be permanent in up to 10% of patients. Treatment of the headache itself often employs the same agents used to treat migraines; ergots, antiemetics, and occasionally steroids. The headache is often refractory and difficult to manage. The condition is associated with obesity and weight loss is sometimes helpful.

A 65-year-old actively seizing woman is brought to the ED by EMS. She was found slumped over at the bus stop bench. EMS personnel state that when they found the woman she was diaphoretic and her speech was garbled. En route to the hospital, she started to seize. As you wheel her to a room, the nurse gives you some of her vital signs which are a BP of 150/90 mm Hg, HR of 115 beats per minute, and oxygen saturation of 96%. Which of the following is the next best step in managing this patient? a. Request a rectal temperature to rule out meningitis. b. Call the CT technologist and tell them you are bringing over a seizing patient. c. Ask for a stat ECG and administer an aspirin. d. Check the patient's fingerstick blood glucose level. e. Intubate the patient.

*D* In a patient who is actively seizing, it is essential to check the blood glucose level. Hypoglycemia is an easily reversible cause of seizure and is corrected with the administration of dextrose, not the usual anticonvulsants. Patients at both extremes of age are particularly susceptible to glucose stress during acute illness.

An undomiciled man of unknown age presents to the ED unresponsive. His ECG is shows unspecified heart block. What is the most appropriate next step in treatment? a. Defibrillation b. Cardiac pacing c. Advanced cardiac life support d. Advanced cardiac life support and rewarming e. Cardiopulmonary bypass

*D* In all critically ill patients, it is important to remove clothing, obtain a rectal temperature and initiate continuous monitoring, including an ECG, while assessing the ABC status of the patient. The clinician must initiate advanced cardiac life support and rewarm the patient once a rectal temperature has confirmed hypothermia. A tympanic temperature is not accurate below 94°F. It is important to remember that severely hypothermic patients who appear dead have a good chance of a normal neurologic outcome with continued resuscitation and rewarming. Remember that a patient is not dead until they're warm and dead! Those at extremes of age, users of sedative hypnotics, undomiciled individuals, and those with chronic illness, altered mental status, or sepsis are at most risk for hypothermia. Rewarming should begin with removing all wet clothing and placing warm blankets over the patient with progressive attempts made to rewarm the patient; mechanical warming blanket, warm IV fluids, gastric lavage/peritoneal warming, and lastly cardiopulmonary bypass.

A 23-year-old G1P0 presents to the ED with vaginal spotting that began earlier in the day. She denies any abdominal pain, trauma, dysuria, or back pain. Her initial vital signs include HR of 90 beats per minute, BP of 125/60 mm Hg, RR of 14 breaths per minute with oxygen saturation of 99% on room air. Her pelvic examination is significant for a scant amount of blood in the posterior vaginal vault and a closed os. The patient has no tenderness upon bimanual examination. She states that she is 6 weeks pregnant. Given this patient's presentation, which of the following ancillary tests must be performed? a. CBC b. Basic metabolic panel c. Coagulation panel d. Type and screen e. Urinalysis

*D* In any patient who is pregnant with vaginal bleeding, it is imperative to obtain a type and screen in order to identify the Rh status of the patient. Rhesus isoimmunization is an immunologic disorder that affects Rh-negative mothers of Rh-positive fetuses. Any transplacental maternal exposure to fetal Rh-positive blood cells can initiate this, whether its origin is traumatic or not. Initial exposure leads to primary sensitization and the production of antibodies. In subsequent pregnancies, these maternal antibodies can then cross the placenta and attack the fetal Rh-positive blood cells. Prevention can be accomplished by giving RhoGAM to all mothers who are Rh-negative. Administration of the immune globulin can be given prophylactically at 28 weeks or at the time of maternal exposure.

A 51-year-old man presents to the ED complaining of nausea and abdominal pain after drinking some "bitter stuff." He is considered one of the "regulars" who is usually at triage with ethanol intoxication. His temperature is 97.9°F, BP is 130/65 mm Hg, HR is 90 beats per minute, RR is 16 breaths per minute, and oxygen saturation is 97% on room air. Physical examination is unremarkable, except for slurred speech and the smell of acetone on the patient's breath. Laboratory results reveal serum sodium 138 mEq/L, potassium 3.5 mEq/L, chloride 105 mEq/L, bicarbonate 23 mEq/L, BUN 10 mg/dL, creatinine 1.7 mg/dL, glucose 85 mg/dL, arterial blood pH 7.37, and lactate 1.4 mEq/L. Urinalysis shows moderate ketones. Which of the following is the most likely diagnosis? a. Diabetic ketoacidosis (DKA) b. Ethanol intoxication c. Methanol intoxication d. Isopropyl alcohol intoxication e. Ethylene glycol intoxication

*D* Isopropyl alcohol is one of the toxic alcohols (ethylene glycol, methanol, and isopropyl alcohol). It is a clear, colorless liquid with a bitter taste. It is commonly used as a rubbing alcohol and as a solvent in haircare products, skin lotion, and home aerosols. Moreover, it is often ingested as an inexpensive and convenient substitute for ethanol. Clinically, GI and CNS complaints predominate. Its GI irritant properties cause patients to complain of abdominal pain, nausea, and vomiting. Pupillary size varies but miosis is commonly observed. Large ingestions can result in coma. Hypotension, although rare, signifies severe poisoning. Characteristically, metabolic acidosis, unlike the other toxic alcohols, is not present. This is because isopropyl alcohol is metabolized to acetone, a ketone, not an acid. It is also the cause for the presence of urinary ketones and the odor on the patient's breath. Isopropyl alcohol intoxication is often remembered by ketosis without acidosis. Another unique finding is the presence of "pseudo renal failure" or isolated false elevation of creatinine with a normal BUN. This results from interference of acetone and acetoacetate by the colorimetric method used to measure the creatinine level.

A 22-year-old college student presents to the ED with a painful, swollen finger. He states that he was playing with a roommate's dog and was bitten 3 days earlier. On physical examination, his heart rate (HR) is 70 beats per minute, blood pressure (BP) is 115/65 mm Hg, respiratory rate (RR) is 16 breaths per minute, and temperature is 101.6°F. Bilaterally, the radial pulses remain equal, and the hands and fingers are intact to sensation. His right second finger is held in flexion, and is symmetrically swollen from the distal tip to the MCP joint. Both your attempt to extend the digit passively and palpate the flexor tendon sheath produce great pain. Based on these findings you make your diagnosis and notify the hand surgeon immediately. What name is given to the criteria used to make the diagnosis? a. Finkelstein b. Trousseau c. Tinel d. Kanavel e. Phalen

*D* Kanavel criteria for flexor tenosynovitis are known as S.T.E.P.: (1) Symmetric swelling of the finger, (2) Tenderness over the flexor tendon sheath, (3) Extension (passive) of the digit is painful, and (4) Posture of the digit is flexed. A tenosynovitis is an infection of the flexor tendon sheath caused by penetrating trauma and dirty wounds (eg, a dog bite). Infection spreads along the tendon sheath; therefore, failure to diagnose and treat a flexor tenosynovitis may lead to loss of function of the affected digit and eventually the entire hand. It is, therefore, a surgical emergency. Treatment includes hand immobilization and elevation, immediate consultation with a hand surgeon, and IV antibiotics. Pain control and tetanus immunization should be provided, if not up-to-date.

You are called to the bedside of a hypotensive patient with altered mental status. The nurse hands you an ECG which shows atrial flutter at 150 beats per minute with 2:1 arteriovenous (AV) block. You feel that the patient is unstable and elect to perform emergency cardioversion. You attach the monitor leads to the patient. What is the critical next step in electrical cardioversion? a. Set the appropriate energy level b. Position conductor pads or paddles on patient c. Charge the defibrillator d. Turn on the synchronization mode e. Administer 25 μg of fentanyl IV

*D* Low-energy cardioversion is very successful in converting atrial flutter to sinus rhythm. Remember, cardioversion is different than defibrillation. Cardioversion is performed on patients with organized cardiac electrical activity with pulses, whereas defibrillation is performed on patients without pulses (VF and VF without a pulse). Patients with heart beats who receive electrical energy during their heart's relative refractory period are at risk for VF. Therefore, cardioversion is a timed shock designed to avoid delivering a shock during the heart's relative refractory period. By activating synchronization mode, the machine will identify the patient's R waves and not deliver electrical energy during these times. The key step when cardioverting is to activate the synchronization mode and confirm the presence of sync markers on the R waves prior to delivering electrical energy.

A 51-year-old diabetic man complains of intense right-ear pain and discharge. On physical examination, his BP is 145/65 mm Hg, HR 91 beats per minute, and temperature 101°F. He withdraws when you retract the pinna of his ear. The external auditory canal is erythematous, edematous, and contains what looks like friable granulation tissue in the external auditory canal. The tympanic membrane is partially obstructed but appears to be erythematous, as well. Which of the following statements regarding this patient's condition is true? a. It is a common complication of otitis externa that afflicts otherwise healthy patients. b. The mainstay of treatment is outpatient with oral antibiotics. c. Patients are usually afebrile and have minimal ear pain. d. It is caused by P aeruginosa. e. Hearing loss is the most common complication.

*D* Malignant or necrotizing otitis externa is a complication of otitis externa that occurs primarily in adult diabetics and debilitated and immunocompromised individuals. It is associated with a high mortality rate. The condition is better described as an osteitis of the underlying bone of the external auditory canal caused by P aeruginosa. It is distinguished by fever, intense ear pain, erythema, edema, and granulation tissue in the external canal. Cranial nerve palsies and trismus can also occur. These patients require hospitalization and treatment with IV antipseudomonal antibiotics and possible surgical debridement. CT and MRI are appropriate studies to evaluate for osteomyelitis. Patients also require immediate ENT consultation.

A 33-year-old woman presents to the ED complaining of fever, vomiting, and gradually worsening RUQ pain. She states that her pain radiates to her back. Her BP is 130/75 mm Hg, HR is 90 beats per minute, temperature is 100.9°F, and RR is 17 breaths per minute. While examining her abdomen you palpate her RUQ and notice that she momentarily stops her inspiration. What is the name of this classic sign? a. Grey-Turner sign b. Kernig sign c. McMurray sign d. Murphy sign e. McBurney sign

*D* Murphy sign is named after the Chicago surgeon, John B. Murphy. The patient is asked to take a deep breath while the examiner applies pressure over the area of the gallbladder. If the gallbladder is inflamed, the descending diaphragm forces it against the examiner's fingertips, causing pain and often a sudden pause to inspiration. A sonographic Murphy sign elicits the same response with an ultrasound probe over the gallbladder.

A 74-year-old lethargic woman is brought to the ED by her family. Her daughter states that the patient has been progressively somnolent over the last week and could not be woken up today. The patient takes medications for diabetes, hypertension, hypothyroidism, and a recent ankle sprain, which is treated with a hydrocodone/acetaminophen combination. In the ED, the patient is profoundly lethargic, responsive only to pain, and has periorbital edema and delayed relaxation of the deep tendon reflexes. Her BP is 145/84 mm Hg, HR is 56 beats per minute, temperature is 94.8°F, and RR is 12 breaths per minute. Which of the following is the most likely diagnosis? a. Hypoglycemia b. Opioid overdose c. Stroke d. Myxedema coma e. Depression

*D* Myxedema coma is a lifethreatening complication of hypothyroidism. Mortality in myxedema coma approaches 20% to 50% even with appropriate management. The patient exhibits classic signs and symptoms of the disease: lethargy or coma, hypothermia, bradycardia, periorbital and nonpitting edema, and a delayed relaxation phase of deep tendon reflexes (areflexia in more severe cases). Myxedema coma can be triggered by sepsis, trauma, surgery, congestive heart failure, prolonged cold exposure, or use of sedatives or narcotics (as seen in this example).

A 27-year-old man complains of chest palpitations and lightheadedness for the past hour. He has no past medical history and is not taking any medications. He drinks a beer occasionally on the weekend and does not smoke cigarettes. His HR is 180 beats per minute, BP is 110/65 mm Hg, and oxygen saturation is 99% on room air. An ECG reveals a HR of 180 beats per minute with a QRS complex of 90 msec with a regular rhythm. There are no discernable P waves. Which of the following is the most appropriate medication to treat this dysrhythmia? a. Digoxin b. Lidocaine c. Amiodarone d. Adenosine e. Bretylium

*D* Narrow-complex tachycardias are defined as rhythms with a QRS complex duration less than 100 ms and a ventricular rate greater than 100 beats per min. Although virtually all narrow-complex tachydysrhythmias originate from a focus above the ventricles, the term supraventricular tachycardia (SVT) is conventionally used to denote those rhythms aside from sinus tachycardia, atrial tachycardia, atrial fibrillation, and atrial flutter (eg, atrioventricular nodal reentry tachycardia and atrioventricular reentry tachycardia). Adenosine, an ultrashortacting AV-nodal blocking agent, is typically used to treat SVTs. Because it is so fast-acting, it must be delivered through a large vein (eg, the antecubital fossa) with a rapid intravenous fluid bolus. In addition to adenosine, maneuvers that increase vagal tone have been shown to slow conduction through the AV node. Some of these maneuvers include carotid sinus massage, Valsalva maneuver, and facial immersion in cold water. Lidocaine and amiodarone, although may be effective in treating narrow-complex tachycardias, are agents generally used to treat wide-complex or ventricular tachycardias.

A 30-year-old man is brought to the ED by police officers. The patient is agitated, vomiting, and complaining of body aches. He states that he is withdrawing from his medication. His vital signs are BP 160/85 mm Hg, RR 20 breaths per minute, HR 107 beats per minute, and temperature 99.7°F. On examination he is diaphoretic, has rhinorrhea, piloerection, and hyperactive bowel sounds. Which of the following substances is this patient most likely withdrawing from? a. Ethanol b. Cocaine c. Nicotine d. Methadone e. Clonidine

*D* Opioid withdrawal initially presents with drug craving, yawning, rhinorrhea, and piloerection and progresses to nausea, vomiting, diarrhea, hyperactive bowels, diaphoresis, myalgias, arthralgias, anxiety, fear, and mild tachycardia. Methadone withdrawal starts approximately 24 hours after the last dose and persists for 3 to 7 days. Heroin withdrawal begins about 6 hours after the last dose and usually fully manifests at 24 hours. Opioid withdrawal is not a life-threatening condition as long as adequate hydration and nutritional support is maintained.

A 25-year-old man is brought into the ED by two police officers because of suspected drug use. The patient is extremely agitated and is fighting the police officers. It takes three hospital staff members and the two police officers to keep him on the stretcher. His vital signs are: BP 150/80 mm Hg, HR 107 beats per minute, temperature 99.7°F, RR 18 breaths per minute, and oxygen saturation 99% on room air. Physical examination is unremarkable except for cool, diaphoretic skin, persistent vertical and horizontal nystagmus, and occasional myoclonic jerks. Which of the following is the most likely diagnosis? a. Cocaine intoxication b. Cocaine withdrawal c. Amphetamine intoxication d. PCP intoxication e. Opiate withdrawal

*D* PCP intoxication is characterized by a wide spectrum of findings. Behavior may be bizarre, agitated, confused, or violent. The hallmark of PCP toxicity is the recurring delusion of superhuman strength and invulnerability resulting from both the anesthetic and dissociative properties of the drug. Patients have broken police handcuffs, fracturing bones in doing so. The major cause of death or injury from PCP is behavioral toxicity leading to suicide and provoked homicide. Typical neurologic signs include nystagmus (horizontal, vertical, or rotary), ataxia, and altered gait. Pupils are usually midsized and reactive, but can be mydriatic or miotic. Bizarre posturing, grimacing, and writhing may be seen. Management is conservative. To prevent self-injury, the patient must be safely restrained. Antipsychotics or benzodiazepines are frequently administered for chemical sedation. PCP intoxication usually ranges from 8 to 16 hours, but can last longer in chronic users.

A 35-year-old woman presents to the ED for the second time. She is complaining of fever, neck stiffness, and photophobia. She was seen in your ED 2 days ago for the same symptoms. At that time, she had a normal neurologic examination, was otherwise well-appearing, and underwent diagnostic LP. The results of her CSF analysis were as follows: Glucose 82 mg/dL, Protein 60 mg/dL, WBC 150/ L (98% lymphocytes), Gram stain No organisms seen. The patient was sent home after a period of observation with presumed viral meningitis. She was told to return if her symptoms were not better in 48 hours. Since then, her fever increased (100.4°F-102.2°F). What is the next most appropriate step in management? a. Administration of acyclovir b. Aggressive antipyretic therapy and observation c. CT scan of the sinuses d. CT of the head and, if no contraindication, repeat LP e. Viral cultures and polymerase chain reaction (PCR) from previously obtained CSF

*D* Patients with CSF analysis consistent with viral meningitis can be managed as outpatients with close follow-up. These patients must be reliable for follow-up, not immunocompromised and otherwise well-appearing. They should be told to return for reevaluation if their symptoms do not improve within 48 hours. If they have not improved, reevaluation including neuroimaging, repeat LP, and treatment with antibiotics is indicated.

A 76-year-old woman with a history of congestive heart failure, coronary artery disease, and an "irregular heart beat" is brought to the ED by her family. She has been complaining of increasing abdominal pain over the past several days. She denies nausea or vomiting and bowel movements remain unchanged. Vitals are HR of 114 beats per minute, BP 110/75 mm Hg, and temperature 98°F. On cardiac examination, her HR is irregularly irregular with no murmur detected. The abdomen is soft, nontender, and nondistended. The stool is heme-positive. This patient is at high risk for which of the following conditions? a. Perforated gastric ulcer b. Diverticulitis c. Acute cholecystitis d. Mesenteric ischemia e. Sigmoid volvulus

*D* Patients with coronary artery disease, valvular heart disease, and arrhythmias, particularly atrial fibrillation, are at high risk for mesenteric ischemia. In addition, age greater than 50 years, congestive heart failure, recent myocardial infarction, critically ill patients with sepsis or hypotension, use of diuretics or vasoconstrictive medications, and hypercoagulable states place patients at higher risk. The most common cause of acute mesenteric ischemia is arterial embolus, which accounts for 50% of cases. The classic finding is "pain out of proportion to examination findings," that is, a patient complains of severe pain but is not particularly tender on examination. A high degree of suspicion for mesenteric ischemia in an elderly patient with abdominal pain is warranted.

A 19-year-old man suffers a single gunshot wound to the left chest and is brought in by his friends. He is complaining of chest pain. On examination, his temperature is 99°F, BP is 70/40 mm Hg, HR is 140 beats per minute, RR is 16 breaths per minute, and oxygen saturation is 96% on room air. He has distended neck veins, but his trachea is not deviated. Lungs are clear to auscultation bilaterally. Heart sounds are difficult to appreciate, but you feel a bounding, regular pulse. Abdomen is soft and nontender. Extremity examination is normal. Two large-bore IV lines are placed and the patient is given 2 L of normal saline. Chest radiograph shows a globular cardiac silhouette, but a normal mediastinum and no pneumothorax. What is the definitive management of this patient? a. Intubation b. Tube thoracostomy c. Pericardiocentesis d. Thoracotomy e. Blood transfusion

*D* Patients with penetrating trauma to the chest with possible cardiac injury and signs of hemodynamic instability need immediate operative thoracotomy. This patient has signs of cardiac tamponade, a collection of blood surrounding the heart and interfering with the heart's ability to contract. He has Beck triad of hypotension, distended neck veins, and muffled heart sounds. His CXR also shows an enlarged heart. An echocardiogram would be helpful in confirming the diagnosis, but since this patient is unstable and echocardiogram may not be readily available, the treatment is immediate thoracotomy in the operating room.

A 40-year-old G1P0101 presents to the ED with suprapubic pain and general malaise for the last 3 days without improvement. Her vitals are significant for HR of 115 beats per minute, BP of 100/60 mm Hg, and temperature of 101°F. Upon physical examination, her abdomen is soft with significant suprapubic tenderness to light palpation but no rebound or guarding. Her pelvic examination reveals a scant amount of dark red vaginal blood and yellow discharge. Her past medical history is significant for an emergent cesarean section 2 weeks ago without any other gynecologic history. What is the most likely diagnosis in this patient? a. Uterine atony b. Retained products of conception c. Uterine inversion d. Endometritis e. Tubo-ovarian torsion

*D* Postpartum bleeding is classified as early, within 24 to 48 hours of delivery, or late, up to 1 to 2 weeks. Causes of early postpartum bleeding include uterine atony, genital tract trauma, retained products of conception, and uterine inversion. Late bleeding episodes may be caused by endometritis or retained products of conception. Patients with endometritis most often present with fever, vaginal discharge, general malaise, and vaginal bleeding. Upon pelvic examination, the uterus will be soft and tender to the touch. The majority of infections are caused by normal vaginal flora, such as anaerobes, enterococci, and streptococci. Patients who do not respond to initial antibiotic therapy may warrant broader spectrum coverage and need to be further evaluated for a pelvic abscess or pelvic thrombophlebitis.

A 26-year-old G1P1001 presents to the ED with vaginal spotting for the last 3 days with occasional left-sided pelvic pain. Her physical examination includes a closed cervical os, scant blood within the vaginal vault with left adnexal tenderness. Given this patient's history and physical examination, you suspect an ectopic pregnancy. In addition to a quantitative β-hCG, which of the following laboratory tests may prove helpful in the evaluation of this patient? a. Estrogen level b. Follicle-stimulating hormone (FSH) level c. Thyrotropin (TSH) level d. Progesterone level e. CBC

*D* Progesterone levels may be helpful in distinguishing ectopic pregnancies. A level of < 5 ng/mL is highly suggestive of an ectopic pregnancy, given that the uterus may not be able to carry the pregnancy. Higher levels indicate that embryo implantation inside the uterus is more likely.

A 26-year-old woman, who was a belted front-seat driver in a headon motor vehicle collision, is brought to the ED. She is speaking but complains of progressively worsening shortness of breath and hemoptysis. Her BP is 135/75 mm Hg, HR is 111 beats per minute, RR is 24 breaths per minute, and oxygen saturation is 96% on non-rebreather. On examination, you note ecchymosis over the right side of her chest. Her breath sounds are equal bilaterally. There is no bony crepitus and the trachea is midline. After placing two large-bore IVs and completing the primary and secondary surveys, you view the chest radiograph shows frank consolidation. Which of the following is the most likely diagnosis? a. Diaphragm rupture b. Hemothorax c. Tension pneumothorax (PTX) d. Pulmonary contusion e. Acute respiratory distress syndrome (ARDS)

*D* Pulmonary contusions usually occur after a blunt traumatic force to the chest that causes injury to the lung parenchyma. This is followed by alveolar edema and hemorrhage. Pulmonary contusion is reported to be present in 30% to 75% of patients with significant blunt chest trauma, most often from automobile collisions with rapid deceleration. Typical radiographic findings begin to appear within minutes of injury and range from patchy, irregular, alveolar infiltrate to frank consolidation.

A 5-month-old boy, ex-34 week preemie, is brought to the ED by his mother who reports that the infant has been breathing with extra effort for the last 2 days. He has no other past medical history and is current on immunizations having received the full course of vaccines at both 2 and 4 months. The mom reports the child has had rhinorrhea and a cough. Upon physical examination, the patient has a temperature of 101.1°F, HR 160 beats per minute, RR 70 breaths per minute, pulse oximetry of 87% on room air. The infant has copious nasal discharge, audible wheezing with diffuse rhonchi and rales upon chest auscultation. He also has intercostal retractions and nasal flaring. A chest radiograph shows increased perihilar markings, hyperinflation and diffuse patchy areas of atelectasis versus infiltrates. Given this patient's history and physical examination, which of the following is the most likely etiology of his symptoms? a. Foreign body aspiration b. Asthma c. Pneumococcal pneumonia d. Respiratory syncytial virus (RSV) e. Parvovirus B19

*D* RSV is a common cause of respiratory distress in infants, especially under 6 months of age, and the most common cause of the clinical condition known as bronchiolitis. Diagnosis is made clinically on the basis of the constellation of symptoms, as this patient demonstrates. Patients who are at risk of increased disease severity are those with chronic lung disease, especially ex-preemies, those with an immunocompromised condition, and those with complex congenital heart disease. These patients are eligible for receiving palivizumab, an injectable antibody that confers passive immunity and can reduce the likelihood of severe lower respiratory tract disease caused by RSV in susceptible individuals. The condition associated with RSV infections overlap with those of other respiratory tract pathogens including rhinovirus, parainfluenzavirus, coronavirus, echovirus, and coxsackievirus. The clinical condition of bronchiolitis is caused by RSV in approximately 70% of cases, especially during the peak months of November to April. The diagnosis is confirmed by a rapid antigen test of a nasal aspirate, but this is typically only useful in patients whose diagnosis is unclear or in whom management may be different based on concomitant conditions. Management is largely supportive. A variety of medications including albuterol, aerosolized epinephrine, and corticosteroids are often administered. However, none have been shown to be consistently beneficial in treating the symptoms or reducing the duration of illness. Respiratory precautions are necessary to limit transmission.

A 55-year-old man with a history of alcoholism and osteoarthritis developed left knee pain several days after a fall from standing height. The patient was brought to the ED by ambulance after being found on a park bench stating he was unable to walk because of the pain. On physical examination, his left knee is warm, diffusely tender, and swollen with a large effusion. He has pain on passive range of motion. His BP is 150/85 mm Hg, HR is 105 beats per minute, temperature is 102.7°F, RR is 16 breaths per minute, and fingerstick glucose is 89 mg/dL. Which of the following is the most appropriate diagnostic test? a. Knee radiographs b. Magnetic resonance imaging (MRI) c. Erythrocyte sedimentation rate (ESR) and C-reactive protein d. Arthrocentesis e. Bone scan

*D* Septic arthritis is an infection of a joint space, most commonly the knee, followed by the hip, shoulder, and wrist. Patient's present with a warm, tender, erythematous, swollen joint and pain with passive range of motion. Fever and chills are common. Arthrocentesis is diagnostic with joint fluid demonstrating a WBC count > 50,000/μL with > 75% granulocytes. Staphylococcus aureus remains the predominant pathogen for all age groups. In young adults gonococcal septic arthritis is common. Patients with septic arthritis should receive a first dose of antibiotics in the ED prior to admission. If a coincident cellulitis is present over the involved joint, arthrocentesis may need to be delayed.

A 61-year-old man presents to the ED with chest wall pain after a motor vehicle collision. He is speaking full sentences, breath sounds are equal bilaterally, and his extremities are well-perfused. His BP is 150/75 mm Hg, HR is 92 beats per minute, and oxygen saturation is 97% on room air. Chest radiography reveals fractures of the seventh and eighth ribs of the right anterolateral chest. He has no other identifiable injuries. Which of the following is the most appropriate treatment for this patient's rib fractures? a. Apply adhesive tape on the chest wall perpendicular to the rib fractures b. Insert a chest tube into the right thorax c. Bring the patient to the OR for surgical fixation d. Analgesia and incentive spirometry e. Observation

*D* Simple rib fractures are the most common form of significant chest injury. Ribs usually break at the point of impact or at the posterior angle, which is structurally the weakest area. The fourth through ninth ribs are most commonly involved. Rib fractures occur more commonly in adults than in children owing to the relative inelasticity of the adult chest wall compared to the more compliant chest wall of children. The presence of two or more rib fractures at any level is associated with a higher incidence of internal injuries. The treatment of patients with simple acute rib fractures includes adequate pain relief and maintenance of pulmonary function. Oral pain medications are usually sufficient for young and healthy patients. Older patients may require better analgesia with opioids, but care must be taken to avoid over sedation. Continuing daily activities and deep breathing is important to ensure ventilation and prevent atelectasis. If there is a question about the patient's ability to cough, breathe deeply, and maintain activity, particularly if two or more ribs are fractured, it is preferable to admit the patient to the hospital for aggressive pulmonary care.

A 69-year-old man is brought to the ED by his son, who states that his father developed left arm and leg weakness this afternoon and now has difficulty walking. The patient states that he has a history of heart palpitations and recently stopped taking his blood-thinning medicine because it was giving him an upset stomach. His BP is 165/90 mm Hg, HR is 97 beats per minute, temperature is 98.9°F, and RR is 16 breaths per minute. You suspect the patient is having a stroke and rush him to the CT scanner. The result of the head CT is shows large hypodensity in the right parietal-occipital region. What percentage of all stroke patients will have this type of stroke? a. 20% b. 40% c. 60% d. 80% e. 95%

*D* The CT image shows a large hypodensity in the right parietal-occipital region representing an ischemic stroke. Ischemic strokes comprise 80% of all strokes, with hemorrhagic strokes accounting for the other 20%. Ischemic events include thrombotic (thrombus forming at the site of an ulcerated atherosclerotic plaque), embolic (thrombus embolized to a distal site), and lacunar (small terminal artery occlusion) strokes. This patient likely had a cardioembolic secondary to atrial fibrillation. Atrial fibrillation is an important risk factor for an embolic stroke, particularly when patients are noncompliant with anticoagulation therapy, as the patient in the vignette. Intracranial bleeding secondary to a hemorrhagic event appears hyperdense on CT scan.

A 3-month-old boy is brought to the ED by his mother because of decreased feeding over the last 24 hours. The patient has been doing well without any prior medical problems until today when the mother noted sweating and irritability, particularly with feeding. In the ED, the patient attempts to feed but within minutes stops and begins to cry. Vital signs include a pulse of 240 beats per minute, RR of 50 breaths per minute, temperature of 98.2°F, and pulse oximetry of 98% on room air. On physical examination, the patient is pale and clammy to touch. Breath sounds are clear on auscultation. Pulses are normal and symmetric in all extremities. An ECG is showsa narrow complex tachycardia at 300 beats per minute with no variability and absent P waves. Which of the following is the most appropriate next step in management? a. Synchronized cardioversion at 0.5 J/kg b. Verapamil 0.1 mg/kg bolus c. Defibrillation at 2 J/kg d. Adenosine at 0.1 mg/kg followed by 0.2 mg/kg if first dose is ineffective e. Carotid massage

*D* The ECG shows a narrow complex tachycardia at 300 beats per minute with no variability and absent P waves. This is diagnostic of supraventricular tachycardia (SVT). SVT is the most common pathologic arrhythmia of childhood. It is sometimes confused with sinus tachycardia, which usually presents at a rate of < 225 beats per minute in infants and < 150 beats per minute in older children and adults. This finding coupled with variability with respirations and evidence of normal P waves will prove to be useful in differentiating sinus tachycardia from SVT. This dysrhythmia is typically well tolerated in young children and infants. SVT will often present with a history of pallor, poor feeding, tachypnea, and lethargy, or irritability. Older children will describe palpitations, lightheadedness, and shortness of breath. Signs of congestive heart failure or shock may be present. SVT can occur in children with no structural lesions and is associated with fever, infection, or sympathomimetic drugs (such as cold medicine or bronchodilators), but is most often idiopathic. Stability is the most important factor in managing patients with SVT. Stable patients usually have normal mental status and only mild symptoms. Unstable patients typically present in congestive heart failure or shock. Adenosine is safe and effective and has a short half-life. It is the first-line therapy in patients with stable SVT. It must be administered rapidly through a large-bore IV in a vein as close to the heart as possible (antecubital is classic and generally adequate) and followed by a 10- to 20-mL normal saline bolus using the double stopcock method to flush it quickly. Adenosine will block conduction at the atrioventricular (AV) node leading to a brief period of asystole, which can be very disconcerting to those receiving or administering the medication.

A 75-year-old woman is transferred to your ED from the local nursing home for fever, cough, and increasing lethargy. Over the past 3 days, the nursing home staff noticed increasing yellow sputum and decreasing urine output from the patient. Her BP is 118/75 mm Hg, RR is 20 breaths per minute, HR is 105 beats per minute, temperature is 100.9°F, and pulse oxygenation is 94% on room air. On examination, the patient appears dehydrated. Auscultation of the lungs reveals bibasilar crackles. Laboratory results reveal WBC 14,500/μL, hematocrit 39%, platelets 250/μL, sodium 132 mEq/L, potassium 3.5 mEq/L, chloride 100 mEq/L, bicarbonate 18 mEq/L, BUN 27 mg/dL, creatinine 1.5 mg/dL, and glucose 85 mg/dL. Serum lactate is 4.7 mol/dL. Based on this patient's presentation, how would you classify her condition? a. Systemic inflammatory response syndrome (SIRS) b. Severe systemic inflammatory response syndrome c. Sepsis d. Severe sepsis e. Septic shock

*D* The basis of sepsis (c) is the presence of infection and the subsequent physiologic alterations in response to that infection, namely the activation of the inflammatory cascade. (a) SIRS is a term used to define this clinical condition, and is considered present if abnormalities in two of the following four clinical parameters exist: (1) body temperature, (2) HR, (3) RR, and (4) peripheral leukocyte count. These criteria include body temperature < 36°C (96.8°F) or > 38°C (100.4°F), rather > 90 beats per minute, RR > 20 breaths per minute or, on blood gas, a PaCO2 less than 32 mm Hg, and WBC count < 4000 cells/mm3 or > 12,000 cells/mm3, or greater than 10% band forms. This patient's condition is most consistent with severe sepsis, which is defined as sepsis with acute organ dysfunction. Some examples of end-organ dysfunction include acute lung injury, encephalopathy, confusion, progressive oagulopathy caused by inability to synthesize clotting factors, elevated unconjugated serum bilirubin, oliguria, anuria, and systolic and diastolic heart failure. Severe SIRS (b) is defined as end-organ dysfunction without evidence of infection. Septic shock (e) is defined as sepsis with persistent hypotension despite fluid resuscitation and resulting tissue hypoperfusion.

A 67-year-old man is brought to the ED by emergency medical service (EMS). His wife states that the patient was doing his usual chores around the house when all of a sudden he started complaining of severe abdominal pain. He has a past medical history of coronary artery disease and hypertension. His BP is 85/70 mm Hg, HR is 105 beats per minute, temperature is 98.9°F, and his RR is 18 breaths per minute. On physical examination, he is diaphoretic and in obvious pain. Upon palpating his abdomen, you feel a large pulsatile mass. An electrocardiogram (ECG) reveals sinus tachycardia. You place the patient on a monitor, administer oxygen, insert two large-bore IVs, and send his blood to the laboratory. His BP does not improve after a 1-L fluid bolus. Which of the following is the most appropriate next step in management? a. Order a CT scan to evaluate his aorta. b. Call the angiography suite and have them prepare the room for the patient. c. Order a portable abdominal radiograph. d. Call surgery and have them prepare the operating room (OR) for an exploratory laparotomy. e. Call the cardiac catheterization laboratory to prepare for stent insertion.

*D* The classic triad of a ruptured abdominal aortic aneurysm (AAA) is pain, hypotension, and a pulsatile abdominal mass. Sometimes patients have only one or two of the components and occasionally may have none. Most patients who are diagnosed with AAA are asymptomatic. However, rupture is often the first manifestation of an AAA. Most patients with a ruptured AAA experience pain in the abdomen, back, or flank. It is usually acute in onset and severe. Approximately 20% of the time, patients present to the ED with syncope. Patients with a ruptured AAA are unstable until their aorta is cross clamped in the OR. Therefore, any hemodynamically unstable patient with a diagnosed or strongly suspected AAA should be taken immediately to the OR

A 19-year-old college student presents to the ED complaining of headache, sore throat, myalgias, and rash that developed over the previous 12 hours. Her BP is 95/60 mm Hg, HR is 132 beats per minute, temperature is 103.9°F, RR is 19 breaths per minute, and oxygen saturation is 98% on room air. She is confused and oriented only to person. Physical examination is remarkable for pain with neck flexion, a petechial and purpuric rash on her extremities, and delayed capillary refill. Which of the following best describes the emergency physicians' priority in managing this patient? a. Collect two sets of blood cultures prior to antibiotic administration. b. Call the patient's parents and have them come immediately to the hospital. c. Call her roommate to gather more information. d. Begin fluid resuscitation, administer intravenous (IV) antibiotics, and perform a lumbar puncture (LP). e. Administer acetaminophen to see if her headache and fever resolve.

*D* The first question an emergency physician asks for each patient is whether a life-threatening process is causing the patient's complaint. Emergency medicine is primarily a complaint-oriented, rather than a disease-oriented specialty. Its emphasis rests on anticipating and recognizing a life-threatening process rather than seeking the diagnosis. The goal is to think about and plan to prevent the life-threatening things from happening or progressing in the patient. The patient in the vignette may have meningitis, a life-threatening condition, or a viral syndrome with dehydration. The initial approach is stabilization and treatment or prevention of a life-threatening process. This patient requires fluid resuscitation for her BP, altered mental status, and delayed capillary refill. Antibiotics should be started immediately and an LP performed once increased ICP is evaluated by funduscopic examination or CT scan. She should be placed in isolation. Her disposition is directed by her response to initial resuscitation and results of the LP.

A 45-year-old man is brought to the ED after a head-on motor vehicle collision. Paramedics at the scene tell you that the front-end of the car is smashed. The patient's BP is 130/80 mm Hg, HR is 100 beats per minute, RR is 15 breaths per minute, and oxygen saturation is 98% on room air. Radiographs of the cervical spine reveal bilateral fractures of the C2 vertebra. The patient's neurologic examination is unremarkable. Which of the following best describes this fracture? a. Colles fracture b. Boxer's fracture c. Jefferson fracture d. Hangman's fracture e. Clay-shoveler's fracture

*D* The hangman's fracture, or traumatic spondylolysis of C2, occurs when the head is thrown into extreme hyperextension as a result of abrupt deceleration, resulting in bilateral fractures of the pedicles. The name "hangman's fracture" was derived from judicial hangings, where the knot of the noose was placed under the chin which caused extreme hyperextension of the head on the neck, resulting in a fracture at C2. However, many hangings resulted in death from strangulation rather than spinal cord damage. Today, the most common cause of a hangman's fracture is the result of head-on automobile collisions.

A 30-year-old man with type 1 diabetes presents to the emergency department (ED). His blood pressure (BP) is 100/70 mm Hg and heart rate (HR) is 140 beats per minute. His blood glucose is 750 mg/dL, potassium level is 5.9 mEq/L, bicarbonate is 5 mEq/L, and arterial pH 7.1. His urine is positive for ketones. Which of the following is the best initial therapy for this patient? a. Give normal saline as a 2-L bolus then administer 20 U of regular insulin subcutaneously. b. Bolus 2 ampules of bicarbonate and administer 10 U of insulin intravenously. c. Give him 5 mg of metoprolol to slow down his heart, start intravenous (IV) hydration, and then give 10 U of regular insulin intravenously. d. Give normal saline in 2-L bolus and then administer 10 U of insulin intravenously followed by an insulin drip and continued hydration. e. Give normal saline in 2-L bolus with 20 mEq/L KCl in each bag.

*D* The mainstay of treatment for DKA is aggressive fluid resuscitation and insulin therapy. The patient should receive 2 L of normal saline within 2 hours of presentation followed by 4 to 6 L over the next 8 to 12 hours depending on the patient's fluid status. In DKA, the average adult has a water deficit of 5 to 10 L. After fluid administration, regular insulin is administered usually first as a 10-U bolus intravenously and then at a rate of 0.1 U/kg/h. Insulin must be administered for ketosis and acidosis to resolve.

A 17-year-old man presents to the ED with a lip laceration. He sustained the laceration during an altercation. On examination, he has a horizontal, 2-cm laceration of the right lower lip crossing the midline. There is significant swelling of the lower lip. The attending asks you, the intern, to perform a mental block for anesthesia. Which of the following statements regarding mental blocks is correct? a. A mental block is performed by injecting lidocaine into the mental nerve foramen. b. The patient requires a right-sided mental block. c. A mental block is not appropriate in this patient because it will lead to worsened swelling of the lower lip swelling. d. A mental block anesthetizes the mental nerve which innervates the lower lip. e. A mental block is performed with a 3-cc syringe attached to an 18-gauge needle.

*D* The mental nerve is a continuation of the inferior alveolar nerve. It emerges from the mental foramen below the second premolar and innervates the mucosa and skin of the lower lip. A mental nerve block is the most appropriate way to anesthetize this patient given his lip swelling.

A 6-day-old infant is brought to the ED by his mom who describes the newborn as breathing fast, poor feeding, and appearing blue. He has no history of fever or vomiting. The patient was born full-term at home to a G4P3 mother with an uncomplicated antenatal course. The mom had prenatal laboratory tests but is unaware of the results. On examination, the patient is lethargic with central and peripheral cyanosis. The rectal temperature is 95.4°F, HR 180 beats per minute, RR 70 breaths per minute, and BP unobtainable in the extremities by automatic pressure meter. The oxygen saturation on room air is 65% which does not improve with administration of 100% oxygen by face mask. Auscultation reveals a harsh 3/6 systolic murmur with an active precordium. Lungs reveal diffuse, bilateral rales and wheezes. Liver edge is palpated 3 to 4 cm below right costal margin. Which of the following is the most important next step in the management of this patient? a. Intubation for administration of 100% oxygen b. STAT portable chest radiograph and electrocardiogram (ECG) c. Administration of IV antibiotics and full sepsis workup d. Administration of prostaglandin bolus followed by continuous drip e. Immediate surgical intervention and activation of extracorporeal membrane oxygenation (ECMO) team

*D* The most important next step is administration of prostaglandin bolus followed by a drip. This patient is in severe cardiac failure resulting from impending closure of his ductus arteriosus. The patient is presenting in a classic time frame, approximately 4 to 7 days after birth. Ductal-dependent congenital heart lesions that present in this manner typically include transposition of the great arteries, truncus arteriosus, total anomalous pulmonary venous return, left heart hypoplasia, and coarctation. The most important next step is administration of a medication that will assist the ductus to remain open while preparing for surgical intervention.

A 78-year-old man presents to the emergency department (ED) complaining of left arm weakness that started 10 minutes ago in the clinic. The patient states that he has a history of hypertension and diabetes, but has never had similar symptoms in the past. He is feeling well otherwise. His blood pressure (BP) is 157/85 mm Hg, heart rate (HR) is 87 beats per minute, temperature is 98.8°F, and respiratory rate (RR) is 14 breaths per minute. His neurologic examination is unremarkable and the patient embarrassingly states that his left arm is no longer weak. Which of the following is the most likely diagnosis? a. Thrombotic stroke b. Conversion disorder c. Migraine with focal neurologic deficit d. Transient ischemic attack (TIA) e. Todd paralysis

*D* The patient had a TIA, which involves neurologic deficits that resolve within 24 hours of onset. TIAs often precede ischemic stroke; up to 50% of patients with a TIA will have a stroke in the next 5 years, with the highest incidence in the first month. It is important to recognize TIAs and to evaluate patients for cardiac or carotid arterial sources of emboli. Although the symptoms often resolve, many patients with a TIA will have evidence of infarction on CT/MRI.

A 47-year-old man presents to urgent care complaining of a 2-day history of sore throat and subjective fever at home. He denies cough or vomiting. His BP is 130/75 mm Hg, HR is 85 beats per minute, temperature is 101°F, and his RR is 14 breaths per minute. He has tonsillar swelling without exudates and bilaterally enlarged and tender lymph nodes of the neck. The rapid streptococcal antigen test is negative. Which of the following is the next best step in management? a. Administer penicillin and discharge the patient. b. Schedule a lymph node biopsy to rule out lymphoma. c. Observe for 6 hours. d. Perform a throat culture, symptomatic care, and treat if results are positive. e. Administer amantadine to patient and all contacts.

*D* The patient has a modified Centor score of 2 (history of fever, tender adenopathy, no cough, age > 45 years). (subtract 1 for being more than 45)

A 25-year-old man presents to the ED complaining of headache for 2 days. He describes the pain as pulsatile and occipital. The patient had an LP 3 days prior and was diagnosed with viral meningitis after 4 days of symptoms. Noncontrast head CT at that time was normal. He improved shortly thereafter with defervescence of his fever and resolution of his constitutional and nuchal symptoms. He states that his new headache is different than his previous in that it is exacerbated by standing or sitting upright and is relieved by sitting down and is not associated with photophobia or neck stiffness. The headache is not relieved by over-the-counter pain medications. He is afebrile and nontoxic appearing. Which of the following is definitive therapy for this patient's headache? a. A 1-L bolus of intravenous (IV) normal saline b. Treatment with standard pharmacologic agents for migraine c. Treatment with meclizine d. Consultation with anesthesia for a blood patch e. Repeat LP to improve symptoms

*D* The patient has a post-LP headache. The headache is thought to be caused by the removal of CSF during LP with a continued leakage of CSF. It is exquisitely sensitive to position and many patients will experience complete relief of pain after being placed in Trendelenburg position. A blood patch is placed by injecting an aliquot of the patient's blood in a sterile fashion just external to the dura mater at the same interspace where the LP occurred. The majority of patients have relief of symptoms with this procedure. Prevention of the post-LP headache includes using a 22-gauge or smaller needle, removing as little fluid as possible, and facing the bevel up when the patient is in the lateral position.

A 22-year-old man calls the ED from a local bar stating that he was punched in the face 10 minutes ago and is holding his front incisor tooth in his hand. He wants to know what is the best way to preserve the tooth. Which of the following is the most appropriate advice to give the caller? a. Place the tooth in a napkin and bring it to the ED b. Place the tooth in a glass of water and bring it to the ED c. Place the tooth in a glass of beer and bring it to the ED d. Pour some water over the tooth and place it immediately back into the socket e. Place the tooth in a glass of milk and bring it to the ED

*D* The patient has an avulsed tooth, which is a dental emergency. When a tooth is missing from a patient, the possibility of aspiration or entrapment in soft tissues should be considered. Avulsed permanent teeth require prompt intervention. The best environment for an avulsed tooth is its socket. Replantation is most successful if the tooth is returned to its socket within 30 minutes of the avulsion. A 1% chance of successful replantation is lost for every minute that the tooth is outside of its socket. The tooth should only be handled by the crown, so as not to disrupt the root. If the patient cannot replant the tooth, then he or she should keep the tooth under his or her tongue or in the buccal pouch so that it is bathed in saliva. If that cannot be achieved, then the tooth can be placed in a cup of milk (e) or in saline. The best transport solution is Hank solution, which is a buffered chemical solution. However, it is typically unavailable in this setting.

A 55-year-old woman with a past medical history of diabetes walks into the emergency department (ED) stating that her tongue and lips feel like they are swollen. During the history, she tells you that her doctor justnstarted her on a new blood pressure (BP) medication. Her only other medication is a baby aspirin. Her vitals at triage are: BP 130/70 mm Hg, heart rate (HR) 85 beats per minute, respiratory rate (RR) 16 breaths per minute, oxygen saturation 99% on room air, and temperature 98.7°F. On physical examination, you detect mild lip and tongue swelling. Over the next hour, you notice that not only are her tongue and lips getting more swollen, but her face is starting to swell, too. What is the most likely inciting agent? a. Metoprolol b. Furosemide c. Aspirin d. Lisinopril e. Diltiazem

*D* The patient has angioedema, a rare, but significant side effect of angiotensin-converting enzyme (ACE-I) inhibitors. This type of angioedema is usually limited to the lips, tongue, and face.

A 22-year-old man presents to the ED with a 3-day history of rash, fever, malaise, and mouth sores. He has been unable to eat because of mouth pain. He denies arthralgias, penile discharge, new medications, drug allergies, or prior similar episodes. On examination, the patient's BP is 100/60 mm Hg, HR is 110 beats per minute, RR is 20 breaths per minute, and temperature is 102°F. The patient appears alert but uncomfortable. He has multiple vesiculobullous lesions on his conjunctivae and mouth as seen in the image below. Visual acuity is 20/20. Target lesions are found on his palms and soles. What is the most appropriate next step in management? a. Discharge him with analgesics, antihistamines, and mouth rinses. b. Discharge him with acyclovir, analgesics, antihistamines, and mouth rinses. c. Discharge him after 1-L normal saline IV; and prescriptions for analgesics, antihistamines, oral prednisone, and mouth rinses. d. Admit him and administer 1- to 2-L normal saline IV, oral prednisone, analgesics, antihistamines, and mouth rinses. e. Admit him and administer 1- to 2-L normal saline IV; analgesics, and acyclovir IV.

*D* The patient has signs suggestive of Erythema Multiforme (EM), an acute inflammatory skin disease that ranges from a localized eruption (EM minor) to a severe multisystem illness (EM major) with extensive vesiculobullous lesions and erosion of the mucous membranes, known as Stevens-Johnson syndrome (SJS). It affects all age groups with the highest incidence in males 20 to 40 years of age. SJS has significant morbidity and a mortality rate of approximately 10%. Death is usually a result of infection and dehydration. As in this case, patients with severe disease should be admitted. Therapy consists of IV fluids, oral prednisone, analgesics, antihistamines, mouth rinses, and skin care. While no causative factor can be found in 50% of cases, known triggers include infection, especially Mycoplasma and herpes simplex virus, drugs, especially anticonvulsants and antibiotics, and malignancies.

A 58-year-old woman is brought to the ED by paramedics complaining of worsening left arm and leg weakness. She reports a history of hypertension, diabetes, and smoking. She denies any past surgeries. Her BP is 165/83 mm Hg, HR is 110 beats per minute, temperature is 98.4°F, RR is 18 breaths per minute, pulse oxymetry is 98% on room air, and capillary glucose is 147 mg/dL. On examination, the patient's speech is slurred and you notice a left-sided facial droop. Her left arm and leg strength is 2/5 and there is decreased sensation. The patient's head CT is normal. It has been 130 minutes since the onset of symptoms. Which of the following is the most appropriate next step in management? a. Observation since she is out of the thrombolytic window. b. Administer nitroprusside to lower her BP then give thrombolytics. c. Administer heparin only since she is out of the thrombolytic window. d. Administer thrombolytic therapy. e. Administer aspirin only since she is out of the thrombolytic window.

*D* The patient is a good candidate for fibrinolytic therapy. She is having an acute ischemic stroke (in the distribution of the middle cerebral artery), has no contraindications to the therapy and is being evaluated within the 3-hour (180-minute) therapeutic window from the onset of symptoms.

A 23-year-old woman presents to the ED complaining of abdominal pain, nausea, and vomiting. She has a history of depression but is not currently taking any antidepressant medications. Upon further questioning, the patient states that she ingested a bottle of pills in her medicine cabinet approximately 3 hours ago. Her BP is 115/65 mm Hg, HR is 101 beats per minute, temperature is 100.1°F, RR is 29 breaths per minute, and oxygen saturation is 100% on room air. Physical examination is unremarkable except for mild diffuse abdominal tenderness. Laboratory results reveal a white blood cell (WBC) count of 10,300/μL, hematocrit 46%, platelets 275/μL, aspartate transaminase (AST) 70 U/L, alanine transaminase (ALT) 85 U/L, alkaline phosphatase 75 U/L, sodium 143 mEq/L, potassium 3.7 mEq/L, chloride 98 mEq/L, bicarbonate 8 mEq/L, blood urea nitrogen (BUN) 22 mg/dL, creatinine 0.9 mg/dL, and glucose 85 mg/dL. Arterial blood gas values on room air are pH 7.51, PCO2 11 mm Hg, and PO2 134 mm Hg. Which of the following substances did this patient most likely ingest? a. Diphenhydramine (Benadryl) b. Ibuprofen c. Acetaminophen d. Aspirin e. Pseudoephedrine

*D* The patient most likely ingested aspirin. Patients with an acute salicylate overdose may present with nausea, vomiting, tinnitus, fever, diaphoresis, and confusion. Salicylates are capable of producing several types of acid-base disturbances. Acute respiratory alkalosis, without hypoxia, is caused by salicylate stimulation of the respiratory center in the brainstem. If the patient is hypoxic, salicylate induced noncardiogenic pulmonary edema should be considered. Within 12 to 24 hours after ingestion, the acid-base status in an untreated patient shifts toward an anion gap metabolic acidosis because of interference with the Krebs cycle, uncoupling oxidative-phosphorylation, and increased fatty acid metabolism. A mixed respiratory alkalosis and metabolic acidosis is typically seen in adults.

A 35-year-old woman presents to the ED complaining of headache and blurry vision. She has had daily headaches for 3 months associated with blurry vision. She is afebrile, not losing weight, and has a normal neurologic examination, including fundoscopy. You ask when her last menstrual period was and she states she has not menstruated for 5 months and is not taking oral contraceptive pills. She also complains of galactorrhea. Noncontrast head CT is normal. An LP is performed and reveals a normal opening pressure. Which of the following is the most appropriate next step in managing the patient's headaches? a. Repeat head CT with administration of IV contrast. b. Initiation of therapy with bromocriptine. c. Evaluation of CSF for xanthochromia and RBCs. d. Treatment of her headache with analgesia and a MRI. e. Repeat LP with removal of 15-mL CSF for therapeutic benefit.

*D* The patient presents with symptoms consistent with a prolactin-secreting pituitary adenoma. The appropriate imaging modality to diagnose a pituitary adenoma is with high resolution MRI with thin cuts through the sella. Women often present with amenorrhea, infertility, and galactorrhea. Men will present with decreased libido. In both cases, extension beyond the sella may present with visual field defects or other mass-related symptoms.

A 42-year-old man presents to the ED after sustaining an electric shock when he was changing a wall outlet. The patient complains of pain in the finger that was shocked. Visual inspection reveals localized erythema at the distal tip but good capillary refill, 2+ radial, and ulnar pulses and full range of motion of that hand and extremity. Chest auscultation reveals a normal S1 and S2 with clear and equal breath sounds bilaterally. There are no other signs of trauma. What diagnostic test should be performed next in the complete evaluation of this patient? a. Urinalysis b. Basic metabolic panel c. Chest radiograph d. Electrocardiogram (ECG) e. Arterial Doppler

*D* The patient sustained a low-voltage (< 1000 V) electrical injury from the wall socket. All patients sustaining such an injury warrant an ECG and cardiac monitoring in the ED. In the United States, household wiring has 120 V of alternating current with a frequency (number of switches from positive to negative) of 60 Hz. Alternating current causes continuous muscle contraction and stimulation that may cause ventricular fibrillation. Direct current usually just causes a single muscle spasm and is the less dangerous of the two, depending upon the voltage. The current is equal to the voltage over the resistance (Ohm's law). This patient sustained a superficial thermal burn and most likely did not become part of the circuit given that there were not any clear entry or exit wounds as seen in direct-contact injuries. Disposition of this patient should include localized wound care, close follow-up, and instructions to return if there are any worsening symptoms.

A 23-year-old woman presents to the ED in moderate pain in her left lower quadrant (LLQ). She states that the pain began suddenly and is associated with nausea and vomiting. She had a bout of diarrhea yesterday. This is the second time this month that she experienced pain in this location, however, never with this severity. Her BP is 120/75 mm Hg, HR is 101 beats per minute, temperature is 99.5°F, and RR is 18 breaths per minute. She has a tender LLQ on abdominal examination and a tender adnexa on pelvic examination. Which of the following is the most appropriate diagnostic test for the patient? a. CT scan b. MRI c. X-ray d. Doppler ultrasound e. Laparoscopy

*D* The patient's clinical picture is consistent with ovarian torsion. This phenomenon is most common in women in their mid-twenties. It is caused by the ovary twisting on its stalk, which leads to occlusion of venous draining from the ovary. This leads to ovarian edema, hemorrhage, and necrosis. Most occur in the presence of an enlarged ovary (ie, as a result of cyst, abscess, or tumor). Patients may give a history of similar pain that resolved spontaneously. The first choice to diagnose ovarian torsion is with Doppler ultrasound to demonstrate decreased or absent blood flow to the ovary. It can also identify an ovarian mass. If suspicion is high for ovarian torsion, the patient may immediately undergo laparoscopy, which is diagnostic and potentially therapeutic.

A 41-year-old man is brought into the ED after having a witnessed tonic-clonic seizure. He is alert and oriented and states that he has not taken his seizure medication for the last week. His BP is 140/75 mmHg, HR is 88 beats per minute, temperature is 99.7°F, and his RR is 16 breaths per minute. On examination you notice that his arm is internally rotated and adducted. He cannot externally rotate the arm and any movement of his shoulder elicits pain. Which of the following is the most likely diagnosis? a. Humerus fracture b. Clavicular fracture c. Scapular fracture d. Posterior shoulder dislocation e. Anterior shoulder dislocation

*D* The patient's clinical presentation is consistent with a posterior shoulder dislocation. Posterior dislocations are rare and account for only 2% of all glenohumeral dislocations. Posterior dislocations are traditionally associated with seizure patients and lightening injuries. However, the most common dislocation seen in postseizure patients is an anterior dislocation. Classically, the patient holds the dislocated arm across the chest in adduction and internal rotation. Abduction is limited and external rotation is blocked. Radiographs may reveal a "light bulb" sign, which is the light bulb appearance when the humeral head is profiled in internal rotation.

A 49-year-old man presents to the ED with nausea, vomiting, and abdominal pain that began approximately 2 days ago. The patient states that he usually drinks a six-pack of beer daily, but increased his drinking to two six-packs daily over the last week because of pressures at work. He notes decreased appetite over the last 3 days and states he has not had anything to eat in 2 days. His BP is 125/75 mm Hg, HR is 105 beats per minute, and RR is 20 breaths per minute. You note generalized abdominal tenderness on examination. Laboratory results reveal the following: White blood cells (WBC) 9000/μL, Sodium 131 mEq/L, Hematocrit 48%, Potassium 3.5 mEq/L, Platelets 210/μL Chloride 101 mEq/L, AST 85 U/L, Bicarbonate 10 mEq/L, ALT 60 U/L, BUN, 9 mg/dL, Alkaline phosphatase 75 U/L Creatinine 0.5 mg/dL, Total bilirubin 0.5 mg/dL, Glucose 190 mg/dL, Lipase 40 IU, Nitroprusside test weakly positive for ketones Which of the following is the mainstay of therapy for patients with this condition? a. Normal saline (NS) solution b. Half normal saline (1/2 NS) c. Glucose solution (D5W) d. Solution containing both saline and glucose (D5/NS or D5 1/2 NS) e. The type of solution is irrelevant

*D* The patient's presentation is consistent with alcoholic ketoacidosis (AKA). This is an acute metabolic acidosis that typically occurs in people who (1) chronically abuse alcohol and have a recent history of binge drinking, (2) have had little or no recent food intake, and (3) have had persistent vomiting. AKA is characterized by elevated serum ketone levels and a high anion gap (↑AG = Na − [HCO3 + Cl] > 12). A concomitant metabolic alkalosis is common, secondary to vomiting and volume depletion. AKA is the result of (1) starvation with glycogen depletion and counterregulatory hormone production; (2) a raised NADH/NAD+ ratio (related to the metabolism of ethanol); and (3) volume depletion, resulting in ketogenesis. The typical symptoms and physical findings relate to volume depletion and chronic alcohol abuse and include nausea, vomiting, abdominal pain, and/or hematemesis. The fruity odor of ketones may be present on the patient's breath. The patient's mental status may be impaired. Also associated with the presentation are dyspnea, tremulousness, and dizziness. Rarely do patients present with muscle pain, fever, diarrhea, syncope, seizure, or melena. In AKA, the β-hydroxybutyrate (β-OH)/ acetyl acetate (AcAc) formation ratio is 5:1. The nitroprusside reaction (Acetest) may be negative or only weakly positive for serum ketones because nitroprusside reacts with AcAc, but not with β-OH. Therefore, ketosis may be more severe than would be inferred from a nitroprusside reaction alone. Once the diagnosis of alcoholic ketoacidosis is established, the mainstay of treatment is (d) hydration with 5% dextrose in normal saline (D5NS or 1/2 NS). With initial therapy, ketone formation shifts toward the production of AcAc so that measured ketone levels rise, although β-OH levels decrease. Carbohydrate and fluid replacement reverse the pathophysiologic derangements that lead to AKA by increasing serum insulin levels and suppressing the release of glucagon and other counterregulatory hormones. Dextrose stimulates the oxidation of NADH and aids in normalizing the NADH/NAD+ ratio. As additional treatment, thiamine supplementation should be given as prophylaxis against Wernicke encephalopathy. In general, exogenous insulin is contraindicated in the treatment of AKA because it may cause life threatening hypoglycemia in patients with depleted glycogen stores.

A 2-week-old girl is brought to the ED with the complaint of eye discharge. The patient was born full-term to a mother who received scant prenatal care, but states she was treated for a vaginal infection during her pregnancy. The infant has been feeding well and gaining weight. She has had no fever. Mother notes that the patient has had a mild, dry sounding cough. She has had no vomiting or irritability. Her examination is nonfocal and benign except for bilateral redness of the conjunctivae, palpebral more than bulbar, and bilateral purulent discharge. She has her 2-week office visit scheduled for 2 days from now. Which of the following is the most appropriate course of action? a. Initiate a full sepsis workup and administer ceftriaxone. b. Obtain a blood and urine and eye culture and treat systemically for gonorrhea infection. c. Perform a conjunctival scraping for direct fluorescent antibody, culture, and begin topical erythromycin. d. Perform a conjunctival scraping for direct fluorescent antibody, culture, and begin systemic treatment with erythromycin. e. Reassure that this is likely to be a clogged tear duct, which is a common and self limiting condition in such a well-appearing infant, and plan for close follow-up.

*D* The patient's presentation is suspicious for chlamydial conjunctivitis. Even without any respiratory symptoms at the time of presentation, a significant percentage of these patients harbor the organism in their nasopharynx and go on to develop chlamydial pneumonia at a later date. Therefore, treatment for 14 to 21 days with oral erythromycin is appropriate. The classic description of chlamydial pneumonia is a dry, staccato cough.

While playing in his family's annual Thanksgiving Day touch football game, a 41-year-old man fell onto his outstretched hand upon attempting to make the winning catch. He presents to the emergency department (ED) complaining of severe wrist pain. Which of the following carpal bones is most frequently injured in a fall on the outstretched hand? a. Triquetrum b. Lunate c. Capitate d. Scaphoid e. Pisiform

*D* The scaphoid is the most common carpal bone injured in a fall on the outstretched hand (a "FOOSH" injury). On examination, patients exhibit tenderness at the anatomic snuffbox and pain referred to the anatomic snuffbox with longitudinal compression of the thumb. In the setting of a tender thumb, a thumb splint should be applied until films can be repeated after 1 week since initial films may not reveal the fracture. The application of the splint is especially important, because of the scaphoid's high risk for avascular necrosis.

A 32-year-old firefighter presents to the ED in acute respiratory distress. He was taken to the ED shortly after extinguishing a large fire in a warehouse. His initial vitals include a HR of 90 beats per minute, a BP of 120/55 mm Hg, and an RR of 18 breaths per minute with an oxygen saturation of 98% on 2-L nasal cannula. An ECG shows a first-degree heart block. Upon physical examination, there are diffuse rhonchi bilaterally. The patient is covered in soot and the hairs in his nares are singed. Given this clinical presentation, which of the following may be responsible for this patient's respiratory distress? a. Reactive airway disease b. Foreign body aspiration c. Decompression sickness d. Thermal burns e. Pneumothorax

*D* The singed nares seen in this patient should give you a clue to the possibility of severe thermal burns. Although there is minimal external involvement, damage from the heat may extend deep into the pulmonary system through inspiration. This results in a severe inflammatory reaction causing pneumonitis. Early intubation is critical.

A 44-year-old woman is undergoing a diagnostic evaluation for 3 hours of abdominal pain. She had two similar episodes in the past 2 months. She is tolerating oral intake and is afebrile. As part of this evaluation, a diagnostic ultrasound is performed and is shown below. Which of the following is the most likely diagnosis? a. Nephrolithiasis b. Pancreatic pseudocyst c. Ovarian cysts d. Cholelithiasis e. Liver abscess

*D* The ultrasound shows a gallbladder with multiple echogenic gallstones associated with well defined acoustic shadows. This image is typical for cholelithiasis. If the patient was febrile and thought to have cholecystitis, then one may also observe gallbladder distention, wall thickening, and pericholecystic fluid in addition to gallstones. The most common clinical manifestation of cholelithiasis is biliary colic.

A 20-year-old man presents to the ED with multiple stab wounds to his chest. His BP is 85/50 mm Hg and HR is 123 beats per minute. Two large-bore IVs (intravenous) are established and running wide open. On examination, the patient is mumbling incomprehensibly, has good air entry on lung examination, and you notice jugular venous distension. As you are listening to his heart, the nurse calls out that the patient has lost his pulse and that she cannot get a BP reading. Which of the following is the most appropriate next step in management? a. Atropine b. Epinephrine c. Bilateral chest tubes d. ED thoracotomy e. Pericardiocentesis

*D* The vignette describes a traumatic arrest after penetrating chest trauma. The most likely cause is cardiac tamponade, which occurs in approximately 2% of anterior penetrating chest traumas. Clinically, patients present with hypotension, JVD, and muffled heart sounds. These three signs are called Beck triad. In addition, tachycardia is often present. JVD may not be present if there is marked hypovolemia. The most effective method for relieving acute pericardial tamponade in the trauma setting is thoracotomy and incision of the pericardium with removal of blood from the pericardial sac. The indications to perform an ED thoracotomy generally include blunt or penetrating trauma patients who lose their vital signs in transport to or in the ED. Patients with penetrating wounds have a significantly better chance of surviving with thoracotomy; also patients with stab wounds are more likely to do better than GSWs.

An 11-year-old girl presents to the ED with a 3-day history of decreased energy and difficulty climbing stairs. The patient had previously complained of pain in her lower extremities last week and was seen by her primary-care physician when she was diagnosed with muscle cramping. She has no complaints of focal tenderness or history of trauma. On examination, the patient is afebrile with normal vital signs. Her neurologic examination is significant for marked lower extremity weakness and absent deep tendon reflexes. Her sensory examination is normal. The upper extremities reveal normal strength and sensation throughout. Which of the following would be most helpful to confirm the suspected diagnosis? a. Emergency magnetic resonance imaging (MRI) of the spine b. Deep muscle biopsy c. Electrolytes looking for decreased potassium d. Lumbar puncture e. Edrophonium challenge

*D* This is a case of Guillain-Barré syndrome (GBS). GBS is characterized by progressive, symmetric muscle weakness, typically beginning caudally and progressing cephalad. It is also associated with reduced or absent deep tendon reflexes. It is thought to be a postinfectious condition (viral illnesses in the preceding weeks are reported in the majority of cases) that is pathologically characterized by acute demyelination. It is typically self-resolving but can be treated by IVIG or even plasmapheresis in severe cases. Care is supportive, which may ultimately involve mechanical ventilation in cases where paralysis ascends to involve respiratory muscles. A variant of GBS is known as Miller Fisher disease in which there is predominant involvement of facial nerves. There is no specific serologic test for GBS but lumbar puncture is typically performed and demonstrates increased protein with no pleocytosis.

A 3-year-old African American boy with a history of sickle-cell disease presents to the ED after he developed a low-grade fever, runny nose, and an erythematous discoloration of both cheeks. His vital signs are heart rate (HR) 110 beats per minute, respiratory rate (RR) of 24 breaths per minute, and pulse oximetry of 98% on room air. The patient looks well and is in no acute distress. You note a macular lesion in both cheeks. The rash is not pruritic and there is no associated cellulitis or suppuration. What is the most serious complication to consider in this patient? a. Osteomyelitis b. Viral encephalitis c. Pneumonia d. Aplastic anemia e. Meningitis

*D* This is a case of erythema infectiosum or Fifth disease. Infection by parvovirus B19 produces this pattern of a "slapped cheek" appearance. It is characterized by an eruption that presents initially as an erythematous malar blush followed by an erythematous maculopapular eruption on the extensor surfaces of extremities that evolves into a reticulated, lacy, mottled appearance. Fever and other symptoms may be present but are uncommon. In patients with chronic hemolytic anemias like sickle-cell disease, aplastic anemia is a serious complication. Pregnant women should avoid exposure to this virus, since it may cause fetal hydrops in 10% of cases.

A 2-year-old boy is brought to the ED by his parents stating that he is limping. The mother states that he was fine yesterday but woke up today and would not bear weight. He had a normal active day yesterday with no notable falls. On examination, the patient is in mild distress. His vital signs are a temperature of 101°F, HR 120 beats per minute, RR 24 breaths per minute, blood pressure (BP) 90/55 mm Hg. He has mild nasal congestion. He is able to move his left lower extremity only a small amount and has discomfort with range of motion of his left hip. He is unable to bear weight. There is no swelling, rash, warmth, or erythema. White blood cell (WBC), C-reactive protein (CRP), and erythrocyte sedimentation rate (ESR) are all normal. Radiograph of the hips bilaterally and left femur and knee are all negative. After high dose ibuprofen in the ED, the patient is able to bear weight. What is the most appropriate management of this patient? a. Admit for IV antibiotics. b. Consult orthopedics after ultrasound for aspiration of hip in the operating room (OR). c. Bone scan to evaluate for osteomyelitis. d. Nonsteroidal anti-inflammatory drugs (NSAIDs) and reassurance to parents with follow-up in 24 hours. e. Splint leg for treatment of occult fracture through growth plate than can not be visualized on initial x-rays.

*D* This is a case of toxic synovitis, also called transient synovitis, of the hip. Despite its name, this is a benign and self-limiting condition that typically responds to rest and NSAIDs. It is thought to be a postinfectious inflammatory process, though the true cause is unknown. The course is usually from several days to a couple of weeks. Though fever is not a typical part of most patients with toxic synovitis, it can be present and low grade or could potentially be unrelated (note this child has concurrent upper respiratory infection [URI] symptoms). It is most important to distinguish this condition from a septic arthritis. The main differences are that the joint can often be gently maneuvered in toxic synovitis whereas in true septic joints even the smallest amount of joint motion typically produces intense pain and patients will strongly resist full range of motion.

A 6-month-old girl is brought to the ED because of persistent crying for the past 6 hours. Her teenage father informs you that she has been inconsolable since awaking from her nap. No recent illness, trauma, fever, or other complaints are reported. On physical examination the patient is alert, awake, and crying. You note swelling, deformity, and tenderness of the left femur. When inquired about this finding, the caretaker responds, "Her leg got stuck between the rails of her crib." Radiographs show a minimally displaced spiral fracture of the left femur. Which of the following is the next best step in management? a. Genetic workup for osteogenesis imperfecta and other bone abnormalities b. Orthopedic consultation for closed reduction c. Serum electrolytes including calcium and phosphate d. Perform skeletal survey and contact Child Protective Services e. Placement of posterior splint and discharge home with orthopedic follow-up

*D* This is a case where nonaccidental trauma (NAT) should be considered. A spiral fracture of the femur in a non-ambulating child is highly suspicious of NAT. Metaphyseal corner fractures, fractures of the posterior ribs, sternum scapula or spinous processes, or multiple fractures in various stages of healing may be the only presentation of child abuse. Any type of trauma that does not fit the mechanism should raise the suspicion and alert the physician. In this case, it is unlikely that the patient was able to fit her leg through the crib rails and break her femur. Although in ambulating children it is not uncommon to have spiral fractures of the femur while running. All states have mandatory reporting of child abuse. According to the AAP (American Academy of Pediatrics), the skeletal survey is the initial test of choice for all children suspected of being abused. It is also mandatory to contact the local Child Protective Services to further investigate the situation.

A 44-year-old man presents to the ED complaining of right foot pain. He states while playing basketball 2 weeks ago, he stepped on a nail that punctured through his sneaker and cut his great toe. He went immediately to the ED, where an x-ray was taken and confirmed negative, in addition to being administered tetanus prophylaxis. On physical examination, his toe is swollen, erythematous, and tender to palpation. There is no obvious break in the skin or abscess present. His BP is 120/75 mm Hg, HR is 80 beats per minute, temperature is 100.4°F, and RR is 16 breaths per minute. Which of the following organisms is the most likely pathogen? a. Neisseria gonorrhoeae b. Staphylococcus epidermidis c. Sporothrix schenckii d. Pseudomonas aeruginosa e. Salmonella sp.

*D* This is a typical scenario for osteomyelitis. Conventional radiography on the day of injury is insensitive to the detection of osteomyelitis; even 1 week after the injury x-ray diagnosis is limited. The most common pathogen in osteomyelitis is S aureus. However, Pseudomonas is responsible for bone and joint infections in three settings. First, a puncture wound through a shoe. Pseudomonas does not grow on the puncture object, but rather is associated with the shoe itself and may be inoculated into the bone as the sharp object passes through the colonized shoe into the wound. Second, prosthetic devices implanted for orthopedic surgery. Third is IV drug use.

A 45-year-old woman presents with right eye pain and redness for 1 day. She has photophobia and watery discharge from the eye. She does not wear glasses or contact lenses and has no prior eye problems. On examination, the patient's visual acuity is 20/20 in the left eye and 20/70 in the right eye. She has conjunctival injection around the cornea and clear watery discharge. On slitlamp examination, the lids, lashes, and anterior chamber are normal. When fluorescein is applied, a branching, white-colored epithelial defect is seen. The remainder of the head examination is normal and the patient has no cutaneous lesions. Which of the following is the most appropriate treatment for this patient? a. Admission for intravenous (IV) antibiotics b. Admission for IV antiviral agents c. Topical steroids d. Topical antiviral medication e. Immediate ophthalmology consultation

*D* This patient has a corneal epithelial disease caused by the herpes simplex virus. The hallmark of this disease is the branching or dendritic ulcer. Patients may also present without corneal involvement, but will have typical herpetic skin lesions in the eyelids and conjunctiva. Patients should be treated with topical antivirals, such as trifluridine, with topical antibiotics added to prevent secondary bacterial infection.

A 30-year-old G2P2 presents to the ED with acute onset of lower abdominal pain associated with vaginal bleeding that began 2 hours prior to arrival. She denies any prior medical history but does report having a tubal ligation after the birth of her second child. Her vitals are significant for a heart rate (HR) of 120 beats per minute and a blood pressure (BP) of 90/60 mm Hg. On physical examination, the patient has right adnexal tenderness with blood in the posterior vaginal vault. Her cervical os is closed. Given this patient's history and physical examination, which of the following is the most likely diagnosis? a. Appendicitis b. Pelvic inflammatory disease (PID) c. Placenta previa d. Ectopic pregnancy e. Abruptio placentae

*D* This patient has a significant risk factor for having an ectopic pregnancy. Tubal ligations raise the likelihood of having an ectopic pregnancy by providing an outlet for improper implantation of an embryo into the abdominal cavity or somewhere outside of the uterus. Implantation most commonly occurs in the fallopian tubes, 95% of the time. Again, a β-hCG is crucial in the beginning stages in the workup of this patient. In a normal pregnancy, the β-hCG doubles every 2 days and typically only increases by two-thirds in ectopic pregnancies. Progesterone levels also differ and may be helpful. A transvaginal ultrasound is also warranted to determine the size and location of the ectopic and any associated free fluid indicating rupture. β-hCG levels dictate whether a transvaginal (> 1500 mIU/mL) or transabdominal (> 6500 mIU/mL) approach should be used. Smaller, nonruptured ectopics may be treated with methotrexate. However, this patient is hemodynamically compromised and has most likely ruptured. Surgical intervention is warranted in these cases. Patients with ectopic pregnancies may also present with back or flank pain, syncope or peritonitis in cases with significant abdominal hemorrhage. Risk factors also include previous ectopics, intrauterine devices, PID, sexually transmitted diseases (STDs), in vitro fertilization, and recent elective abortion. The incidence of a coexisting heterotopic pregnancy is about 1/30,000 but increases to 1/8000 in women on fertility drugs.

A 19-year-old woman presents with bilateral lower-abdominal pain, fever, nausea, vomiting, and general malaise. Her last menstrual period was 5 days ago. Vitals are HR 98 beats per minute, BP 110/65 mm Hg, RR 18 breaths per minute, and temperature of 102.7°F. Pelvic examination demonstrates exquisite cervical motion tenderness and right adnexal tenderness. Laboratory reports are notable for a WBC 15,000/μL, an ESR of 95 mm/h, and a negative urine β-human chorionic gonadotropin (β-hCG). Transvaginal ultrasound demonstrates a right complex mass with cystic and solid components. Which of the following is the most appropriate next step in management? a. Ceftriaxone IM plus a 14-day course of oral doxycycline and follow-up in the gynecology clinic. b. Oral ofloxacin plus metronidazole and follow-up in the gynecology clinic. c. Analgesics for a ruptured ovarian cyst and follow-up in the gynecology clinic. d. Admission for IV antibiotics and possible laparoscopic drainage. e. Admission for emergent medical or surgical treatment of an ectopic pregnancy.

*D* This patient has a tubo-ovarian abscess (TOA), a common and potentially fatal complication of pelvic inflammatory disease (PID). While PID can usually be treated with outpatient antibiotics, the Center for Disease Control (CDC) lists several guidelines for inpatient admission, including: (1) cases in which surgical emergencies (eg, appendicitis) cannot be excluded; (2) pregnancy; (3) failure of outpatient therapy; (4) inability to tolerate oral intake; (5) severe illness, including nausea, vomiting, or high fever; and (6) TOA. This patient has several of these factors and should, therefore, be admitted for further management. IV antibiotics are curative in 60% to 80% of cases and must be selected to cover N gonorrhoeae and Chlamydia trachomatis, the most common pathogens responsible for PID, as well as bacteroides, the most common cause of TOA. Surgical drainage or salpingectomy and oophorectomy may be required in resistant cases.

An 84-year-old woman with a history of metastatic breast cancer presents to the ED with new-onset dyspnea and exercise intolerance for the past week. She denies fever, chest pain, or cough. On examination, her temperature is 100.3°F, BP is 70/50 mm Hg, HR is 110 beats per minute, RR is 20 breaths per minute, and oxygen saturation is 93% on room air. As your colleague is performing a physical examination, you place the portable ultrasound on the heart and see a thin echo-free area around the heart with right atrium and right ventricular collapse. Which of the following is the most likely diagnosis? a. Pulmonary embolism b. Congestive heart failure c. Massive myocardial infarction (MI) d. Cardiac tamponade e. Dehydration

*D* This patient has cardiac tamponade from metastatic breast cancer. Bedside ultrasound is often diagnostic. The echo-free area around the heart is a pericardial effusion. It is important to realize that the presence or absence of pericardial effusion is not diagnostic of tamponade; right atrial and ventricular collapse, on the other hand, are more specific ultrasound signs for tamponade. Clinical findings should always be considered in making the diagnosis. Patients with tamponade usually have tachycardia, low systolic BP, and narrow pulse pressure. Tamponade must always be considered with trauma to the chest, as well as patients with metastatic malignancy, pericarditis, uremia, and those patients on anticoagulation.

A 61-year-old woman with a history of congestive heart failure (CHF) is at a family picnic when she starts complaining of shortness of breath. Her daughter brings her to the ED where she is found to have an oxygen saturation of 85% on room air with rales halfway up both of her lung fields. Her BP is 185/90 mm Hg and pulse rate is 101 beats per minute. On examination, her jugular venous pressure (JVP) is 6 cm above the sternal angle. There is lower extremity pitting edema. Which of the following is the most appropriate first-line medication to lower cardiac preload? a. Metoprolol b. Morphine sulfate c. Nitroprusside d. Nitroglycerin e. Oxygen

*D* This patient has decompensated CHF with pulmonary edema. Nitroglycerin is the most effective and most rapid means of reducing preload in a patient with CHF. Nitrates decrease myocardial preload and, to a lesser extent, afterload. Nitrates increase venous capacitance, including venous pooling, which decreases preload and myocardial oxygen demand. It is most beneficial when the patient who presents with CHF is also hypertensive. It is administered sublingually, intravenously, or transdermally.

A 62-year-old man presents to the ED after he was found talking to himself by witnesses on a nearby street. Upon arrival, the patient appears confused and is actively hallucinating. His initial vitals include an irregular HR of 80 to 110 beats per minute, an RR of 14 breaths per minute, a BP of 160/80 mm Hg with an oxygen saturation of 97% on room air. An ECG indicates atrial fibrillation. The patient can be redirected but states that he is distracted by colorful, floating images in the room. Given this patient's presentation, what is the most likely etiology of his symptoms? a. Acute psychotic disorder b. Malingering c. Conversion disorder d. Digoxin overdose e. Antidepressant overdose

*D* This patient is experiencing a visual hallucination, most typical of a medical rather than psychiatric etiology. Digoxin is a common precipitant of these symptoms and may begin with yellow-blue changes in vision, known as van Gogh vision. Digoxin directly binds the Na-K ATPase which increases sodium and calcium levels, increasing the contractility of the heart. Hallucinations are often an early symptom of digoxin overdose. Treatment for this includes a protein fragment that binds this medication.

A 34-year-old woman with no known medical problems is having a sushi dinner with her husband. Halfway through dinner, she begins scratching her arms and her husband notices that her face is flushed. The itching intensifies and she begins to feel chest pain, shortness of breath, and dizziness. On arrival to the ED, she can barely talk. Her temperature is 100°F, BP is 85/50 mm Hg, HR is 125 beats per minute, and RR is 26 breaths per minute, and oxygen saturation is 91% on room air. Which of the following is the most likely diagnosis? a. Hypovolemic shock b. Neurogenic shock c. Cardiogenic shock d. Anaphylactic shock e. Septic shock

*D* This patient is in anaphylactic shock from a food allergy while dining. Anaphylaxis is a severe systemic hypersensitivity reaction leading to shock from hypotension and respiratory compromise. The diagnosis is made clinically. This patient's reaction began classically with urticarial symptoms of pruritus and flushing. She then progressed to shock with hypotension and respiratory edema. She should be treated immediately with oxygen, intramuscular or IV epinephrine, corticosteroids, diphenhydramine, and IV fluids. Supplies should also be ready for intubation and surgical cricothyrotomy.

An 82-year-old man with a history of COPD and hypertension presents with shortness of breath and fever. His medications include albuterol, ipratropium, prednisone, hydrochlorothiazide, and atenolol. His temperature is 102.1°F, BP is 70/40 mm Hg, HR is 110 beats per minute, RR is 24 breaths per minute, and oxygen saturation is 91% on room air. The patient is uncomfortable and mumbling incoherently. On chest examination, you appreciate rales on the left side of his chest. His heart is tachycardic, but regular with no murmurs, rubs, or gallops. His abdomen is soft and nontender. You believe this patient is in septic shock from pneumonia and start IV fluids, broad-spectrum antibiotics, and a dopamine drip. His BP remains at 75/50 mm Hg. Which of the following is the most appropriate next step in management? a. D5 normal saline IV bolus b. Phenylephrine IV drip c. Fludrocortisone IV d. Hydrocortisone IV e. Epinephrine IV drip

*D* This patient is in septic shock from pneumonia and also has adrenal crisis. Initial treatment with IV fluids, antibiotics, and dopamine is appropriate. Continued hypotension in a patient on maintenance steroid therapy should make you think of adrenal crisis. Exogenous glucocorticoids suppress hypothalamic release of corticotropin-releasing hormone (CRH) and subsequently anterior pituitary release of adrenocorticotropic hormone (ACTH). The adrenals subsequently atrophy from lack of stimulation. The patient is now faced with an acute stress from pneumonia and sepsis. His adrenals have atrophied and are unable to respond with increased cortisol secretion. Laboratory clues to adrenal crisis include hyponatremia and hyperkalemia caused by a lack of aldosterone. The treatment of adrenal crisis in the face of septic shock is hydrocortisone.

A 24-year-old woman presents to the ED complaining of dizziness and numbness and tingling in her fingertips with decreased range of motion. Her initial vitals include a HR of 100 beats per minute, a RR of 30 breaths per minute with an oxygen saturation of 100% on room air. The patient denies any other symptoms. Upon physical examination, the patient appears anxious, tachypneic with a clawed appearance to both hands which are difficult to range. An arterial blood gas is drawn that shows a pH of 7.55 with a decreased carbon dioxide level and normal bicarbonate level. Which of the following underlying metabolic disturbances is responsible for this patient's symptoms? a. Metabolic acidosis b. Metabolic alkalosis c. Respiratory acidosis d. Respiratory alkalosis e. Hyperthyroidism

*D* This patient is tachypneic, probably as a result of some underlying anxiety, which has resulted in paresthesias, carpal spasm, and tetany. This occurs as a result of an alkalemic environment that decreases ionized calcium levels. The treatment is reassurance, a rebreathing mask allowing for carbon dioxide retention and possibly sedation. The patient's metabolic profile is consistent with a respiratory alkalosis (elevated pH, low carbon dioxide).

A 44-year-old agitated woman is brought to the ED by her husband. He states that she has had fevers to 101°F and a productive cough at home for the last 3 days. Today she became labile, agitated, and complained of abdominal pain. She was recently diagnosed with Graves disease and started on PTU. Her BP is 156/87 mm Hg, HR is 145 beats per minute, temperature is 102.4°F, and RR is 20 breaths per minute. On examination, the patient is agitated, confused, and has rales on auscultation bilaterally. Which of the following is the most likely diagnosis? a. Pheochromocytoma b. Cocaine ingestion c. Heat stroke d. Thyroid storm e. Neuroleptic malignant syndrome

*D* This patient presents with a rare but life-threatening hypermetabolic state of thyroid storm. It occurs in patients with known or undiagnosed hyperthyroidism and is usually triggered by infection, trauma, myocardial infarction, stroke, or noncompliance with anti-hyperthyroid medications. Thyroid storm is a clinical diagnosis. The signs and symptoms of this disorder reflect an overactive sympathetic system and include fever, tachycardia out of the proportion to the fever, GI symptoms, and altered mental status. Patients may also develop high-output heart failure. The clue to the diagnosis in this case is the patient's known hyperthyroidism.

A 58-year-old man presents to the ED complaining of generalized weakness for the last 2 days. He states that a few days ago he had abdominal cramps, vomiting, and diarrhea when his whole family got sick after a picnic. These symptoms resolved a day and a half ago but he has not been eating well and now feels weak all over. The patient has a history of hypertension for which he takes hydrochlorothiazide (HCTZ), which was recently increased. His BP is 144/87 mm Hg, HR is 89 beats per minute, temperature is 98.7°F, and RR is 12 breaths per minute. The physical examination reveals hyporeflexia. His electrocardiogram (ECG) is shows flattened T waves, U waves, and prolonged QT and PR intervals. Which of the following is the most likely diagnosis? a. Hypernatremia b. Hyponatremia c. Hyperkalemia d. Hypokalemia e. Hypercalcemia

*D* This patient presents with hypokalemia, secondary to increased potassium losses through vomiting and diarrhea as well as reduced oral intake. Potassium deficiency results in hyperpolarization of the cell membrane and leads to muscle weakness, hyporeflexia, intestinal ileus, and respiratory paralysis. Characteristic ECG findings include flattened T waves, U waves, and prolonged QT and PR intervals.

A 39-year-old woman is brought into the ED by her family who states that she has had 4 days of diarrhea and has now started acting "crazy" with mood swings and confusion. The family states that she usually takes a medication for a problem with her neck. Her BP is 130/45 mm Hg, HR is 140 beats per minute, temperature is 101.5°F, and her respiratory rate (RR) is 22 breaths per minute. An electrocardiogram (ECG) reveals atrial fibrillation with a normal QRS complex. After you address the airway, breathing, and circulation (ABCs), which of the following is the most appropriate next step in management? a. Administer 2 ampules of bicarbonate to treat for tricyclic antidepressant overdose. b. Administer chlordiazepoxide, thiamine, and folate. c. Administer ceftriaxone and prepare for a lumbar puncture. d. Administer propranolol, propylthiouracil (PTU) then wait an hour to give Lugol iodine solution. e. Administer ciprofloxacin and give a 2-L bolus of normal saline for treatment of dehydration secondary to infectious diarrhea.

*D* Thyroid storm is a medical emergency that will lead to death if not treated in time. The manifestations of thyroid storm include temperature greater than 100°F, tachycardia out of proportion to fever, widened pulse pressure, and dysfunction of the CNS (eg, confusion, agitation), cardiovascular system (eg, high-output congestive heart failure, atrial fibrillation), or gastrointestinal (GI) system (eg, diarrhea, abdominal pain). Thyroid storm is a clinical diagnosis since no confirmatory tests are immediately available. The most important factor in reducing mortality is blocking peripheral adrenergic hyperactivity with propranolol, a β-blocker. PTU is used to inhibit new hormone synthesis in the thyroid and has a small effect on inhibiting peripheral conversion of T4 to T3. Iodine is administered to block hormone release from the thyroid but should be given 1 hour after PTU to prevent organification of the iodine

A 22-year-old woman is brought to the ED by ambulance complaining of sudden onset of severe abdominal pain for 1 hour. The pain is in the RLQ and is not associated with nausea, vomiting, fever, or diarrhea. On the pelvic examination you palpate a tender right adnexal mass. The patient's last menstrual period was 6 weeks ago. Her BP is 95/65 mm Hg, HR is 124 beats per minute, temperature is 99.8°F, and RR is 20 breaths per minute. Which of the following are the most appropriate next steps in management? a. Provide her oxygen via face mask and administer morphine sulfate. b. Administer morphine sulfate, order an abdominal CT with contrast, and call an emergent surgery consult. c. Send the patient's urine for analysis and order an abdominal CT. d. Bolus 2-L NS, order a type and crossmatch and β-hCG, and call gynecology for possible surgery. e. Provide oxygen via face mask, give morphine sulfate, and order a transvaginal ultrasound.

*D* You should be concerned about a ruptured ectopic pregnancy in this patient. She missed her last menstrual period, has severe pain in the lower abdomen, and is hypotensive. This is a life-threatening condition that needs to be managed aggressively. The patient requires fluid resuscitation with 2-L NS. If her BP does not respond to the bolus, then blood should be administered. The patient will most likely be taken to the OR for a salpingectomy/oophorectomy. Risk factors for an ectopic pregnancy include history of pelvic inflammatory disease, prior ectopic pregnancy, pelvic surgery, and intrauterine device (IUD) use.

A 29-year-old man presents to the ED after being stabbed in his neck. The patient is speaking in full sentences. His breath sounds are equal bilaterally. His BP is 130/75 mm Hg, HR is 95 beats per minute, RR is 16 breaths per minute, and oxygen saturation is 99% on room air. The stab wound is located between the angle of the mandible and the cricoid cartilage and violates the platysma. There is blood oozing from the site although there is no expanding hematoma. Which of the following is the most appropriate next step in management? a. Explore the wound and blind clamp any bleeding site. b. Probe the wound looking for injured vessels. c. Apply direct pressure and bring the patient immediately to the OR to explore the zone 1 injury. d. Apply direct pressure and bring the patient immediately to the OR to explore the zone 2 injury. e. Apply direct pressure and bring the patient immediately to the OR to explore the zone 3 injury.

*D* Zone I extends superiorly from the sternal notch and clavicles to the cricoid cartilage. Injuries to this region can affect both neck and mediastinal structures. Zone II is the area between the cricoid cartilage and the angle of the mandible. Zone III extends from the angle of the mandible to the base of the skull. Generally, Zone II injuries are taken directly to the operating room for surgical exploration. Injuries in Zones I and III may be taken to the operating room or managed conservatively using a combination of angiography, bronchoscopy, esophagoscopy, and CT scanning.

A 12-year-old girl presents to the ED with a left index finger laceration. She sustained the injury when her friend accidentally closed the car door on her finger. She has a semicircular laceration distal to the distal interphalangeal (DIP) joint on the volar surface of the left second digit. Sensation distal to the laceration is intact. Range of motion at the DIP joint is intact. The patient hopes to be a hand model in the future and is extremely anxious about the cosmetic result. Which of the following is the best method to provide anesthesia in this patient? a. Application of a tourniquet to control bleeding and irrigation of the wound with 3 cc of lidocaine b. Local infiltration through the wound margins with 3 cc of lidocaine c. Local infiltration through the wound margins with 3 cc of lidocaine with epinephrine d. A digital block at the base of the proximal phalanx with 3 cc of lidocaine e. A digital block at the base of the proximal phalanx with 3 cc of lidocaine with epinephrine

*D*Application of a lidocaine digital block at the base of the finger will block the digital nerves allowing for appropriate anesthesia. The needle is inserted into the web space on either side and anesthesia is deposited anteriorly and posteriorly. This is repeated on the opposite side of the affected digit

A 43-year-old woman presents to the ED with a 3-week history of intermittent headache, nausea, and fatigue. She was seen at her private doctor's office 1 week ago along with her husband and children, who also have similar symptoms. They were diagnosed with a viral syndrome and told to increase their fluid intake. She states that the symptoms began approximately when it started to get cold outside. The symptoms are worse in the morning and improve while she is at work. Her BP is 123/75 mm Hg, HR is 83 beats per minute, temperature is 98.9°F, and oxygen saturation is 98% on room air. Physical examination is unremarkable. You suspect her first diagnosis was incorrect. Which of the following is the most appropriate next step to confirm your suspicion? a. Order a mono spot test b. Perform a nasal pharyngeal swab to test for influenza c. Consult psychiatry to evaluate for malingering d. Order a carboxyhemoglobin (COHb) level e. Order a lead level

*D*The most useful diagnostic test obtainable in a suspected CO poisoning is a COHb level. Normal levels range from 0% to 5%, as CO is a natural by-product of the metabolism of porphyrins. COHb levels average 6% in one-pack-per-day smokers. CO poisoning should be suspected when multiple patients, usually in the same family, present with flu-like symptoms, and were exposed to products of combustion (eg, home heaters/generators). This most commonly occurs in colder, winter months. The mainstay of treatment is the delivery of oxygen. Hyperbaric oxygen is usually used for patients with COHb levels greater than 25%.

A 4-year-old boy is brought to the ED by his mother with a complaint of ear pain since last night. The patient also complains of mild congestion and cough. He has normal oral intake, activity, and urine output. On examination, the patient is alert, interactive and playful. His temperature is 100.6°F, HR 128 beats per minute, RR 26 breaths per minute, and pulse oxymetry is 98% on room air. His right tympanic membrane has slight erythema with several bubbles of clear fluid noted inferiorly. His left tympanic membrane is erythematous and bulging with opaque, purulent fluid noted. There is no neck stiffness, mastoid tenderness, or difficulty breathing. The patient has a primary-care physician who they see regularly. Which of the following is the best treatment for this patient? a. Analgesics only b. A third-generation cephalosporin c. Amoxicillin 40 to 50 mg/kg/day d. Treatment with analgesics initially and consideration of oral high dose amoxicillin in 48 to 72 hours if no resolution of symptoms e. No treatment is needed for this viral illness

*D*This is a case of a viral URI with an acute otitis media (AOM). Treatment for otitis media evolved significantly over the last several years. Otitis has traditionally been treated in the United States with oral antibiotics but there is now data showing that most of these infections resolve spontaneously, even those caused by bacterial organisms. Bacterial otitis media can present similarly but is more typical to have viral symptoms for several days leading to eustachian tube inflammation and dysfunction, which subsequently predisposes a child to secondary bacterial infection from the fluid that cannot drain from the middle ear. In cases of AOM in children over the age of 2 years for whom secondary complications are less likely, the recommended treatment is analgesics (can be topical, oral, or a combination of both) with a 48 to 72 hour period of waiting and observing for improvement or persistence of symptoms. This will avoid unnecessary antibiotics in many children. The remainder should be treated with amoxicillin at a high dose (80-90 mg/kg/d divided into two doses) as first-line therapy.

A 32-year-old man with no past medical problems presents to the ED with palpitations. For the past 2 days he has been feeling weak and over the last 6 hours he has noticed that his heart is racing. He has no chest pain or shortness of breath. He has never felt this way before. His temperature is 98.9°F, BP is 140/82 mm Hg, HR is 180 beats per minute, and RR is 14 breaths per minute. His physical examination is normal. You obtain the following rhythm strip. showing a narrow complex, regular tachycardia What is your first-line treatment for this patient? a. Synchronized cardioversion at 100 J b. Adenosine 6-mg intravenous (IV) push c. Adenosine 12-mg IV push d. Valsalva maneuver e. Verapamil 3-mg IV push

*D*This patient has supraventricular tachycardia (SVT), a narrow complex, regular tachycardia. It is caused by a reentry or an ectopic pacemaker in areas of the heart above the bundle of His, usually the atria. Regular P waves will be present, but may be difficult to discern owing to the very fast rate. The patient in this case has normal vital signs and examination, and is therefore stable. First-line treatment for a patient with stable SVT is vagal maneuvers to slow conduction and prolong the refractory period in the AV node. The Valsalva maneuver can be accomplished by asking the patient to bear down as if they are having a bowel movement and hold the strain for at least 10 seconds. Other vagal maneuvers include carotid sinus massage (after auscultating for carotid bruits) and facial immersion in cold water.

A 61-year-old woman with a history of diabetes and hypertension is brought to the ED by her daughter. The patient states that she started feeling short of breath approximately 12 hours ago and then noticed a tingling sensation in the middle of her chest and became diaphoretic. An ECG reveals ST-depression in leads II, III, and aVF. You believe that the patient had a non-ST-elevation MI. Which of the following cardiac markers begins to rise within 3 to 6 hours of chest pain onset, peaks at 12 to 24 hours, and returns to baseline in 7 to 10 days? a. Myoglobin b. Creatinine kinase (CK) c. Creatinine kinase-MB (CK-MB) d. Troponin I e. Lactic dehydrogenase (LDH)

*D.* myoglobin- rises w/ in 1-2 hrs, peaks at 4-6, baseline at 24hrs CK - rises 3-8 hrs, peaks at 12-24 hrs, normalizes at 3-4 days CK-MB rises 4-6 hrs, peaks at 12-36 hrs, normalizes at 3-4 days LDH rises 12 hrs, peaks at 24-48 hrs, returns to normal 10-14 days

A 28-year-old man presents to the ED complaining of constant vague, diffuse epigastric pain. He describes having a poor appetite and feeling nauseated ever since eating sushi last night. His BP is 125/75 mm Hg, HR is 96 beats per minute, temperature is 100.5°F, and his RR is 16 breaths per minute. On examination, his abdomen is soft and moderately tender in the right lower quadrant (RLQ). Laboratory results reveal a WBC of 12,000/μL. Urinalysis shows 1+ leukocyte esterase. The patient is convinced that this is food poisoning from the sushi and asks for some antacid. Which of the following is the most appropriate next step in management? a. Order a plain radiograph to look for dilated bowel loops. b. Administer 40 cc of Maalox and observe for 1 hour. c. Send the patient for an abdominal ultrasound. d. Order an abdominal CT scan. e. Discharge the patient home with ciprofloxacin.

*D.* Appendicitis is the most common cause of the acute surgical abdomen. It can occur at any age but is most prevalent in the teens and twenties. It is classically described as starting with the vague onset of dull periumbilical pain that migrates to the RLQ. It is associated with anorexia, nausea, and vomiting. A low-grade fever may develop. You should suspect appendicitis in any patient with RLQ pain. Approximately 90% of patients have a WBC > 10,000/mm3 although, the WBC can be normal in appendicitis. Urinalysis can help differentiate urinary tract disease from acute appendicitis, although a mild pyuria may be seen in appendicitis if the appendix is irritating the ureter. Abdominal CT with IV and oral or rectal contrast is reported to have a sensitivity of up to 100% and specificity of 95%. CT findings of appendicitis include an enlarged appendix (> 6 mm), pericecal inflammation, and the presence of an appendicolith.

While discussing a case presentation with a medical student, a nearby patient who just returned from getting an ankle radiograph done yells out in pain. You walk over to him and ask what is wrong. He states that since returning from the radiology suite, his automatic implantable cardioverter defibrillator (AICD) is discharging. You hook him up to the monitor and note that his rhythm is sinus. You observe a third shock while the patient is in sinus rhythm. Which of the following is the most appropriate next step in management? a. Send the patient back to the radiology suite for another radiograph to desensitize his AICD. b. Administer pain medication and wait until the device representative arrives at the hospital to power off the AICD. c. Admit the patient to the telemetry unit to monitor his rhythm and find the cause of his AICD discharge. d. Place a magnet over the AICD generator to inactivate it and thereby prevent further shocks. e. Make a small incision over his chest wall and remove the AICD generator and leads.

*D.* Automatic implantable cardioverter—defibrillators (AICD) are placed in patients who are at high risk for fatal dysrhythmias (eg, ventricular tachycardia and fibrillation) and sudden death. In these patients, AICDs decrease the risk of sudden death from approximately 40% per year to less than 2% per year. Occasionally, it may become necessary to temporarily deactivate an AICD, as in the case of inappropriate shock in the setting of a stable rhythm (such as seen with the patient in question). The patient in this scenario is receiving a shock while he is in sinus rhythm. Some potential causes of inappropriate shock delivery include false sensing of supraventricular tachycardias, muscular activity (eg, shivering), sensing T waves as QRS complexes, unsustained tachydysrhythmias, and component failure. AICDs are generally inactivated by placing a magnet over the AICD generator. Although, there is some variability depending on the generation of the AICD, most EDs have a special donut magnet that is reserved for this function. If the patient subsequently experiences a dysrhythmia in the setting of having his or her defibrillator tuned off the physician should use the bedside defibrillator to treat the patient.

For which of the following patients is an abdominal CT scan contraindicated? a. A 52-year-old man with abdominal pain after blunt trauma, negative focused assessment with sonography for trauma (FAST) examination, BP 125/78 mm Hg, and HR 109 beats per minute b. A 22-year-old female with RLQ pain, negative β-hCG, temperature 100.6°F c. A 45-year-old man with abdominal pain, temperature 100.5°F, WBC 11,200/μL, BP 110/70 mm Hg, HR 110 beats per minute, and lipase 250 IU d. A 70-year-old man with abdominal pain, an 11-cm pulsatile mass in the epigastrium, BP of 70/50 mm Hg, and HR of 110 beats per minute e. A 65-year-old woman with right flank pain that radiates to her groin, microhematuria, BP 165/85 mm Hg, and HR 105 beats per minute

*D.* CT scanning has become an integral part of the ED evaluation of a patient. It is useful to differentiate abdominal pathologies when the history and physical examination are nonspecific, or in confirming a diagnosis suspected by the clinical presentation. Contrast materials for CT examinations in the ED are usually administered via oral and IV routes, which improves visualization of abdominal organs. A patient must be relatively stable to undergo a CT scan. The patient with abdominal pain and a BP of 70/50 mm Hg is not hemodynamically stable. His clinical presentation is consistent with a ruptured abdominal aortic aneurysm. This is a surgical emergency and should be treated in the OR. Obtaining a CT scan first will delay definitive treatment and worsen the patient's hemorrhagic shock and ultimately decrease his chance of survival.

A 42-year-old man found vomiting in the street is brought to the ED by EMS. He has a known history of alcohol abuse with multiple presentations for intoxication. Today, the patient complains of acute onset, persistent chest pain associated with dysphagia, and pain upon flexing his neck. His BP is 115/70 mm Hg, HR is 101 beats per minute, RR is 18 breaths per minute, and oxygen saturation is 97% on room air. As you listen to his heart, you hear a crunching sound. His abdomen is soft with mild epigastric tenderness. The ECG is sinus tachycardia without ST-T-wave abnormalities. On chest x-ray, you note lateral displacement of the left mediastinal pleural. What is the most likely diagnosis? a. Aspiration pneumonia b. Acute pancreatitis c. Pericarditis d. Esophageal perforation e. Aortic dissection

*D.* Esophageal perforation is a potentially life-threatening condition that can result from any Valsalva-like maneuver, including childbirth, coughing, and heavy lifting. Alcoholics are at risk as a result of their frequent vomiting. The most common cause of esophageal perforation is from iatrogenic causes, such as a complication from upper endoscopy. The classic physical examination finding is mediastinal or cervical emphysema. This is noted by feeling air under the skin on palpation of the chest wall or by a *crunching sound heard on auscultation, also known as Hamman sign*. Radiographic signs of pneumomediastinum can be subtle. Lateral displacement of the mediastinal pleura by mediastinal air creates a linear density paralleling the mediastinal contour. On the lateral projection, mediastinal air can be seen in the retrocardiac space

A 79-year-old man was being commemorated at an awards dinner for his 50 years of service at the local bank. While eating a steak dinner, he felt food get stuck in his stomach. He drank a glass of water, but shortly thereafter vomited the water up. He is a bit anxious and decides to come to the ED for further evaluation. His BP is 155/70 mm Hg, HR is 98 beats per minute, RR is 18 breaths per minute, and oxygen saturation is 99% on room air. What is the most common area for an esophageal foreign body to lodge in an adult? a. Aortic arch b. Cricopharyngeus muscle c. Tracheal bifurcation d. Lower esophageal sphincter e. Pyloric sphincter

*D.* Foreign bodies tend to lodge or impact at sites where esophageal narrowing occurs. In the adult, the lower esophageal sphincter is the most common site for impactions. When impaction occurs, patients usually feel some discomfort, sometimes substernal chest pain, anxiety, and progressive dysphagia. Most of the time, the patient can accurately locate the position of the impacted foreign body.

A 71-year-old woman presents to the ED after a reported mechanical fall 2 days ago. Her initial vitals include a HR of 55 beats per minute, a BP of 110/60 mm Hg, an RR of 14 breaths per minute, and an oxygen saturation of 96% on room air. The patient does not appear to be taking deep breaths. Her physical examination is significant for decreased breath sounds bilaterally and tenderness to palpation along the right side of her chest. After initial stabilization, which of the following is the diagnostic test of choice for this patient's condition? a. Chest x-ray b. Chest CT scan c. ECG d. Rib radiographs e. Thoracentesis

*D.* Given the history of trauma, a rib fracture is the most probable etiology of the patient's symptoms in this clinical scenario. Rib fractures usually occur at the point of impact or at the posterior angle, which is the weakest part of the rib. It is important to note that the true danger of rib fractures involves not the rib itself, but the risk of penetrating injury to underlying structures. A rib series is the most effective way to visualize these fractures. Treatment of patients with simple acute rib fractures includes pain relief so that respiratory splinting does not occur, which increases the rate of atelectasis and pneumonia. Chest binders should not be used as they promote hypoventilation. For multiple rib fractures, intercostal nerve blocks may be a more effective means of analgesia. Most rib fractures heal uneventfully within 3 to 6 weeks. The patient should be encouraged to take deep breaths to avoid developing pneumonia.

A 30-year-old obese woman with no significant past medical history presents to the ED complaining of shortness of breath and coughing up blood-streaked sputum. The patient states that she traveled to Moscow a month ago. Upon returning to the United States, the patient developed a persistent cough associated with dyspnea. She was seen by a pulmonologist, who diagnosed her with bronchitis and prescribed an inhaler. However, over the following weeks, the patient's symptoms worsened, and she developed pleuritic chest pain. In the ED, she lets you know that she smokes half a pack per day. Her vitals include a temperature of 99°F, BP of 105/65 mm Hg, HR of 124 beats per minute, RR of 22 breaths per minute, and an oxygen saturation of 94% on room air. Physical examination is noncontributory, except for rales at the left-mid lung. Her ECG reveals sinus tachycardia with large R waves in V1 to V3 and inverted T waves. Given this patient's history and presentation, what is the most likely etiology of her symptoms? a. Mycoplasma pneumoniae ("walking" pneumonia) b. Q fever pneumonia c. Pneumocystis jiroveci pneumonia (PCP) d. PE e. ARDS

*D.* In the history there are details that lead you to suspect pulmonary embolus: obesity, recent travel, progressive dyspnea despite inhaler treatment, and blood-streaked sputum. Objectively, her ECG shows right heart strain (large R waves and inverted T waves in the precordial leads V1-V3), which is because of the heart beating against the high resistance of the pulmonary vasculature causing backflow resulting in right ventricular enlargement. She is tachycardic, tachypneic, and hypoxic, cardinal signs of cardiovascular distress. Either dyspnea, pleuritic chest pain, or tachypnea is present in 95% of patients with a PE. The classic triad of dyspnea, pleuritic chest pain, and hemoptysis is uncommon and present in less than 25% of patients.

A 61-year-old woman was on her way to the grocery store when she started feeling chest pressure in the center of her chest. She became diaphoretic and felt short of breath. On arrival to the ED by EMS, her BP is 130/70 mm Hg, HR is 76 beats per minute, and oxygen saturation is 98% on room air. The nurse gives her an aspirin and an EKG shows ST elevation in II, III, and aVF. Which of the following best describes the location of this patient's myocardial infarction (MI)? a. Anteroseptal b. Anterior c. Lateral d. Inferior e. Posterior

*D.* Inferior wall MIs are characterized by ST elevation in at least two of the inferior leads (II, III, aVF).

A 22-year-old man presents to the ED with a history consistent with an acute MI. His ECG reveals ST elevations and his cardiac biomarkers are positive. He has been smoking half a pack of cigarettes per day for the last 3 months. He drinks alcohol when hanging out with his friends. His grandfather died of a heart attack at 80 years of age. The patient does not have hypertension or diabetes mellitus and takes no prescription medications. A recent cholesterol check revealed normal levels of total cholesterol, lowdensity lipoprotein (LDL), and high-density lipoprotein (HDL). Which of the following is the most likely explanation for his presentation? a. Cigarette smoking b. Family history of heart attack at age 80 years c. Incorrectly placed leads on the ECG d. Undisclosed cocaine use e. Alcohol use

*D.* Often young people are afraid to disclose a history of drug use. Cocaine is well-known to cause acute MI in young, otherwise healthy individuals. Patients with cocaine-related MI often have fixed atherosclerotic lesions. Although these lesions may themselves be of clinical significance, cocaine-induced elevations in pulse and BP increase myocardial work. The additional metabolic requirements that result may convert an asymptomatic obstruction into one of clinical significance. Cocaine use combined with ethanol consumption produces cocaethylene, a longer-acting and more toxic by-product.

A 59-year-old man presents to the ED complaining of vomiting and sharp abdominal pain in the epigastric area that began abruptly this afternoon. He describes feeling nauseated and has no appetite. Laboratory results reveal WBC 18,000/μL, hematocrit 48%, platelets 110/μL, AST 275 U/L, ALT 125 U/L, alkaline phosphatase 75 U/L, amylase 1150 U/L, lipase 1450 IU, LDH 400 U/L, sodium 135 mEq/L, potassium 3.5 mEq/L, chloride 110 mEq/L, bicarbonate 20 mEq/L, BUN 20 mg/dL, creatinine 1.5 mg/dL, and glucose 250 mg/dL. Which of the following laboratory results correlate with the poorest prognosis? a. Amylase 950, lipase 1250, LDH 400 b. Lipase 1250, LDH 400, bicarbonate 20 c. Lipase 1250, creatinine 1.5, potassium 3.5 d. WBC 18,000, LDH 400, glucose 250 e. WBC 18,000, amylase 950, lipase 1250

*D.* The patient's clinical picture is consistent with acute pancreatitis. Ranson developed criteria that help predict mortality rates in patients with pancreatitis. The presence of more than three criteria equals 1% mortality, while the presence of six or more criteria approaches 100% mortality. Ranson criteria at admission are age > 55, WBC > 16,000, glucose > 200, LDH > 350, AST > 250. Within 48 hours of admission, hematocrit fall > 10%, BUN rise > 5, serum calcium < 8, arterial PO2 < 60, base deficit > 4, and fluid sequestration > 6 L. The patient in the case fulfills four of Ranson criteria and has approximately 15% mortality risk. Note that lipase and amylase are not part of Ranson criteria despite being relevant in the diagnosis of acute pancreatitis. The mnemonic GA LAW & CHOBBS can be used to remember this criteria. Stands for Glucose, Age, LDH, AST and WBC; Calcium, Hematocrit, Oxygen, BUN, Base, Sequestration.

A 41-year-old woman presents to the ED complaining of pain in her RUQ that is steady but gets worse with eating over the past 2 days. The pain also radiates to the right side of her midback. She denies vomiting. Her only medication is an oral contraceptive. Her BP is 140/75 mm Hg, HR is 80 beats per minute, temperature is 98.7°F, and RR is 16 breaths per minute. Laboratory tests are within normal limits. An abdominal ultrasound reveals stones in her gallbladder, but no thickened wall or pericholecystic fluid. What is the most likely diagnosis? a. Cholangitis b. Urolithiasis c. Cholecystitis d. Biliary colic e. Peptic ulcer disease

*D.* The patient's clinical picture is consistent with biliary colic because of the passage of small stones from the gallbladder through the cystic duct into the common bile duct. The term colic is a misnomer in that these patients usually have a steady pain rather than an intermittent pain. Pain is present in the RUQ and often is referred to the base of the scapula. Laboratory results are commonly normal. Ultrasound reveals stones in the gallbladder without other pathologies. Patients can usually be sent home with pain medications and are instructed to avoid fatty foods.

An 81-year-old diabetic woman with a history of atrial fibrillation is transferred to your emergency department (ED) from the local nursing home. The note from the facility states that the patient is complaining of abdominal pain, having already vomited once. Her vital signs in the ED are temperature 100.1°F, blood pressure (BP) 105/75 mm Hg, heart rate (HR) 95 beats per minute, and respiratory rate (RR) 18 breaths per minute. You examine the patient and focus on her abdomen. Considering that the patient has not stopped moaning in pain since arriving to the ED, you are surprised to find that her abdomen is soft on palpation. You decide to order an abdominal radiographic series. Which of the findings on plain abdominal film is strongly suggestive of mesenteric infarction? a. Sentinel loop of bowel b. No gas in the rectum c. Presence of an ileus d. Pneumatosis intestinalis e. Air-fluid levels

*D.* There is a high suspicion that this patient has mesenteric ischemia, secondary to a thromboembolism from her atrial fibrillation. The typical patient with mesenteric ischemia may initially present with "pain that is out of proportion to the examination" (ie, although the patient is in pain, the abdomen is neither rigid, nor significantly tender on physical examination). Abdominal distention and peritoneal signs are late findings and signal the presence of bowel infarction. Plain abdominal radiographs are usually obtained early on in the workup. Although rare, the finding of gas in either the bowel wall (pneumatosis intestinalis) or in the portal venous system is strongly suggestive of intestinal infarct. This is a surgical emergency! (a, b, c, and e) These are possible findings but less specific

A 63-year-old insurance agent is brought to the ED by paramedics for shortness of breath and a RR of 31 breaths per minute. The patient denies chest pain, fever, vomiting, or diarrhea. His wife says he ran out of his "water pill" 1 week ago. His BP is 185/90 mm Hg, HR is 101 beats per minute, oxygen saturation is 90% on room air, and temperature is 98.9°F. There are crackles midway up both lung fields and 2+ pitting edema midway up his legs. An ECG shows sinus tachycardia. The patient is sitting up and able to speak to you. After placing the patient on a monitor and inserting an IV, which of the following is the most appropriate next step in management? a. Obtain blood cultures, complete blood cell (CBC) count, and begin empiric antibiotic therapy. b. Order a STAT portable CXR c. Administer oxygen via nasal cannula and have the patient chew an aspirin. d. Administer oxygen via non-rebreather, furosemide, nitroglycerin, and consider noninvasive respiratory therapy. e. Rapid sequence endotracheal intubation.

*D.*The patient has acute CHF exacerbation with acute pulmonary edema (APE). Although not always apparent at presentation, it is important to find the cause of the exacerbation. This patient, for instance, has been noncompliant with his medications. Treatment begins by assessing the airway, breathing, and circulation (ABCs). Initial stabilization is aimed at maintaining airway control and adequate ventilation. Preload and afterload reduction is integral with nitroglycerin being the agent of choice, provided that the patient is not hypotensive. Volume reduction with diuretics is also critical to lower BP and cardiac-filling pressures. Noninvasive airway techniques (eg, bilevel positive airways pressure [BiPAP], continuous positive airway pressure [CPAP]) also aid in improving oxygen exchange, reducing the work of breathing, and decreasing left ventricular preload and afterload by raising intrathoracic pressure in the compromised but not agonal APE patients.

A 29-year-old woman presents to the ED for hyperventilation. Her initial vitals include an RR of 28 breaths per minute with an oxygen saturation of 100% on room air. She is able to speak in full sentences and tells you that she cannot breathe and that her hands and feet are cramping up. She denies any trauma, past medical history, or illicit drug use. Chest auscultation reveals clear breath sounds bilaterally. A subsequent chest radiograph is normal. Upon reevaluation, the patient reports that she is breathing better. Her vitals include an RR of 12 breaths per minute with an oxygen saturation of 100% on room air. Which of the following conditions is most likely the etiology of this patient's symptoms? a. Pneumothorax b. Hemopneumothorax c. Pleural effusion d. Anxiety attack e. Asthma exacerbation

*D.*The patient has normal vitals, a normal chest radiograph, and is stable. In fact, her symptoms self-resolve with time and without intervention. Only with this information can one be comfortable with making the diagnosis of an anxiety attack as the precipitant to the patient's symptoms. A history of a stressor may be helpful, but it is important to note that these symptoms are not under the voluntary control of the patient, and often patients may not even be able to identify a specific stressor. It is important to remember that the diagnosis of anxiety in the ED is a diagnosis of exclusion. Her extremity symptoms are typical of carpal-pedal spasm seen with tetany, a result of a transient decrease in calcium serum levels secondary to a respiratory alkalosis.

A 60-year-old man with a history of alcohol abuse presents to the ED with hematemesis for 1 day. He denies abdominal or chest pain. On physical examination, his eyes appear reddened which he attributes to having drunken heavily the night before (he also reveals vomiting several times after this recent binge). Vital signs are: HR 115 beats per minute, BP 130/85 mm Hg, RR 18 breaths per minute, and temperature 99.5°F. Chest radiograph is unremarkable. Laboratory results reveal a WBC 10,000/μL, hemoglobin 14 mg/dL, hematocrit 40%, and platelets 210/μL. Which diagnosis is endoscopic evaluation most likely to confirm? a. Esophageal varices b. Boerhaave syndrome c. Curling ulcer d. Perforated gastric ulcer e. Mallory-Weiss tear

*E* A Mallory-Weiss tear usually follows a forceful bout of retching and vomiting and involves a 1- to 4-cm area of the mucosa or submucosa of the GI tract; 75% of cases occur in the stomach with the remainder near the gastroesophageal (GE) junction. Bleeding is usually mild and self-limited. However, 3% of deaths from upper GI bleeds result from Mallory-Weiss tears.

A 23-year-old man presents to the ED complaining of left hand pain. He states that he was mad at a friend and punched the wall in his bedroom. Immediately after he punched the wall, he felt intense pain in his left hand. On physical examination, you note swelling and tenderness over the fifth metacarpal. When you ask him to make a fist, his fifth finger rotates to lie on top of his fourth finger. What is the name of this type of fracture? a. Colles fracture b. Smith fracture c. Scaphoid fracture d. Galeazzi fracture e. Boxer's fracture

*E* A boxer's fracture is a fracture of the neck of the fifth metacarpal. It is one of the most common fractures of the hand and usually occurs from a direct impact to the hand (eg, a punch with a closed fist).

A 43-year-old man presents to the ED complaining of progressively worsening abdominal pain over the past 2 days. The pain is constant and radiates to his back. He also describes nausea and vomiting and states he usually drinks 6 pack of beer daily, but has not had a drink for 2 days. His BP is 144/75 mm Hg, HR is 101 beats per minute, temperature is 99.8°F, and RR is 14 breaths per minute. He is lying on his side with his knees flexed. Examination shows voluntary guarding and tenderness to palpation of his epigastrium. Laboratory results reveal WBC 10,500/μL, hematocrit 51%, platelets 225/μL, and lipase 620 IU. An abdominal radiograph reveals a nonspecific bowel gas pattern. There is no free air under the diaphragm. Which of the following is the most appropriate next step in management? a. Observe in the ED b. Send home with antibiotic therapy c. Admit to the hospital for endoscopy d. Admit to the hospital for exploratory laparotomy e. Admit to the hospital for medical management and supportive care

*E* Abdominal pain that radiates to the back and is associated with nausea, vomiting, epigastric tenderness, and an elevated lipase in a patient with a history of ethanol abuse all point to a diagnosis of alcoholic pancreatitis. Although patients with mild pancreatitis, no evidence of systemic complications, and a low likelihood of biliary tract disease may be managed as outpatients, this patient requires admission for the potentially rapid progression of symptoms, severity of pain, and possible unreliability of the patient. The initial treatment for acute pancreatitis is supportive: bowel rest, fluid resuscitation, and analgesia. Ninety percent of patients recover without complications. Surgery is reserved for complications of alcoholic pancreatitis, such as pseudocysts, phlegmons, and abscesses.

An 18-year-old woman is brought to the ED by her mother. The patient is diaphoretic and vomiting. Her mom states that she thinks her daughter tried to commit suicide. The patient admits to ingesting a few handfuls of extra-strength Tylenol approximately 3 hours ago. Her temperature is 99.1°F, BP is 105/70 mm Hg, HR is 92 beats per minute, RR is 17 breaths per minute, and oxygen saturation is 99% on room air. On examination, her head and neck are unremarkable. Cardiovascular and pulmonary examinations are within normal limits. She is mildly tender in her right upper quadrant but there is no rebound or guarding. Bowel sounds are normoactive. She is alert and oriented and has no focal deficits on neurologic examination. You administer 50 g of activated charcoal. At this point, she appears well and has no complaints. Her 4-hour serum acetaminophen (APAP) concentration returns at 350 μg/mL. You plot the level on the nomogram seen below. Which of the following is the most appropriate next step in management? a. Discharge home with instructions to return if symptoms return b. Observe for 6 hours and if the patient still has no complaints discharge her home c. Admit to the hospital for serial abdominal examinations d. Admit to the psychiatry unit and keep on suicide watch while performing serial abdominal examinations e. Begin NAC and admit to the hospital

*E* Acetaminophen is one of the most commonly used analgesic-antipyretic medications and causes more hospitalizations after overdose than by any other pharmaceutical agent. Risk of hepatotoxicity is best established by plotting the acetaminophen concentration on the acetaminophen nomogram. Acetaminophen concentration must be measured between 4 and 24 hours after ingestion and then plotted on the nomogram. Patients with acetaminophen concentrations on or above the treatment line should be treated. This patient has a 4-hour serum acetaminophen concentration of 350 μg/mL. According to the nomogram, at 4 hours any concentration above 200 μg/mL should be treated. Therefore, the patient should be started on NAC and admitted to the hospital. During her admission, she should be evaluated by a psychiatrist regarding her attempted suicide.

A 24-year-old man is brought to the ED after being shot once in the abdomen. On arrival, his BP is 100/60 mm Hg, HR is 115 beats per minute, and RR is 22 breaths per minute. His airway is patent and you hear breath sounds bilaterally. On abdominal examination, you note a single bullet entry wound approximately 1 cm to the right of the umbilicus. During the log roll, you see a single bullet exit wound approximately 3 cm to the right of the lumbar spine. His GCS score is 15. The patient's BP is now 85/65 mm Hg and HR is 125 beats per minute after 2 L of fluid. Which of the following is the most appropriate next step in management? a. Probe the entry wound to see if it violates the peritoneum b. Perform a FAST examination c. Perform a DPL d. Take the patient directly to the CT scanner e. Take the patient directly to the OR

*E* An important concern with anterior abdominal GSW is to determine whether the missile traversed the peritoneal cavity. Patients with transabdominal GSWs nearly all have intra-abdominal injuries requiring surgery. Most of the time, this can be determined by approximating the trajectory. Therefore, a hole in both the anterior and posterior abdomen highly suggests a transabdominal trajectory. If there are a single or odd number of holes, a plain film may help estimate the trajectory. In cases of tangential or multiple GSWs, it may be impossible to determine trajectory with any certainty. In a patient with evidence of peritoneal penetration, a missile tract that clearlyenters the abdominal cavity, or has a positive diagnostic study (eg, DPL, FAST, CT) in a tangential wound, he or she should undergo exploratory laparotomy. The standard algorithm for penetrating abdominal trauma recommends that any patient with unstable vital signs be taken directly to the OR to undergo an exploratory laparotomy. If their vital signs are stable, they should undergo further diagnostic studies, such as a FAST examination, DPL, or CT scan.

A 42-year-old man who is actively seizing is brought to the ED by EMS after a massive ingestion of an unknown substance. The man is known to have a history of acquired immunodeficiency syndrome (AIDS). An intravenous (IV) line is established and anticonvulsant therapy is administered. After high doses of diazepam, phenobarbital, and phenytoin, it is determined that the seizures are refractory to standard anticonvulsant therapy. Which of the following substances did this patient most likely ingest? a. Cocaine b. Diphenhydramine c. Tricyclic antidepressant d. Camphor e. Isoniazid (INH)

*E* An overdose of any of these agents can lead to seizures. However, INH is notorious for causing seizures that are refractory to standard therapy. Marked acidosis and respiratory compromise may also be present. Pyridoxine (vitamin B6) is the treatment of choice for INH overdose. INH is used for the treatment of tuberculosis, which is seen with a greater incidence in patients with AIDS. All of the other substances listed as answer choices should respond to standard therapy with benzodiazepines.

A 55-year-old man with hypertension and end-stage renal disease requiring hemodialysis presents with 2 days of painless hematochezia. He reports similar episodes of bleeding in the past, which were attributed to angiodysplasia. He denies abdominal pain, nausea, vomiting, diarrhea, and fever. His vitals include HR of 90 beats per minute, BP of 145/95 mm Hg, RR of 18 breaths per minute, and temperature of 98°F. His abdomen is soft and nontender and his stool is grossly positive for blood. Which of the following statements are true regarding angiodysplasia? a. They are responsible for over 50% of acute lower GI bleeding. b. They are more common in younger patients. c. Angiography is the most sensitive method for identifying angiodysplasias. d. They are less common in patients with end-stage renal disease. e. The majority of angiodysplasias are located on the right side of the colon.

*E* Angiodysplasias, also known as arteriovenous malformations, are small ectatic blood vessels in the submucosa of the GI tract. More than half of angiodysplasias are located on the right side of the colon.

A 51-year-old man with a long history of hypertension presents to the ED complaining of intermittent chest palpitations lasting for a week. He denies chest pain, shortness of breath, nausea, and vomiting. He recalls feeling similar episodes of palpitations a few months ago but they resolved. His blood pressure (BP) is 130/75 mm Hg, heart rate (HR) is 130 beats per minute, respiratory rate (RR) is 16 breaths per minute, and oxygen saturation is 99% on room air. An ECG shows a fib. Which of the following is the most appropriate next step in management? a. Sedate patient for immediate synchronized cardioversion with 100 Joules b. Prepare patient for the cardiac catheterization laboratory c. Administer Coumadin d. Administer amiodarone e. Administer diltiazem

*E* Atrial fibrillation (AF) is a rhythm disturbance of the atria that results in irregular, chaotic, ventricular waveforms. This chaotic activity can lead to reduced cardiac output from a loss of coordinated atrial contractions and a rapid ventricular rate, both of which may limit diastolic filling and stroke volume of the ventricles. Atrial fibrillation may be chronic or paroxysmal, lasting minutes to days. On the ECG, fibrillatory waves are seen and accompanied by an irregular QRS pattern. The main ED treatment for atrial fibrillation is rate control. This can be accomplished by many agents, but the agent most commonly used is diltiazem, a CCB with excellent AV nodal blocking effects

A 34-year-old construction worker is brought to the ED by EMS after falling 30 ft from a scaffold. His vital signs are HR 124 beats per minute, BP 80/40 mm Hg, and oxygen saturation 93% on 100% oxygen. He has obvious head trauma with a scalp laceration overlying a skull fracture on his occiput. He does not speak when asked his name, his respirations are poor and you hear gurgling with each attempted breath. Auscultation of the chest reveals diminished breath sounds on the right. There is no jugular venous distension (JVD) or anterior chest wall crepitus. His pelvis is unstable with movement laterally to medially and you note blood at the urethral meatus. His right leg is grossly deformed at the knee and there is an obvious fracture of his left arm. Which of the following is the most appropriate next step in management? a. Insert a 32F chest tube into the right thoracic cavity. b. Perform a DPL to rule out intra-abdominal hemorrhage. c. Create two Burr holes into the cranial vault to treat a potential epidural hematoma. d. Immediately reduce the extremity injuries and place in a splint until the patient is stabilized. e. Plan for endotracheal intubation of the airway with in-line stabilization of the cervical spine.

*E* Based on the principles of advanced trauma life support (ATLS), injured patients are assessed and treated in a fashion that establishes priorities based on their presenting vital signs, mental status, and injury mechanism. The approach to trauma care consists of a primary survey, rapid resuscitation, and a more thorough secondary survey followed by diagnostic testing. The goal of the primary survey is to quickly identify and treat immediately life-threatening injuries. The assessment of the ABCDEs (airway, breathing, circulation, neurologic disability, exposure) is a model that should be followed in all patients.

A 27-year-old man presents to the ED extremely agitated complaining of mild chest pain and dyspnea. He states that he was snorting cocaine all afternoon. You place him on a monitor and get his vital signs. His BP is 215/130 mm Hg, HR is 112 beats per minute, temperature is 100.1°F, RR is 17 breaths per minute, and oxygen saturation is 98% on room air. An ECG reveals sinus tachycardia at a rate of 116. Which of the following is the most appropriate medication to administer? a. Haloperidol b. Labetalol c. Esmolol d. Diltiazem e. Diazepam

*E* Benzodiazepines (eg, diazepam) should be used as the first-line agent for nearly all cocaine toxicities. Many effects of cocaine are thought to be mediated through CNS stimulation by the release or inhibiting the reuptake of catecholamines. The effects that acute cocaine intoxication has on the heart include coronary vasoconstriction with increasing myocardial oxygen demand. The goal of lowering the BP in this patient is to reverse the vasoconstriction by norepinephrine at peripheral α-adrenergic receptors. Benzodiazepines restore the CNS inhibitory tone on the peripheral nervous system. The use of β-adrenergic antagonists should be avoided with cocaine because of paradoxical hypertension and coronary artery vasoconstriction because of unopposed α-adrenergic receptor stimulation.

A 35-year-old man who is employed as a forklift operator was found sitting outside of a warehouse. He came stumbling out complaining of dizziness and headaches. Coworkers in an adjoining warehouse also complained of headache and nausea. After collapsing outside, he regained consciousness immediately but appeared confused. In the ED, his BP is 100/54 mm Hg, HR is 103 beats per minute, temperature is 100°F, and RR is 23 breaths per minute. Physical examination is unremarkable. Laboratory results reveal WBC 10,500/μL, hematocrit 45%, platelets 110/μL, sodium 137 mEq/L, potassium 4 mEq/L, chloride 103 mEq/L, bicarbonate 21 mEq/L, BUN 8 mg/dL, creatinine 0.5 mg/dL, and glucose 89 mg/dL. Arterial blood gas results are pH 7.32, PCO2 32 mm Hg, and PO2 124 mm Hg. Which of the following is the most likely diagnosis? a. Methemoglobinemia b. Hypoglycemic syncope c. Hydrocarbon poisoning d. Opioid overdose e. CO poisoning

*E* CO poisoning is the leading cause of poisoning morbidity and mortality in the United States. People are exposed to CO through fires, vehicle exhaust, home generators, and the metabolism of methylene chloride. Workers also become symptomatic from use of propane-powered equipment indoors, such as forklifts and ice skating resurfaces. Often, other people in the area have similar complaints. The earliest symptoms are nonspecific and readily confused with other illnesses such as viral syndromes. Mild symptoms include headache, nausea, and dizziness. Severe symptoms include chest pain, palpitations, and seizures. Diagnosis is made by detecting elevated CO in the blood. Normal levels range from 0% to 5%, while smokers may be as high as 10%. Treatment includes immediate oxygen therapy. Hyperbaric oxygen is the treatment of choice for patients with significant CO exposures.

A 23-year-old woman presents to the ED complaining of pain with urination. She has no other complaints. Her symptoms started 3 weeks ago. During this time, she has been to the clinic twice, with negative urine cultures each time. Her condition has not improved with antibiotic therapy with sulfonamides or quinolones. Physical examination is normal. Which of the following organisms is most likely responsible for the patient's symptoms? a. Staphylococcus aureus b. Herpes simplex virus c. Trichomonas vaginalis d. Escherichia coli e. Chlamydia trachomatis

*E* Chlamydia trachomatis urethritis accounts for 5% to 20% of cases of dysuria, and its presence may be suspected when urine cultures are sterile. If clinical symptoms and urinalysis point to a UTI, urine cultures are sterile, and standard antibiotic regimens effective against most urinary pathogens fail, consider Chlamydia urethritis.

An 18-year-old woman presents to the ED w ith a lower lip lacerationsecondary to a fall. The patient reports she fell forward after her heel caught in a storm drain. In addition to small facial abrasions, she has a 3-cm-deep laceration on the inner aspect of the lower lip. You plan to close the laceration to control bleeding and prevent food impaction. Which of the following suture material is the best choice in this patient? a. 2-0 nylon b. 6-0 nylon c. 3-0 monofilament absorbable d. 1-0 absorbable chromic gut e. 5-0 absorbable chromic gut

*E* Choosing a suture material is based on tissue type, tissue tension, and whether or not sutures will require removal. Oral mucosa is rapidly healing and under moderate tension. Additionally, because the inner lip mucosa does not require a cosmetic result, absorbable sutures should be used. Therefore, absorbable chromic gut is the best suture choice. A 5-0 chromic gut will provide sufficient tensile strength to match that of the oral mucosa.

A 67-year-old man is brought to the ED by his wife who states that her husband's face looks different and that he has been nauseated, vomiting, and unsteady on his feet since yesterday. The patient also states that he has been having blurry vision and difficulty swallowing, in addition to feeling like the room is tilting from side to side. The patient attributes this to eating leftover salmon last night. The patient's past medical history is notable for obesity and hypertension. His BP is 187/89 mm Hg, HR is 86 beats per minute, temperature is 99.3°F, and RR is 13 breaths per minute. On examination, you find a right facial droop, diplopia, vertical nystagmus, and severe ataxia. Which of the following is the most likely diagnosis? a. Lacunar infarct b. BPV c. Labyrinthitis d. Posterior cerebral artery occlusion e. Vertebrobasilar artery occlusion

*E* Do not get confused with the multiple signs and symptoms in this case! They involve three distinct areas of the brain; the brainstem (facial droop, dysphagia, vertigo, and vertical nystagmus), cerebellum (ataxia, vertigo, and vertical nystagmus), and visual cortex (diplopia). All of these anatomical areas are supplied by the posterior circulation, specifically the vertebrobasilar artery. A mnemonic to help remember the presentation of a vertebrobasilar stroke is the "three D's": dizziness (vertigo), dysphagia, and diplopia. There are cerebellar and cranial nerve deficits observed on both sides of the body.

A 5-year-old boy is brought to the ED by his mom for a forehead laceration. She reports that he slipped on ice and hit his head. There was no loss of consciousness. The child is alert and active. He has a 3-cm-forehead laceration which crosses the hairline. What is the most appropriate method of wound closure in this patient? a. Shave the hair surrounding the laceration and close with interrupted sutures. b. Shave the hair surrounding the laceration and close with topical skin adhesive (Dermabond). c. Close with staples. d. Close with Steri-Strips. e. Close with interrupted sutures.

*E* Facial lacerations should be closed with a 6-0 nylon suture in interrupted fashion.

A 45-year-old man lacerated his right forehead after an altercation in a local bar. Instead of seeking medical attention, the patient applied super glue to his wound. He successfully stopped the bleeding, but some of the glue got into his right eye and now he comes to the ED with difficulty opening his right eye. What is the most appropriate treatment of this patient? a. Call ophthalmology immediately. b. Wash eye with acetone. c. Wash eye with normal saline. d. Use forceps to remove all the glue from the eye. e. Apply erythromycin ointment.

*E* Fortunately, super glue or crazy glue (cyanoacrylate) exposure to the eye is usually not permanently damaging to the eye. Corneal irritation can occur from the hard, irregularly shaped glue. To prevent this abrasive effect, large amounts of erythromycin ointment should be applied to the eye and eyelids as a lubricant. Large clumps of glue can be removed manually.

A 27-year-old G2P1001 presents to the ED with vaginal spotting. She reports occasional abdominal cramping associated with the bleeding, without fetal product passage, approximately 6 hours prior to presentation. She is currently in her first trimester and reports that her previous pregnancy was uncomplicated. She has been compliant with her prenatal vitamins and does not report any other medications or drug use. Her vitals are within normal limits and her physical examination is significant for a closed cervical os. Which of the following is the most likely diagnosis? a. Placenta previa b. Inevitable abortion c. Complete abortion d. Septic abortion e. Threatened abortion

*E* Given that this patient has a closed cervical os with no history of passing tissue, it is most likely a threatened abortion. This patient will need further ultrasound and β-hCG evaluation in 1 to 2 days to follow the pregnancy. Progesterone levels can also aid in distinguishing an ectopic pregnancy. Proper bleeding precautions should also be given and an Rh status determined.

A 39-year-old woman is brought to the ED by EMS on a backboard and cervical collar after being involved in a motor vehicle collision. The primary survey is unremarkable. However, the secondary survey reveals a deformity of the right arm involving the upper arm and elbow. Which of the following associations are paired incorrectly? a. Olecranon fracture and ulnar nerve injury b. Posterior elbow dislocation and ulnar and median nerve injury c. Anterior elbow dislocation and brachial artery injury d. Supracondylar fracture and brachial artery and median nerve injury e. Humeral shaft fracture and axillary nerve injury

*E* Humeral shaft fractures most commonly occur from a direct blow to the mid-upper arm. The fracture usually involves the middle third of the humeral shaft. The most common associated injury is damage to the radial nerve that causes wrist drop and loss of sensation in the first dorsal web space. Ulnar and median nerve injuries may also occur but are much less common

A 27-year-old woman presents to the ED 6 hours after the onset of body aches, abdominal cramping, and diarrhea. She is currently visiting relatives and normally lives in another state. She regularly takes six to eight tablets daily of hydrocodone for chronic low-back pain, sumatriptan for migraines, and amitriptyline and paroxetine for bulimia nervosa. Her BP is 130/80 mm Hg, HR is 100 beats per minute, temperature is 98.6°F, RR is 16 breaths per minute, and oxygen saturation is 99% on room air. Examination shows diaphoresis, dilated pupils, and piloerection. Neurologically she is moving all four extremities and you do not note tremors. She is alert and cooperative but seems restless. She denies hallucinations or suicidal ideations. She becomes very angry when you ask her for the phone numbers of her regular physicians. Which of the following is the most likely explanation of her symptoms? a. Anticholinergic overdose b. TCA intoxication c. Ethanol withdrawal d. Serotonin syndrome e. Opiate withdrawal

*E* Hydrocodone is an opioid used for pain relief. Opioid withdrawal occurs in tolerant individuals when opioid exposure is discontinued or an antagonist is administered. The effects of withdrawal are secondary to increased sympathetic discharge, which is responsible for the clinical signs and symptoms. Although these can be significant, they are typically not life-threatening. Withdrawal is associated with CNS excitation, tachypnea, and mydriasis. Pulse and BP may be elevated. The patient may present with nausea, vomiting, diarrhea, abdominal cramps, myalgias, and insomnia. Examination often reveals piloerection, yawning, lacrimation, rhinorrhea, and diaphoresis. Neurologic manifestations include restlessness, agitation, and anxiety, but cognition and mental status are unaffected. Dysphoria and drug craving are usually prominent.

A 75-year-old woman is brought to the ED by EMS after she had a witnessed seizure on the street. A bystander reports that the patient fell to the ground, had tonic-clonic activity, and was drooling. Her BP is 162/85 mm Hg, HR is 95 beats per minute, temperature is 99.4°F, and RR is 16 breaths per minute. On examination the patient is unresponsive and has a bleeding superficial scalp laceration. Which of the following electrolyte disturbances is least likely to cause a seizure? a. Hypoglycemia b. Hyperglycemia c. Hyponatremia d. Hypernatremia e. Hypokalemia

*E* Hypokalemia presents with muscle weakness and characteristic ECG changes, such as flat T waves and U waves, and is not associated with seizures or altered mental status.

A 33-year-old G3P2002 presents to the ED complaining of vaginal bleeding that started earlier in the day. She has gone through two pads over the last 12 hours. She describes mild lower abdominal pain. Her BP is 105/75 mm Hg, HR is 78 beats per minute, temperature is 98.9°F, and RR is 14 breaths per minute. Pelvic examination reveals clotted blood in the vaginal vault with a closed cervical os. Transvaginal ultrasound documents an intrauterine pregnancy. Which of the following is the correct diagnosis? a. Embryonic gestation b. Complete abortion c. Incomplete abortion d. Inevitable abortion e. Threatened abortion

*E* In threatened abortion, the patient has vaginal bleeding with a closed internal os in the first 20 weeks of gestation. The risk of miscarriage is approximately 35% to 50%, depending on the patient population and severity of symptoms.

During a busy day working in the ED, five patients come in all of whom you believe require an LP to evaluate for meningitis. From the following list of patients below, which one can you forgo a CT scan and proceed directly to an LP? a. A 65-year-old man with fever and headache b. A 49-year-old woman with acquired immunodeficiency syndrome (AIDS) c. A 74-year-old man with new right lower extremity motor weakness d. A 19-year-old man with a fever who is lethargic and disoriented e. A 51-year-old woman who is febrile and complains of neck stiffness

*E* Indications for brain CT scanning prior to LP include: patients who are older than 60 years, patients who are immunocompromised, patients with known CNS lesions, patients who presented with a seizure or who had a seizure within 1 week of presentation, patients with abnormal level of consciousness, patients with focal findings on neurologic examination, and patients with papilledema seen on physical examination with clinical suspicion of elevated ICP. All of the patients in this scenario presented fit one of these contraindications except for the 51-year-old woman with fever and neck stiffness.

A 67-year-old man is brought to the ED in respiratory distress. His initial vitals include a HR of 112 beats per minute, a BP of 145/88 mm Hg, and an RR of 18 breaths per minute with an oxygen saturation of 92% on room air. He is also febrile at 102°F. After obtaining IV access, placing the patient on a monitor, and administering oxygen via nasal cannula, a chest radiograph is performed and shows patchy alveolar infiltrates with consolidation in the lower lobes. On review of systems, the patient tells you that he had five to six watery bowel movements a day for the last 2 days with a few bouts of emesis. Which of the following infectious etiologies is most likely responsible for the patient's presentation? a. Streptococcus pneumoniae b. Haemophilus influenzae c. Mycoplasma pneumoniae d. Chlamydophila pneumoniae e. Legionella pneumophila

*E* Legionella pneumophila is an intracellular organism that lives in aquatic environments. The organism may live in ordinary tap water and has probably been underdiagnosed in a number of community outbreaks. It is typically seen in the elderly and immunocompromised. Legionnaire disease is more common in the summer, especially in August. Patients often experience a prodrome of 1 to 2 days of mild headache and myalgias, followed by high fever, chills, and multiple rigors. Cough is present in 90% of cases. Other pulmonary manifestations include dyspnea, pleuritic chest pain, and hemoptysis. GI symptoms include nausea, vomiting, diarrhea, and anorexia. Neurologic symptoms include headache, altered mental status, and rarely, focal symptoms. Urine antigen testing is highly specific and sensitive and, if available, very rapid in making the diagnosis.

A 37-year-old woman presents to urgent care complaining of general weakness and blurry vision over the last month. She states that she feels great in the morning, but by dinner she has trouble cooking and complains of double vision. Her husband notices that her eyelids sometimes look droopy in the evening. On physical examination, her cranial nerves are intact, pupils are equal and reactive, extraocular muscles are intact; however, you notice slight ptosis. What is the most likely diagnosis of this patient? a. Botulism b. Lambert-Eaton myasthenic syndrome c. Ophthalmoplegia of the third cranial nerve d. Guillain-Barré syndrome e. Myasthenia gravis

*E* Myasthenia gravis is an autoimmune disease of the neuromuscular junction that is more common in women in their twenties and thirties and men in their seventies and eighties. Myasthenia gravis results from autoantibodies directed against the nicotinic acetylcholine receptor at the neuromuscular junction. This leads to destruction of acetylcholine receptors and competition with acetylcholine for the remaining receptors. Muscular weakness and fatigability are the hallmarks of myasthenia gravis. Ocular symptoms manifest early and include ptosis, diplopia, and muscle weakness. Symptoms usually worsen as the day progresses. Diagnosis is usually made by the edrophonium or Tensilon test.

A 71-year-old man presents to the ED with diffuse, crampy abdominal pain that began 1 hour after eating lunch today. The pain is intermittent over the last 8 hours with increasing severity. He also complains of nausea and chills, and vomited once on his way to the ED. He has not had a bowel movement or flatus since the pain began. His past medical history includes prostate cancer, left total hip replacement, appendectomy 25 years ago, right iliac artery aneurysm repair 5 years ago, incisional hernia repair 4 years ago, and irritable bowel syndrome. Which of the following is the most common cause of SBO in adults? a. Bezoar b. Neoplasm c. Incarcerated hernia d. Gallstone ileus e. Adhesion

*E* Overall, adhesions, hernias, and cancer account for more than 90% of cases of SBO. Postoperative adhesions are the most common cause of SBO

A 79-year-old man presents to the ED by paramedics with the chief complaint of agitation and confusion over the previous 12 hours. He has a past medical history of schizophrenia and is not taking any of his antipsychotics. His BP is 135/85 mm Hg, HR is 119 beats per minute, RR is 18 breaths per minute, oxygen saturation is 97% on room air, and fingerstick glucose is 135 mg/dL. Because of his agitation at triage, he was placed in wrist restraints. At this time, he is calm but confused. Examination reveals warm and clammy skin and 4-mm pupils that are equal and reactive. His cardiac examination reveals tachycardia and no murmurs. His lungs are clear to auscultation and his abdomen is soft and nontender. He is able to move all of his extremities. Which of the following is the most appropriate next step in management? a. Administer haloperidol or lorazepam b. Consult psychiatry c. Order a CT scan of his head d. Send a urine toxicologic screen e. Obtain a rectal temperature

*E* Patients frequently present to the ED with agitation. It is important to discern what is causing their agitation; the range of etiologies is expansive, from ethanol intoxication to intracerebral bleeding. The approach to the emergency patient always begins with the ABCs (airway, breathing, and circulation). In addition, the vital signs must be obtained early in a patient's assessment in order to reveal potentially life-threatening conditions. The patient in the vignette presents with agitation and tachycardia. Although it is tempting to attribute his agitation to his untreated schizophrenia, doing this without investigating medical causes of agitation can be disastrous. Finding out that the patient's temperature is 103.1°F, for instance, will lead you down a different clinical path than if his temperature is 98°F. This patient was ultimately diagnosed with meningitis.

A 27-year-old man bought to the ED by paramedics after a motor vehicle collision. His RR is 45 breaths per minute, oxygen saturation is 89%, HR is 112 beats per minute, and BP is 115/75 mm Hg. You auscultate his chest and hear decreased breath sounds on the left. Which of the following is the most appropriate next step in management? a. Order a STAT chest radiograph b. Perform a pericardiocentesis c. Perform a DPL d. Perform an ED thoracotomy e. Perform a tube thoracostomy

*E* Patients with a traumatic PTX typically present with shortness of breath, chest pain, and tachypnea. The physical examination may reveal decreased or absent breath sounds over the involved side, as well as subcutaneous emphysema. Any patient with respiratory symptoms in the setting of a PTX should be treated with a tube thoracostomy (chest tube).The preferred site for insertion is the fourth or fifth intercostal space at the anterior or midaxillary line. The tube should be positioned posteriorly and toward the apex so that it can effectively remove both air and fluid.

A 55-year-old man presents to the ED complaining of mild diffuse abdominal pain. He states that he underwent a routine colonoscopy yesterday and was told "everything is fine." The pain began upon waking up and is associated with some nausea. He denies fever, vomiting, diarrhea, and rectal bleeding. His BP is 143/71 mm Hg, HR is 87 beats per minute, temperature is 98.9°F, and RR is 16 breaths per minute. His abdomen is tense but only mildly tender. You order baseline laboratory tests. His chest radiograph is shows air below R diaphragm. Which of the following is the most likely diagnosis? a. Ascending cholangitis b. Acute pulmonary edema c. Acute liver failure d. Pancreatitis e. Pneumoperitoneum

*E* Potential complications of colonoscopy include hemorrhage, perforation, retroperitoneal abscess, pneumoscrotum, pneumothorax, volvulus, and infection. Perforation of the colon with pneumoperitoneum is usually evident immediately, but can take several hours to manifest. Perforation is usually secondary to intrinsic disease of the colon (eg, diverticulitis) or to vigorous manipulation during colonoscopy. Most patients require immediate laparotomy. However, expectant management is appropriate in some patients with a late presentation (1-2 days later), or without signs of peritonitis. The radiograph in the figure demonstrates air under the diaphragm, which is pathognomonic for pneumoperitoneum.

An 85-year-old man is rambling incoherently and not eating at his nursing home. Records indicate that he has a past medical history of hypertension, diabetes, dementia, and benign prostatic hypertrophy. On arrival to the ED, the patient is combative and oriented only to name. His temperature is 101°F rectally, BP is 85/50 mm Hg, HR is 125 beats per minute, RR is 22 breaths per minute, and blood sugar is 154 mg/dL. He appears uncomfortable and cachectic. His lungs are clear to auscultation, with scant crackles at the bases, and his abdomen is soft, nontender, and nondistended. He has a Foley catheter in place draining cloudy, white urine. He has no peripheral edema. Which of the following is the most likely diagnosis? a. Hypovolemic shock b. Neurogenic shock c. Cardiogenic shock d. Anaphylactic shock e. Septic shock

*E* Septic shock is a clinical syndrome of hypoperfusion, hypotension, and multiorgan dysfunction caused by infection. This patient is clearly in shock with hypotension, tachycardia, tachypnea, and acute mental status changes. He also has fever and pus in his urine, making a urine infection the likely source of infection. He requires immediate hydration, broad-spectrum antibiotics, possible intubation, pressors, and admission to the intensive care unit. Remember, elderly patients with comorbid conditions, such as diabetes are more prone to developing sepsis. In addition, patients with indwelling lines, such as Foley catheters, are at an even higher risk for infection.

A 42-year-old IV drug user presents to the ED with fever, chills, pleuritic chest pain, myalgias, and general malaise. The patient's vitals includ a HR of 110 beats per minute, BP of 110/65 mm Hg, RR of 18 breaths per minute, and temperature of 103°F. Physical examination is notable for retinal hemorrhages, petechiae on the conjunctivae and mucus membranes, a faint systolic ejection murmur, and splenomegaly. Which of the following is the most likely diagnosis? a. Sick sinus syndrome b. Myocarditis c. Pericarditis d. Cardiac tamponade e. Endocarditis

*E* Symptoms of endocarditis are nonspecific and vary widely, but the most common include fever (85%) and malaise (80%). In IV drug users, fever is present 98% of the time. Other symptoms include weakness, myalgias, dyspnea, chest pain, cough, headache, and anorexia. Neurologic signs and symptoms (eg, confusion, personality changes, decreased level of consciousness, and focal motor deficits) are seen in 30% to 40% of patients. Vasculitic lesions, including petechiae, splinter hemorrhages, tender fingertip nodules (Osler nodes), and nontender palmar plaques (Janeway lesions) are seen in 35% of patients. Splenomegaly, new heart murmur, and retinal hemorrhages may also be detected on physical examination. Risk factors for infective endocarditis include rheumatic or congenital heart disease, calcific degenerative valve disease, prosthetic heart valve, mitral valve prolapse, a history of IV drug use, or a history of endocarditis. Although any valve can be affected, IV drug use is the most common cause of right-sided endocarditis. The recurrence rate in these patients is 41%, significantly higher than the rate of less than 20% in non-IV drug use patients.

A 25-year-old stockbroker presents to the ED complaining of 6 weeks of daily headaches. Her headaches are band-like in distribution and are not associated with nausea, vomiting, visual phenomena, or neurologic symptoms. Normally they respond to acetaminophen, but they have increased in frequency in the past week as she stopped taking a medication that had been prescribed to prevent them. What type of primary headache syndrome is the patient likely experiencing? a. Migraine headaches b. Cluster type headaches c. Trigeminal neuralgia d. Postherpetic neuralgia e. Tension headache

*E* Tension headaches often occur daily, and classically cause bilateral occipital pain that is described as a band tightening around the head. In general, people experience a constant, dull pain that is nonthrobbing and without the ancillary features associated with migraines (visual phenomena, aura, neurologic complaints, nausea, vomiting). There is often secondary contraction of the neck and scalp musculature. First-line treatment includes NSAIDs and acetaminophen. If the headaches occur frequently enough to cause dysfunction in daily activities, patients may benefit from preventive therapy, such as amitriptyline, desipramine, or propranolol.

An 87-year-old man is brought to the ED on a long board and in a cervical collar after falling down a flight of steps. He denies losing consciousness. On arrival, his vital signs include a HR of 99 beats per minute, BP of 160/90 mm Hg, and RR of 16 breaths per minute. He is alert and speaking in full sentences. Breath sounds are equal bilaterally. Despite an obvious right arm fracture, his radial pulses are 2+ and symmetric. When examining his cervical spine, he denies tenderness to palpation and you do not feel any bony deformities. Which of the following is a true statement? a. Epidural hematomas are very common in the elderly age population. b. Cerebral atrophy in the elderly population provides protection against subdural hematomas. c. Increased elasticity of their lungs, allows elderly patients to recover from thoracic trauma more quickly than younger patients. d. The most common cervical spine fracture in this age group is a wedge fracture of the sixth cervical vertebra. e. Despite lack of cervical spine tenderness, imaging of his cervical spine is warranted.

*E* The Canadian C-spine rule for radiography in alert and stable patients following blunt head or neck trauma identified age greater than 65 years as a high risk factor for C-spine injury, even among those with stable vital signs and a GCS score of 15. Therefore, C-spine imaging in all such elderly patients is warranted.

A 70-year-old woman presents to the ED with dark stool for 3 weeks. She occasionally notes bright red blood mixed with the stool. Review of systems is positive for decreased appetite, constipation, and a 10-lb weight loss over 2 months. She denies abdominal pain, nausea, vomiting, and fever, but feels increased weakness and fatigue. She also describes a raspy cough with white sputum production over the previous 2 weeks. Examination reveals a pale female with a supine BP of 115/60 mm Hg, HR of 90 beats per minute. Standing BP is 100/50 mm Hg, with a pulse of 105 beats per minute. Which of the following is the most likely diagnosis? a. Hemorrhoids b. Diverticulitis c. Mallory-Weiss tear d. Diverticulosis e. Adenocarcinoma

*E* The combination of lower GI bleed with weight loss and decreased appetite points toward carcinoma, most likely adenocarcinoma of the colon. The lack of esophageal, abdominal, or rectal pain makes the other choices unlikely, as does the lack of associated symptoms (nausea, vomiting, or fever). Anemia or rectal bleeding in an elderly person should be assumed to be malignancy until proven otherwise.

A 22-year-old man presents to the ED after being ejected from his vehicle following a high-speed motor vehicle collision. Upon arrival, his BP is 85/55 mm Hg and HR is 141 beats per minute. Two large-bore IVs are placed in the antecubital veins and lactated ringers solution is being administered. After 3 L of crystalloid fluid, the patient's BP is 83/57 mm Hg. Which of the following statements is most appropriate regarding management of a hypotensive trauma patient who fails to respond to initial volume resuscitation? a. It is important to wait for fully cross-matched blood prior to transfusion b. Whole blood should be used rather than packed red blood cells (RBCs) c. Blood transfusion should begin after 4 L of crystalloid infusion d. Type O blood that is Rh-negative should be transfused e. Type O blood that is Rh-positive should be transfused

*E* The decision to begin blood transfusion in a trauma patient is based on the initial response to crystalloid volume resuscitation. Blood products should be administered if vital signs transiently improve or remain unstable despite resuscitation with 2 to 3 L of crystalloid fluid. However, if there is obvious major blood loss and the patient is unstable, blood transfusion should be started concomitantly with crystalloid administration. The main purpose in transfusing blood is to restore the oxygen carrying capacity of the intravascular volume. Fully cross-matched blood is preferable (eg, type B, Rh-negative, antibody negative); however, this process may take more than 1 hour, which is inappropriate for the unstable trauma. patient. Type-specific blood (eg, type A, Rh negative, unknown antibody) can be provided by most blood banks within 30 minutes. This blood is compatible with ABO and Rh blood types, but may be incompatible with other antibodies. If type-specific blood is unavailable, type O packed cells are indicated for patients who are unstable. Men should be administered type O, Rh-positive blood. To reduce sensitization and future complications, type O, Rh-negative blood is reserved for women of childbearing age.

A 32-year-old otherwise healthy man develops dizziness, nausea, and confusion after running a race. Emergency medical services (EMS) are available on-site and the patient is given intravenous (IV) fluids. He is oriented to person and time, but not place. He appears generally confused and it is difficult to obtain contact information from him. The patient is brought to the ED whereupon his symptoms generally resolve, except for diffuse muscle fatigue. Laboratory tests are drawn at this time, which are essentially normal, except for minimally elevated hepatic transaminases. Given this patient's symptoms and laboratory evaluation, what is the most likely diagnosis? a. Heat syncope b. Heat edema c. Rhabdomyolysis d. Heat stroke e. Heat exhaustion

*E* The diagnosis of heat exhaustion is initially made upon clinical presentation. Patients may have general malaise, fatigue, frontal headache, impaired judgment, diaphoresis, nausea, and show signs of dehydration with tachycardia and orthostatic hypotension. Heat exhaustion may progress to heat stroke and lies along a spectrum in which intermediate cases may often be difficult to delineate; however, the treatment is essentially the same. It is important to bring the patient to a cool location and start fluid replacement slowly as to prevent cerebral edema, especially in younger patients. To help distinguish between heat exhaustion and heat stroke, hepatic transaminases are helpful. Elevations to several thousand units may be seen in heat exhaustion or healthy runners after a marathon, whereas in heat stroke the levels are elevated to tens of thousands.

A 54-year-old man is brought to the ED by his wife for bizarre behavior. The wife complains that her husband has not been acting like his usual self over the last several days. She states that he has not had any change in sleep, appetite, or activity level. She also recalls that her husband complained of morning headaches for the last 2 months. The patient is otherwise in good health and does not take any medications. His BP is 135/87 mm Hg, HR is 76 beats per minute, temperature is 98.9°F, and RR is 14 breaths per minute. His examination is otherwise unremarkable. Which of the following is the most likely diagnosis? a. Migraine headache b. Tension headache c. Subarachnoid hemorrhage d. Pseudotumor cerebri e. Frontal lobe mass

*E* The most likely diagnosis in this patient is a space-occupying lesion in the frontal lobe of the brain. Brain tumors can present with morning headaches associated with nausea and vomiting. Neurologic examination is normal in most patients. Papilledema might provide an important clue of increased ICP and the presence of a brain mass. Frontal lobe tumors typically involve personality changes as seen in this patient.

A 42-year-old diabetic man presents to the ED with 3 days of rapidly worsening scrotal and perineal pain. His HR is 115 beats per minute, BP is 135/90 mm Hg, RR is 18 breaths per minute, and temperature is 102.9°F. Laboratory results are notable for a WBC of 18,000/μL. Physical examination demonstrates a necrotizing infection of the scrotum and perineal subcutaneous fascia. Which of the following is the most appropriate next step in management? a. 14-Day course of levofloxacin and early urology follow-up. b. Oral metronidazole and cephalexin with early urology follow-up. c. Pain medications, daily warm soaks, and follow-up with an urologist. d. Aggressive fluid resuscitation, bedside incision and drainage, and urology consult. e. Surgical debridement, broad-spectrum IV antibiotics, and hospital admission.

*E* The patient has Fournier gangrene, a polymicrobial necrotizing fasciitis of the perineal subcutaneous fascia and male genitalia. Treatment is aggressive fluid resuscitation, surgical debridement, broad-spectrum antibiotics, and possibly hyperbaric oxygen therapy. These patients require hospital admission. If not promptly treated, the overall mortality rate of these patients is approximately 20%.

A 23-year-old man presents to the ED complaining of finger pain. He states that while playing football, he went to catch a pass and the ball hit the tip of his finger and bent his finger backward. He thinks his finger is just "jammed." On examination, you notice that the distal phalanx is flexed and there is swelling and tenderness over the distal interphalangeal (DIP) joint, as seen below. In addition, he cannot extend his distal finger at the DIP joint. An x-ray does not reveal a fracture. Which of the following is the most appropriate way to manage this injury? a. Place a dorsal splint so that the proximal interphalangeal (PIP) and DIP joints are immobile, remove splint in 1 to 2 weeks. b. Place a dorsal splint so that the PIP and DIP joints are immobile, remove splint in 1 week. c. Buddy tape the finger. d. Place a dorsal splint to immobilize the DIP joint, remove splint in 1 to 2 weeks. e. Place a dorsal splint to immobilize the DIP joint, remove splint in 6 to 8 weeks.

*E* The patient has a mallet finger or a rupture of the extensor tendon that inserts into the base of the distal phalanx. This type of injury occurs often from a sudden forceful flexion of an extended finger when an object, such as a football, strikes the tip of the finger. This is the common mechanism among athletes. Clinically, patients present with pain and swelling over the DIP joint, which is held in flexion of up to 40 degrees because of loss of the extensor mechanism. The most important aspect in managing these injuries is to keep the DIP joint in continuous extension until healing occurs. Therefore, a splint should be applied so that only the DIP is immobilized in extension for 6 to 8 weeks. The PIP and MCP joints should be mobile. Any disruption of the immobile DIP joint can result in improper healing.

A 65-year-old man presents to the ED with a headache, drowsiness, and confusion. He has a history of long-standing hypertension. His BP is 230/120 mm Hg, pulse 87 beats per minute, RR 18 breaths per minutes, and oxygen saturation 97% on room air. On examination, you note papilledema. A head CT scan is performed and there is no evidence of ischemia or hemorrhage. Which of the following is the most appropriate method to lower his BP? a. Administer propofol for rapid reduction in BP. b. Administer mannitol for rapid reduction in BP and intracranial pressure (ICP). c. Administer a high-dose diuretic to reduce preload. d. Administer labetalol until his BP is 140/80 mm Hg. e. Administer labetalol until his BP is 180/100 mm Hg.

*E* The patient has hypertensive encephalopathy, which is defined by a rapid rise in BP that is accompanied by neurologic changes. Patients typically present with a systolic BP > 220 mm Hg and diastolic BP > 110 mm Hg. Neurologic findings include severe headache, vomiting, drowsiness, confusion, seizure, blindness, focal neurologic deficits, or coma. Hypertensive emergency is a medical emergency. The goal of therapy is to stop and reverse the progression of end-organ dysfunction while maintaining organ perfusion and avoiding complications. Reduction in BP should be done rapidly but carefully. It is important to avoid dropping the pressure too low as this may lead to cerebral ischemia. The immediate goal is to reduce the mean arterial BP by 20% to 30% of pretreatment levels over the first hour of therapy. This can be accomplished by labetalol, a β1-, β2-, and α1-receptor blocker. Another useful medication is nitroprusside, which is a better choice if the patient's BP is being monitored through an intra-arterial line. Nitroprusside can cause a reflex tachycardia.

A 43-year-old man presents to the ED after falling approximately 6 ft from the roof of his garage. The patient states that he landed on his feet but then fell to the ground. You assess his ABC (airway, breathing, and circulation), which are all normal. Vital signs are stable. Your secondary survey reveals a swollen and tender right heel. A radiograph reveals a fractured calcaneus. About 6 hours after the initial fall, the patient starts complaining of a constant burning in his right foot. You examine the foot and elicit pain with passive movement. There is decreased two-point discrimination. His dorsalis pedis pulse is 2+. Which of the following is the most appropriate next step in management? a. Place ice on the foot and administer analgesia. b. Order another radiograph to look for an occult fracture. c. Elevate the leg and place an elastic bandage around the foot. d. Order a duplex ultrasound for suspicion of a deep vein thrombosis (DVT). e. Measure the intracompartmental pressure of the foot.

*E* The patient is showing signs and symptoms of compartment syndrome. The syndrome occurs as a result of an increase in pressure within a confined osseofascial space that impedes neurovascular function. The end result is necrosis and damage to tissues. It can occur after crush injuries, circumferential burns, hemorrhage, edema, or any process that increases compartment pressure. Clinically, the patient complains of pain that is out of proportion to the injury. Physical examination may reveal swelling, sensory deficits, and pain with passive motion. The presence of a pulse does not rule out compartment syndrome. Late findings include pallor of the skin, diminished or absent pulses, and a cool extremity. The only way to diagnose compartment syndrome is to measure intracompartmental pressure with a Stryker device. A pressure greater than 30 mm Hg is considered diagnostic and requires a fasciotomy to avoid tissue damage.

A 29-year-old woman presents to the ED complaining of a generalized headache over the last 2 months. She has seen many doctors for it but has yet to get a correct diagnosis. She describes the headache as moderate in intensity and worse with eye movement. Occasionally, it awakes her from sleep and is worse when tying her shoes. She is scared because her vision gets blurry for a few minutes everyday. Her only medication is acetaminophen and an oral contraceptive. Her BP is 140/75 mm Hg, HR is 75 beats per minute, temperature is 98.9°F, and RR is 16 breaths per minute. On physical examination you appreciate papilledema. Which of the following is the most appropriate next step in management? a. Consult neurosurgery. b. Administer 2 g of ceftriaxone then perform an LP rule out meningitis. c. Order a magnetic resonance imaging (MRI) to look for a carotid artery dissection. d. Diagnose a migraine headache and prescribe her a triptan. e. Perform a CT scan and if negative perform an LP specifically to measure the opening pressure.

*E* The patient most likely has IIH (pseudotumor cerebri), a neurologic disease seen primarily in young obese women of childbearing age. Clinically, patients complain of a generalized headache of gradual onset and moderate severity. It may worsen with eye movements or with the Valsalva maneuver. Visual complaints are common and may occur several times a day and can become permanent in 10% of patients. Patients typically have papilledema and visual field defects on physical examination. Diagnosis is made by a normal neuroimaging scan (eg, head CT scan) and an elevated intracerebral pressure (> 200 mm H2O) measured by the opening pressure from an LP. This is also therapeutic.

A 36-year-old woman presents to the ED with sudden onset of leftsided chest pain and mild shortness of breath that began the night before. She was able to fall asleep without difficulty but woke up in the morning with persistent pain that is worsened upon taking a deep breath. She walked up the stairs at home and became very short of breath, which made her come to the ED. Two weeks ago, she took a 7-hour flight from Europe and since then has left-sided calf pain and swelling. What is the most common ECG finding for this patient's presentation? a. S1Q3T3 pattern b. Atrial fibrillation c. Right-axis deviation d. Right-atrial enlargement e. Tachycardia or nonspecific ST-T-wave changes

*E* The patient most likely has a pulmonary thromboembolism (PE) that embolized from a thrombus in her left calf. The diagnosis of PE is usually made with a CT angiogram, echocardiogram, or a ventilation-perfusion scan. The most common ECG abnormalities in the setting of PE are tachycardia and nonspecific ST-T- wave abnormalities.

A 68-year-old man presents to the ED 4 hours after an upper endoscopy was performed for 5 months of progressive dysphagia. During the procedure, a 1-cm ulcerated lesion was found and biopsied. Now, the patient complains of severe neck and chest pain. His vitals are as follows: BP 135/80 mm Hg, HR 123 beats per minute, RR 26 breaths per minute, and temperature 101°F. On physical examination, he appears diaphoretic and in moderate distress with crepitus in the neck and a crunching sound over the heart. You obtain an electrocardiogram (ECG), which is notable for sinus tachycardia. After obtaining a surgical consult, which of the following is the next best step in management? a. Perform an immediate bronchoscopy. b. Give aspirin 325 mg and obtain a cardiology consult for possible cardiac catheterization. c. Repeat the endoscopy to evaluate the biopsy site. d. Perform an immediate thoracotomy. e. Order an immediate esophagram with water-soluble agent.

*E* The patient most likely has an esophageal perforation, a serious, life-threatening complication of endoscopy that must be identified and treated quickly. Although sometimes reported as a result of forceful vomiting (eg, Boerhaave syndrome), the most common cause is iatrogenic. These usually occur as a complication of GI procedures, including upper endoscopy, dilation, sclerotherapy, and even NG tube placement or endotracheal intubation. The signs and symptoms may include chest pain near the rupture site, fever, respiratory distress, hoarseness, or dysphagia. Most patients have mediastinal or cervical emphysema, which may be noted by palpation or by a crunching sound heard during auscultation (ie, Hamman sign). An immediate esophagram with a water-soluble agent (eg, Gastrografin) is indicated

A 33-year-old carpenter was working on a construction project to build a new house. While he was using a high-pressure paint gun, he inadvertently injected his left index finger. On arrival to the ED, he complains of intense hand pain. On examination, you note a 2-mm wound over the second proximal phalange. He has full range of motion and brisk capillary refill. Radiographs of the finger show soft tissue swelling, a small amount of subcutaneous air, but no fracture. His tetanus is up to date. Which of the following is the most appropriate disposition for this patient? a. Place the hand in a radial gutter splint and have the patient follow up with an orthopedic surgeon in 1 week. b. Discharge home with pain medication and have the patient return for repeat radiographs in 1 week. c. Order a CT scan of the finger to confirm that there is no occult fracture before discharging the patient home. d. Place the hand in a radial gutter splint, prescribe a 10-day course of antibiotics, and have the patient follow up with an orthopedic surgeon in 1 week. e. Place the hand in a radial gutter splint, administer broad-spectrum antibiotics, and admit the orthopedic service for operative debridement.

*E* The patient sustained a high-pressure injection injury of his finger—this is a surgical emergency. These injuries may involve extensive tissue loss and are associated with a high infection rate. Most of these injuries involve grease, paint, or other industrial toxins. Paint generates a large, early inflammatory response resulting in a high percentage of amputations. Within several hours after the digit has been injected, the extremity becomes painful and swollen. Initially, there may be anesthesia and even vascular insufficiency of the extremity. In the late stages, marked breakdown of the skin occurs resulting in ulcers and draining sinuses. If the material injected into the extremity is radiopaque, it is possible to determine its degree of spread. Management involves splinting and elevating the extremity, administration of antibiotics, tetanus prophylaxis updated as indicated, analgesia, and immediate orthopedic consultation.

A 45-year-old man is on his way to work and loses his footing while walking up a flight of steps. He feels excruciating pain at the back of his ankle and cannot ambulate. He arrives to the ED and states that he felt a snap in his leg during the injury. He has a past medical history of hypertension and hypercholesterolemia. He spends most of his free time in front of the television watching horror movies. On examination, you note swelling of the distal calf. Which of the following is likely to be positive in this individual? a. Homan sign b. Lachman test c. McMurray test d. Ballotable patella e. Thompson test

*E* The patient's diagnosis is an Achilles tendon rupture. The individual gives a history of a sudden excruciating pain and having heard or felt a pop or snap. This entity is most common in sedentary, middle-aged men or in athletes. The diagnosis can be made with the Thompson test. The patient is placed in the prone position. With normal function, squeezing the calf produces plantar flexion of the foot. With a complete tear of the Achilles tendon, plantar flexion will not occur. If doubt remains, magnetic resonance imaging (MRI) or ultrasound can be used to confirm the diagnosis.

5-year-old boy who fell from the monkey bars and landed on his left elbow is brought into the ED for evaluation. The patient has no significant medical history. On physical examination, you note the patient is holding his left arm in an adducted position. There is obvious swelling around the elbow with decreased range of motion secondary to pain. He complains of hand numbness, but the motor and vascular examination is normal. The radiograph shown below shows posterior displacement of the capitellum with evidence of a dark shadow posterior to the distal humerus. Which of the following is the most serious complication associated with this injury? a. Transection of the brachial artery b. Malunion of distal humerus c. Motor deficit from injury to the ulnar nerve d. Chronic arthritis of the elbow e. Chronic deformity of the hands and fingers caused by contractures

*E* The patient's presentation and radiographic findings are consistent with a supracondylar fracture. Supracondylar fractures are the most common type of elbow fracture in children and important to recognize early because of the risk of injury to the arteries and nerves that pass through this area. Patients typically present with pain, swelling, and decreased range of motion with the arm held in adduction. Most fractures occur after a fall on an outstretched hand with the distal humerus displacing posteriorly. Posterior fractures are classified in three categories. Type I fractures show only an increased anterior fat pad sign and evidence of a posterior fat pad which is always pathologic, although not specific for this condition. Type II fractures have an obvious non displaced fracture. Type III show posterior displacement of the capitellum and have no cortical contact between the fracture fragments. Types II and III require reduction and fixation. The most serious complication of supracondylar fractures is Volkmann ischemic contracture. This occurs when high-pressure builds up in the forearm compartments leading to a compartment syndrome. It can also be caused by kinking of the brachial artery with subsequent ischemia if not repaired. If the condition is not addressed, there is potential for permanent damage to nerves and muscles of the forearm leading to contractures. Patients who develop pain upon passive extension of the fingers, forearm tenderness, or refuse to open the hand have a very high risk of developing this condition.

You are notified that emergency medical service (EMS) is bringing in a patient who collapsed 5 minutes ago in his house and was intubated at the scene by paramedics. On arrival to the ED, you confirm ET placement and continue cardiopulmonary resuscitation (CPR). You connect the patient to the cardiac monitor and see the rhythm below. Which of the following is the most appropriate next step in management? a.Perform synchronized cardioversion at 100 J. b. Immediately defibrillate at 200 J. c. Confirm the rhythm in two leads, begin CPR, then defibrillate at 200 J. d. Confirm the rhythm in two leads, begin CPR, then administer amiodarone. e. Confirm the rhythm in two leads, begin CPR, then administer epinephrine and atropine.

*E* The rhythm is asystole. This rhythm represents complete cessation of myocardial electrical activity. Although asystole may occur early in cardiac arrest because of progressive bradycardia, it generally represents the end-stage rhythm after prolonged cardiac arrest secondary to VF or PEA. Because the potential exists for an organized rhythm or VF to appear as asystole in a single lead, asystole should always be confirmed in at least two limb leads. It may be difficult to distinguish between extremely fine VF and asystole. Treatment includes CPR, intubation, IV access, and the administration of epinephrine or vasopressin and atropine.

A 32-year-old G2P1001 presents to the ED with severe abdominal pain and vaginal bleeding. She reports that she is currently in her third trimester and that she has been diagnosed and treated for preeclampsia. Her vitals are significant for a BP of 195/100 mm Hg. Upon physical examination, her abdomen is distended and hard to the touch. Given this patient's history and physical examination, what emergent intervention is most appropriate? a. Ultrasound b. Pelvic examination c. Tocolytics d. Blood transfusion e. Maternal/fetal monitoring with possible delivery

*E* There is a high probability that this patient is suffering from a uterine rupture or abruptio placentae. Monitoring should be instilled immediately with preparation for delivery. Emergent obstetrical consultation is warranted for repair and possible Cesarean-section. Risk factors include trauma, hypertension, recreational drug use, smoking, multiparity, and advanced maternal age

A 75-year old man complaining of chest pain is brought in to the ED by paramedics. He is barely able to speak to you because he is short of breath. The nurse immediately attaches him to the monitor, starts an IV, and gives him oxygen. His temperature is 98.9°F, BP is 70/40 mm Hg, HR is 140 beats per minute, RR is 28 breaths per minute, and oxygen saturation is 95% on room air. On examination, he is in mild distress. His heart is irregular and tachycardic. His lungs are clear to auscultation, with rales at the bases, bilaterally. An electrocardiogram (ECG) is shown below. What is your first-line treatment for this patient? a. Heparin drip b. Diltiazem 10 mg IV push c. Metoprolol 5 mg IV push d. Digoxin 0.5 mg IV e. Synchronized cardioversion at 100 J

*E* This ECG shows atrial fibrillation with rapid ventricular response (RVR). Normally, one area of the atria depolarizes and causes uniform contraction of the atria. In atrial fibrillation, multiple areas of the atria continuously depolarize and contract, leading to multiple atrial impulses and an irregular ventricular response. Atrial fibrillation reduces the effectiveness of atrial contractions and may lead to or worsen heart failure in patients with left ventricular failure. Treatment of atrial fibrillation is dependent on whether or not the patient is stable or not. This patient is clinically unstable; the atrial fibrillation with RVR has pushed him into heart failure and he is hypotensive and tachypneic. Unstable patients like this should undergo synchronized cardioversion. Synchronized cardioversion is performed at 100 J and then at 200 J if the first attempt fails.

An 82-year-old man presents to the ED feeling weak and dizzy. He has a past medical history of hypertension and diabetes and both are well controlled on hydrochlorothiazide, benazepril, atenolol, and metformin. On review of systems, he denies chest pain, gastrointestinal (GI) bleeding, and syncope, but states that he feels short of breath. His temperature is 98.6°F orally, BP is 86/60 mm Hg, HR is 44 beats per minute, RR is 18 breaths per minute, oxygen saturation is 98% on room air, and glucose is 116 mg/dL. He is immediately connected to the cardiac monitor. Which of the following choices best describes the ECG that shows p waves marching through? a. Normal sinus rhythm b. First-degree AV block c. Second-degree Mobitz I (Wenckebach) AV block d. Second-degree Mobitz II AV block e. Third-degree AV block

*E* This ECG shows third degree, or complete AV block. Note that there is no relationship between the P waves and QRS complexes. The P waves occur regularly, but since there is no AV conduction, the ventricles do not respond to the P waves. An escape pacemaker at a rate slower than the atrial rate drives the ventricles, producing regular QRS complexes independent of the P waves

A 2-year-old girl is brought into the ED with 2 days of fever and runny nose. Today she developed a dry, harsh, "barking" cough. Her temperature is 103.3°F, HR 123 beats per minute, RR 25 breaths per minute, and pulse oximetry of 98% on room air. On evaluation you note an alert, responsive but somewhat anxious female, in moderate respiratory distress with stridor and suprasternal retractions at rest. She has nasal congestion, equal air entry bilaterally with no rales or wheezes and a normal ear and throat examination. After receiving a single treatment of racemic epinephrine, she feels better and her work of breathing improves. Which of the following is the most appropriate next step in management? a. Chest radiograph b. CBC and blood culture c. Soft tissue radiograph of the neck d. Administer broad-spectrum antibiotics e. Administer a dose of steroids

*E* This is a case of laryngotracheitis or croup. It is the most common cause of stridor and upper respiratory obstruction in children 6 months to 3 years old. It usually begins with constitutional symptoms and subsequently patients develop the characteristic "seal-like" cough. The etiology is most commonly the parainfluenza virus. Diagnosis is usually clinical. Treatment includes racemic epinephrine, particularly when stridor is present. Following administration of racemic epinephrine, patients should be observed for 3 to 4 hours to monitor for a rebound effect after the epinephrine wears off. Steroids are administered as well to reduce the swelling of the larynx. Dexamethasone is the commonly administered steroid. Some home-based treatments, such as taking a child outdoors to breath cool air (as croup presents most typically in the winter) or into the bathroom to breath the steam produced when running hot water in the shower both seem to aid in symptomatic relief.

A 26-year-old man with a long history of epilepsy is brought to the ED for a recent seizure. While in the ED, he is rhythmically moving his right leg and is unresponsive. Which of the following best describes this seizure pattern? a. Petit mal seizure b. Generalized tonic-clonic seizure c. Partial seizure with secondary generalization d. Simple partial seizure e. Complex partial seizure

*E* This is a complex partial seizure, also known as temporal seizure; although it does not necessarily originate in the temporal lobe. It is characterized by focal electrical discharges (partial seizure), such as clonic leg activity and alteration of consciousness. --petit mal = abscence seizure

A 21-month-old girl, previously well, presents with 7 days of fever and rash. She has been seen previously during this episode by her primary physician and diagnosed with a "viral illness." On further questioning, the mother indicates that the patient has had red eyes, but no discharge. She has had no vomiting, diarrhea, cough, congestion, or complaints of pain. She has, however, seemed very irritable and fussy throughout the last few days and cannot seem to get comfortable. On examination, the patient is highly irritable and intermittently consolable. Vital signs reveal Temperature 101°F rectally, HR 170 beats per minute, RR 22 breaths per minute, BP 100/60 mm Hg. Her conjunctivae are mildly injected with no purulent discharge. The oropharynx is clear though she has dry, cracked lips. There are two anterior cervical nodes measuring 2.5 cm each. The heart is tachycardic without a murmur. The lungs are clear and abdomen is soft and nontender with no hepatosplenomegaly. The skin reveals a diffuse, blanching, erythematous, macular rash. The extremities have no swelling or tenderness. Laboratory evaluation reveals WBC 13,500/μL, Hgb 9.5 mg/dL, platelets 870/μL, CRP 89, ESR 85, electrolytes are normal, liver function tests reveal aspartate aminotransferase (AST) 110 U/L, alanine aminotransferase ALT 88 U/L. Which of the following is the most appropriate next step in the management of this patient? a. Consult cardiology for statim (STAT) echocardiogram b. Additional laboratory tests including Epstein-Barr viral titers, strep test, antinuclear antibody, bone marrow biopsy c. Reassurance to the parents that the initial diagnosis was probably accurate d. Administer IV antibiotics and perform lumbar puncture e. Admit for administration of intravenous immunoglobulin (IVIG) and aspirin

*E* This is a patient with Kawasaki disease (KD). This entity is defined by the following criteria: fever lasting for a minimum of 5 days, plus four out of five of the following: cervical lymphadenopathy of greater than 1.5 cm; dry, cracked lips or other oral mucous membrane involvement (strawberry tongue); truncal, nonvesicular rash; nonpurulent conjunctivitis; swollen or edematous hands and feet. KD is a systemic inflammatory vasculitis of unknown etiology. The characteristic constellation of symptoms, as this patient meets, requires immediate action to prevent complications of the disease. Another common finding, and one which many practitioners use to help in the diagnosis of "atypical" cases, is intense irritability of the child. The most serious sequelae involve coronary artery aneurysms. These occur in approximately 20% to 25% of untreated patients. This number is reduced to around 4% to 5% if treatment occurs within the first 10 days of symptom onset. Additional treatment with aspirin is recommended both for its antiplatelet and antiinflammatory effects

A 30-year-old chef presents to the ED with a complete fingertip amputation. She says her knife slipped while she was chopping vegetables. She brought the amputated pulp in a plastic bag. On examination, the patient has a clean, 5-mm diameter dermal slice on the distal volar tip of the second digit. There is profuse nonpulsatile bleeding. There is no trauma to the DIP joint, proximal interphalangeal (PIP) joint or the nail. Her most recent tetanus booster was 1 year ago. Which of the following is the most appropriate next step in management? a. Consult hand surgery for replantation of the pulp in the operating room. b. Irrigate the hand and replant distal tip in the ED as soon as possible. c. Perform a digital block with lidocaine and epinephrine to control bleeding. d. Immediately place the amputated tip on ice to prevent cell death. e. Discharge the patient with a pressure dressing and splint after the wound is irrigated and the bleeding is controlled.

*E* This patient has a < 10 mm dermal slice that is generally managed with bleeding control, nonadherent dressing, and a finger splint to prevent reinjury and decrease pain. Dressing changes have to be performed every 24 to 48 hours. Fingertip amputation > 10 mm, exposure of the bone, or involvement of the nail may require a hand surgeon consultation. However, this patient's dermal slice is 5 mm and does not involve the phalanx or nail, therefore, it can be managed conservatively. Pressure dressing is typically sufficient to control bleeding in dermal tip amputations. If hemostasis is not achieved with direct pressure, commercial hemostatic agents can be used.

A 31-year-old nurse in your hospital has noticed a lesion in her left eye. She denies change in vision, pain, fevers, or discharge. A picture of her eye is shows a triangular growth of tissue from the bulbar conjunctiva to the periphery of the cornea. Which of the following is the most likely diagnosis? a. Hordeolum b. Chalazion c. Dacryocystitis d. Pinguecula e. Pterygium

*E* This patient has a pterygium, a triangular growth of tissue from the bulbar conjunctiva to the periphery of the cornea. It is more common on the nasal side of the cornea and may affect one or both eyes. Pterygium is associated with exposure to wind, dust, and sunlight. Most cases are asymptomatic and can be followed by an ophthalmologist. In cases where the pterygium grows into the visual axis or restricts extraocular motion, surgical excision is indicated

A car pulls up to your ED and drops off a 19-year-old man who was shot in the chest. The man tells you his name and complains of right-sided chest pain and difficulty breathing. On primary survey, his airway is patent and his oropharynx has no blood or displaced teeth. He is breathing at 32 beats per minute with retractions and an oxygen saturation of 88% on 15 L of oxygen. There is a bullet wound to his right mid-chest with another wound in his back. His trachea is deviated to the left. On auscultation, he has diminished breath sounds on the right side. Which of the following is the most appropriate next step in management? a. Stat portable chest x-ray b. Intubation c. Perform ED thoracotomy d. Call the surgical service e. Needle decompression

*E* This patient has a tension pneumothorax. Secondary to the gunshot, air has entered the pleural space, secondary to the gunshot, and caused the right lung to collapse. This air cannot escape and pressure continues to increase, pushing the right lung into the mediastinum, causing the trachea to shift to the left. If this process is not corrected, venous return and cardiac output can be compromised and the patient will die. Classic symptoms of tension pneumothorax include dyspnea, tachypnea, tracheal deviation to the uninjured side, absent breath sounds on the injured side, and hypotension. Treatment of a tension pneumothorax is immediate needle decompression using a large 14- or 16-gauge IV catheter inserted into the pleural space (ie, second intercostals space in the midclavicular line). Air should come out of the catheter and the patient's clinical condition should improve. A tube thoracostomy should be performed after the needle decompression.

A 48-year-old man with a medical history of cirrhosis caused by hepatitis C has been vomiting bright red blood for 1 day. On arrival to the ED, the patient is confused and unable to provide more information. His family states that he has been vomiting large amounts of bright red blood every 4 hours and has no prior history of GI bleeding. The nurses hook him up to the monitor and start two large-bore IV lines. His BP is 75/43 mm Hg, HR is 130 beats per minute, RR is 24 breaths per minute, and oxygen saturation is 98% on room air. His abdomen is soft with no masses. His rectal examination reveals bright red blood. Which type of fluid is most critical in his resuscitation? a. 7% Sodium chloride b. 0.9% Sodium chloride c. Type and cross-matched blood d. Type-specific blood e. Type O, Rh-positive blood

*E* This patient has significant hypotension and tachycardia with mental status changes and needs type O, Rh-positive blood now. His hypovolemic shock is most likely caused by a brisk upper GI bleeding, secondary to variceal bleeding. While his blood is being processed in the laboratory, including type and cross, complete blood count, chemistry panel, liver function tests, and coagulation profile, a nasogastric tube should be placed and octreotide administered. A gastroenterologist and surgeon should also be consulted. Type O, Rh-negative blood is reserved for women in their childbearing years to prevent potential Rh sensitization.

An 18-year-old college student with no past medical history presents to the ED with a diffuse rash. She also describes having a headache, fever, and arthralgias for 3 days. On examination, her temperature is 101.2°F, BP is 120/63 mm Hg, HR is 110 beats per minute, RR is 24 breaths per minute, and oxygen saturation is 98% on room air. The patient is alert and oriented to person, place, and time. She has nuchal rigidity and photophobia. Her gums are oozing blood. Her abdomen is soft and nontender and her skin has a diffuse, petechial rash. You are concerned about meningococcemia and immediately start ceftriaxone and vancomycin. Her laboratory results reveal a white blood cell (WBC) count of 13,400/μL, hematocrit 36%, platelets 80/μL, PTT 60 seconds, international normalized ratio (INR) 1.9, and fibrinogen 250 g/L. Which of the following is the most appropriate next step in management? a. IV heparin b. Transfuse cryoprecipitate c. Transfuse packed red blood cells (RBC) d. Transfuse platelets e. Transfuse fresh frozen plasma (FFP)

*E* This patient has signs of disseminated intravascular coagulation (DIC) secondary to meningococcemia. She should be treated with FFP for elevated coagulation times (INR and PTT) with signs of active bleeding (oozing gums). FFP will provide lost clotting factors and help to control the bleeding.

A 21-year-old man presents to the ED with a red eye. The patient complains of rhinorrhea and a nonproductive cough, but has no eye pain or discharge. He also has no associated ecchymosis, bony tenderness of the orbit, or pain on extraocular eye movement. His vision is normal, extraocular movements are intact, and intraocular pressure (IOP) is 12. A picture of his eye is shows ______. What is the most appropriate management of this condition? a. Call ophthalmology immediately. b. Administer 1% atropine. c. Elevate patient's head. d. Administer ophthalmic timolol. e. Reassurance only.

*E* This patient has subconjunctival hemorrhage caused by conjunctival vessel rupture from coughing. This common ED complaint can result spontaneously or from Valsalva induced pressure spikes (such as coughing or bearing down), trauma, and hypertension.

You are the physician staffing a clinic in the Rocky Mountains when 29-year-old male mountain-climber presents with nausea, vomiting, and dizziness. Upon review of systems, he denies fever, cough, abdominal pain, or dysuria but does report anorexia and some ankle swelling. What diuretic is the drug of choice in this patient? a. Furosemide b. Hydrochlorothiazide c. Bumetanide d. Fosinopril e. Acetazolamide

*E* This patient is an otherwise healthy male with no reason to have congestive heart failure (CHF), renal failure, hepatic failure, or cardiomyopathy. Instead, he is an avid mountainclimber, which should hint at acute mountain sickness (AMS) in this clinical scenario. Early on, symptoms may mimic an acute viral syndrome with nausea, vomiting, headache, and anorexia. However, these symptoms progress to include peripheral edema, oliguria, retinal hemorrhages, and finally high-altitude pulmonary or cerebral edema. The initial treatment for all of these conditions across the spectrum of AMS, include gradual descent. A descent of 1500 to 3000 ft reverses high-altitude sickness in most cases. Supplemental oxygen is indicated in all cases. Diuretics, such as acetazolamide have been proven effective not only for treatment but for prophylaxis. Symptomatic treatment for vomiting and headache may also be indicated. Hyperbaric oxygen therapy is indicated in severe cases. Risk factors for AMS include, but are not limited to, rapid ascent, chronic obstructive pulmonary disease (COPD), sickle-cell disease, cold exposure, heavy exertion, and sleeping at higher altitudes.

A 4-year-old boy is brought to the ED by his parents who state that he is having difficulty breathing. The patient has a 1-week history of fever, congestion, and cough. Over the last 2 days the patient was tired-appearing with intermittent vomiting and persistently increased RR despite administration of acetaminophen. On presentation, his vital signs are temperature 100.5°F, HR 185 beats per minute, RR 50 breaths per minute, BP 75/40 mm Hg, and pulse oxymetry of 88% on room air. He is ill-appearing and listless. He has diffuse rales noted on auscultation, pulses are weak and thready, and his liver is palpable 3 to 4 cm below the right costal margin. After several attempts at a peripheral IV, the patient becomes increasingly somnolent. Which of the following is the most appropriate method of obtaining access in this patient? a. Internal jugular central line b. Femoral vein central line c. Saphenous vein cutdown d. Large-bore IV in antecubital fossa e. Intraosseous needle

*E* This patient is in shock. Attempts at peripheral IV access are acceptable, but should be limited to two attempts within 60 seconds and then intraosseous (IO) insertion should be attempted. IO lines can be performed quickly and reliably. The preferred sites for insertion are the proximal tibia, followed by the distal tibia and proximal femur. In adults, there is a sternal IO system that has gained popularity for its ease of placement and use of flexible tubing once in place. All medications can be administered through the IO line and onset of action is similar to venous administration.

A 58-year-old man presents to the ED with progressive dyspnea over the course of 1 week. Upon arrival, he is able to speak in full sentences and states that he stopped taking all of his medications recently. Initial vitals include a HR of 92 beats per minute, a BP of 180/100 mm Hg, and an RR of 16 breaths per minute with an oxygen saturation of 94% on room air. Upon physical examination, the patient has bibasilar crackles, jugular venous distention, and pedal edema. Which of the following medication regimens was the patient most likely on? a. Loop diuretic only b. Aspirin only c. Loop diuretic and β-blocker d. Calcium channel blocker e. Loop diuretic, β-blocker, and angiotensin-converting enzyme (ACE) inhibitor

*E* This patient is showing signs and symptoms of CHF, which is classified as either right- or left-sided. Right-sided heart failure manifests as jugular venous distention, ascites, and peripheral edema. Left-sided heart failure manifests as pulmonary edema or shock. This patient has both as left-sided failure often leads to right-sided failure. Outpatient management for CHF includes a β-blocker to decrease cardiac stress and improve contractility, a loop diuretic to aid in diuresis of excess fluid; and an ACE inhibitor for both BP management and renal protective effects.

A 35-year-old man presents to the ED with right hand swelling, pain, and erythema that began 3 days ago. He denies any trauma, sick contacts, insect bites, or recent travel. The patient's vitals are significant for an oral temperature of 101°F. Upon physical examination, you note an area of erythema surrounding multiple punctate lacerations over the right third and fourth metacarpophalangeal (MCP) joints with localized tenderness. The patient is neurovascularly intact with limited flexion caused by the swelling and pain. Given the above presentation, what is the most appropriate disposition for this patient? a. Suture and close follow-up with a hand surgeon b. Suture and prescription for oral antibiotics c. Wound irrigation and prescription for oral antibiotics d. Wound irrigation and tetanus prophylaxis e. Admission for intravenous (IV) antibiotics

*E* This patient most likely sustained a closed-fist injury, which have high infection rates and evidence of poor wound healing. Wounds sustained by punches to the jaw and human bites, also known as "fight bites," are classically over the metacarpal joints. Penetration deep into the joint space and infection is common given the positioning of the hand during the injury, human oral flora, and delay in seeking treatment. Infected wounds are polymicrobial and specifically include Eikenella corrodens, a facultative anaerobic gram-negative rod harbored in human dental plaque. It acts synergistically with aerobic organisms to increase the morbidity of these injuries. The joint spaces must be examined under full range of motion to detect any tendon lacerations or presence of foreign bodies. Hand radiographs should also be obtained to examine for any bony involvement. IV antibiotics and admission is the appropriate disposition. The antibiotics of choice are penicillin and second-generation cephalosporins with broader coverage in the immunocompromised. The wounds should be left open with a sterile dressing, splinted in the position of function (hand-holding-glass position) and elevated. Human bites have resulted in the transmission of hepatitis B, hepatitis C, syphilis, and herpes. Although HIV is present in human saliva, it is in relatively small amounts and considered a low risk for transmission. Appropriate antivirals and testing should be considered in these patients.

A 36-year-old immigrant woman is brought to the ED from her workplace. She was found to be agitated and behaving bizarrely. The patient's past medical history and medications are unknown. Her BP is 162/92 mm Hg, HR is 140 beats per minute, temperature is 101.8°F, and RR is 18 breaths per minute. On examination, the patient is delirious, tremulous, and has a large goiter. Which of the following is the most appropriate management of this patient? a. Administer dantrolene. b. Administer acetaminophen and broad coverage antibiotics. c. Protect airway, administer iodine. d. Administer diazepam. e. Protect airway, administer acetaminophen, propranolol, and PTU.

*E* This patient presents in a hyper-adrenergic state, altered mental status, and a large goiter, placing the diagnosis of thyroid storm on top of the differential. The patient is likely to have undiagnosed hyperthyroidism. This is a clinical diagnosis and has to be treated empirically and rapidly since mortality is high despite treatment. The management of thyroid storm involves supportive care (airway protection, oxygenation, IV hydration) and specific therapy to treat adrenergic symptoms and to decrease synthesis and release of thyroid hormone. β-Adrenergic blockers are given to reverse adrenergic hyperactivity. PTU blocks de novo synthesis of thyroid hormone. Iodine blocks release of the preformed hormone but can be given only after PTU has taken effect otherwise it will promote further hormone production

A 21-year-old college student is brought to the ED by her roommate who states that the patient has been very sleepy today. She has a history of diabetes and has not refilled her medication in over a week. Her BP is 95/61 mm Hg, HR is 132 beats per minute, temperature is 99.7°F, and RR is 20 breaths per minute. Her fingerstick glucose is 530 mg/dL. Which of the following choices most closely matches what you would expect to find on her arterial blood gas with electrolytes and urinalysis? a. pH 7.38, anion gap 5, normal urinalysis b. pH 7.57, anion gap 21, presence of glucose and leukocytes in urine c. pH 7.47, anion gap 12, presence of glucose and ketones in urine d. pH 7.26, anion gap 12, presence of glucose and ketones in urine e. pH 7.26, anion gap 21, presence of glucose and ketones in urine

*E* This patient presents with an anion gap metabolic acidosis, glucosuria, and ketonuria, which is consistent with diabetic ketoacidosis (DKA). DKA is an acute, life-threatening disorder occurring in patients with insulin insufficiency. It results in hyperglycemia, ketosis, and osmotic diuresis and clinically presents with gastrointestinal (GI) distress, polyuria, fatigue, dehydration, mental confusion, lethargy, or coma. When the diagnosis of DKA is clinically suspected and hyperglycemia is confirmed by elevated fingerstick glucose, the results of a blood gas and urinalysis confirm the diagnosis. In DKA, the liver metabolizes free fatty acids into ketone bodies for alternative fuel in the setting of cellular glucose underutilization. The result is ketonuria and anion gap metabolic acidosis (pH < 7.4 and HCO3 < 24). Glucosuria, the result of hyperglycemiarelated osmotic diuresis, is another manifestation of DKA. The anion gap is calculated by subtracting Cl− and HCO3 − from Na. A normal anion gap is 8 to 12 mEq/L. An elevated gap is a result of an increased concentration of unmeasured anions. In DKA, the elevated anion gap is caused by the production of ketones.

A 46-year-old woman is brought to the ED by her husband for 1 day of worsening confusion. The patient has a history of systemic lupus erythematosus (SLE) and takes chronic oral steroids. She has not been feeling well for the last few days. Her BP is 167/92 mm Hg, HR is 95 beats per minute, temperature is 100.3°F, and RR is 16 breaths per minute. On examination the patient is oriented to name and has diffuse petechiae on her torso and extremities. Laboratory results reveal hematocrit 23%, platelets 17,000/μL, BUN 38 mg/dL, creatinine 1.9 mg/dL. Which of the following is the most likely diagnosis? a. Henoch-Schönlein purpura b. Disseminated intravascular coagulopathy c. Von Willebrand disease d. Idiopathic thrombocytopenic purpura e. Thrombotic thrombocytopenic purpura (TTP)

*E* This patient presents with four of the five symptoms classically associated with TTP. These include thrombocytopenia, hemolytic anemia, neurologic deficits, renal impairment, and fever. TTP develops with fibrin-strand deposition in small vessels that attract platelets leading to platelet thrombi and thrombocytopenia. Passing RBCs get sheared in occluded vessels resulting in microangiopathic hemolytic anemia. Renal and neurologic impairment occur as a result of the lodging of thrombi in respective circulations. Plasmapheresis decreases TTP mortality from 90% to 10%. Adjunct therapies include fresh frozen plasma infusion and steroids. It is important to realize that although patients may be severely thrombocytopenic, platelet infusion is contraindicated since it exacerbates the underlying cycle of thrombogenesis. Risk factors for TTP include pregnancy, autoimmune disorders, drugs, infection, and malignancy. Hemolytic-uremic syndrome (HUS) is a closely related entity usually seen in children. There is pronounced renal dysfunction without altered mentation.

A 23-year-old construction worker is brought by ambulance to the ED with bilateral knee pain. He reports mixing cement the day before and kneeling in the process. The patient states that his jeans were soaked through most of the day but did not attempt to wash the cement off. Upon physical examination, you see marked tissue necrosis of both knees extending to the bone in some places. Which chemical was this patient most likely exposed to? a. Hydrocarbon b. Phenol c. Ammonia d. Formic acid e. Lime

*E* This patient was exposed to lime, which is present in cement. When water interacts with dry cement, hydrolysis occurs forming an alkali with a pH of 10 to 12. Alkali burns form a liquefaction necrosis, causing quick dissolution of involved tissue. Acid burns form a coagulation necrosis, which is somewhat slower in nature. The best treatment is intense irrigation upon initial contact, a ritual performed by the more experienced individuals who work with lime. This patient will need further debridement and wound care because of his exposure.

A 67-year-old woman presents to the ED complaining of a 2-day history of general malaise, subjective fevers, chills, diffuse headache, and rightsided jaw pain. She also notes diminished vision in her right eye. Her symptoms are minimally relieved with acetaminophen. She denies any sick contacts. The patient's vitals include an oral temperature of 100.6°F, HR is 95 beats per minute, BP is 130/75 mm Hg, and RR is 16 breaths per minute with oxygen saturation of 99% on room air. She is tender on the right side of her scalp. You initiate empirical treatment. Which of the following tests will confirm your diagnosis? a. Influenza A/B assay b. Rapid strep test c. Erythrocyte sedimentation rate (ESR) d. Complete blood count (CBC) e. Temporal artery biopsy

*E* This patient's clinical picture is consistent with temporal arteritis (TA). Patients are usually middle aged females (age > 50) who present with malaise, fevers, and headache. A complete physical examination would have revealed temporal artery tenderness to palpation. This patient also complains of symptoms consistent with polymyalgia rheumatica, a general achiness that may become confused with influenza. Temporal or giant-cell arteritis is a granulomatous inflammation that involves the large- and medium-sized arteries of the body, commonly the carotid artery and its branches. Symptoms are produced as a result of ischemia to the organs fed by the branches of the artery. Visual loss in one eye, transient diplopia and jaw claudication are common symptoms when the branches of the internal and external carotid are affected. A temporal artery biopsy is the diagnostic test of choice and will confirm the diagnosis. However, it is important to note that TA is segmental in nature and false-negatives do occur. Treatment up until the time of biopsy should include high-dose glucocorticoids, namely prednisone or methylprednisolone. This does not alter biopsy results and may prevent progression of the disease. Hospitalization is warranted in patients with severe debilitation or impending visual loss and may require high-dose steroids.

You are notified by the EMS dispatcher that there is a multiple-car collision on the local highway with many injuries. He states that there are two people dead at the scene, one person is critically injured and hypotensive, and three people have significant injuries, but with stable vital signs. Which of the following is the leading cause of death and disability in trauma victims? a. Abdominal injury b. Thoracic injury c. Back injury d. Cervical injury e. Head injury

*E* Trauma is the leading cause of death between the ages of 1 and 44 years. Many of these injuries are treatable mainly because the patients are young and otherwise healthy. The primary role of the emergency physician is to assess, resuscitate, and stabilize the trauma patient by priority. There are three peak times for trauma deaths. The first, classified as immediate death, is the period with the greatest number of fatalities. This occurs within seconds to minutes of the injury and these patients generally die at the scene. The cause is most commonly because of massive head injury, followed by high cervical spine injury with spinal cord disruption, cardiac and great vessel rupture, and airway obstruction. The second peak period, classified as early death, occurs within minutes to a few hours of injury. This is the period called the "golden hour" where intervention is critical and significantly reduces the morbidity and mortality rate in these patients. Death in these patients is generally secondary to subdural and epidural hematomas. Other causes of death in this group include ruptured spleen, lacerated liver, hypovolemic shock, fracture of pelvis or multiple long bones, hemopneumothorax, tension pneumothorax, cardiac tamponade, and aortic dissection or rupture. The third peak period, classified as delayed death, occurs days to weeks following the initial injury. Death in these patients is usually a result of multisystem organ failure and sepsis.

A 27-year-old pregnant woman, in her third trimester, is brought to the ED after being involved in a low-speed motor vehicle collision. The patient was wearing a seat belt in the back seat of a car that was struck in the front by another car. Her BP is 120/70 mm Hg and HR is 107 beats per minute. Her airway is patent, breath sounds equal bilaterally, and skin is warm with 2+ pulses. FAST examination is negative for free fluid. Evaluation of the fetus reveals appropriate fetal HR and fetal movement. Repeat maternal BP is 120/75 mm Hg. Which of the following is the most appropriate next step in management? a. Perform an immediate cesarean section in the OR. b. Perform an immediate cesarean section in the ED. c. CT scan of the abdomen and pelvis to rule out occult injury. d. Discharge the patient if laboratory testing is normal. e. Monitor the patient and fetus for a minimum of 4 hours.

*E* Trauma occurs in up to 7% of all pregnancies and is the leading cause of maternal death. It is important to focus the primary examination on the patient and evaluate the fetus in the secondary examination. The ABCs are followed in usual fashion. Once the patient is deemed stable, the fetus should be evaluated. Fetal evaluation focuses on the fetal HR and fetal movement. Minor trauma to the patient does not rule out injury to the fetus. Therefore, it is important to monitor the fetus. Cardiotocographic observation of the viable fetus is recommended for a minimum of 4 hours to detect any intrauterine pathology. The minimum should be extended to 24 hours if, at any time during the first 4 hours, there are more than three uterine contractions per hour, persistent uterine tenderness, a non-reassuring fetal monitor strip, vaginal bleeding, rupture of the membranes, or any serious maternal injury is present.

A 47-year-old man is brought into the ED after falling 20 ft from a ladder. His HR is 110 beats per minute, BP is 110/80 mm Hg, RR is 20 breaths per minute, and oxygen saturation is 100% on face mask. He is able to answer your questions without difficulty. His chest is clear with bilateral breath sounds, abdomen is nontender, pelvis is stable, and the FAST examination is negative You note a large scrotal hematoma and blood at the urethral meatus. Which of the following is the most appropriate next step in management? a. Scrotal ultrasound b. Kidney-ureter-bladder (KUB) radiograph c. IV pyelogram d. Retrograde cystogram e. Retrograde urethrogram

*E* Urethral injuries make up approximately 10% of genitourinary trauma. Anterior urethral injuries are most often attributed to falls with straddle injuries or a blunt force to the perineum. Approximately 95% of posterior urethral injuries are secondary to pelvic fractures. Signs and symptoms of urethral injury include perineal pain, inability to void, gross hematuria, blood at the urethral meatus, perineal or scrotal swelling or ecchymosis, and an absent, high-riding, or boggy prostate. A retrograde urethrogram is the study of choice when there is suspicion of a urethral injury. This procedure is performed by inserting an 8F urinary catheter 2 cm into the meatus and inflating the catheter balloon with 2 cc saline to create a seal. Then, 30 cc of radiopaque contrast is administered and a radiograph is obtained looking for extravasation of contrast from the urethra.

You walk into the examining room to interview a patient. He refused giving vital signs at the triage station until he saw a doctor because he was afraid that the nurse would inject poison into him. He appears agitated and starts raising his voice as soon as you are in the room. You ask him in a calm voice if you could help him and he replies by shouting obscenities and throwing a box of gloves at you. You start to walk toward him but he says he will kill you if you get any closer. Which of the following is the most appropriate next step in management? a. Tell the patient he is being uncooperative and is to leave the ED immediately. b. Have the nurse go into the room alone to try to calm the patient. c. Let the patient sit in the room for another hour and see if he calms down. d. Ask the nurse to prepare an injection of lorazepam that you will give to sedate the patient. e. Alert hospital security that there is a violent patient and prepare to place the patient in physical restraints.

*E* Violent behavior requires immediate restraint. Hospital security and the police are best trained to subdue violent patients with the least chance of staff or patient injury. Patients who are threatening or who demonstrate violent behavior should be disrobed, gowned, and searched for weapons. Patients whose behavior suggests the potential for violence should be approached cautiously with adequate security force nearby. The physician should stand in a location that neither threatens the patient nor blocks the exit of the patient or the physician from the room. Physical restraints are frequently required for the violent or severely agitated psychotic, delirious, or intoxicated patient who is a danger to themselves or others. Pharmacologic restraints (haloperidol, lorazepam) are also useful in obtaining behavior control and should be considered once the initial evaluation has been completed.

A 24-year-old woman presents to the ED after being sexually assaulted. She is a college student with no past medical history. Her BP is 130/75 mm Hg, HR is 91 beats per minute, temperature is 98.6°F, and RR is 16 breaths per minute. On physical examination you observe vaginal trauma and scattered bruising and abrasions. Which of the following medications should be offered to the patient in this scenario? a. Ceftriaxone, azithromycin, metronidazole, antiretrovirals, emergency contraception b. Ceftriaxone, tetanus, metronidazole, antiretrovirals, emergency contraception c. Ceftriaxone, azithromycin, tetanus, metronidazole, emergency contraception d. Ceftriaxone, azithromycin, tetanus, antiretrovirals, emergency contraception e. Ceftriaxone, azithromycin, tetanus, metronidazole, antiretrovirals, emergency contraception

*E* When a sexual assault patient is evaluated in the ED, the EP not only has the standard responsibility to care for the patient's immediate physical and psychological health, but he/she must also consider how the encounter may affect the patient's life considerably once discharged from the ED. Once life-threatening injuries are addressed, EPs are responsible for collecting physical evidence necessary for prosecuting the assailant by conducting a sexual assault or "rape kit" with the patient's consent. Most medications provided to sexually assaulted patients are provided as prophylaxis against sexually transmitted infections (STIs), pregnancy, and tetanus.

A 61-year-old man with a history of depression and hypertension is brought to the ED by EMS for altered mental status. The patient's wife states that he stopped taking his fluoxetine 1 month ago and now only takes metoprolol for his hypertension. The patient's BP is 75/40 mm Hg, HR is 39 beats per minute, RR is 14 breaths per minute, oxygen saturation is 99% on 100% oxygen, and fingerstick glucose is 81 mg/dL. The patient is awake and moaning, responding only to deep stimuli. You suspect an overdose of metoprolol. You endotracheally intubate the patient for airway control. Which of the following is the most appropriate next step in management? a. Syrup of ipecac, normal saline bolus, epinephrine b. Cardioversion with 200 J then administer epinephrine c. Cardioversion with 200 J then administer atropine d. Normal saline bolus, atropine, epinephrine, and activated charcoal e. Normal saline bolus, atropine, glucagon, and activated charcoal

*E* β-Adrenergic receptor blockers (eg, Metoprolol) are commonly prescribed medications for hypertension. β-Adrenergic antagonist overdose is often benign, with about 33% of patients remaining asymptomatic. This is partially explained by the fact that β-adrenergic antagonism is often well tolerated in healthy persons who do not rely on sympathetic stimulation to maintain cardiac output. Conversely, those with cardiac abnormalities may rely on sympathetic stimulation to maintain HR or cardiac output. The hallmark of β-adrenergic receptor blocker toxicity is bradycardia with hypotension. Patients may also exhibit conduction and rhythm abnormalities. Onset of toxicity usually occurs within 4 hours of ingestion. If a patient remains asymptomatic after 4 hours there is a low risk for subsequent morbidity unless a delayed-release preparation is involved.

A 27-year-old man who is otherwise healthy presents to the ED with a laceration on his thumb that he sustained while cutting a bagel. You irrigate and repair the wound and are about to discharge the patient when he asks you if he can receive an ECG. It is not busy in the ED so you perform the ECG which shows prolonged PR interval Which of the following is the most appropriate next step in management? a. Admit the patient for placement of a pacemaker. b. Admit the patient for a 24-hour observation period. c. Administer aspirin and send cardiac biomarkers. d. Repeat the ECG because of incorrect lead placement. e. Discharge the patient home.

*E*. The patient's ECG shows a sinus rhythm at a rate of 70 with first-degree heart block. First-degree heart block is defined as prolonged conduction of atrial impulses without the loss of any impulse. On an ECG this translates to a PR interval greater than 200 ms with a narrow QRS complex (less than 120 msec). First-degree heart block is often a normal variant without clinical significance, occurring in 1% to 2% of healthy young adults. This variant requires no specific treatment

Which of the following is the best test to confirm your diagnosis of MS? a. Edrophonium test b. Angiogram of the carotid arteries c. Lumbar puncture (LP) and cerebrospinal fluid (CSF) analysis d. Obtain a head CT e. Obtain a head magnetic resonance imaging (MRI)

*E*Demyelinating MS lesions are often well demonstrated on MRI but cannot be visualized on CT scan.

A 41-year-old man is brought into the ED by paramedics in cardiopulmonary arrest. A friend states that the patient is a long-time user of IV heroin. You look at the monitor and see that the patient has pulseless electrical activity (PEA). Cardiopulmonary resuscitation is being performed and the patient is intubated. You decide to administer epinephrine to the patient, but realize that he does not have IV access. Which of the following drugs is ineffective when administered through an endotracheal (ET) tube? a. Atropine b. Naloxone c. Lidocaine d. Epinephrine e. Sodium bicarbonate

*E*Endotracheal administration of drugs is indicated whenever there is a need for emergent pharmacologic intervention in the absence of other access routes, such as IV orintraosseous. There are a limited number of emergency medications that can be administered safely and effectively via the endotracheal route. These include naloxone, atropine, versed, epinephrine, and lidocaine. This is remembered by the mnemonic NAVEL. Specific medications shown to be unsafe include sodium bicarbonate, isoproterenol, and bretylium. The endotracheal dosage of a medication should be at least equivalent to the IV route, and is usually 2 to 2.5 times the IV dose is administered. The patient in the scenario should receive epinephrine via the ET while IV access is established.

A 55-year-old man presents to the ED with fever, drooling, trismus, and a swollen neck. He reports a foul taste in his mouth since a tooth extraction 2 days ago. On physical examination, the patient appears anxious. He has bilateral submandibular swelling and elevation and protrusion of the tongue. He appears "bull-necked" with tense and markedly tender edema and brawny induration of the upper neck, and he is tender over the lower second and third molars. There is no cervical lymphadenopathy. His vital signs are: HR 105 beats per minute, BP 140/85 mm Hg, RR 26 breaths per minute, and temperature 102°F. Which of the following is the most appropriate next step in management? a. Administer a dose of IV antibiotics and obtain a soft-tissue radiograph of the neck. b. Administer a dose of IV antibiotics then perform an incision and drainage at the bedside. c. Begin steroids to decrease inflammation and obtain an ear, nose, and throat (ENT) consult. d. Discharge the patient with oral antibiotics and ENT follow-up. e. Secure his airway, start IV antibiotics, and obtain an ENT consult.

*E*Ludwig Angina (LA) is a potentially life-threatening cellulitis of the connective tissue of the floor of the mouth and neck that begins in the submandibular space. An infected or recently extracted tooth is present in most cases. Typically, it is a polymicrobial infection involving aerobic and anaerobic bacteria of the mouth. The most commonly isolated organisms include streptococci, staphylococci, and bacteroides species. The most common physical findings in LA are bilateral submandibular swelling and tongue protrusion or elevation. A tense edema and brawny induration of the neck above the hyoid may be present and is described as a "bull neck." Marked tendernes to palpation of the neck and subcutaneous emphysema may be noted. Trismus and fever are often present, but usually no palpable fluctuance or cervical lymphadenopathy. The involved teeth may be tender to palpation. Patients with LA should never be left alone as airway impairment can suddenly occur. Signs of impending airway compromise include stridor, tachypnea, dyspnea, drooling, and agitation. The upper airway may be distorted making endotracheal intubation difficult or impossible. Cricothyrotomy may also be difficult and increases the risk of spreading infection into the mediastinum. Fiberoptic nasotracheal intubation is preferred.

A tall, thin 18-year-old man presents to the ED with acute onset of dyspnea while at rest. The patient reports sitting at his desk when he felt a sharp pain on the right side of his chest that worsened with inspiration. His past medical history is significant for peptic ulcer disease. He reports taking a 2-hour plane trip a month ago. His initial vitals include a HR of 100 beats per minute, a BP of 120/60 mm Hg, an RR of 16 breaths per minute, and an oxygen saturation of 97% on room air. On physical examination, you note decreased breath sounds on the right side. Which of the following tests should be performed next? a. Electrocardiogram (ECG) b. D-dimer c. Ventilation perfusion scan (V/Q scan) d. Upright abdominal radiograph e. Chest radiograph

*E.* A spontaneous pneumothorax typically presents with ipsilateral pleuritic chest pain and dyspnea while at rest. Physical findings tend to correlate with the degree of symptoms. Mild tachycardia, decreased breath sounds to auscultation, or hyperresonance to percussion are the most common findings. It typically occurs in healthy young men of taller than average stature without a precipitating factor. Mitral valve prolapse and Marfan syndrome are also associated with pneumothoraces. The most common condition associated with secondary spontaneous pneumothorax is COPD. Although suggested by this patient's symptoms, the diagnosis of pneumothorax is generally made with a chest radiograph. The classic sign is the appearance of a thin, visceral, pleural line lying parallel to the chest wall, separated by a radiolucent band that is devoid of lung markings. If clinical suspicion is high with a negative initial chest x-ray, inspiratory and expiratory films, or a lateral decubitus film may be taken to evaluate for lung collapse.

An 81-year-old woman presents to the ED with acute onset of shortness of breath just before arrival. She refuses to answer questions for the interview, but repeatedly states that she is feeling short of breath. Her initial vitals include a HR of 89 beats per minute, a BP of 168/76 mm Hg, and an RR of 18 breaths per minute with an oxygen saturation of 89% on room air. A portable chest x-ray appears normal. Her physical examination is unremarkable, except for a systolic ejection murmur. Intravenous (IV) access is successfully obtained. After placing the patient on oxygen and a monitor, which of the following should be performed first? a. Evaluation of troponin level b. Evaluation of D-dimer level c. Rectal temperature d. Repeat chest x-ray e. ECG

*E.* All patients with chest pain and shortness of breath should receive an ECG. It is a quick, noninvasive test that often provides substantive information. An ECG will show that this patient is having a large anterolateral wall myocardial infraction (MI) affecting much of her left ventricle, the reason for her heart murmur. An ECG must be performed in those crucial first moments so that the proper care can be delivered. This example reminds us of the importance of keeping the differential diagnosis broad in patients that present with respiratory distress. The other procedures may be done in a timely manner, but do not necessarily need to be performed as the next most critical step.

A 48-year-old man with a past medical history of hepatitis C and cirrhosis presents to the ED complaining of acute onset abdominal pain and chills. His BP is 118/75 mm Hg, HR is 105 beats per minute, RR is 16 breaths per minute, temperature is 101.2°F rectally, and oxygen saturation is 97% on room air. His abdomen is distended, and diffusely tender. You decide to perform a paracentesis and retrieve 1 L of cloudy fluid. Laboratory analysis of the fluid shows a neutrophil count of 550 cells/mm3. Which of the following is the most appropriate choice of treatment? a. Metronidazole b. Vancomycin c. Sulfamethoxazole/trimethoprim (SMX/TMP) d. Neomycin and lactulose e. Cefotaxime

*E.* Analysis of abdominal fluid and clinical presentation are consistent with spontaneous bacterial peritonitis (SBP). It is recommended to start antibiotic treatment for SBP if the neutrophil count is greater than 250 cells/mm3. Causative organisms include gram-negative enterococcus such as E coli and Klebsiella, as well as Streptococcus sp., and Streptococcus pneumoniae. Therefore, the most appropriate antibiotic for treatment is a third-generation cephalosporin, such as cefotaxime

A 22-year-old woman is brought to the ED by paramedics who state that they found the patient hunched over on a park bench barely breathing. The patient is rousable only to painful stimuli. Her initial vitals include a HR of 78 beats per minute, a BP of 125/58 mm Hg, and a respiratory rate of 6 breaths per minute with an oxygen saturation of 94% on 2-L nasal cannula. Upon physical examination, the patient has clear breath sounds bilaterally and no signs of trauma. Her pupils are 2 mm bilaterally and reactive to light. Which of the following agents may be used to restore this patient's respirations? a. Oxygen b. Flumazenil c. Anticholinergic inhaler treatment d. β2-Agonist nebulized treatment e. Naloxone

*E.* Attention to airway and breathing is of particular importance in opioid intoxication, as indicated by the patient's pinpoint pupils, because respiratory and central nervous system (CNS) depression are the most common life-threatening complications. Naloxone is a μ-opioid receptor competitive antagonist and its rapid blockade of those receptors reverses the depressive effects of opioids.

A 56-year-old woman with a history of ovarian cancer presents to the ED with acute onset of right-sided chest pain, shortness of breath, and dyspnea. Her BP is 131/75 mm Hg, HR is 101 beats per minute, respirations are 18 breaths per minute, and oxygen saturation is 97% on room air. You suspect this patient has a pulmonary embolism (PE). Which of the following tests is most likely to be abnormal? a. Arterial blood gas b. Oxygen saturation c. ECG d. Chest radiograph e. D-dimer

*E.* D-dimer is a degradation product produced by plasmin-mediated proteolysis of cross-linked fibrin. There are two types of D-dimer assays. Those with greatest sensitivity are the enzyme-linked immunosorbent assays and the turbidimetric assays. Because of their high negative predictive value, D-dimer levels are typically used to rule out the diagnosis of PE. In conjunction with a low pretest probability, a negative D-dimer is predictive of not having a PE. Therefore, in this patient with a high probability of PE (eg, exhibiting dyspnea, chest pain, tachycardia, malignancy), it is likely that the D-dimer will be abnormal

A 32-year-old man is brought into the ED by EMS with fever, shortness of breath, and stridor. The patient was treated yesterday in the ED for a viral syndrome. His BP is 90/50 mm Hg, HR is 110 beats per minute, temperature is 101.2°F, and his RR is 28 breaths per minute. A chest radiograph reveals a widened mediastinum. The patient is endotracheally intubated, given a 2-L bolus of normal saline, and started on antibiotics. His BP improves to 110/70 mm Hg and he is transferred to the intensive care unit (ICU). You see a friend that accompanied the patient to the hospital and ask him some questions. You find out that the patient is a drum maker and works with animal hides. What is the most likely organism that is responsible for the patient's presentation? a. Streptococcus pneumoniae b. Corynebacterium diphtheriae c. Coxiella burnetii d. Haemophilus influenzae e. Bacillus anthracis

*E.* Inhalation anthrax is a rare, but life-threatening disease, with mortality rates exceeding 90%. It is caused by inhaling B anthracis spores into the lungs. Initially, the patient develops flu-like symptoms. Within 24 to 48 hours, the clinical course may abruptly deteriorate to septic shock, respiratory failure, and mediastinitis. Chest x-ray may reveal a widened mediastinum. Death usually results within 3 days. Anthrax is normally a disease of sheep, cattle, and horses. As there is no evidence for human-to-human transmission, disease in humans occurs when spores are inhaled. Working with untreated animal hides increases the risk for anthrax exposure.

A 24-year-old man woke up from sleep 1 hour ago with severe pain in his right testicle. He states that he is sexually active with multiple partners. On examination, the right scrotum is swollen, tender, and firm. You cannot elicit a cremasteric reflex. His BP is 145/75 mm Hg, HR is 103 beats per minute, RR is 14 breaths per minute, temperature is 98.9°F, and oxygen saturation is 99% on room air. Which of the following is the most appropriate next step in management? a. Administer one dose of ceftriaxone and doxycycline for 10 days and have him follow-up with a urologist. b. Swab his urethra, send a culture for gonorrhea and Chlamydia, and treat if positive. c. Send a urinalysis and treat for a urinary tract infection (UTI) if positive. d. Treat the patient for epididymitis and have him return if symptoms persist. e. Order a statim (STAT) color Doppler ultrasound and urologic consultation.

*E.* Testicular torsion is a surgical emergency. There are two peak periods in which torsion is likely to occur, the first year of life and at puberty. Manual detorsion can be attempted in most cases while arranging for definitive care. After appropriate analgesia, the anterior testicle should be twisted laterally, like opening a book. Color Doppler ultrasound is the best test of choice in most hospitals. Immediate evaluation and referral to a urologist is essential.

A 75-year-old man goes out to shovel snow from his driveway. After 5 minutes of shoveling, he feels short of breath, chest pain, and then passes out. He awakens minutes later to his wife shaking him. In the ED, he denies chest pain or dyspnea. His BP is 160/85 mm Hg, HR is 71 beats per minute, and oxygen saturation is 97% on room air. On examination, you hear a harsh systolic ejection murmur. An ECG reveals a sinus rhythm with left ventricular hypertrophy. Which of the following is the most likely diagnosis? a. Asystolic cardiac arrest b. Brugada syndrome c. Subclavian steal syndrome d. PE e. Aortic stenosis

*E.* The classic triad of aortic stenosis is dyspnea, chest pain, and exertional syncope. Syncope is a result of either inadequate cerebral perfusion or occasional dysrhythmias. The classic auscultory finding is a harsh systolic ejection murmur that is best heard in the second right intercostal space with radiation to the carotid arteries. Syncope in the setting of a new systolic murmur always should raise the suspicion for aortic stenosis as the etiology. The ECG usually reveals left ventricular hypertrophy.

A 19-year-old woman presents to the ED with 1 hour of acute onset progressively worsening pain in her RLQ. She developed nausea shortly after the pain and vomited twice over the last hour. She had similar but less severe pain 2 weeks ago that resolved spontaneously. Her BP is 123/78 mm Hg, HR is 99 beats per minute, temperature is 99.1°F, and her RR is 16 breaths per minute. On physical examination, the patient appears uncomfortable, not moving on the gurney. Her abdomen is nondistended, diffusely tender, worst in the RLQ. Pelvic examination reveals a normal sized uterus and moderate right-sided adnexal tenderness. Laboratory results reveal WBC 10,000/μL, hematocrit 38%, and a negative urinalysis and β-hCG. Pelvic ultrasound reveals an enlarged right ovary with decreased flow. Which of the following is the most appropriate management for this patient? a. Admit to the gynecology service for observation b. Administer IV antibiotics and operate once inflammation resolves c. Attempt manual detorsion d. Order an abdominal CT e. Immediate laparoscopic surgery

*E.* The differential diagnosis in a woman with RLQ pain is expansive and includes GI pathology, such as appendicitis, inflammatory bowel disease, diverticulitis, and hernia. Gynecologic pathology includes ectopic pregnancy, tubo-ovarian abscess, ruptured corpus luteum cyst, and ovarian torsion. It is often difficult to initially distinguish between gastrointestinal (GI) and gynecologic (GYN) pathology and which diagnostic test, abdominal CT, or a pelvic ultrasound, is warranted. Often, the decision is based on the pelvic examination. The patient in the question exhibits adnexal tenderness and therefore received a pelvic ultrasound that revealed a unilateral enlarged ovary with decreased flow, indicative of ovarian torsion. Ovarian torsion is a gynecologic emergency and conservative management has no place in the treatment decision of suspected torsion even if pain improves in the ED. Failure to surgically correct this entity may result in ischemia and subsequent necrosis of the involved ovary. Therefore, the mainstay of therapy is laparoscopy or laparotomy.

A 71-year-old obese man is brought to the ED complaining of constant left middle quadrant abdominal pain with radiation into his back. His past medical history is significant for hypertension, peripheral vascular disease, peptic ulcer disease, kidney stones, and gallstones. He smokes a pack of cigarettes and consumes a pint of vodka daily. His BP is 145/80 mm Hg, HR is 90 beats per minute, temperature is 98.9°F, and RR is 16 breaths per minute. Abdominal examination is unremarkable. An ECG is read as sinus rhythm with a HR of 88 beats per minute. An abdominal radiograph reveals normal loops of bowel and curvilinear calcification of the aortic wall. Which of the following is the most likely diagnosis? a. Biliary colic b. Nephrolithiasis c. Pancreatitis d. Small bowel obstruction (SBO) e. Abdominal aortic aneurysm

*E.* The patient presents with multiple risk factors for an AAA: age > 60, male gender, hypertension, cigarette smoking, and peripheral artery disease. Classically, AAA presents with constant abdominal pain, often localizing to the left middle or lower quadrant with radiation to the back. Physical examination may reveal a pulsatile abdominal mass. Patients can present unstable if the aneurysm leaks or ruptures requiring emergent management in the OR. Evidence of an AAA is seen on plain radiograph approximately 66% to 75% of the time. The most common findings are curvilinear calcification of the aortic wall or a paravertebral soft tissue mass. Ultrasound and CT are the best diagnostic tools for the stable patient.

An undomiciled 41-year-old man walks into the ED complaining of abdominal pain, nausea, and vomiting. He tells you that he has been drinking beer continuously over the previous 18 hours. On examination, his vitals are BP 150/75 mm Hg, HR 104 beats per minute, RR 16 breaths per minute, oxygen saturation 97% on room air, temperature of 99.1°F rectally, and finger stick glucose 81 mg/dL. The patient is alert and oriented, his pupils anicteric. You notice gynecomastia and spider angiomata. His abdomen is soft but tender in the RUQ. Laboratory tests reveal an AST of 212 U/L, ALT 170 U/L, alkaline phosphatase of 98 U/L, total bilirubin of 1.9 mg/dL, international normalized ratio (INR) of 1.3, WBC 12,000/μL. Urinalysis shows 1+ protein. Chest x-ray is unremarkable. Which of the following is the most appropriate next step in management? a. Place a nasogastric tube in the patient's stomach to remove any remaining ethanol b. Order a HIDA scan to evaluate for acute cholecystitis c. Administer hepatitis B immune globulin d. Send viral hepatitis titers e. Supportive care by correcting any fluid and electrolyte imbalances

*E.* The patient's clinical presentation is consistent with alcoholic hepatitis, which is a potentially severe form of alcohol-induced liver disease. Most people remain subclinical, but the presentation ranges from nausea and vomiting to fulminant hepatitis and liver failure. Laboratory tests usually reveal moderate elevations of AST and ALT. Usually in alcoholic hepatitis, the AST is greater than the ALT (think "scotch" and "tonic" for AST > ALT). The patient exhibits stigmata of chronic alcohol disease seen by gynecomastia and spider angiomata.

As you palpate the right upper quadrant (RUQ) of a 38-year-old woman's abdomen, you notice that she stops her inspiration for a brief moment. During the history, the patient states that over the last 2 days she gets pain in her RUQ that radiates to her back shortly after eating. Her vitals include a temperature of 100.4°F, HR of 95 beats per minute, BP of 130/75 mm Hg, RR of 16 breaths per minute. What is the initial diagnostic modality of choice for this disorder? a. Plain film radiograph b. Computed tomography (CT) scan c. Magnetic resonance imaging (MRI) d. Radioisotope cholescintigraphy (HIDA scan) e. Ultrasonography

*E.* The patient's history and physical examination is consistent with acute cholecystitis. Because of the poor predictive value of the history, physical and laboratory findings in cholecystitis, the most important test for diagnosis is a strong clinical suspicion and ultrasound imaging. It is rapid and noninvasive. Ultrasound may show the presence of gallstones as small as 2 mm, gallbladder wall thickening and distention, and pericholecystic fluid

A 59-year-old man presents to the ED with left-sided chest pain and shortness of breath that began 1 hour ago. Initial vital signs are BP 85/45 mm Hg, HR 105 beats per minute, RR 20 breaths per minute, and oxygen saturation 94% on room air. An ECG is shows ST elevation in leads V1-V4. Which of the following is the most appropriate definitive treatment? a. Administer metoprolol or diltiazem b. Electrical cardioversion c. Administer calcium gluconate d. Thrombolytic therapy e. Percutaneous angioplasty

*E.* The preferred treatment for an ST-elevation MI is primary percutaneous intervention (eg, angioplasty, coronary stent). It has shown to improve long term mortality over (d) thrombolytic therapy

As you are examining the patient described in the previous question (Mobitz II ekg), he starts to complain of chest discomfort and shortness of breath and has another syncopal episode. His ECG is shown below. Which of the following is the most appropriate next step in management? a. Call cardiology consult b. Cardiovert the patient c. Administer metoprolol d. Administer amiodarone e. Apply transcutaneous pacemaker

*E.* The rhythm strip findings are consistent with third-degree AV block, also called complete heart block. It is characterized by absent conduction through the AV node, resulting in the dissociation of atrial and ventricular rhythms. The ECG shows independent P waves and QRS complexes. Mobitz type II often progresses to third degree heart block, as seen in this case. The immediate step in managing complete heart block is applying a transcutaneous pacemaker for ventricular pacing as a temporizing measure. However, patients need implantable ventricular pacemakers for definitive management. In addition, the underlying cause of the block needs to be addressed.

A 67-year-old woman is brought to the ED by paramedics complaining of dyspnea, fatigue, and palpitations. Her BP is 80/50 mm Hg, heart is 139 beats per minute, and RR is 20 breaths per minute. Her skin is cool and she is diaphoretic. Her lung examination reveals bilateral crackles and she is beginning to have chest pain. Her ECG shows a narrow complex irregular rhythm with a rate in the 140s. Which of the following is the most appropriate immediate treatment for this patient? a. Diltiazem b. Metoprolol c. Digoxin d. Coumadin e. Synchronized cardioversion

*E.* This patient is hypotensive and exhibits signs and symptoms of heart failure (dyspnea, fatigue, respiratory crackles, and chest pain) and is in atrial fibrillation (irregular, narrow complex). Any patient with unstable vital signs with a tachydysrhythmia should receive a dose of sedation and undergo synchronized cardioversion starting at 100 J.

A 59-year-old man presents to the ED with left-sided chest pain and shortness of breath that began 2 hours prior to arrival. He states the pain is pressure-like and radiates down his left arm. He is diaphoretic. His BP is 160/80 mm Hg, HR 86 beats per minute, and RR 15 breaths per minute. ECG reveals 2-mm ST-segment elevation in leads I, aVL, V3 to V6. Which of the following is an absolute contraindication to receiving thrombolytic therapy? a. Systolic BP greater than 180 mm Hg b. Patient on Coumadin and aspirin c. Total hip replacement 3 months ago d. Peptic ulcer disease e. Previous hemorrhagic stroke

*E.* Thrombolytic therapy (clotbusters) can be administered to patients having an acute ST-elevation MI that is within 6 to 12 hours from symptom onset. Contraindications to fibrinolytic therapy are those that increase the risk of hemorrhage. The most catastrophic complication is intracranial hemorrhage. Absolute contraindications include: • Previous hemorrhagic stroke • Known intracranial neoplasm • Active internal bleeding (excluding menses) • Suspected aortic dissection or pericarditis

A 23-year-old man is brought to the ED after collapsing during a basketball game. His BP is 80/palp. His father died from a "heart-condition" at a young age. He is placed on a monitor and the rhythm shows wide complex QRS. Which of the following choices best describes this rhythm? a. Ventricular fibrillation b. Atrial fibrillation c. Wolf-Parkinson-White syndrome d. Supraventricular tachycardia e. Torsades de pointes

*E.* Torsades de pointes ("twisting of the points") is a life-threatening uncommon variant of ventricular tachycardia (VT). The ventricular rate can range from 150 to 250 beats per minute. It may occur secondary to medications that prolong the QT interval, such as some antipsychotics. It is also caused by electrolyte disturbances and congenital prolonged-QT syndrome. This congenital form typically presents in childhood or early adulthood and is precipitated by catecholamine excess, such as released during exercise or with the administration of certain medications.

A 29-year-old man presents to the ED complaining of RLQ pain for 24 hours. He states that the pain first began as a dull feeling around his umbilicus and slowly migrated to his right side. He has no appetite, is nauseated, and vomited twice. His BP is 130/75 mm Hg, HR is 95 beats per minute, temperature is 100.9°F, and his RR is 16 breaths per minute. His WBC is 14,000/μL. As you palpate the LLQ of the patient's abdomen, he states that his RLQ is painful. What is the name of this sign? a. Blumberg sign b. Psoas sign c. Obturator sign d. Raynaud sign e. Rovsing sign

*E.*Rovsing sign is the referred tenderness to the RLQ when the LLQ is palpated. It is seen with acute appendicitis.

An 82-year-old woman is brought to the ED by her daughter for worsening fatigue, dizziness, and lightheadedness. The patient denies chest pain or shortness of breath. She has not started any new medications. Her BP is 140/70 mm Hg, HR is 37 beats per minute, and RR is 15 breaths per minute. An IV is started and blood is drawn. An ECG shows P waves marching throughout. Which of the following is the most appropriate next step in management? a. Bed rest for the next 48 hours and follow-up with her primary-care physician. b. Administer aspirin, order a set of cardiac enzymes, and admit to the cardiac care unit (CCU). c. Place a magnet on her chest to turn off her pacemaker. d. Admit for Holter monitoring and echocardiogram. e. Place on a cardiac monitor, place external pacing pads on the patient, and admit to the CCU.

*E.*The patient's ECG reveals third-degree complete heart block. It is a disorder of the cardiac conduction system, where there is no conduction through the AV node. This may occur secondary to MI, drug intoxication, infection, or infiltrative diseases. On the ECG complete heart block is represented by QRS complexes being conducted at their own rate and independent of the P waves. Individuals with second-degree type II or third-degree complete heart block are considered unstable. External pacing pads should be placed on them, followed by a transvenous pacer if their BP is unstable. They may require a permanent pacemaker for irreversible complete heart block.

A 24-year-old woman is brought to the ED after being found on a nearby street hunched over and in mild respiratory distress. Upon arrival, she is tachypneic at 24 breaths per minute with an oxygen saturation of 97% on face mask oxygen administration. Upon physical examination, the patient appears to be in mild distress with supraclavicular retractions. Scattered wheezing is heard throughout bilateral lung fields. Which of the following medications should be administered first? a. Corticosteroids b. Magnesium sulfate c. Epinephrine d. Anticholinergic nebulizer treatment e. β2-Agonist nebulizer treatment

*E.*This patient is suffering from an acute asthma attack. This is a reversible bronchospasm initiated by a variety of environmental factors that produce a narrowing and inflammation of the bronchial airways. The first-line treatment in order to open the airways includes a β2-agonist, which acts to decrease bronchospasm of the smooth muscle.

Which factors will the nurse consider when calculating the CURB-65 score for a patient with pneumonia (select all that apply)? a. Age b. Blood pressure c. Respiratory rate d. Oxygen saturation e. Presence of confusion f. Blood urea nitrogen (BUN) level

-Age -Blood pressure -Respiratory rate -Oxygen saturation -Blood urea nitrogen (BUN) level

A patient develops neutropenia after receiving chemotherapy. Which information about ways to prevent infection will the nurse include in the teaching plan (select all that apply)? a. Cook food thoroughly before eating. b. Choose low fiber, low residue foods. c. Avoid public transportation such as buses. d. Use rectal suppositories if needed for constipation. e. Talk to the oncologist before having any dental work done.

-Cook food thoroughly before eating. -Avoid public transportation such as buses. -Talk to the oncologist before having any dental work done.

A patient with suspected neurogenic shock after a diving accident has arrived in the emergency department. A cervical collar is in place. Which actions should the nurse take (select all that apply)? a. Prepare to administer atropine IV. b. Obtain baseline body temperature. c. Infuse large volumes of lactated Ringer's solution. d. Provide high-flow oxygen (100%) by non-rebreather mask. e. Prepare for emergent intubation and mechanical ventilation.

-Prepare to administer atropine IV. -Obtain baseline body temperature. -Provide high-flow oxygen (100%) by non-rebreather mask. -Prepare for emergent intubation and mechanical ventilation.

Which preventive actions by the nurse will help limit the development of systemic inflammatory response syndrome (SIRS) in patients admitted to the hospital (select all that apply)? a. Use aseptic technique when caring for invasive lines or devices. b. Ambulate postoperative patients as soon as possible after surgery. c. Remove indwelling urinary catheters as soon as possible after surgery. d. Advocate for parenteral nutrition for patients who cannot take oral feedings. e. Administer prescribed antibiotics within 1 hour for patients with possible sepsis.

-Use aseptic technique when caring for invasive lines or devices. -Ambulate postoperative patients as soon as possible after surgery. -Remove indwelling urinary catheters as soon as possible after surgery. -Administer prescribed antibiotics within 1 hour for patients with possible sepsis.

Risk factors for lung bronchoscopy

-patient cooperation -lack of skilled personnel -Unstable angina -uncontrolled ventricular arrhythmias -uncorrected hypoxia/hypercapnia -severe asthma (irritable airways) -severe coagulopathy significant tracheal obstruction

The nurse assesses a patient with chronic obstructive pulmonary disease (COPD) who has been admitted with increasing dyspnea over the last 3 days. Which finding is most important for the nurse to report to the health care provider?

. Respirations are 36 breaths/minute The increase in respiratory rate indicates respiratory distress and a need for rapid interventions such as administration of oxygen or medications.

1. You are caring for a patient with esophageal cancer. Which task could be delegated to a UAP? 1. Assisting the patient with oral hygiene 2. Observing the patient's response to feedings 3. Facilitating expression of grief or anxiety 4. Initiating daily weighings

1. Ans: 1 Oral hygiene is within the scope of duties of the UAP. It is the responsibility of the nurse to observe response to treatments and to help the patient deal with loss or anxiety. The UAP can be directed to weigh the patient but should not be expected to know when to initiate that measurement. Focus: Delegation

To prevent the debilitating cycle of fatigue-depression-fatigue that can occur in patients with cancer, an appropriate nursing intervention is to:

1. Have the patient rest after any major energy expenditure. 2. Encourage the patient to implement a daily walking program. 3. Teach the patient to ignore the fatigue to maintain normal daily activities. 4. Prevent the development of depression by informing the patient to expect fatigue during cancer treatment.

Advantages of Fiberoptic bronchoscopy

1. Improved access to airways 2. Topical anesthesia, IV sedation 3. Patient comfort and safety 4. Fiberoptic scope can be inserted via nasal or oral route or thru tracheostomy or ET tube

After 3 weeks of radiation therapy, a patient has lost 10 pounds and does not eat well because of mucositis. An appropriate nursing diagnosis for the patient is:

1. Risk for infection related to poor nutrition. 2. Ineffective self-health management related to refusal to eat. 3. Imbalanced nutrition: Less than body requirements related to oral inflammation and ulceration. 4. Ineffective health maintenance related to lack of knowledge of nutritional requirements during radiation therapy.

When planning a community education program related to prevention of the cancer with the highest death rates in both women and men, the nurse will stress the importance of:

1. Smoking cessation. 2. Screening with colonoscopy. 3. Regular examination of reproductive organs. 4. Use of sunscreen and protection from ultraviolet light.

Indications Rigid Bronchoscopy

1. infant bronchoscopy removal of foreign bodies 2. Significant Bleeding /massive hemoptysis -increased suction

10. For a patient who is receiving chemotherapy, which laboratory result is of particular importance? 1. White blood cell (WBC) count 2. Prothrombin time and partial thromboplastin time 3. Electrolyte levels 4. Blood urea nitrogen level

10. Ans: 1 WBC count is especially important, because chemotherapy can cause decreases in WBCs, particularly neutrophils (known as neutropenia), which leave the patient vulnerable to infection. The other tests are important in the total management but are less directly specific to chemotherapy in general. Focus: Prioritization

11. For care of a patient who has oral cancer, which task would be appropriate to delegate to an LPN/LVN? 1. Assisting the patient to brush and floss 2. Explaining when brushing and flossing are contraindicated 3. Giving antacids and sucralfate suspension as ordered 4. Recommending saliva substitutes

11. Ans: 3 Giving medications is within the scope of practice of the LPN/LVN. Assisting the patient in brushing and flossing should be delegated to the UAP. Explaining contraindications is the responsibility of the RN. Recommendations for saliva substitutes should come from the physician or pharmacist. Focus: Delegation

12. When staff assignments are made for the care of patients who are receiving chemotherapy, what is the major consideration regarding chemotherapeutic drugs? 1. During preparation, drugs may be absorbed through the skin or inhaled. 2. Many chemotherapeutic drugs are vesicants. 3. Chemotherapeutic drugs are frequently given through central venous access devices. 4. Oral and venous routes of administration are the most common.

12. Ans: 1 Chemotherapy drugs should be given by nurses who have received additional training in how to safely prepare and deliver the drugs and protect themselves and others from exposure. The other options express concerns, but the general principles of drug administration apply. Focus: Assignment

13. You have just received the morning report from the night shift nurse. List the order of priority for assessing and caring for the following patients. 1. A patient who developed tumor lysis syndrome around 5:00 AM 2. A patient with frequent reports of breakthrough pain over the past 24 hours 3. A patient scheduled for exploratory laparotomy this morning 4. A patient with anticipatory nausea and vomiting for the past 24 hours _______,_______, _______, _______

13. Ans: 1, 3, 2, 4 Tumor lysis syndrome is an emergency involving electrolyte imbalances and potential renal failure. A patient scheduled for surgery should be assessed and prepared for surgery. A patient with breakthrough pain needs assessment, and the physician may need to be contacted for a change of dosage or medication. Anticipatory nausea and vomiting has a psychogenic component that requires assessment, teaching, reassurance, and administration of antiemetics. Focus: Prioritization

15. You are caring for an older woman with hepatic cancer. The UAP informs you that the patient's level of consciousness is diminished compared to earlier in the shift. Prioritize the steps of assessment and intervention related to this patient's change of mental status. 1. Take vital signs, including pulse, respirations, blood pressure, and temperature. 2. Check responsiveness and level of consciousness. 3. Obtain a blood glucose reading. 4. Check electrolyte values. 5. Check ammonia level. 6. Check the patency of existing IV lines. 7. Administer oxygen if needed and check pulse oximeter readings. _______, _______,_______, _______, _______, _______, _______

15. Ans: 2, 7, 1, 3, 6, 4, 5 Determine level of consciousness and responsiveness, and changes from baseline. Oxygen should be administered immediately in the presence of respiratory distress or risk for decreased oxygenation and perfusion. Pulse oximetry can be used for continuous monitoring. Adequate pulse, blood pressure, and respirations are required for cerebral perfusion. Increased temperature may signal infection or sepsis. Blood glucose levels should be checked even if the patient is not diabetic. Severe hypoglycemia should be immediately treated per protocol. A patent IV line may be needed for delivery of emergency drugs. Electrolyte and ammonia levels are relevant data for this patient, and abnormalities in these parameters may be contributing to change in mental status. (Note: Laboratory results [i.e., electrolytes and ammonia levels] may be concurrently available; however, you should train yourself to systematically look at data. Look at electrolytes first because these are more commonly ordered. In some cases, you may actually have to remind the physician to order the ammonia level if the patient with a hepatic disorder is having a change in mental status.) Focus: Prioritization

16. Which members of the health care team (advanced practice nurse, MD, RN, LPN/LVN) should perform the tasks related to care of patients who are at risk for breast cancer? (There may be more than one professional who could complete the task.) 1. Perform the clinical breast examination. _________________________________________ 2. Teach about breast self-examination. _________________________________________ 3. Make a nursing diagnosis based on the assessment data. _________________________________________ 4. Assess the patient's belief about and use of complementary and alternative therapies. _________________________________________ 5. Reinforce the importance of a baseline screening mammogram starting at age 40. _________________________________________ 6. Explain the results of the mammogram to the patient. _________________________________________

16. Ans: 1. Advanced practice nurse, MD, 2. Advanced practice nurse, MD, RN, 3. Advanced practice nurse, RN, 4. Advanced practice nurse, MD, RN, 5. Advanced practice nurse, MD, RN, LPN/LVN, 6. MD Advanced practice nurses could do any of the tasks; however, explaining results of a mammogram may be handled by the supervising physician, especially if complex follow-up is needed (e.g., surgery). Physicians could do any of the tasks except they do not make nursing diagnoses. RNs could do tasks 2, 3, 4, and 5 but usually do not do clinical breast examination, unless specially trained, and do not interpret results of diagnostic tests for patients. LPNs/LVNs could reinforce standard information about screening recommendations. The RN should make the nursing diagnoses, and the LPN/LVN assists in planning and implementing the interventions. Focus: Assignment

17. For a patient with osteogenic sarcoma, which laboratory value causes you the most concern? 1. Sodium level of 135 mEq/L 2. Calcium level of 13 mg/dL 3. Potassium level of 4.9 mEq/L 4. Hematocrit of 40%

17. Ans: 2 Potentially life-threatening hypercalcemia can occur in cancers with destruction of bone. Other laboratory values are pertinent for overall patient management but are less specific to bone cancers. Focus: Prioritization

18. Which two cancer patients could potentially be placed together as roommates? 1. A patient with a neutrophil count of 1000/mm3 2. A patient who underwent debulking of a tumor to relieve pressure 3. A patient who just underwent a bone marrow transplantation 4. A patient who has undergone laminectomy for spinal cord compression _______,_______

18. Ans: 2, 4 Debulking of tumor and laminectomy are palliative procedures. These patients can be placed in the same room. The patient with a low neutrophil count and the patient who has had a bone marrow transplantation need protective isolation. Focus: Assignment

19. An athletic young man was recently diagnosed with Ewing sarcoma. He has pain, lowgrade fever, and anemia. The surgeon recommends amputation of the right lower leg for an operable tumor. The patient tells you he is leaving the hospital to go on a long hiking trip. What is the priority nursing diagnosis? 1. Acute Pain related to tumor invasion of soft tissue 2. Fatigue related to anemia 3. Ineffective Coping related to loss of body image 4. Noncompliance related to personal values

19. Ans: 3 The patient is not coping with the recent diagnosis of cancer and prospect of losing his leg. His decision to go hiking may be a form of denial, or possibly a veiled suicide threat. It is also possible that he has decided not to have any treatment; however, you need to make additional assessment about his decision and actions and help him to discuss alternatives and consequences. The other diagnoses may also apply, but if he leaves the hospital there will be no chance to address any other issues. Focus: Prioritization

2. A 56-year-old patient comes to the walk-in clinic reporting scant rectal bleeding and intermittent diarrhea and constipation for the past several months. There is a history of polyps and a family history of colorectal cancer. While you are trying to teach about colonoscopy, the patient becomes angry and threatens to leave. What is the priority diagnosis? 1. Diarrhea/Constipation related to altered bowel patterns 2. Deficient Knowledge related to the disease process and diagnostic procedure 3. Risk for Deficient Fluid Volume related to rectal bleeding and diarrhea 4. Anxiety related to unknown outcomes and perceived threats to body integrity

2. Ans: 4 The patient's physical condition is currently stable, but emotional needs are affecting his or her ability to receive the information required to make an informed decision. The other diagnoses are relevant, but if the patient leaves the clinic the interventions may be delayed or ignored. Focus: Prioritization

21. People at risk are the target populations for cancer screening programs. Which of these asymptomatic patients need extra encouragement to participate in cancer screening? (Select all that apply.) 1. A 21-year-old white American woman who is sexually inactive, for a Pap test 2. A 30-year-old Asian-American woman, for an annual mammogram 3. A 45-year-old African-American man, for a prostate-specific antigen test 4. A 50-year-old African-American man, for a fecal occult blood test 5. A 50-year-old white American woman, for a colonoscopy 6. A 70-year-old Asian-American woman with normal results on three previous Pap tests, for a Pap test

21. Ans: 1, 3, 4, 5 Women age 21 or over should have annual Pap smears, regardless of sexual activity. African-American men should begin prostate-specific antigen testing at age 45. Colonoscopy and annual fecal occult blood testing are recommended for those with average risk starting at age 50. Annual mammograms are recommended for women over the age of 40. Women age 65 or older who have normal results on previous Pap tests may forego additional screenings for cervical cancer. Focus: Prioritization

Several patients call the urology clinic requesting appointments with the health care provider as soon as possible. Which patient will the nurse schedule to be seen first? a. 22-year-old who has noticed a firm, nontender lump on his scrotum b. 35-year-old who is concerned that his scrotum "feels like a bag of worms" c. 40-year-old who has pelvic pain while being treated for chronic prostatitis d. 70-year-old who is reporting frequent urinary dribbling after a prostatectomy

22-year-old who has noticed a firm, nontender lump on his scrotum The patient's age and symptoms suggest possible testicular cancer. Some forms of testicular cancer can be very aggressive, so the patient should be evaluated by the health care provider as soon as possible. Varicoceles do require treatment, but not emergently. Ongoing pelvic pain is common with chronic prostatitis. Urinary dribbling is a common problem after prostatectomy.

22. A patient with lung cancer develops syndrome of inappropriate antidiuretic hormone secretion (SIADH). After reporting symptoms of weight gain, weakness, and nausea and vomiting to the physician, you would anticipate which initial order for the treatment of this patient? 1. A fluid bolus 2. Fluid restrictions 3. Urinalysis 4. Sodium-restricted diet

22. Ans: 2 Hyponatremia is a concern; therefore, fluid restrictions would be ordered. Urinalysis is less pertinent; however, the nurse should monitor for changes in urine specific gravity. The diet may need to include sodium supplements. Fluid bolus is unlikely to be ordered for patients with SIADH; however, IV normal saline or hypertonic saline solutions may be given very cautiously. Focus: Prioritization

Several patients call the outpatient clinic and ask to make an appointment as soon as possible. Which patient should the nurse schedule to be seen first? a. 44-year-old with sickle cell anemia who says "my eyes always look sort of yellow" b. 23-year-old with no previous health problems who has a nontender lump in the axilla c. 50-year-old with early-stage chronic lymphocytic leukemia who reports chronic fatigue d. 19-year-old with hemophilia who wants to learn to self-administer factor VII replacement

23-year-old with no previous health problems who has a nontender lump in the axilla The patient's age and presence of a nontender axillary lump suggest possible lymphoma, which needs rapid diagnosis and treatment. The other patients have questions about treatment or symptoms that are consistent with their diagnosis but do not need to be seen urgently.

23. In the care of a patient with neutropenia, what tasks can be delegated to a UAP? (Select all that apply.) 1. Taking vital signs every 4 hours 2. Reporting temperature of more than 100.4° F (38° C) 3. Assessing for sore throat, cough, or burning with urination 4. Gathering the supplies to prepare the room for protective isolation 5. Reporting superinfections, such as candidiasis 6. Practicing good hand-washing technique

23. Ans: 1, 2, 4, 6 Measuring vital signs and reporting on specific parameters, practicing good hand washing, and gathering equipment are within the scope of duties for a UAP. Assessing for symptoms of infections and superinfections is the responsibility of the RN. Focus: Delegation

24. A primary nursing responsibility is the prevention of lung cancer by assisting patients in cessation of smoking or other tobacco use. Which task would be appropriate to delegate to an LPN/LVN? 1. Development of a "quit plan" 2. Explanation of how to apply a nicotine patch 3. Discussion of strategies to avoid relapse 4. Suggestion of ways to deal with urges for tobacco

24. Ans: 2 An LPN/LVN is versed in medication administration and able to teach patients standardized information. The other options require more in-depth assessment, planning, and teaching, which should be performed by the RN. Helping patients with smoking cessation is a Core Measure. Focus: Delegation

25. You are providing end-of-life-care for a patient with terminal liver cancer. The patient is weak and restless. Her skin is cool and mottled. Dyspnea develops and the patient appears anxious and frightened. What should you do? 1. Obtain an order for morphine elixir. 2. Alert the rapid response team and call the physician for orders. 3. Deliver breaths at 20/min with a bag-valve mask and prepare for intubation. 4. Sit quietly with the patient and offer emotional support and comfort.

25. Ans: 1 Morphine elixir is the therapy of choice because it is thought to reduce anxiety and the subjective sensation of air hunger. It also increases venous capacitance. End-of-life-care should not include aggressive measures such as intubation or resuscitation. Support and comfort are always welcome, but in this case you should not sit quietly because there is an option that would offer some physical relief for the patient. Focus: Prioritization

26. You are asked to float to a different nursing unit. During report, you are told that the patient is receiving IV administration of vincristine (Oncovin) that should be completed within the next 15 minutes. The IV site is intact, and the patient is not having any problems with the infusion. You are not certified in chemotherapy administration. What is your priority action? 1. Ask the nurse to stay until the infusion is finished, because you are not certified. 2. Assess the IV site; check the progress of the infusion and the patient's condition. 3. Contact the charge nurse and explain that you are not chemotherapy certified. 4. Look up drug side effects and monitor, because the infusion is almost complete.

26. Ans: 3 Explain that you are not chemotherapy certified so that the charge nurse can quickly rearrange the patient assignments. You can assess the patient, site, and infusion; however, you do not have the expertise to recognize the side effects of the medication or to give specialized care that may be needed. Asking the nurse to stay is not the best solution, because the care of the patient and the effects of the medication continue after the infusion has been completed. Looking up the side effects of the drug is okay for your own information, but you are still not qualified to deal with this situation. In addition, knowing how to properly discontinue the infusion and dispose of the equipment are essential for your own safety and the safety of others. Focus: Prioritization

27. You are caring for a patient with uterine cancer who is being treated with intracavitary radiation therapy. The UAP reports that the patient insisted on ambulating to the bathroom and now "something feels like it is coming out." What is the priority action? 1. Assess the UAP's knowledge; explain the rationale for strict bed rest. 2. Assess for dislodgment; use forceps to retrieve and a lead container to store as needed. 3. Assess the patient's knowledge of the treatment plan and her willingness to participate. 4. Notify the physician about the potential or confirmed dislodgment of the radiation implant.

27. Ans: 2 If the radiation implant has obviously been expelled (i.e., is on the bed linens), use a pair of forceps to place the radiation source in a lead container. The other options would be appropriate after safety of the patient and personnel are ensured. Focus: Prioritization, supervision

28. You are the charge nurse. Two hours into the shift you discover that two nurses have switched patients because Nurse A does "not like to take care of patients with prostate cancer." Which action should you take first? 1. Insist that they switch back to the original patient assignments and talk to each of them at the end of the shift. 2. Allow them this flexibility; as long as the patients are well cared for it doesn't matter if the assignments are changed. 3. Ask Nurse A to explain her position regarding prostate cancer patients and seek alternatives to prevent future issues. 4. Explain to Nurse A and B that all patients deserve kindness and care regardless of their condition or the nurses' personal feelings.

28. Ans: 3 You must try to find out what Nurse A is thinking and feeling. If you can discover the underlying issue, there is a better chance that you can help her (e.g., referral to counseling or in-service training). You should try to avoid being too draconian with your staff by insisting that they switch back to the original assignments, or too condescending by lecturing them about patients' rights. Nurses frequently can and do switch patients to help each other out, but the charge nurse should always be informed prior to making the switch. Focus: Assignment, supervision

Which patient should the nurse assess first after receiving change-of-shift report? a. 60-year-old patient whose new ileostomy has drained 800 mL over the previous 8 hours b. 50-year-old patient with familial adenomatous polyposis who has occult blood in the stool c. 40-year-old patient with ulcerative colitis who has had six liquid stools in the previous 4 hours d. 30-year-old patient who has abdominal distention and an apical heart rate of 136 beats/minute

30-year-old patient who has abdominal distention and an apical heart rate of 136 beats/minute The patient's abdominal distention and tachycardia suggest hypovolemic shock caused by problems such as peritonitis or intestinal obstruction, which will require rapid intervention. The other patients should also be assessed as quickly as possible, but the data do not indicate any life-threatening complications associated with their diagnoses.

After receiving change-of-shift report for several patients with neutropenia, which patient should the nurse assess first? a. 56-year-old with frequent explosive diarrhea b. 33-year-old with a fever of 100.8° F (38.2° C) c. 66-year-old who has white pharyngeal lesions d. 23-year old who is complaining of severe fatigue

33-year-old with a fever of 100.8° F (38.2° C) Any fever in a neutropenic patient indicates infection and can quickly lead to sepsis and septic shock. Rapid assessment and (if prescribed) initiation of antibiotic therapy within 1 hour are needed. The other patients also need to be assessed but do not exhibit symptoms of potentially life-threatening problems.

5. When care assignments are being made for patients with alterations related to gastrointestinal (GI) cancer, which patient would be the most appropriate to assign to an LPN/LVN under the supervision of a team leader RN? 1. A patient with severe anemia secondary to GI bleeding 2. A patient who needs enemas and antibiotics to control GI bacteria 3. A patient who needs preoperative teaching for bowel resection surgery 4. A patient who needs central line insertion for chemotherapy

5. Ans: 2 Administering enemas and antibiotics is within the scope of practice of LPNs/LVNs. Although some states an facilities may allow the LPN/LVN to administer blood, in general, administering blood, providing preoperative teaching, and assisting with central line insertion are the responsibilities of the RN. Focus: Assignment

6. A community health center is preparing a presentation on the prevention and detection of cancer. Which health care professional (RN, LPN/LVN, nurse practitioner,nutritionist) should be assigned responsibility for the following tasks? 1. Explain screening examinations and diagnostic testing for common cancers. ___________________________ 2. Discuss how to plan a balanced diet and reduce fats and preservatives. ___________________________ 3. Prepare a poster on the seven warning signs of cancer. ___________________________ 4. Discuss how to perform breast or testicular self-examinations. ___________________________ 5. Describe strategies for reducing risk factors such as smoking and obesity. ___________________________

6. Ans: 1. Nurse practitioner, 2. Nutritionist, 3. LPN/LVN, 4. Nurse practitioner, 5. RN The nurse practitioner is often the provider who performs the physical examinations and recommends diagnostic testing. The nutritionist can give information about diet. The LPN/LVN will know the standard seven warning signs and can educate through standard teaching programs. The RN has primary responsibility for educating people about risk factors. Focus: Assignment

7. The physician tells the patient with cancer that there will be an initial course of treatment with continued maintenance treatments and ongoing observation for signs and symptoms over a prolonged period of time. Which patient statement would concern you the most? 1. "My symptoms will eventually be cured; I'm so happy that I don't have to worry any longer." 2. "My doctor is trying to help me control the symptoms; I am grateful for the extension of time with my family." 3. "My pain will be relieved, but I am going to die soon; I would like to have control over my own life and death." 4. "Initially, I may have to take some time off of work for my treatments; I can probably work full time in the future."

7. Ans: 3 Further assess what the patient means by having "control over my own life and death." This could be an indirect statement of suicidal intent. A patient who believes he will be cured should also be assessed for misunderstanding what the physician said; however, the patient may need to use denial as a temporary defense mechanism. The patient's acknowledgment that the treatments are for control of symptoms or plans for the immediate future suggest an understanding of what the physician said. Focus: Prioritization

8. For a patient who is experiencing side effects of radiation therapy, which task would be the most appropriate to delegate to the UAP? 1. Helping the patient to identify patterns of fatigue 2. Recommending participation in a walking program 3. Reporting the amount and type of food consumed from the tray 4. Checking the skin for redness and irritation after the treatment

8. Ans: 3 The UAP can observe the amount that the patient eats (or what is gone from the tray) and report to the nurse. Assessing patterns of fatigue and skin reaction is the responsibility of the RN. The initial recommendation for exercise should come from the physician. Focus: Delegation

9. For a patient receiving the chemotherapeutic drug vincristine (Oncovin), which side effects should be reported to the physician? 1. Fatigue 2. Nausea and vomiting 3. Paresthesia 4. Anorexia

9. Ans: 3 Paresthesia is a side effect associated with some chemotherapy drugs such as vincristine. The physician can modify the dosage or discontinue the drug. Fatigue, nausea, vomiting, and anorexia are common side effects of many chemotherapy medications. The nurse can assist the patient by planning for rest periods, giving antiemetics as ordered, and encouraging small meals containing high-protein and highcalorie foods. Focus: Prioritization

The nurse receives change-of-shift report on the following four patients. Which patient should the nurse assess first? a. A 23-year-old patient with cystic fibrosis who has pulmonary function testing scheduled b. A 46-year-old patient on bed rest who is complaining of sudden onset of shortness of breath c. A 77-year-old patient with tuberculosis (TB) who has four antitubercular medications due in 15 minutes d. A 35-year-old patient who was admitted the previous day with pneumonia and has a temperature of 100.2° F (37.8° C)

A 46-year-old patient on bed rest who is complaining of sudden onset of shortness of breath

The nurse receives change-of-shift report on the following four patients. Which patient should the nurse assess first?

A 46-year-old patient on bed rest who is complaining of sudden onset of shortness of breath Patients on bed rest who are immobile are at high risk for deep vein thrombosis (DVT). Sudden onset of shortness of breath in a patient with a DVT suggests a pulmonary embolism and requires immediate assessment and action such as oxygen administration.

The nurse in the clinic notes elevated prostate specific antigen (PSA) levels in the laboratory results of these patients. Which patient's PSA result is most important to report to the health care provider? a. A 38-year-old who is being treated for acute prostatitis b. A 48-year-old whose father died of metastatic prostate cancer c. A 52-year-old who goes on long bicycle rides every weekend d. A 75-year-old who uses saw palmetto to treat benign prostatic hyperplasia (BPH)

A 48-year-old whose father died of metastatic prostate cancer The family history of prostate cancer and elevation of PSA indicate that further evaluation of the patient for prostate cancer is needed. Elevations in PSA for the other patients are not unusual.

The nurse in a health clinic receives requests for appointments from several patients. Which patient should be seen by the health care provider first? a. A 48-year-old man who has perineal pain and a temperature of 100.4° F b. A 58-year-old man who has a painful erection that has lasted over 6 hours c. A 38-year-old man who states he had difficulty maintaining an erection last night d. A 68-year-old man who has pink urine after a transurethral resection of the prostate (TURP) 3 days ago

A 58-year-old man who has a painful erection that has lasted over 6 hours Priapism can cause complications such as necrosis or hydronephrosis, and this patient should be treated immediately. The other patients do not require immediate action to prevent serious complications.

The nurse is reviewing the medical records for five patients who are scheduled for their yearly physical examinations in September. Which patients should receive the inactivated influenza vaccination (select all that apply)?

A 76-year-old nursing home resident A 36-year-old female patient who is pregnant e. A 30-year-old patient who takes corticosteroids for rheumatoid arthritis

Thoracentesis

A needle puncture into the pleural space in the chest cavity to remove pleural fluid for diagnostic or therapeutic reasons Often done by the radiologist, can be done bed side, U/S guide is becoming more common.

Previous administrations of chemotherapy agents to a cancer patient have resulted in diarrhea. Which dietary modification should the nurse recommend? A. A bland, low-fiber diet B. A high-protein, high-calorie diet C. A diet high in fresh fruits and vegetables D. A diet emphasizing whole and organic foods

A. A bland, low-fiber diet Patients experiencing diarrhea secondary to chemotherapy and/or radiation therapy often benefit from a diet low in seasonings and roughage before the treatment. Foods should be easy to digest and low in fat. Fresh fruits and vegetables are high in fiber and should be minimized during treatment. Whole and organic foods do not prevent diarrhea.

A patient who is having an acute exacerbation of multiple sclerosis has a prescription for methylprednisolone (Solu-Medrol) 160 mg IV. The label on the vial reads: methylprednisolone 125 mg in 2 mL. How many mL will the nurse administer?

ANS: 2.56 With a concentration of 125 mg/2 mL, the nurse will need to administer 2.56 mL to obtain 160 mg of methylprednisolone. DIF: Cognitive Level: Understand (comprehension) REF: 1430-1431 TOP: Nursing Process: Implementation MSC: NCLEX: Physiological Integrity

A hospitalized 31-year-old patient with a history of cluster headache awakens during the night with a severe stabbing headache. Which action should the nurse take first? a. Start the ordered PRN oxygen at 6 L/min. b. Put a moist hot pack on the patient's neck. c. Give the ordered PRN acetaminophen (Tylenol). d. Notify the patient's health care provider immediately.

ANS: A Acute treatment for cluster headache is administration of 100% oxygen at 6 to 8 L/min. If the patient obtains relief with the oxygen, there is no immediate need to notify the health care provider. Cluster headaches last only 60 to 90 minutes, so oral pain medications have minimal effect. Hot packs are helpful for tension headaches but are not as likely to reduce pain associated with a cluster headache. DIF: Cognitive Level: Apply (application) REF: 1417 OBJ: Special Questions: Prioritization TOP: Nursing Process: Implementation MSC: NCLEX: Physiological Integrity

A patient who is diagnosed with cervical cancer that is classified as Tis, N0, M0 asks the nurse what the letters and numbers mean. Which response by the nurse is most appropriate? a. "The cancer involves only the cervix." b. "The cancer cells look almost like normal cells." c. "Further testing is needed to determine the spread of the cancer." d. "It is difficult to determine the original site of the cervical cancer."

ANS: A Cancer in situ indicates that the cancer is localized to the cervix and is not invasive at this time. Cell differentiation is not indicated by clinical staging. Because the cancer is in situ, the origin is the cervix. Further testing is not indicated given that the cancer has not spread.

8. A 31-year-old woman who has multiple sclerosis (MS) asks the nurse about risks associated with pregnancy. Which response by the nurse is accurate? a. "MS symptoms may be worse after the pregnancy." b. "Women with MS frequently have premature labor." c. "MS is associated with an increased risk for congenital defects." d. "Symptoms of MS are likely to become worse during pregnancy."

ANS: A During the postpartum period, women with MS are at greater risk for exacerbation of symptoms. There is no increased risk for congenital defects in infants born of mothers with MS. Symptoms of MS may improve during pregnancy. Onset of labor is not affected by MS. DIF: Cognitive Level: Understand (comprehension) REF: 1429 TOP: Nursing Process: Implementation MSC: NCLEX: Health Promotion and Maintenance

he nurse advises a patient with myasthenia gravis (MG) to a. perform physically demanding activities early in the day. b. anticipate the need for weekly plasmapheresis treatments. c. do frequent weight-bearing exercise to prevent muscle atrophy. d. protect the extremities from injury due to poor sensory perception.

ANS: A Muscles are generally strongest in the morning, and activities involving muscle activity should be scheduled then. Plasmapheresis is not routinely scheduled, but is used for myasthenia crisis or for situations in which corticosteroid therapy must be avoided. There is no decrease in sensation with MG, and muscle atrophy does not occur because although there is muscle weakness, they are still used. DIF: Cognitive Level: Apply (application) REF: 1437 TOP: Nursing Process: Implementation MSC: NCLEX: Physiological Integrity

A patient has been taking phenytoin (Dilantin) for 2 years. Which action will the nurse take when evaluating for adverse effects of the medication? a. Inspect the oral mucosa. b. Listen to the lung sounds. c. Auscultate the bowel tones. d. Check pupil reaction to light.

ANS: A Phenytoin can cause gingival hyperplasia, but does not affect bowel tones, lung sounds, or pupil reaction to light. DIF: Cognitive Level: Apply (application) REF: 1424 TOP: Nursing Process: Evaluation MSC: NCLEX: Physiological Integrity

Which action will the nurse include in the plan of care for a patient with impaired functioning of the left glossopharyngeal nerve (CN IX) and the vagus nerve (CN X)? a. Withhold oral fluid or foods. b. Provide highly seasoned foods. c. Insert an oropharyngeal airway. d. Apply artificial tears every hour.

ANS: A The glossopharyngeal and vagus nerves innervate the pharynx and control the gag reflex. A patient with impaired function of these nerves is at risk for aspiration. An oral airway may be needed when a patient is unconscious and unable to maintain the airway, but it will not decrease aspiration risk. Taste and eye blink are controlled by the facial nerve. DIF: Cognitive Level: Apply (application) REF: 1347 TOP: Nursing Process: Planning MSC: NCLEX: Physiological Integrity

The nurse is caring for a patient with colon cancer who is scheduled for external radiation therapy to the abdomen. Which information obtained by the nurse would indicate a need for patient teaching? a. The patient swims a mile 3 days a week. b. The patient snacks frequently during the day. c. The patient showers everyday with a mild soap. d. The patient has a history of dental caries with amalgam fillings.

ANS: A The patient is instructed to avoid swimming in salt water or chlorinated pools during the treatment period. The patient does not need to change habits of eating frequently or showering with a mild soap. A history of dental caries will not impact the patient who is scheduled for abdominal radiation.

The nurse observes a patient ambulating in the hospital hall when the patient's arms and legs suddenly jerk and the patient falls to the floor. The nurse will first a. assess the patient for a possible head injury. b. give the scheduled dose of divalproex (Depakote). c. document the timing and description of the seizure. d. notify the patient's health care provider about the seizure.

ANS: A The patient who has had a myoclonic seizure and fall is at risk for head injury and should first be evaluated and treated for this possible complication. Documentation of the seizure, notification of the seizure, and administration of antiseizure medications are also appropriate actions, but the initial action should be assessment for injury. DIF: Cognitive Level: Apply (application) REF: 1423 OBJ: Special Questions: Prioritization TOP: Nursing Process: Implementation MSC: NCLEX: Physiological Integrity

To assess the functioning of the trigeminal and facial nerves (CNs V and VII), the nurse should a. shine a light into the patient's pupil. b. check for unilateral eyelid drooping. c. touch a cotton wisp strand to the cornea. d. have the patient read a magazine or book.

ANS: A The trigeminal and facial nerves are responsible for the corneal reflex. The optic nerve is tested by having the patient read a Snellen chart or a newspaper. Assessment of pupil response to light and ptosis are used to check function of the oculomotor nerve. DIF: Cognitive Level: Understand (comprehension) REF: 1347 TOP: Nursing Process: Assessment MSC: NCLEX: Physiological Integrity

A widowed mother of four school-age children is hospitalized with metastatic ovarian cancer. The patient is crying and tells the nurse that she does not know what will happen to her children when she dies. Which response by the nurse is most appropriate? a. "Why don't we talk about the options you have for the care of your children?" b. "I'm sure you have friends that will take the children when you can't care for them." c. "For now you need to concentrate on getting well and not worrying about your children." d. "Many patients with cancer live for a long time, so there is still time to plan for your children."

ANS: A This response expresses the nurse's willingness to listen and recognizes the patient's concern. The responses beginning "Many patients with cancer live for a long time" and "For now you need to concentrate on getting well" close off discussion of the topic and indicate that the nurse is uncomfortable with the topic. In addition, the patient with metastatic ovarian cancer may not have a long time to plan. Although it is possible that the patient's friends will take the children, more assessment information is needed before making plans.

When caring for a patient who is pancytopenic, which action by unlicensed assistive personnel (UAP) indicates a need for the nurse to intervene? a. The UAP assists the patient to use dental floss after eating. b. The UAP adds baking soda to the patient's saline oral rinses. c. The UAP puts fluoride toothpaste on the patient's toothbrush. d. The UAP has the patient rinse after meals with a saline solution.

ANS: A Use of dental floss is avoided in patients with pancytopenia because of the risk for infection and bleeding. The other actions are appropriate for oral care of a pancytopenic patient.

A patient with Parkinson's disease is admitted to the hospital for treatment of pneumonia. Which nursing interventions will be included in the plan of care (select all that apply)? a. Use an elevated toilet seat. b. Cut patient's food into small pieces. c. Provide high-protein foods at each meal. d. Place an armchair at the patient's bedside. e. Observe for sudden exacerbation of symptoms.

ANS: A, B, D Because the patient with Parkinson's has difficulty chewing, food should be cut into small pieces. An armchair should be used when the patient is seated so that the patient can use the arms to assist with getting up from the chair. An elevated toilet seat will facilitate getting on and off the toilet. High-protein foods will decrease the effectiveness of L-dopa. Parkinson's is a steadily progressive disease without acute exacerbations. DIF: Cognitive Level: Apply (application) REF: 1436-1437 TOP: Nursing Process: Planning MSC: NCLEX: Physiological Integrity

A 27-year-old patient who has been treated for status epilepticus in the emergency department will be transferred to the medical nursing unit. Which equipment should the nurse have available in the patient's assigned room (select all that apply)? a. Side-rail pads b. Tongue blade c. Oxygen mask d. Suction tubing e. Urinary catheter f. Nasogastric tube

ANS: A, C, D The patient is at risk for further seizures, and oxygen and suctioning may be needed after any seizures to clear the airway and maximize oxygenation. The bed's side rails should be padded to minimize the risk for patient injury during a seizure. Use of tongue blades during a seizure is contraindicated. Insertion of a nasogastric (NG) tube is not indicated because the airway problem is not caused by vomiting or abdominal distention. A urinary catheter is not required unless there is urinary retention .DIF: Cognitive Level: Apply (application) REF: 1426 TOP: Nursing Process: Planning MSC: NCLEX: Physiological Integrity

The nurse at the clinic is interviewing a 64-year-old woman who is 5 feet, 3 inches tall and weighs 125 pounds (57 kg). The patient has not seen a health care provider for 20 years. She walks 5 miles most days and has a glass of wine 2 or 3 times a week. Which topics will the nurse plan to include in patient teaching about cancer screening and decreasing cancer risk (select all that apply)? a. Pap testing b. Tobacco use c. Sunscreen use d. Mammography e. Colorectal screening

ANS: A, C, D, E The patient's age, gender, and history indicate a need for screening and/or teaching about colorectal cancer, mammography, Pap smears, and sunscreen. The patient does not use excessive alcohol or tobacco, she is physically active, and her body weight is healthy.

Which nursing diagnosis is expected to be appropriate for a patient who has a positive Romberg test? a. Acute pain b. Risk for falls c. Acute confusion d. Ineffective thermoregulation

ANS: B A positive Romberg test indicates that the patient has difficulty maintaining balance with the eyes closed. The Romberg does not test for orientation, thermoregulation, or discomfort. DIF: Cognitive Level: Apply (application) REF: 1348 TOP: Nursing Process: Diagnosis MSC: NCLEX: Physiological Integrity

An unconscious male patient has just arrived in the emergency department after a head injury caused by a motorcycle crash. Which order should the nurse question? a. Obtain x-rays of the skull and spine. b. Prepare the patient for lumbar puncture. c. Send for computed tomography (CT) scan. d. Perform neurologic checks every 15 minutes.

ANS: B After a head injury, the patient may be experiencing intracranial bleeding and increased intracranial pressure, which could lead to herniation of the brain if a lumbar puncture is performed. The other orders are appropriate. DIF: Cognitive Level: Apply (application) REF: 1349 | 1352 TOP: Nursing Process: Implementation MSC: NCLEX: Physiological Integrity

Several patients have been hospitalized for diagnosis of neurologic problems. Which patient will the nurse assess first? a. Patient with a transient ischemic attack (TIA) returning from carotid duplex studies b. Patient with a brain tumor who has just arrived on the unit after a cerebral angiogram c. Patient with a seizure disorder who has just completed an electroencephalogram (EEG) d. Patient prepared for a lumbar puncture whose health care provider is waiting for assistance

ANS: B Because cerebral angiograms require insertion of a catheter into the femoral artery, bleeding is a possible complication. The nurse will need to check the pulse, blood pressure, and the catheter insertion site in the groin as soon as the patient arrives. Carotid duplex studies and EEG are noninvasive. The nurse will need to assist with the lumbar puncture as soon as possible, but monitoring for hemorrhage after cerebral angiogram has a higher priority. DIF: Cognitive Level: Analyze (analysis) REF: 1351-1352 OBJ: Special Questions: Prioritization; Multiple Patients TOP: Nursing Process: Planning MSC: NCLEX: Physiological Integrity

The nurse supervises the care of a patient with a temporary radioactive cervical implant. Which action by unlicensed assistive personnel (UAP), if observed by the nurse, would require an intervention? a. The UAP flushes the toilet once after emptying the patient's bedpan. b. The UAP stands by the patient's bed for 30 minutes talking with the patient. c. The UAP places the patient's bedding in the laundry container in the hallway. d. The UAP gives the patient an alcohol-containing mouthwash to use for oral care.

ANS: B Because patients with temporary implants emit radioactivity while the implants are in place, exposure to the patient is limited. Laundry and urine/feces do not have any radioactivity and do not require special precautions. Cervical radiation will not affect the oral mucosa, and alcohol-based mouthwash is not contraindicated.

Which finding would the nurse expect when assessing the legs of a patient who has a lower motor neuron lesion? a. Spasticity b. Flaccidity c. No sensation d. Hyperactive reflexes

ANS: B Because the cell bodies of lower motor neurons are located in the spinal cord, damage to the neuron will decrease motor activity of the affected muscles. Spasticity and hyperactive reflexes are caused by upper motor neuron damage. Sensation is not impacted by motor neuron lesions. DIF: Cognitive Level: Understand (comprehension) REF: 1338 TOP: Nursing Process: Assessment MSC: NCLEX: Physiological Integrity

The home health nurse cares for a patient who has been receiving interferon therapy for treatment of cancer. Which statement by the patient indicates a need for further assessment? a. "I have frequent muscle aches and pains." b. "I rarely have the energy to get out of bed." c. "I experience chills after I inject the interferon." d. "I take acetaminophen (Tylenol) every 4 hours."

ANS: B Fatigue can be a dose-limiting toxicity for use of biologic therapies. Flulike symptoms, such as muscle aches and chills, are common side effects with interferon use. Patients are advised to use acetaminophen every 4 hours.

A hospitalized patient who has received chemotherapy for leukemia develops neutropenia. Which observation by the nurse would indicate a need for further teaching? a. The patient ambulates several times a day in the room. b. The patient's visitors bring in some fresh peaches from home. c. The patient cleans with a warm washcloth after having a stool. d. The patient uses soap and shampoo to shower every other day.

ANS: B Fresh, thinned-skin fruits are not permitted in a neutropenic diet because of the risk of bacteria being present. The patient should ambulate in the room rather than the hospital hallway to avoid exposure to other patients or visitors. Because overuse of soap can dry the skin and increase infection risk, showering every other day is acceptable. Careful cleaning after having a bowel movement will help prevent skin breakdown and infection.

A 39-year-old patient with a suspected herniated intervertebral disc is scheduled for a myelogram. Which information is most important for the nurse to communicate to the health care provider before the procedure? a. The patient is anxious about the test. b. The patient has an allergy to shellfish. c. The patient has back pain when lying flat. d. The patient drank apple juice 4 hours earlier.

ANS: B Iodine-containing contrast medium is injected into the subarachnoid space during a myelogram. The health care provider may need to modify the postmyelogram orders to prevent back pain, but this can be done after the procedure. Clear liquids are usually considered safe up to 4 hours before a diagnostic or surgical procedure. The patient's anxiety should be addressed, but this is not as important as the iodine allergy. DIF: Cognitive Level: Apply (application) REF: 1352 OBJ: Special Questions: Prioritization TOP: Nursing Process: Assessment MSC: NCLEX: Physiological Integrity

The home health registered nurse (RN) is planning care for a patient with a seizure disorder related to a recent head injury. Which nursing action can be delegated to a licensed practical/vocational nurse (LPN/LVN)? a. Make referrals to appropriate community agencies. b. Place medications in the home medication organizer. c. Teach the patient and family how to manage seizures. d. Assess for use of medications that may precipitate seizures.

ANS: B LPN/LVN education includes administration of medications. The other activities require RN education and scope of practice. DIF: Cognitive Level: Apply (application) REF: 1426 OBJ: Special Questions: Delegation TOP: Nursing Process: Planning MSC: NCLEX: Safe and Effective Care Environment

Which intervention will the nurse include in the plan of care for a patient with primary restless legs syndrome (RLS) who is having difficulty sleeping? a. Teach about the use of antihistamines to improve sleep. b. Suggest that the patient exercise regularly during the day. c. Make a referral to a massage therapist for deep massage of the legs. d. Assure the patient that the problem is transient and likely to resolve.

ANS: B Nondrug interventions such as getting regular exercise are initially suggested to improve sleep quality in patients with RLS. Antihistamines may aggravate RLS. Massage does not alleviate RLS symptoms and RLS is likely to progress in most patients. DIF: Cognitive Level: Apply (application) REF: 1427 TOP: Nursing Process: Planning MSC: NCLEX: Physiological Integrity

A 73-year-old patient with Parkinson's disease has a nursing diagnosis of impaired physical mobility related to bradykinesia. Which action will the nurse include in the plan of care? a. Instruct the patient in activities that can be done while lying or sitting. b. Suggest that the patient rock from side to side to initiate leg movement. c. Have the patient take small steps in a straight line directly in front of the feet. d. Teach the patient to keep the feet in contact with the floor and slide them forward.

ANS: B Rocking the body from side to side stimulates balance and improves mobility. The patient will be encouraged to continue exercising because this will maintain functional abilities. Maintaining a wide base of support will help with balance. The patient should lift the feet and avoid a shuffling gait. DIF: Cognitive Level: Apply (application) REF: 1437 TOP: Nursing Process: Planning MSC: NCLEX: Physiological Integrity

A patient who is being treated for stage IV lung cancer tells the nurse about new-onset back pain. Which action should the nurse take first? a. Give the patient the prescribed PRN opioid. b. Assess for sensation and strength in the legs. c. Notify the health care provider about the symptoms. d. Teach the patient how to use relaxation to reduce pain.

ANS: B Spinal cord compression, an oncologic emergency, can occur with invasion of tumor into the epidural space. The nurse will need to assess the patient further for symptoms such as decreased leg sensation and strength and then notify the health care provider. Administration of opioids or use of relaxation may be appropriate but only after the nurse has assessed for possible spinal cord compression.

The nurse administers an IV vesicant chemotherapeutic agent to a patient. Which action is most important for the nurse to take? a. Infuse the medication over a short period of time. b. Stop the infusion if swelling is observed at the site. c. Administer the chemotherapy through a small-bore catheter. d. Hold the medication unless a central venous line is available.

ANS: B Swelling at the site may indicate extravasation, and the IV should be stopped immediately. The medication generally should be given slowly to avoid irritation of the vein. The size of the catheter is not as important as administration of vesicants into a running IV line to allow dilution of the chemotherapeutic drug. These medications can be given through peripheral lines, although central vascular access devices (CVADs) are preferred.

Which action will the nurse plan to take for a 40-year-old patient with multiple sclerosis (MS) who has urinary retention caused by a flaccid bladder? a. Decrease the patient's evening fluid intake. b. Teach the patient how to use the Credé method. c. Suggest the use of adult incontinence briefs for nighttime only. d. Assist the patient to the commode every 2 hours during the day.

ANS: B The Credé method can be used to improve bladder emptying. Decreasing fluid intake will not improve bladder emptying and may increase risk for urinary tract infection (UTI) and dehydration. The use of incontinence briefs and frequent toileting will not improve bladder emptying. DIF: Cognitive Level: Apply (application) REF: eNCP 59-3 TOP: Nursing Process: Planning MSC: NCLEX: Physiological Integrity

When a 74-year-old patient is seen in the health clinic with new development of a stooped posture, shuffling gait, and pill rolling-type tremor, the nurse will anticipate teaching the patient about a. oral corticosteroids. b. antiparkinsonian drugs. c. magnetic resonance imaging (MRI). d. electroencephalogram (EEG) testing.

ANS: B The diagnosis of Parkinson's is made when two of the three characteristic manifestations of tremor, rigidity, and bradykinesia are present. The confirmation of the diagnosis is made on the basis of improvement when antiparkinsonian drugs are administered. This patient has symptoms of tremor and bradykinesia. The next anticipated step will be treatment with medications. MRI and EEG are not useful in diagnosing Parkinson's disease, and corticosteroid therapy is not used to treat it. DIF: Cognitive Level: Apply (application) REF: 1434 TOP: Nursing Process: Planning MSC: NCLEX: Physiological Integrity

A high school teacher who has just been diagnosed with epilepsy after having a generalized tonic-clonic seizure tells the nurse, "I cannot teach anymore, it will be too upsetting if I have a seizure at work." Which response by the nurse is best? a. "You might benefit from some psychologic counseling." b. "Epilepsy usually can be well controlled with medications." c. "You will want to contact the Epilepsy Foundation for assistance." d. "The Department of Vocational Rehabilitation can help with work retraining."

ANS: B The nurse should inform the patient that most patients with seizure disorders are controlled with medication. The other information may be necessary if the seizures persist after treatment with antiseizure medications is implemented. DIF: Cognitive Level: Apply (application) REF: 1422 TOP: Nursing Process: Implementation MSC: NCLEX: Psychosocial Integrity

A chemotherapy drug that causes alopecia is prescribed for a patient. Which action should the nurse take to maintain the patient's self-esteem? a. Tell the patient to limit social contacts until regrowth of the hair occurs. b. Encourage the patient to purchase a wig or hat and wear it once hair loss begins. c. Teach the patient to gently wash hair with a mild shampoo to minimize hair loss. d. Inform the patient that hair usually grows back once the chemotherapy is complete.

ANS: B The patient is taught to anticipate hair loss and to be prepared with wigs, scarves, or hats. Limiting social contacts is not appropriate at a time when the patient is likely to need a good social support system. The damage occurs at the hair follicle and will occur regardless of gentle washing or use of a mild shampoo. The information that the hair will grow back is not immediately helpful in maintaining the patient's self-esteem.

ollowing a thymectomy, a 62-year-old male patient with myasthenia gravis receives the usual dose of pyridostigmine (Mestinon). An hour later, the patient complains of nausea and severe abdominal cramps. Which action should the nurse take first? a. Auscultate the patient's bowel sounds. b. Notify the patient's health care provider. c. Administer the prescribed PRN antiemetic drug. d. Give the scheduled dose of prednisone (Deltasone).

ANS: B The patient's history and symptoms indicate a possible cholinergic crisis. The health care provider should be notified immediately, and it is likely that atropine will be prescribed. The other actions will be appropriate if the patient is not experiencing a cholinergic crisis. DIF: Cognitive Level: Apply (application) REF: 1438-1439 OBJ: Special Questions: Prioritization TOP: Nursing Process: Implementation MSC: NCLEX: Physiological Integrity

A hospitalized patient complains of a bilateral headache, 4/10 on the pain scale, that radiates from the base of the skull. Which prescribed PRN medications should the nurse administer initially? a. Lorazepam (Ativan) b. Acetaminophen (Tylenol) c. Morphine sulfate (Roxanol) d. Butalbital and aspirin (Fiorinal)

ANS: B The patient's symptoms are consistent with a tension headache, and initial therapy usually involves a nonopioid analgesic such as acetaminophen, which is sometimes combined with a sedative or muscle relaxant. Lorazepam may be used in conjunction with acetaminophen but would not be appropriate as the initial monotherapy. Morphine sulfate and butalbital and aspirin would be more appropriate for a headache that did not respond to a nonopioid analgesic. DIF: Cognitive Level: Apply (application) REF: 1414 OBJ: Special Questions: Prioritization TOP: Nursing Process: Implementation MSC: NCLEX: Physiological Integrity

Which cerebrospinal fluid analysis result will be most important for the nurse to communicate to the health care provider? a. Specific gravity 1.007 b. Protein 65 mg/dL (0.65 g/L) c. Glucose 45 mg/dL (1.7 mmol/L) d. White blood cell (WBC) count 4 cells/μL

ANS: B The protein level is high. The specific gravity, WBCs, and glucose values are normal. DIF: Cognitive Level: Understand (comprehension) REF: 1353 TOP: Nursing Process: Implementation MSC: NCLEX: Physiological Integrity

The nurse will assess a 67-year-old patient who is experiencing a cluster headache for a. nuchal rigidity. b. unilateral ptosis. c. projectile vomiting. d. throbbing, bilateral facial pain.

ANS: B Unilateral eye edema, tearing, and ptosis are characteristic of cluster headaches. Nuchal rigidity suggests meningeal irritation, such as occurs with meningitis. Although nausea and vomiting may occur with migraine headaches, projectile vomiting is more consistent with increased intracranial pressure (ICP). Unilateral sharp, stabbing pain, rather than throbbing pain, is characteristic of cluster headaches. DIF: Cognitive Level: Understand (comprehension) REF: 1414 TOP: Nursing Process: Assessment MSC: NCLEX: Physiological Integrity

7. When obtaining a health history and physical assessment for a 36-year-old female patient with possible multiple sclerosis (MS), the nurse should a. assess for the presence of chest pain. b. inquire about urinary tract problems. c. inspect the skin for rashes or discoloration. d. ask the patient about any increase in libido.

ANS: B Urinary tract problems with incontinence or retention are common symptoms of MS. Chest pain and skin rashes are not symptoms of MS. A decrease in libido is common with MS. DIF: Cognitive Level: Apply (application) REF: 1429 TOP: Nursing Process: Assessment MSC: NCLEX: Physiological Integrity

A patient with Hodgkin's lymphoma who is undergoing external radiation therapy tells the nurse, "I am so tired I can hardly get out of bed in the morning." Which intervention should the nurse add to the plan of care? a. Minimize activity until the treatment is completed. b. Establish time to take a short walk almost every day. c. Consult with a psychiatrist for treatment of depression. d. Arrange for delivery of a hospital bed to the patient's home.

ANS: B Walking programs are used to keep the patient active without excessive fatigue. Having a hospital bed does not necessarily address the fatigue. The better option is to stay as active as possible while combating fatigue. Fatigue is expected during treatment and is not an indication of depression. Minimizing activity may lead to weakness and other complications of immobility.

The nurse teaches a patient with cancer of the liver about high-protein, high-calorie diet choices. Which snack choice by the patient indicates that the teaching has been effective? a. Lime sherbet b. Blueberry yogurt c. Cream cheese bagel d. Fresh strawberries and bananas

ANS: B Yogurt has high biologic value because of the protein and fat content. Fruit salad does not have high amounts of protein or fat. Lime sherbet is lower in fat and protein than yogurt. Cream cheese is low in protein.

The nurse teaches a patient who is scheduled for a prostate needle biopsy about the procedure. Which statement, if made by the patient, indicates that teaching was effective? a. "The biopsy will remove the cancer in my prostate gland." b. "The biopsy will determine how much longer I have to live." c. "The biopsy will help decide the treatment for my enlarged prostate." d. "The biopsy will indicate whether the cancer has spread to other organs."

ANS: C A biopsy is used to determine whether the prostate enlargement is benign or malignant, and determines the type of treatment that will be needed. A biopsy does not give information about metastasis, life expectancy, or the impact of cancer on the patient's life.

Which nursing action will be included in the care for a patient who has had cerebral angiography? a. Monitor for headache and photophobia. b. Keep patient NPO until gag reflex returns. c. Check pulse and blood pressure frequently. d. Assess orientation to person, place, and time.

ANS: C Because a catheter is inserted into an artery (such as the femoral artery) during cerebral angiography, the nurse should assess for bleeding after this procedure. The other nursing assessments are not necessary after angiography. DIF: Cognitive Level: Apply (application) REF: 1351 TOP: Nursing Process: Planning MSC: NCLEX: Physiological Integrity

The nurse receives change-of-shift report on the oncology unit. Which patient should the nurse assess first? a. 35-year-old patient who has wet desquamation associated with abdominal radiation b. 42-year-old patient who is sobbing after receiving a new diagnosis of ovarian cancer c. 24-year-old patient who received neck radiation and has blood oozing from the neck d. 56-year-old patient who developed a new pericardial friction rub after chest radiation

ANS: C Because neck bleeding may indicate possible carotid artery rupture in a patient who is receiving radiation to the neck, this patient should be seen first. The diagnoses and clinical manifestations for the other patients are not immediately life threatening.

While the nurse is transporting a patient on a stretcher to the radiology department, the patient begins having a tonic-clonic seizure. Which action should the nurse take? a. Insert an oral airway during the seizure to maintain a patent airway. b. Restrain the patient's arms and legs to prevent injury during the seizure. c. Time and observe and record the details of the seizure and postictal state. d. Avoid touching the patient to prevent further nervous system stimulation.

ANS: C Because the diagnosis and treatment of seizures frequently are based on the description of the seizure, recording the length and details of the seizure is important. Insertion of an oral airway and restraining the patient during the seizure are contraindicated. The nurse may need to move the patient to decrease the risk of injury during the seizure. DIF: Cognitive Level: Apply (application) REF: 1422 TOP: Nursing Process: Implementation MSC: NCLEX: Physiological Integrity

A 49-year-old patient with multiple sclerosis (MS) is to begin treatment with glatiramer acetate (Copaxone). Which information will the nurse include in patient teaching? a. Recommendation to drink at least 4 L of fluid daily b. Need to avoid driving or operating heavy machinery c. How to draw up and administer injections of the medication d. Use of contraceptive methods other than oral contraceptives

ANS: C Copaxone is administered by self-injection. Oral contraceptives are an appropriate choice for birth control. There is no need to avoid driving or drink large fluid volumes when taking glatiramer. DIF: Cognitive Level: Apply (application) REF: 1430 TOP: Nursing Process: Implementation MSC: NCLEX: Physiological Integrity

During the neurologic assessment, the patient is unable to respond verbally to the nurse but cooperates with the nurse's directions to move his hands and feet. The nurse will suspect a. cerebellar injury. b. a brainstem lesion. c. frontal lobe damage. d. a temporal lobe lesion.

ANS: C Expressive speech is controlled by Broca's area in the frontal lobe. The temporal lobe contains Wernicke's area, which is responsible for receptive speech. The cerebellum and brainstem do not affect higher cognitive functions such as speech. DIF: Cognitive Level: Apply (application) REF: 1339 TOP: Nursing Process: Assessment MSC: NCLEX: Physiological Integrity

A patient who has severe pain associated with terminal pancreatic cancer is being cared for at home by family members. Which finding by the nurse indicates that teaching regarding pain management has been effective? a. The patient uses the ordered opioid pain medication whenever the pain is greater than 5 (0 to 10 scale). b. The patient agrees to take the medications by the IV route in order to improve analgesic effectiveness. c. The patient takes opioids around the clock on a regular schedule and uses additional doses when breakthrough pain occurs. d. The patient states that nonopioid analgesics may be used when the maximal dose of the opioid is reached without adequate pain relief.

ANS: C For chronic cancer pain, analgesics should be taken on a scheduled basis, with additional doses as needed for breakthrough pain. Taking the medications only when pain reaches a certain level does not provide effective pain control. Although nonopioid analgesics also may be used, there is no maximum dose of opioid. Opioids are given until pain control is achieved. The IV route is not more effective than the oral route, and usually the oral route is preferred.

A 40-year-old patient is diagnosed with early Huntington's disease (HD). When teaching the patient, spouse, and children about this disorder, the nurse will provide information about the a. use of levodopa-carbidopa (Sinemet) to help reduce HD symptoms. b. prophylactic antibiotics to decrease the risk for aspiration pneumonia. c. option of genetic testing for the patient's children to determine their own HD risks. d. lifestyle changes of improved nutrition and exercise that delay disease progression.

ANS: C Genetic testing is available to determine whether an asymptomatic individual has the HD gene. The patient and family should be informed of the benefits and problems associated with genetic testing. Sinemet will increase symptoms of HD because HD involves an increase in dopamine. Antibiotic therapy will not reduce the risk for aspiration. There are no effective treatments or lifestyle changes that delay the progression of symptoms in HD. DIF: Cognitive Level: Apply (application) REF: 1440 TOP: Nursing Process: Implementation MSC: NCLEX: Physiological Integrity

The nurse is caring for a patient who has been diagnosed with stage I cancer of the colon. When assessing the need for psychologic support, which question by the nurse will provide the most information? a. "How long ago were you diagnosed with this cancer?" b. "Do you have any concerns about body image changes?" c. "Can you tell me what has been helpful to you in the past when coping with stressful events?" d. "Are you familiar with the stages of emotional adjustment to a diagnosis like cancer of the colon?"

ANS: C Information about how the patient has coped with past stressful situations helps the nurse determine usual coping mechanisms and their effectiveness. The length of time since the diagnosis will not provide much information about the patient's need for support. The patient's knowledge of typical stages in adjustment to a critical diagnosis does not provide insight into patient needs for assistance. Because surgical interventions for stage I cancer of the colon may not cause any body image changes, this question is not appropriate at this time

The nurse determines that teaching about management of migraine headaches has been effective when the patient says which of the following? a. "I can take the (Topamax) as soon as a headache starts." b. "A glass of wine might help me relax and prevent a headache." c. "I will lie down someplace dark and quiet when the headaches begin." d. "I should avoid taking aspirin and sumatriptan (Imitrex) at the same time."

ANS: C It is recommended that the patient with a migraine rest in a dark, quiet area. Topiramate (Topamax) is used to prevent migraines and must be taken for several months to determine effectiveness. Aspirin or other nonsteroidal antiinflammatory medications can be taken with the triptans. Alcohol may precipitate migraine headaches. DIF: Cognitive Level: Apply (application) REF: 1416 | 1419 TOP: Nursing Process: Evaluation MSC: NCLEX: Physiological Integrity

External-beam radiation is planned for a patient with cervical cancer. What instructions should the nurse give to the patient to prevent complications from the effects of the radiation? a. Test all stools for the presence of blood. b. Maintain a high-residue, high-fiber diet. c. Clean the perianal area carefully after every bowel movement. d. Inspect the mouth and throat daily for the appearance of thrush.

ANS: C Radiation to the abdomen will affect organs in the radiation path, such as the bowel, and cause frequent diarrhea. Careful cleaning of this area will help decrease the risk for skin breakdown and infection. Stools are likely to have occult blood from the inflammation associated with radiation, so routine testing of stools for blood is not indicated. Radiation to the abdomen will not cause stomatitis. A low-residue diet is recommended to avoid irritation of the bowel when patients receive abdominal radiation.

The nurse is caring for a patient with left-sided lung cancer. Which finding would be most important for the nurse to report to the health care provider? a. Hematocrit 32% b. Pain with deep inspiration c. Serum sodium 126 mEq/L d. Decreased breath sounds on left side

ANS: C Syndrome of inappropriate antidiuretic hormone (and the resulting hyponatremia) is an oncologic metabolic emergency and will require rapid treatment in order to prevent complications such as seizures and coma. The other findings also require intervention, but are common in patients with lung cancer and not immediately life threatening.

The nurse is caring for a patient who smokes 2 packs/day. To reduce the patient's risk of lung cancer, which action by the nurse is best? a. Teach the patient about the seven warning signs of cancer. b. Plan to monitor the patient's carcinoembryonic antigen (CEA) level. c. Discuss the risks associated with cigarettes during every patient encounter. d. Teach the patient about the use of annual chest x-rays for lung cancer screening.

ANS: C Teaching about the risks associated with cigarette smoking is recommended at every patient encounter because cigarette smoking is associated with multiple health problems. A tumor must be at least 0.5 cm large before it is detectable by current screening methods and may already have metastasized by that time. Oncofetal antigens such as CEA may be used to monitor therapy or detect tumor reoccurrence, but are not helpful in screening for cancer. The seven warning signs of cancer are actually associated with fairly advanced disease

After change-of-shift report on the oncology unit, which patient should the nurse assess first? a. Patient who has a platelet count of 82,000/µL after chemotherapy b. Patient who has xerostomia after receiving head and neck radiation c. Patient who is neutropenic and has a temperature of 100.5° F (38.1° C) d. Patient who is worried about getting the prescribed long-acting opioid on time

ANS: C Temperature elevation is an emergency in neutropenic patients because of the risk for rapid progression to severe infections and sepsis. The other patients also require assessments or interventions, but do not need to be assessed as urgently. Patients with thrombocytopenia do not have spontaneous bleeding until the platelets are 20,000/µL. Xerostomia does not require immediate intervention. Although breakthrough pain needs to be addressed rapidly, the patient does not appear to have breakthrough pain.

A 22-year-old patient seen at the health clinic with a severe migraine headache tells the nurse about having other similar headaches recently. Which initial action should the nurse take? a. Teach about the use of triptan drugs. b. Refer the patient for stress counseling. c. Ask the patient to keep a headache diary. d. Suggest the use of muscle-relaxation techniques.

ANS: C The initial nursing action should be further assessment of the precipitating causes of the headaches, quality, and location of pain, etc. Stress reduction, muscle relaxation, and the triptan drugs may be helpful, but more assessment is needed first. DIF: Cognitive Level: Apply (application) REF: 1419 OBJ: Special Questions: Prioritization TOP: Nursing Process: Implementation MSC: NCLEX: Physiological Integrity

A patient who is scheduled for a right breast biopsy asks the nurse the difference between a benign tumor and a malignant tumor. Which answer by the nurse is correct? a. "Benign tumors do not cause damage to other tissues." b. "Benign tumors are likely to recur in the same location." c. "Malignant tumors may spread to other tissues or organs." d. "Malignant cells reproduce more rapidly than normal cells."

ANS: C The major difference between benign and malignant tumors is that malignant tumors invade adjacent tissues and spread to distant tissues and benign tumors never metastasize. The other statements are inaccurate. Both types of tumors may cause damage to adjacent tissues. Malignant cells do not reproduce more rapidly than normal cells. Benign tumors do not usually recur.

During a routine health examination, a 40-year-old patient tells the nurse about a family history of colon cancer. Which action should the nurse take next? a. Teach the patient about the need for a colonoscopy at age 50. b. Teach the patient how to do home testing for fecal occult blood. c. Obtain more information from the patient about the family history. d. Schedule a sigmoidoscopy to provide baseline data about the patient.

ANS: C The patient may be at increased risk for colon cancer, but the nurse's first action should be further assessment. The other actions may be appropriate, depending on the information that is obtained from the patient with further questioning.

During the teaching session for a patient who has a new diagnosis of acute leukemia the patient is restless and is looking away, never making eye contact. After teaching about the complications associated with chemotherapy, the patient asks the nurse to repeat all of the information. Based on this assessment, which nursing diagnosis is most appropriate for the patient? a. Risk for ineffective adherence to treatment related to denial of need for chemotherapy b. Acute confusion related to infiltration of leukemia cells into the central nervous system c. Risk for ineffective health maintenance related to anxiety about new leukemia diagnosis d. Deficient knowledge: chemotherapy related to a lack of interest in learning about treatment

ANS: C The patient who has a new cancer diagnosis is likely to have high anxiety, which may impact learning and require that the nurse repeat and reinforce information. The patient's history of a recent diagnosis suggests that infiltration of the leukemia is not a likely cause of the confusion. The patient asks for the information to be repeated, indicating that lack of interest in learning and denial are not etiologic factors.

A patient with cancer has a nursing diagnosis of imbalanced nutrition: less than body requirements related to altered taste sensation. Which nursing action is most appropriate? a. Add strained baby meats to foods such as casseroles. b. Teach the patient about foods that are high in nutrition. c. Avoid giving the patient foods that are strongly disliked. d. Add extra spice to enhance the flavor of foods that are served.

ANS: C The patient will eat more if disliked foods are avoided and foods that the patient likes are included instead. Additional spice is not usually an effective way to enhance taste. Adding baby meats to foods will increase calorie and protein levels, but does not address the issue of taste. The patient's poor intake is not caused by a lack of information about nutrition

A patient with metastatic cancer of the colon experiences severe vomiting following each administration of chemotherapy. Which action, if taken by the nurse, is most appropriate? a. Have the patient eat large meals when nausea is not present. b. Offer dry crackers and carbonated fluids during chemotherapy. c. Administer prescribed antiemetics 1 hour before the treatments. d. Give the patient two ounces of a citrus fruit beverage during treatments.

ANS: C Treatment with antiemetics before chemotherapy may help prevent nausea. The patient should eat small, frequent meals. Offering food and beverages during chemotherapy is likely to cause nausea. The acidity of citrus fruits may be further irritating to the stomach.

The charge nurse is observing a new staff nurse who is assessing a patient with a traumatic spinal cord injury for sensation. Which action indicates a need for further teaching of the new nurse about neurologic assessment? a. The new nurse tests for light touch before testing for pain. b. The new nurse has the patient close the eyes during testing. c. The new nurse asks the patient if the instrument feels sharp. d. The new nurse uses an irregular pattern to test for intact touch

ANS: C When performing a sensory assessment, the nurse should not provide verbal clues. The other actions by the new nurse are appropriate. DIF: Cognitive Level: Apply (application) REF: 1328 OBJ: Special Questions: Delegation TOP: Nursing Process: Evaluation MSC: NCLEX: Safe and Effective Care Environment

A patient with a large stomach tumor that is attached to the liver is scheduled to have a debulking procedure. Which information should the nurse teach the patient about the outcome of this procedure? a. Pain will be relieved by cutting sensory nerves in the stomach. b. Relief of pressure in the stomach will promote better nutrition. c. Tumor growth will be controlled by the removal of malignant tissue. d. Tumor size will decrease and this will improve the effects of other therapy.

ANS: D A debulking surgery reduces the size of the tumor and makes radiation and chemotherapy more effective. Debulking surgeries do not control tumor growth. The tumor is debulked because it is attached to the liver, a vital organ (not to relieve pressure on the stomach). Debulking does not sever the sensory nerves, although pain may be lessened by the reduction in pressure on the abdominal organs.

Which medication taken by a patient with restless legs syndrome should the nurse discuss with the patient? a. Multivitamin (Stresstabs) b. Acetaminophen (Tylenol) c. Ibuprofen (Motrin, Advil) d. Diphenhydramine (Benadryl)

ANS: D Antihistamines can aggravate restless legs syndrome. The other medications will not contribute to restless legs syndrome. DIF: Cognitive Level: Apply (application) REF: 1427 TOP: Nursing Process: Implementation MSC: NCLEX: Physiological Integrity

A patient has been assigned the nursing diagnosis of imbalanced nutrition: less than body requirements related to painful oral ulcers. Which nursing action will be most effective in improving oral intake? a. Offer the patient frequent small snacks between meals. b. Assist the patient to choose favorite foods from the menu. c. Provide teaching about the importance of nutritional intake. d. Apply the ordered anesthetic gel to oral lesions before meals.

ANS: D Because the etiology of the patient's poor nutrition is the painful oral ulcers, the best intervention is to apply anesthetic gel to the lesions before the patient eats. The other actions might be helpful for other patients with impaired nutrition, but would not be as helpful for this patient.

Which information about a 60-year-old patient with MS indicates that the nurse should consult with the health care provider before giving the prescribed dose of dalfampridine (Ampyra)? a. The patient has relapsing-remitting MS. b. The patient walks a mile a day for exercise. c. The patient complains of pain with neck flexion. d. The patient has an increased serum creatinine level.

ANS: D Dalfampridine should not be given to patients with impaired renal function. The other information will not impact whether the dalfampridine should be administered. DIF: Cognitive Level: Apply (application) REF: 1431 TOP: Nursing Process: Assessment MSC: NCLEX: Physiological Integrity

A 76-year-old patient is being treated with carbidopa/levodopa (Sinemet) for Parkinson's disease. Which information is most important for the nurse to report to the health care provider? a. Shuffling gait b. Tremor at rest c. Cogwheel rigidity of limbs d. Uncontrolled head movement

ANS: D Dyskinesia is an adverse effect of the Sinemet, indicating a need for a change in medication or decrease in dose. The other findings are typical with Parkinson's disease. DIF: Cognitive Level: Apply (application) REF: 1435 OBJ: Special Questions: Prioritization TOP: Nursing Process: Planning MSC: NCLEX: Physiological Integrity

A patient with suspected meningitis is scheduled for a lumbar puncture. Before the procedure, the nurse will plan to a. enforce NPO status for 4 hours. b. transfer the patient to radiology. c. administer a sedative medication. d. help the patient to a lateral position.

ANS: D For a lumbar puncture, the patient lies in the lateral recumbent position. The procedure does not usually require a sedative, is done in the patient room, and has no risk for aspiration. DIF: Cognitive Level: Apply (application) REF: 1352 TOP: Nursing Process: Planning MSC: NCLEX: Physiological Integrity

A 62-year-old patient who has Parkinson's disease is taking bromocriptine (Parlodel). Which information obtained by the nurse may indicate a need for a decrease in the dose? a. The patient has a chronic dry cough. b. The patient has four loose stools in a day. c. The patient develops a deep vein thrombosis. d. The patient's blood pressure is 92/52 mm Hg.

ANS: D Hypotension is an adverse effect of bromocriptine, and the nurse should check with the health care provider before giving the medication. Diarrhea, cough, and deep vein thrombosis are not associated with bromocriptine use. DIF: Cognitive Level: Apply (application) REF: 1435 TOP: Nursing Process: Evaluation MSC: NCLEX: Physiological Integrity

Which information about a 72-year-old patient who has a new prescription for phenytoin (Dilantin) indicates that the nurse should consult with the health care provider before administration of the medication? a. Patient has generalized tonic-clonic seizures. b. Patient experiences an aura before seizures. c. Patient's most recent blood pressure is 156/92 mm Hg. d. Patient has minor elevations in the liver function tests.

ANS: D Many older patients (especially with compromised liver function) may not be able to metabolize phenytoin. The health care provider may need to choose another antiseizure medication. Phenytoin is an appropriate medication for patients with tonic-clonic seizures, with or without an aura. Hypertension is not a contraindication for phenytoin therapy. DIF: Cognitive Level: Apply (application) REF: 1424 TOP: Nursing Process: Implementation MSC: NCLEX: Physiological Integrity

The nurse is caring for a patient receiving intravesical bladder chemotherapy. The nurse should monitor for which adverse effect? a. Nausea b. Alopecia c. Mucositis d. Hematuria

ANS: D The adverse effects of intravesical chemotherapy are confined to the bladder. The other adverse effects are associated with systemic chemotherapy.

hich nursing diagnosis is of highest priority for a patient with Parkinson's disease who is unable to move the facial muscles? a. Activity intolerance b. Self-care deficit: toileting c. Ineffective self-health management d. Imbalanced nutrition: less than body requirements

ANS: D The data about the patient indicate that poor nutrition will be a concern because of decreased swallowing. The other diagnoses may also be appropriate for a patient with Parkinson's disease, but the data do not indicate that they are current problems for this patient. DIF: Cognitive Level: Apply (application) REF: 1436 OBJ: Special Questions: Prioritization TOP: Nursing Process: Analysis MSC: NCLEX: Physiological Integrity

A patient who has ovarian cancer is crying and tells the nurse, "My husband rarely visits. He just doesn't care." The husband indicates to the nurse that he never knows what to say to help his wife. Which nursing diagnosis is most appropriate for the nurse to add to the plan of care? a. Compromised family coping related to disruption in lifestyle b. Impaired home maintenance related to perceived role changes c. Risk for caregiver role strain related to burdens of caregiving responsibilities d. Dysfunctional family processes related to effect of illness on family members

ANS: D The data indicate that this diagnosis is most appropriate because poor communication among the family members is affecting family processes. No data suggest a change in lifestyle or its role as an etiology. The data do not support impairment in home maintenance or a burden caused by caregiving responsibilities.

The nurse reviews the laboratory results of a patient who is receiving chemotherapy. Which laboratory result is most important to report to the health care provider? a. Hematocrit of 30% b. Platelets of 95,000/µL c. Hemoglobin of 10 g/L d. White blood cell (WBC) count of 2700/µL

ANS: D The low WBC count places the patient at risk for severe infection and is an indication that the chemotherapy dose may need to be lower or that WBC growth factors such as filgrastim (Neupogen) are needed. Although the other laboratory data indicate decreased levels, they do not indicate any immediate life-threatening adverse effects of the chemotherapy.

A patient with leukemia is considering whether to have hematopoietic stem cell transplantation (HSCT). The nurse will include which information in the patient's teaching plan? a. Transplant of the donated cells is painful because of the nerves in the tissue lining the bone. b. Donor bone marrow cells are transplanted through an incision into the sternum or hip bone. c. The transplant procedure takes place in a sterile operating room to minimize the risk for infection. d. Hospitalization will be required for several weeks after the stem cell transplant procedure is performed.

ANS: D The patient requires strict protective isolation to prevent infection for 2 to 4 weeks after HSCT while waiting for the transplanted marrow to start producing cells. The transplanted cells are infused through an IV line, so the transplant is not painful, nor is an operating room or incision required.

A patient receiving head and neck radiation for larynx cancer has ulcerations over the oral mucosa and tongue and thick, ropey saliva. Which instructions should the nurse give to this patient? a. Remove food debris from the teeth and oral mucosa with a stiff toothbrush. b. Use cotton-tipped applicators dipped in hydrogen peroxide to clean the teeth. c. Gargle and rinse the mouth several times a day with an antiseptic mouthwash. d. Rinse the mouth before and after each meal and at bedtime with a saline solution.

ANS: D The patient should rinse the mouth with a saline solution frequently. A soft toothbrush is used for oral care. Hydrogen peroxide may damage tissues. Antiseptic mouthwashes may irritate the oral mucosa and are not recommended.

The nurse performing a focused assessment of left posterior temporal lobe functions will assess the patient for a. sensation on the left side of the body. b. voluntary movements on the right side. c. reasoning and problem-solving abilities. d. understanding written and oral language.

ANS: D The posterior temporal lobe integrates the visual and auditory input for language comprehension. Reasoning and problem solving are functions of the anterior frontal lobe. Sensation on the left side of the body is located in the right postcentral gyrus. Voluntary movement on the right side is controlled in the left precentral gyrus. DIF: Cognitive Level: Apply (application) REF: 1339 TOP: Nursing Process: Assessment MSC: NCLEX: Physiological Integrity

The nurse teaches a postmenopausal patient with stage III breast cancer about the expected outcomes of cancer treatment. Which patient statement indicates that the teaching has been effective? a. "After cancer has not recurred for 5 years, it is considered cured." b. "The cancer will be cured if the entire tumor is surgically removed." c. "Cancer is never considered cured, but the tumor can be controlled with surgery, chemotherapy, and radiation." d. "I will need to have follow-up examinations for many years after I have treatment before I can be considered cured."

ANS: D The risk of recurrence varies by the type of cancer. Some cancers are considered cured after a shorter time span or after surgery, but stage III breast cancer will require additional therapies and ongoing follow-up.

Which prescribed intervention will the nurse implement first for a patient in the emergency department who is experiencing continuous tonic-clonic seizures? a. Give phenytoin (Dilantin) 100 mg IV. b. Monitor level of consciousness (LOC). c. Obtain computed tomography (CT) scan. d. Administer lorazepam (Ativan) 4 mg IV.

ANS: D To prevent ongoing seizures, the nurse should administer rapidly acting antiseizure medications such as the benzodiazepines. A CT scan is appropriate, but prevention of any seizure activity during the CT scan is necessary. Phenytoin will also be administered, but it is not rapidly acting. Patients who are experiencing tonic-clonic seizures are nonresponsive, although the nurse should assess LOC after the seizure. DIF: Cognitive Level: Apply (application) REF: 1424 OBJ: Special Questions: Prioritization TOP: Nursing Process: Implementation MSC: NCLEX: Physiological Integrity

Which action should the nurse take when caring for a patient who is receiving chemotherapy and complains of problems with concentration? a. Teach the patient to rest the brain by avoiding new activities. b. Teach that "chemo-brain" is a short-term effect of chemotherapy. c. Report patient symptoms immediately to the health care provider. d. Suggest use of a daily planner and encourage adequate rest and sleep.

ANS: D Use of tools to enhance memory and concentration such as a daily planner and adequate rest are helpful for patients who develop "chemo-brain" while receiving chemotherapy. Patients should be encouraged to exercise the brain through new activities. Chemo-brain may be short- or long-term. There is no urgent need to report common chemotherapy side effects to the provider.

The priority nursing assessment for a 72-year-old patient being admitted with a brainstem infarction is a. reflex reaction time. b. pupil reaction to light. c. level of consciousness. d. respiratory rate and rhythm.

ANS: D Vital centers that control respiration are located in the medulla, and these are the priority assessments because changes in respiratory function may be life threatening. The other information will also be collected by the nurse, but it is not as urgent. DIF: Cognitive Level: Apply (application) REF: 1339 OBJ: Special Questions: Prioritization TOP: Nursing Process: Assessment MSC: NCLEX: Physiological Integrity

Which laboratory result will the nurse expect to show a decreased value if a patient develops heparin-induced thrombocytopenia (HIT)? a. Prothrombin time b. Erythrocyte count c. Fibrinogen degradation products d. Activated partial thromboplastin time

Activated partial thromboplastin time Platelet aggregation in HIT causes neutralization of heparin, so that the activated partial thromboplastin time will be shorter and more heparin will be needed to maintain therapeutic levels. The other data will not be affected by HIT.

The nurse notes that a patient who was admitted with diabetic ketoacidosis has rapid, deep respirations. Which action should the nurse take? a. Give the prescribed PRN lorazepam (Ativan). b. Start the prescribed PRN oxygen at 2 to 4 L/min. c. Administer the prescribed normal saline bolus and insulin. d. Encourage the patient to take deep, slow breaths with guided imagery.

Administer the prescribed normal saline bolus and insulin.

Which action for a patient with neutropenia is appropriate for the registered nurse (RN) to delegate to a licensed practical/vocational nurse (LPN/LVN)? a. Assessing the patient for signs and symptoms of infection b. Teaching the patient the purpose of neutropenic precautions c. Administering subcutaneous filgrastim (Neupogen) injection d. Developing a discharge teaching plan for the patient and family

Administering subcutaneous filgrastim (Neupogen) injection Administration of subcutaneous medications is included in LPN/LVN education and scope of practice. Patient education, assessment, and developing the plan of care require RN level education and scope of practice.

A patient with acute dyspnea is scheduled for a spiral computed tomography (CT) scan. Which information obtained by the nurse is a priority to communicate to the health care provider before the CT?

Allergy to shellfish Because iodine-based contrast media is used during a spiral CT, the patient may need to have the CT scan without contrast or be premedicated before injection of the contrast media.

What question should the nurse ask while assessing the nutritional-metabolic health pattern in a patient suspected to have lung cancer? 1 "Do you feel tired often?" 2 "Do you have any chest pain?" 3 "Do you have difficulty breathing?" 4 "Do you have difficulty swallowing?"

Answer4

A patient is diagnosed with a hamartoma and asks the nurse what type of tumor it is. What is the best response by the nurse? 1 It is a slow-growing congenital tumor. 2 It is a benign tumor arising in the bronchi. 3 It is a tumor that originates from the visceral pleura. 4 It is a tumor that consists of columnar cystic spaces

Answer: 1

A patient has clinical manifestations that correlate with a diagnosis of lung cancer. For what test does the nurse initially prepare the patient? 1 Biopsy 2 Chest x-ray 3 Computed tomography 4 Sputum cytology studies

Answer: 2

A patient with lung cancer scheduled for a surgical resection received radiation therapy preoperatively. About which outcome does the nurse educate the patient after the radiation therapy? 1 Reduction in the tumor mass 2 Relief of dyspnea and hemoptysis 3 Removal of lesions obstructing the airway 4 Delay extension of the tumor into the airway lumen

Answer: 2

During the treatment of a patient with lung cancer, the primary health care provider asks the nurse to administer porfimer intravenously. For which procedure does the nurse prepare the patient? 1 Airway stenting 2 Photodynamic therapy 3 Bronchoscopic laser therapy 4 Prophylactic cranial radiation

Answer: 2

The nurse who is caring for a patient with lung cancer anticipates that the tumor has spread to the mediastinum. Which finding helps the nurse to reach this conclusion? 1 Presence of dysphagia 2 Presence of pericardial effusion 3 Obstruction of the superior vena cava 4 Unilateral paralysis of the diaphragm

Answer: 2

What is the characteristic feature of stage IIIA lung cancer? 1 Tumor involving the heart 2 Tumor involving the pleura 3 Tumor involving the bronchus 4 Tumor involving the contralateral lymph nodes

Answer: 2

When planning appropriate nursing interventions for a patient with metastatic lung cancer and a 60-pack-per-year history of cigarette smoking, the nurse recognizes that the smoking most likely has decreased the patient's underlying respiratory defenses because of impairment of what? 1 Cough reflex 2 Mucociliary clearance 3 Reflex bronchoconstriction 4 Ability to filter particles from the air

Answer: 2

Which type of primary lung cancer presents with a very rapid growth of cancer cells? 1 Adenocarcinoma 2 Small cell carcinoma 3 Large cell carcinoma 4 Squamous cell carcinoma

Answer: 2

A patient is suspected to have stage IIA lung cancer. About what diagnostic tests will the nurse educate the patient that will be used to confirm the diagnosis? Select all that apply. 1 Chest x-ray 2 Computed tomography 3 Sputum cytologic studies 4 Magnetic resonance imaging 5 Positron emission tomography

Answer: 2,4,5

A nurse preparing educational information about lung cancer notes that what is the primary risk factor related to the development of lung cancer? 1 Genetics 2 Chewing tobacco 3 Cigarette smoking 4 Occupational exposure

Answer: 3

A patient has paraneoplastic syndrome. What type of lung cancer does the patient have? 1 Adenocarcinoma 2 Large cell carcinoma 3 Small cell carcinoma 4 Squamous cell carcinoma

Answer: 3

The nurse is caring for a patient with lung cancer with a tumor that has spread to the regional lymph nodes. Which stage of non-small cell lung cancer is detected in the patient? 1 Stage IA 2 Stage IIB 3 Stage IIIA 4 Stage IVA

Answer: 3

A 71-year-old patient with metastatic lung cancer has a decrease in oxygen saturation to 87% while completing activities of daily living. What is the priority nursing action? 1 Maintain strict bed rest 2 Administer a bronchodilator 3 Obtain a stat arterial blood gas 4 Apply supplemental oxygen with increased activity levels as needed (PRN)

Answer: 4

A patient with lung cancer is suspected to have mediastinal involvement. For what clinical manifestation does the nurse assess in this patient? 1 Dyspnea 2 Dysphagia 3 Pleural effusion 4 Pericardial effusion

Answer: 4

The patient had video-assisted thoracic surgery (VATS) to perform a lobectomy. What does the nurse know is the reason for using this type of surgery? 1 The patient has lung cancer. 2 The incision will be medial sternal or lateral. 3 Chest tubes will not be needed postoperatively. 4 Less discomfort and faster return to normal activity

Answer: 4

Which management technique will the interprofessional care team implement for a patient with lung cancer?

Answer: radiation

The nurse is assessing a patient who has been found to have a diffuse infiltrate on a chest x-ray. The patient states, "I am afraid that I have cancer." The nurse will assess the patient for which most common symptom of lung cancer? 1 Hoarseness 2 Hypercalcemia 3 Persistent cough 4 Anorexia and weight loss

Answer:3

The nurse is caring for an older frail adult patient with stage II non-small cell lung cancer who may not be a candidate for a surgical procedure. About what alternative should the nurse educate the patient? 1 Airway stenting 2 Chemotherapy 3 Photodynamic therapy 4 Radiofrequency ablation

Answer:4

The nurse is collecting the data of a patient with lung cancer. What findings should the nurse assess while observing cognitive-perceptual health patterns?

Answer:Bone pain

After the complete assessment of a patient with lung cancer, the nurse finds that the patient has edema of neck and face. Which complication does the nurse expect in the patient?

Answer:Superior vena cava syndrome

A 42-year-old male patient has had a herniorrhaphy to repair an incarcerated inguinal hernia. Which patient teaching will the nurse provide before discharge? a. Soak in sitz baths several times each day. b. Cough 5 times each hour for the next 48 hours. c. Avoid use of acetaminophen (Tylenol) for pain. d. Apply a scrotal support and ice to reduce swelling.

Apply a scrotal support and ice to reduce swelling. A scrotal support and ice are used to reduce edema and pain. Coughing will increase pressure on the incision. Sitz baths will not relieve pain and would not be of use after this surgery. Acetaminophen can be used for postoperative pain.

A patient has been assigned the nursing diagnosis of imbalanced nutrition: less than body requirements related to painful oral ulcers. Which nursing action will be most effective in improving oral intake? a. Offer the patient frequent small snacks between meals. b. Assist the patient to choose favorite foods from the menu. c. Provide teaching about the importance of nutritional intake. d. Apply the ordered anesthetic gel to oral lesions before meals.

Apply the ordered anesthetic gel to oral lesions before meals.

Which care activity for a patient with a paralytic ileus is appropriate for the registered nurse (RN) to delegate to unlicensed assistive personnel (UAP)? a. Auscultation for bowel sounds b. Nasogastric (NG) tube irrigation c. Applying petroleum jelly to the lips d. Assessment of the nares for irritation

Applying petroleum jelly to the lips UAP education and scope of practice include patient hygiene such as oral care. The other actions require education and scope of practice appropriate to the RN.

A patient has acute bronchitis with a nonproductive cough and wheezes. Which topic should the nurse plan to include in the teaching plan?

Appropriate use of cough suppressants Cough suppressants are frequently prescribed for acute bronchitis. Because most acute bronchitis is viral in origin, antibiotics are not prescribed unless there are systemic symptoms

A patient who is taking a potassium-wasting diuretic for treatment of hypertension complains of generalized weakness. It is most appropriate for the nurse to take which action? a. Assess for facial muscle spasms. b. Ask the patient about loose stools. c. Suggest that the patient avoid orange juice with meals. d. Ask the health care provider to order a basic metabolic panel.

Ask the health care provider to order a basic metabolic panel.

A 58-year-old man with erectile dysfunction (ED) tells the nurse he is interested in using sildenafil (Viagra). Which action should the nurse take first? a. Assure the patient that ED is common with aging. b. Ask the patient about any prescription drugs he is taking. c. Tell the patient that Viagra does not always work for ED. d. Discuss the common adverse effects of erectogenic drugs.

Ask the patient about any prescription drugs he is taking. Because some medications can cause ED and patients using nitrates should not take sildenafil, the nurse should first assess for prescription drug use. The nurse may want to teach the patient about realistic expectations and adverse effects of sildenafil therapy, but this should not be the first action. Although ED does increase with aging, it may be secondary to medication use or cardiovascular disease.

After a total proctocolectomy and permanent ileostomy, the patient tells the nurse, "I cannot manage all these changes. I don't want to look at the stoma." What is the best action by the nurse? a. Reassure the patient that ileostomy care will become easier. b. Ask the patient about the concerns with stoma management. c. Develop a detailed written list of ostomy care tasks for the patient. d. Postpone any teaching until the patient adjusts to the ileostomy.

Ask the patient about the concerns with stoma management. Encouraging the patient to share concerns assists in helping the patient adjust to the body changes. Acknowledgment of the patient's feelings and concerns is important rather than offering false reassurance. Because the patient indicates that the feelings about the ostomy are the reason for the difficulty with the many changes, development of a detailed ostomy care plan will not improve the patient's ability to manage the ostomy. Although detailed ostomy teaching may be postponed, the nurse should offer teaching about some aspects of living with an ostomy.

A 27-year-old man who has testicular cancer is being admitted for a unilateral orchiectomy. The patient does not talk to his wife and speaks to the nurse only to answer the admission questions. Which action is best for the nurse to take? a. Teach the patient and the wife that impotence is unlikely after unilateral orchiectomy. b. Ask the patient if he has any questions or concerns about the diagnosis and treatment. c. Document the patient's lack of communication on the chart and continue preoperative care. d. Inform the patient's wife that concerns about sexual function are common with this diagnosis.

Ask the patient if he has any questions or concerns about the diagnosis and treatment. The initial action by the nurse should be assessment for any anxiety or questions about the surgery or postoperative care. The nurse should address the patient, not the spouse, when discussing the diagnosis and any possible concerns. Without further assessment of patient concerns, the nurse should not offer teaching about complications after orchiectomy. Documentation of the patient's lack of interaction is not an adequate nursing action in this situation.

A 25-year-old male patient calls the clinic complaining of diarrhea for 24 hours. Which action should the nurse take first? a. Inform the patient that laboratory testing of blood and stools will be necessary. b. Ask the patient to describe the character of the stools and any associated symptoms. c. Suggest that the patient drink clear liquid fluids with electrolytes, such as Gatorade or Pedialyte. d. Advise the patient to use over-the-counter loperamide (Imodium) to slow gastrointestinal (GI) motility.

Ask the patient to describe the character of the stools and any associated symptoms. The initial response by the nurse should be further assessment of the patient. The other responses may be appropriate, depending on what is learned in the assessment.

When assessing a patient with chronic obstructive pulmonary disease (COPD), the nurse finds a new onset of agitation and confusion. Which action should the nurse take first? a. Notify the health care provider. b. Check pupils for reaction to light. c. Attempt to calm and reorient the patient. d. Assess oxygenation using pulse oximetry.

Assess oxygenation using pulse oximetry.

A patient with a tracheostomy has a new order for a fenestrated tracheostomy tube. Which action should the nurse include in the plan of care in collaboration with the speech therapist?

Assess the ability to swallow before using the fenestrated tube. Because the cuff is deflated when using a fenestrated tube, the patient's risk for aspiration should be assessed before changing to a fenestrated tracheostomy tube.

A 71-year-old male patient tells the nurse that growing old causes constipation so he has been using a suppository for constipation every morning. Which action should the nurse take first? a. Encourage the patient to increase oral fluid intake. b. Assess the patient about risk factors for constipation. c. Suggest that the patient increase intake of high-fiber foods. d. Teach the patient that a daily bowel movement is unnecessary.

Assess the patient about risk factors for constipation. The nurse's initial action should be further assessment of the patient for risk factors for constipation and for his usual bowel pattern. The other actions may be appropriate but will be based on the assessment.

A 71-year-old patient had an abdominal-perineal resection for colon cancer. Which nursing action is most important to include in the plan of care for the day after surgery? a. Teach about a low-residue diet. b. Monitor output from the stoma. c. Assess the perineal drainage and incision. d. Encourage acceptance of the colostomy stoma.

Assess the perineal drainage and incision. Because the perineal wound is at high risk for infection, the initial care is focused on assessment and care of this wound. Teaching about diet is best done closer to discharge from the hospital. There will be very little drainage into the colostomy until peristalsis returns. The patient will be encouraged to assist with the colostomy, but this is not the highest priority in the immediate postoperative period.

A newly admitted patient is diagnosed with hyponatremia. When making room assignments, the charge nurse should take which action? a. Assign the patient to a room near the nurse's station. b. Place the patient in a room nearest to the water fountain. c. Place the patient on telemetry to monitor for peaked T waves. d. Assign the patient to a semi-private room and place an order for a low-salt diet.

Assign the patient to a room near the nurse's station.

A patient with bacterial pneumonia has rhonchi and thick sputum. What is the nurse's most appropriate action to promote airway clearance? a. Assist the patient to splint the chest when coughing. b. Teach the patient about the need for fluid restrictions. c. Encourage the patient to wear the nasal oxygen cannula. d. Instruct the patient on the pursed lip breathing technique.

Assist the patient to splint the chest when coughing.

The oxygen saturation (SpO2) for a patient with left lower lobe pneumonia is 90%. The patient has rhonchi, a weak cough effort, and complains of fatigue. Which action is a priority for the nurse to take? a. Position the patient on the left side. b. Assist the patient with staged coughing. c. Place a humidifier in the patient's room. d. Schedule a 2-hour rest period for the patient.

Assist the patient with staged coughing.

Following a laryngectomy a patient coughs violently during suctioning and dislodges the tracheostomy tube. Which action should the nurse take first?

Attempt to reinsert the tracheostomy tube with the obturator in place The first action should be to attempt to reinsert the tracheostomy tube to maintain the patient's airway

Which action should the nurse take first when a patient complains of acute chest pain and dyspnea soon after insertion of a centrally inserted IV catheter? a. Notify the health care provider. b. Offer reassurance to the patient. c. Auscultate the patient's breath sounds. d. Give the prescribed PRN morphine sulfate IV.

Auscultate the patient's breath sounds.

Which intervention will be included in the nursing care plan for a patient with immune thrombocytopenic purpura (ITP)? a. Assign the patient to a private room. b. Avoid intramuscular (IM) injections. c. Use rinses rather than a soft toothbrush for oral care. d. Restrict activity to passive and active range of motion.

Avoid intramuscular (IM) injections. IM or subcutaneous injections should be avoided because of the risk for bleeding. A soft toothbrush can be used for oral care. There is no need to restrict activity or place the patient in a private room.

What can the nurse do to facilitate cancer prevention for the patient in the promotion stage of cancer development? A. Teach the patient to exercise daily. B. Teach the patient promoting factors to avoid. C. Tell the patient to have the cancer surgically removed now. D. Teach the patient which vitamins will improve the immune system.

B. Teach the patient promoting factors to avoid. The promotion stage of cancer is characterized by the reversible proliferation of the altered cells. Changing the lifestyle to avoid promoting factors (dietary fat, obesity, cigarette smoking, and alcohol consumption) can reduce the chance of cancer development. Daily exercise and vitamins alone will not prevent cancer. Surgery at this stage may not be possible without a critical mass of cells, and this advice would not be the nurse's role.

Cancer Survivorship

Be aware of late and long-term effects of cancer: Secondary cancer Cognitive changes Cardiovascular/sexual dysfunction Psychosocial effects Nurses can help cancer survivors. Educate health care workers about needs of cancer survivors. Teach survivors to look for late effects. Promote healthy behaviors. Good nutrition Regular exercise Appropriate health screening Encourage survivors to seek any help they need.

Support patient and family by

Being available, especially during difficult times Exhibiting a caring attitude Listening actively to fears and concerns Providing relief from distressing symptoms Providing information about cancer Maintaining relationship based on trust and confidence Being honest Using touch to exhibit caring Assisting patient in setting realistic short-term and long-term goals Assisting in maintaining usual lifestyle Maintaining hope

Procedures done with Fiberoptic Bronchoscopy

Biopsies Bronchoalveolar lavage Therapeutic- removal of mucus plugs

When necessary to do a percutaneous needle biopsy

Biopsy of peripheral lung CT guidance and usually lesions>1cm 90-95% specificity for malignant

The nurse teaches a patient with cancer of the liver about high-protein, high-calorie diet choices. Which snack choice by the patient indicates that the teaching has been effective? a. Lime sherbet b. Blueberry yogurt c. Cream cheese bagel d. Fresh strawberries and bananas

Blueberry yogurt

The nurse is caring for a patient receiving an initial dose of chemotherapy to treat a rapidly growing metastatic colon cancer. The nurse is aware that this patient is at risk for tumor lysis syndrome (TLS) and will monitor the patient closely for which abnormality associated with this oncologic emergency? a. hypokalemia b. hypouricemia c. hypocalcemia d. hypophosphatemia

C. Rationale: TLS is a metabolic complication characterized by rapid release of intracellular components in response to chemotherapy. This can rapidly lead to acute renal injury. The hallmark signs of TLS are hyperuricemia, hyperphosphatemia, hyperkalemia, and hypocalcemia.

A 31-year-old man who works for a moving company presents to the ED because he thought he was having a heart attack. He does not smoke, and jogs 3 days a week. His father died of a heart attack in his sixties. He describes a gradual onset of chest pain that is worse with activity and resolves when he is at rest. His HR is 68 beats per minute, BP is 120/70 mm Hg, and RR is 14 breaths per minute. On examination, his lungs are clear and there is no cardiac murmur. You palpate tenderness over the left sternal border at the third and fourth rib. An ECG reveals sinus rhythm at a rate of 65. A chest radiograph shows no infiltrates or pneumothorax. Which of the following is the most appropriate next step in management? a. Administer aspirin and send for a troponin. b. Administer aspirin, clopidogrel, and heparin and admit for acute coronary syndrome (ACS). c. Administer ibuprofen and reassure the patient that he is not having a heart attack. d. Inject corticosteroid into the costochondral joint to reduce inflammation. e. Observe the patient for 6 hours.

C. The patient has costochondritis, an inflammatory process of the costochondral or costosternal joints that causes localized pain and tenderness. Any of the seven costochondral junctions may be affected, and more than one site is affected in 90% of cases. The second to fifth costochondral junctions are most commonly involved. In contrast to myocardial ischemia or infarction, costochondritis is a benign cause of chest pain, often with an insidious onset, and is an important consideration in the differential diagnosis for chest pain. Of note, 5% to 7% of patients with cardiac ischemia also have chest-wall tenderness. The onset is often insidious. Chest-wall pain with a history of repeated minor trauma or unaccustomed activity (eg, painting, moving furniture) is common. The goal of therapy is to reduce inflammation. NSAIDs are typically prescribed

The health care provider prescribes the following interventions for a patient with acute prostatitis caused by E. coli. Which intervention should the nurse question? a. Instruct patient to avoid sexual intercourse until treatment is complete. b. Administer ibuprofen (Advil) 400 mg every 8 hours as needed for pain. c. Catheterize the patient as needed if symptoms of urinary retention develop. d. Give trimethoprim/sulfamethoxazole (Bactrim) DS 1 tablet daily for 28 days.

Catheterize the patient as needed if symptoms of urinary retention develop. Although acute urinary retention may occur, insertion of a catheter through an inflamed urethra is contraindicated and the nurse will anticipate that the health care provider will need to insert a suprapubic catheter. The other actions are appropriate

To evaluate the effectiveness of the pantoprazole (Protonix) ordered for a patient with systemic inflammatory response syndrome (SIRS), which assessment will the nurse perform? a. Auscultate bowel sounds. b. Palpate for abdominal pain. c. Ask the patient about nausea. d. Check stools for occult blood.

Check stools for occult blood. Proton pump inhibitors are given to decrease the risk for stress ulcers in critically ill patients. The other assessments also will be done, but these will not help in determining the effectiveness of the pantoprazole administration.

Which action will the admitting nurse include in the care plan for a 30-year old woman who is neutropenic? a. Avoid any injections. b. Check temperature every 4 hours. c. Omit fruits or vegetables from the diet. d. Place a "No Visitors" sign on the door.

Check temperature every 4 hours. The earliest sign of infection in a neutropenic patient is an elevation in temperature. Although unpeeled fresh fruits and vegetables should be avoided, fruits and vegetables that are peeled or cooked are acceptable. Injections may be required for administration of medications such as filgrastim (Neupogen). The number of visitors may be limited and visitors with communicable diseases should be avoided, but a "no visitors" policy is not needed.

A patient comes to the clinic complaining of frequent, watery stools for the last 2 days. Which action should the nurse take first? a. Obtain the baseline weight. b. Check the patient's blood pressure. c. Draw blood for serum electrolyte levels. d. Ask about any extremity numbness or tingling.

Check the patient's blood pressure.

The nurse is caring for a patient suffering from anorexia secondary to chemotherapy. Which strategy would be most appropriate for the nurse to use to increase the patient's nutritional intake? a. Increase intake of liquids at mealtime to stimulate the appetite. b. Serve three large meals per day plus snacks between each meal. c. Avoid the use of liquid protein supplements to encourage eating at mealtime. d. Add items such as skim milk powder, cheese, honey, or peanut butter to selected foods.

D. Rationale: The nurse can increase the nutritional density of foods by adding items high in protein and/or calories (such as peanut butter, skim milk powder, cheese, honey, or brown sugar) to foods the patient will eat. Increasing fluid intake at mealtime fills the stomach with fluid and decreases the desire to eat. Small frequent meals are best tolerated. Supplements can be helpful.

A patient has been diagnosed with Burkitt's lymphoma. In the initiation stage of cancer, the cells genetic structure is mutated. Exposure to what may have functioned as a carcinogen for this patient? A. Bacteria B. Sun exposure C. Most chemicals D. Epstein-Barr virus

D. Epston-Barr virus Burkitt's lymphoma consistently shows evidence of the presence of Epstein-Barr virus in vitro. Bacteria do not initiate cancer. Sun exposure causes cell alterations leading to melanoma and squamous and basal cell skin carcinoma. Long-term exposure to certain chemicals (e.g., ethylene oxide, chloroform, benzene) is known to initiate cancer.

When caring for the patient with cancer, what does the nurse understand as the response of the immune system to antigens of the malignant cells? A. Metastasis B. Tumor angiogenesis C. Immunologic escape D. Immunologic surveillance

D. Immunologic surveillance Immunologic surveillance is the process where lymphocytes check cell surface antigens and detect and destroy cells with abnormal or altered antigenic determinants to prevent these cells from developing into clinically detectable tumors. Metastasis is increased growth rate of the tumor, increased invasiveness, and spread of the cancer to a distant site in the progression stage of cancer development. Tumor angiogenesis is the process of blood vessels forming within the tumor itself. Immunologic escape is the cancer cells' evasion of immunologic surveillance that allows the cancer cells to reproduce.

Which nursing diagnosis is most appropriate for a patient experiencing myelosuppression secondary to chemotherapy for cancer treatment? a. Acute pain b. Hypothermia c. Powerlessness d. Risk for infection

D. Risk for infection Myelosuppression is accompanied by a high risk of infection and sepsis. Hypothermia, powerlessness, and acute pain are also possible nursing diagnoses for patients undergoing chemotherapy, but the threat of infection is paramount.

The long-term care nurse is evaluating the effectiveness of protein supplements for an older resident who has a low serum total protein level. Which assessment finding indicates that the patient's condition has improved? a. Hematocrit 28% b. Absence of skin tenting c. Decreased peripheral edema d. Blood pressure 110/72 mm Hg

Decreased peripheral edema

A patient who has been receiving a heparin infusion and warfarin (Coumadin) for a deep vein thrombosis (DVT) is diagnosed with heparin-induced thrombocytopenia (HIT) when her platelet level drops to 110,000/µL. Which action will the nurse include in the plan of care? a. Use low-molecular-weight heparin (LMWH) only. b. Administer the warfarin (Coumadin) at the scheduled time. c. Teach the patient about the purpose of platelet transfusions. d. Discontinue heparin and flush intermittent IV lines using normal saline.

Discontinue heparin and flush intermittent IV lines using normal saline. All heparin is discontinued when the HIT is diagnosed. The patient should be instructed to never receive heparin or LMWH. Warfarin is usually not given until the platelet count has returned to 150,000/µL. The platelet count does not drop low enough in HIT for a platelet transfusion, and platelet transfusions increase the risk for thrombosis.

A 58-year-old patient who has been recently diagnosed with benign prostatic hyperplasia (BPH) tells the nurse that he does not want to have a transurethral resection of the prostate (TURP) because it might affect his ability to maintain an erection during intercourse. Which action should the nurse take? a. Provide teaching about medications for erectile dysfunction (ED). b. Discuss that TURP does not commonly affect erectile function. c. Offer reassurance that sperm production is not affected by TURP. d. Discuss alternative methods of sexual expression besides intercourse.

Discuss that TURP does not commonly affect erectile function. ED is not a concern with TURP, although retrograde ejaculation is likely and the nurse should discuss this with the patient. Erectile function is not usually affected by a TURP, so the patient will not need information about penile implants or reassurance that other forms of sexual expression may be used. Because the patient has not asked about fertility, reassurance about sperm production does not address his concerns.

The nurse is caring for a patient who smokes 2 packs/day. To reduce the patient's risk of lung cancer, which action by the nurse is best? a. Teach the patient about the seven warning signs of cancer. b. Plan to monitor the patient's carcinoembryonic antigen (CEA) level. c. Discuss the risks associated with cigarettes during every patient encounter. d. Teach the patient about the use of annual chest x-rays for lung cancer screening.

Discuss the risks associated with cigarettes during every patient encounter.

The nurse is caring for a patient who has a right-sided chest tube after a right lower lobectomy. Which nursing action can the nurse delegate to the unlicensed assistive personnel (UAP)? a. Document the amount of drainage every eight hours. b. Obtain samples of drainage for culture from the system. c. Assess patient pain level associated with the chest tube. d. Check the water-seal chamber for the correct fluid level.

Document the amount of drainage every eight hours.

The health care provider orders the following interventions for a 67-kg patient who has septic shock with a BP of 70/42 mm Hg and oxygen saturation of 90% on room air. In which order will the nurse implement the actions? (Put a comma and a space between each answer choice [A, B, C, D, E].) a. Obtain blood and urine cultures. b. Give vancomycin (Vancocin) 1 g IV. c. Start norepinephrine (Levophed) 0.5 mcg/min. d. Infuse normal saline 2000 mL over 30 minutes. e. Titrate oxygen administration to keep O2 saturation >95%.

E D C A B

A chemotherapy drug that causes alopecia is prescribed for a patient. Which action should the nurse take to maintain the patient's self-esteem? a. Tell the patient to limit social contacts until regrowth of the hair occurs. b. Encourage the patient to purchase a wig or hat and wear it once hair loss begins. c. Teach the patient to gently wash hair with a mild shampoo to minimize hair loss. d. Inform the patient that hair usually grows back once the chemotherapy is complete.

Encourage the patient to purchase a wig or hat and wear it once hair loss begins.

A patient with respiratory failure has a respiratory rate of 6 breaths/minute and an oxygen saturation (SpO2) of 88%. The patient is increasingly lethargic. Which intervention will the nurse anticipate? a. Administration of 100% oxygen by non-rebreather mask b. Endotracheal intubation and positive pressure ventilation c. Insertion of a mini-tracheostomy with frequent suctioning d. Initiation of continuous positive pressure ventilation (CPAP)

Endotracheal intubation and positive pressure ventilation

A patient with Hodgkin's lymphoma who is undergoing external radiation therapy tells the nurse, "I am so tired I can hardly get out of bed in the morning." Which intervention should the nurse add to the plan of care? a. Minimize activity until the treatment is completed. b. Establish time to take a short walk almost every day. c. Consult with a psychiatrist for treatment of depression. d. Arrange for delivery of a hospital bed to the patient's home.

Establish time to take a short walk almost every day.

A patient who is taking rifampin (Rifadin) for tuberculosis calls the clinic and reports having orange discolored urine and tears. Which is the best response by the nurse? a. Ask if the patient is experiencing shortness of breath, hives, or itching. b. Ask the patient about any visual abnormalities such as red-green color discrimination. c. Explain that orange discolored urine and tears are normal while taking this medication. d. Advise the patient to stop the drug and report the symptoms to the health care provider.

Explain that orange discolored urine and tears are normal while taking this medication.

Infiltrative emergencies: Cardiac tamponade

Fluid accumulation in pericardial sac, constriction of pericardium by tumor, or pericarditis Manifestations Heavy feeling over the chest Shortness of breath Dysphagia Tachycardia Cough, hoarseness, hiccups Extreme anxiety ↓ Level of consciousness Pulsus paradoxus Distant, muted heart sounds Nausea, vomiting, excessive perspiration Nursing Care: Management aimed at decreasing fluid around heart and includes Surgical establishment of pericardial window Indwelling pericardial catheter

The nurse provides preoperative instruction for a patient scheduled for a left pneumonectomy for cancer of the lung. Which information should the nurse include about the patient's postoperative care? a. Positioning on the right side b. Bed rest for the first 24 hours c. Frequent use of an incentive spirometer d. Chest tube placement with continuous drainage

Frequent use of an incentive spirometer

The nurse provides preoperative instruction for a patient scheduled for a left pneumonectomy for cancer of the lung. Which information should the nurse include about the patient's postoperative care?

Frequent use of an incentive spirometer Frequent deep breathing and coughing are needed after chest surgery to prevent atelectasis

A patient with cardiogenic shock has the following vital signs: BP 102/50, pulse 128, respirations 28. The pulmonary artery wedge pressure (PAWP) is increased and cardiac output is low. The nurse will anticipate an order for which medication? a. 5% human albumin b. Furosemide (Lasix) IV c. Epinephrine (Adrenalin) drip d. Hydrocortisone (Solu-Cortef)

Furosemide (Lasix) IV The PAWP indicates that the patient's preload is elevated, and furosemide is indicated to reduce the preload and improve cardiac output. Epinephrine would further increase heart rate and myocardial oxygen demand. 5% human albumin would also increase the PAWP. Hydrocortisone might be considered for septic or anaphylactic shock.

A 78-kg patient with septic shock has a urine output of 30 mL/hr for the past 3 hours. The pulse rate is 120/minute and the central venous pressure and pulmonary artery wedge pressure are low. Which order by the health care provider will the nurse question? a. Give PRN furosemide (Lasix) 40 mg IV. b. Increase normal saline infusion to 250 mL/hr. c. Administer hydrocortisone (Solu-Cortef) 100 mg IV. d. Titrate norepinephrine (Levophed) to keep systolic BP >90 mm Hg.

Give PRN furosemide (Lasix) 40 mg IV. Furosemide will lower the filling pressures and renal perfusion further for the patient with septic shock. The other orders are appropriate.

The nurse assesses a patient who has been hospitalized for 2 days. The patient has been receiving normal saline IV at 100 mL/hr, has a nasogastric tube to low suction, and is NPO. Which assessment finding would be a priority for the nurse to report to the health care provider? a. Oral temperature of 100.1° F b. Serum sodium level of 138 mEq/L (138 mmol/L) c. Gradually decreasing level of consciousness (LOC) d. Weight gain of 2 pounds (1 kg) above the admission weight

Gradually decreasing level of consciousness (LOC)

A patient with leukemia is considering whether to have hematopoietic stem cell transplantation (HSCT). The nurse will include which information in the patient's teaching plan? a. Transplant of the donated cells is painful because of the nerves in the tissue lining the bone. b. Donor bone marrow cells are transplanted through an incision into the sternum or hip bone. c. The transplant procedure takes place in a sterile operating room to minimize the risk for infection. d. Hospitalization will be required for several weeks after the stem cell transplant procedure is performed.

Hospitalization will be required for several weeks after the stem cell transplant procedure is performed.

Which statement by the patient indicates that the teaching has been effective for a patient scheduled for radiation therapy of the larynx?

I will need to buy a water bottle to carry with me." Xerostomia can be partially alleviated by drinking fluids at frequent intervals

The nurse provides discharge teaching for a patient who has two fractured ribs from an automobile accident. Which statement, if made by the patient, would indicate that teaching has been effective?

I will use the incentive spirometer every hour or two during the day." Prevention of the complications of atelectasis and pneumonia is a priority after rib fracture

The nurse obtains information about a hospitalized patient who is receiving chemotherapy for colorectal cancer. Which information about the patient alerts the nurse to discuss a possible change in therapy with the health care provider? a. Poor oral intake b. Frequent loose stools c. Complaints of nausea and vomiting d. Increase in carcinoembryonic antigen (CEA)

Increase in carcinoembryonic antigen (CEA)

An older patient with cardiogenic shock is cool and clammy and hemodynamic monitoring indicates a high systemic vascular resistance (SVR). Which intervention should the nurse anticipate doing next? a. Increase the rate for the dopamine (Intropin) infusion. b. Decrease the rate for the nitroglycerin (Tridil) infusion. c. Increase the rate for the sodium nitroprusside (Nipride) infusion. d. Decrease the rate for the 5% dextrose in normal saline (D5/.9 NS) infusion.

Increase the rate for the sodium nitroprusside (Nipride) infusion. Nitroprusside is an arterial vasodilator and will decrease the SVR and afterload, which will improve cardiac output. Changes in the D5/.9 NS and nitroglycerin infusions will not directly decrease SVR. Increasing the dopamine will tend to increase SVR.

The nurse assesses the chest of a patient with pneumococcal pneumonia. Which finding would the nurse expect? a. Increased tactile fremitus b. Dry, nonproductive cough c. Hyperresonance to percussion d. A grating sound on auscultation

Increased tactile fremitus

A 45-year-old patient is admitted to the emergency department with severe abdominal pain and rebound tenderness. Vital signs include temperature 102° F (38.3° C), pulse 120, respirations 32, and blood pressure (BP) 82/54. Which prescribed intervention should the nurse implement first? a. Administer IV ketorolac (Toradol) 15 mg. b. Draw blood for a complete blood count (CBC). c. Obtain a computed tomography (CT) scan of the abdomen. d. Infuse 1 liter of lactated Ringer's solution over 30 minutes.

Infuse 1 liter of lactated Ringer's solution over 30 minutes. The priority for this patient is to treat the patient's hypovolemic shock with fluid infusion. The other actions should be implemented after starting the fluid infusion.

A patient with possible disseminated intravascular coagulation arrives in the emergency department with a blood pressure of 82/40, temperature 102° F (38.9° C), and severe back pain. Which physician order will the nurse implement first? a. Administer morphine sulfate 4 mg IV. b. Give acetaminophen (Tylenol) 650 mg. c. Infuse normal saline 500 mL over 30 minutes. d. Schedule complete blood count and coagulation studies.

Infuse normal saline 500 mL over 30 minutes. The patient's blood pressure indicates hypovolemia caused by blood loss and should be addressed immediately to improve perfusion to vital organs. The other actions also are appropriate and should be rapidly implemented, but improving perfusion is the priority for this patient.

A nurse is caring for a patient with shock of unknown etiology whose hemodynamic monitoring indicates BP 92/54, pulse 64, and an elevated pulmonary artery wedge pressure. Which collaborative intervention ordered by the health care provider should the nurse question? a. Infuse normal saline at 250 mL/hr. b. Keep head of bed elevated to 30 degrees. c. Hold nitroprusside (Nipride) if systolic BP <90 mm Hg. d. Titrate dobutamine (Dobutrex) to keep systolic BP >90 mm Hg.

Infuse normal saline at 250 mL/hr. The patient's elevated pulmonary artery wedge pressure indicates volume excess. A saline infusion at 250 mL/hr will exacerbate the volume excess. The other actions are appropriate for the patient.

A 71-year-old patient who has benign prostatic hyperplasia (BPH) with urinary retention is admitted to the hospital with elevated blood urea nitrogen (BUN) and creatinine. Which prescribed therapy should the nurse implement first? a. Infuse normal saline at 50 mL/hr. b. Insert a urinary retention catheter. c. Draw blood for a complete blood count. d. Schedule a pelvic computed tomography (CT) scan.

Insert a urinary retention catheter. The patient data indicate that the patient may have acute kidney injury caused by the BPH. The initial therapy will be to insert a catheter. The other actions are also appropriate, but they can be implemented after the acute urinary retention is resolved.

Which nursing interventions included in the care of a mechanically ventilated patient with acute respiratory failure can the registered nurse (RN) delegate to an experienced licensed practical/vocational nurse (LPN/LVN) working in the intensive care unit? a. Assess breath sounds every hour. b. Monitor central venous pressures. c. Place patient in the prone position. d. Insert an indwelling urinary catheter.

Insert an indwelling urinary catheter.

Which information will the nurse teach a patient who has chronic prostatitis? a. Ibuprofen (Motrin) should provide good pain control. b. Prescribed antibiotics should be taken for 7 to 10 days. c. Intercourse or masturbation will help relieve symptoms. d. Cold packs used every 4 hours will decrease inflammation.

Intercourse or masturbation will help relieve symptoms. Ejaculation helps drain the prostate and relieve pain. Warm baths are recommended to reduce pain. Nonsteroidal antiinflammatory drugs (NSAIDs) are frequently prescribed but usually do not offer adequate pain relief. Antibiotics for chronic prostatitis are taken for 4 to 12 weeks.

A patient with a chronic cough has a bronchoscopy. After the procedure, which intervention by the nurse is most appropriate?

Keep the patient NPO until the gag reflex returns Risk for aspiration and maintaining an open airway is the priority. Because a local anesthetic is used to suppress the gag/cough reflexes during bronchoscopy, the nurse should monitor for the return of these reflexes before allowing the patient to take oral fluids or food.

A 64-year-old woman who has chronic constipation asks the nurse about the use of psyllium (Metamucil). Which information will the nurse include in the response? a. Absorption of fat-soluble vitamins may be reduced by fiber-containing laxatives. b. Dietary sources of fiber should be eliminated to prevent excessive gas formation. c. Use of this type of laxative to prevent constipation does not cause adverse effects. d. Large amounts of fluid should be taken to prevent impaction or bowel obstruction.

Large amounts of fluid should be taken to prevent impaction or bowel obstruction. A high fluid intake is needed when patients are using bulk-forming laxatives to avoid worsening constipation. Although bulk-forming laxatives are generally safe, the nurse should emphasize the possibility of constipation or obstipation if inadequate fluid intake occurs. Although increased gas formation is likely to occur with increased dietary fiber, the patient should gradually increase dietary fiber and eventually may not need the psyllium. Fat-soluble vitamin absorption is blocked by stool softeners and lubricants, not by bulk-forming laxatives.

Complications of Cancer Oncologic Emergencies

Life-threatening Occur as the result of disease or treatment Emergencies can be Obstructive Metabolic Infiltrative

A patient who has non-Hodgkin's lymphoma is receiving combination treatment with rituximab (Rituxan) and chemotherapy. Which patient assessment finding requires the most rapid action by the nurse? a. Anorexia b. Vomiting c. Oral ulcers d. Lip swelling

Lip swelling Lip swelling in angioedema may indicate a hypersensitivity reaction to the rituximab. The nurse should stop the infusion and further assess for anaphylaxis. The other findings may occur with chemotherapy, but are not immediately life threatening.

Which information will the nurse teach a 23-year-old patient with lactose intolerance? a. Ice cream is relatively low in lactose. b. Live-culture yogurt is usually tolerated. c. Heating milk will break down the lactose. d. Nonfat milk is a better choice than whole milk.

Live-culture yogurt is usually tolerated. Lactose-intolerant individuals can usually eat yogurt without experiencing discomfort. Ice cream, nonfat milk, and milk that has been heated are all high in lactose.

A patient receives 3% NaCl solution for correction of hyponatremia. Which assessment is most important for the nurse to monitor for while the patient is receiving this infusion? a. Lung sounds b. Urinary output c. Peripheral pulses d. Peripheral edema

Lung sounds

After a transurethral resection of the prostate (TURP), a 64-year-old patient with continuous bladder irrigation complains of painful bladder spasms. The nurse observes clots in the urine. Which action should the nurse take first? a. Increase the flow rate of the bladder irrigation. b. Administer the prescribed IV morphine sulfate. c. Give the patient the prescribed belladonna and opium suppository. d. Manually instill and then withdraw 50 mL of saline into the catheter.

Manually instill and then withdraw 50 mL of saline into the catheter. The assessment suggests that obstruction by a clot is causing the bladder spasms, and the nurse's first action should be to irrigate the catheter manually and to try to remove the clots. IV morphine will not decrease the spasm, although pain may be reduced. Increasing the flow rate of the irrigation will further distend the bladder and may increase spasms. The belladonna and opium suppository will decrease bladder spasms but will not remove the obstructing blood clot.

The nurse notes that a patient has incisional pain, a poor cough effort, and scattered rhonchi after a thoracotomy. Which action should the nurse take first? a. Assist the patient to sit upright in a chair. b. Splint the patient's chest during coughing. c. Medicate the patient with prescribed morphine. d. Observe the patient use the incentive spirometer.

Medicate the patient with prescribed morphine.

A nurse is assessing a newly admitted patient with chronic heart failure who forgot to take prescribed medications and seems confused. The patient complains of "just blowing up" and has peripheral edema and shortness of breath. Which assessment should the nurse complete first? a. Skin turgor b. Heart sounds c. Mental status d. Capillary refill

Mental status

Which intervention will the nurse include in the plan of care for a patient who has cardiogenic shock? a. Check temperature every 2 hours. b. Monitor breath sounds frequently. c. Maintain patient in supine position. d. Assess skin for flushing and itching.

Monitor breath sounds frequently. Since pulmonary congestion and dyspnea are characteristics of cardiogenic shock, the nurse should assess the breath sounds frequently. The head of the bed is usually elevated to decrease dyspnea in patients with cardiogenic shock. Elevated temperature and flushing or itching of the skin are not typical of cardiogenic shock.

The nurse notes a serum calcium level of 7.9 mg/dL for a patient who has chronic malnutrition. Which action should the nurse take next? a. Monitor ionized calcium level. b. Give oral calcium citrate tablets. c. Check parathyroid hormone level. d. Administer vitamin D supplements.

Monitor ionized calcium level.

Which nursing action will the nurse include in the plan of care for a 35-year-old male patient admitted with an exacerbation of inflammatory bowel disease (IBD)? a. Restrict oral fluid intake. b. Monitor stools for blood. c. Ambulate four times daily. d. Increase dietary fiber intake.

Monitor stools for blood. Because anemia or hemorrhage may occur with IBD, stools should be assessed for the presence of blood. The other actions would not be appropriate for the patient with IBD. Because dietary fiber may increase gastrointestinal (GI) motility and exacerbate the diarrhea, severe fatigue is common with IBD exacerbations, and dehydration may occur.

Which action can the registered nurse (RN) who is caring for a critically ill patient with multiple IV lines delegate to an experienced licensed practical/vocational nurse (LPN/LVN)? a. Administer IV antibiotics through the implantable port. b. Monitor the IV sites for redness, swelling, or tenderness. c. Remove the patient's nontunneled subclavian central venous catheter. d. Adjust the flow rate of the 0.9% normal saline in the peripheral IV line.

Monitor the IV sites for redness, swelling, or tenderness.

Following successful treatment of Hodgkin's lymphoma for a 55-year-old woman, which topic will the nurse include in patient teaching? a. Potential impact of chemotherapy treatment on fertility b. Application of soothing lotions to treat residual pruritus c. Use of maintenance chemotherapy to maintain remission d. Need for follow-up appointments to screen for malignancy

Need for follow-up appointments to screen for malignancy The chemotherapy used in treating Hodgkin's lymphoma results in a high incidence of secondary malignancies; follow-up screening is needed. The fertility of a 55-year-old woman will not be impacted by chemotherapy. Maintenance chemotherapy is not used for Hodgkin's lymphoma. Pruritus is a clinical manifestation of lymphoma, but should not be a concern after treatment.

During change-of-shift report, the nurse is told that a patient has been admitted with dehydration and hypotension after having vomiting and diarrhea for 4 days. Which finding is most important for the nurse to report to the health care provider? a. New onset of confusion b. Heart rate 112 beats/minute c. Decreased bowel sounds d. Pale, cool, and dry extremities

New onset of confusion The changes in mental status are indicative that the patient is in the progressive stage of shock and that rapid intervention is needed to prevent further deterioration. The other information is consistent with compensatory shock.

Which diet choice by the patient with an acute exacerbation of inflammatory bowel disease (IBD) indicates a need for more teaching? a. Scrambled eggs b. White toast and jam c. Oatmeal with cream d. Pancakes with syrup

Oatmeal with cream During acute exacerbations of IBD, the patient should avoid high-fiber foods such as whole grains. High-fat foods also may cause diarrhea in some patients. The other choices are low residue and would be appropriate for this patient

The nurse administers prescribed therapies for a patient with cor pulmonale and right-sided heart failure. Which assessment would best evaluate the effectiveness of the therapies? a. Observe for distended neck veins. b. Auscultate for crackles in the lungs. c. Palpate for heaves or thrills over the heart. d. Review hemoglobin and hematocrit values.

Observe for distended neck veins.

Obstructive emergencies: Superior vena cava syndrome

Obstruction by tumor or thrombosis Signs include Facial and periorbital edema Distention of veins of head, neck, and chest Seizures Headache

During a routine health examination, a 40-year-old patient tells the nurse about a family history of colon cancer. Which action should the nurse take next? a. Teach the patient about the need for a colonoscopy at age 50. b. Teach the patient how to do home testing for fecal occult blood. c. Obtain more information from the patient about the family history. d. Schedule a sigmoidoscopy to provide baseline data about the patient.

Obtain more information from the patient about the family history.

The nurse documents the vital signs for a patient admitted 2 days ago with gram-negative sepsis: temperature 101.2° F, blood pressure 90/56 mm Hg, pulse 92, respirations 34. Which action should the nurse take next? a. Give the scheduled IV antibiotic. b. Give the PRN acetaminophen (Tylenol). c. Obtain oxygen saturation using pulse oximetry. d. Notify the health care provider of the patient's vital signs.

Obtain oxygen saturation using pulse oximetry.

A nurse is caring for an obese patient with right lower lobe pneumonia. Which position will be best to improve gas exchange? a. On the left side b. On the right side c. In the tripod position d. In the high-Fowler's position

On the left side

Which information about a patient who is receiving cisatracurium (Nimbex) to prevent asynchronous breathing with the positive pressure ventilator requires immediate action by the nurse? a. Only continuous IV opioids have been ordered. b. The patient does not respond to verbal stimulation. c. There is no cough or gag when the patient is suctioned. d. The patient's oxygen saturation fluctuates between 90% to 93%.

Only continuous IV opioids have been ordered.

The clinic nurse teaches a patient with a 42 pack-year history of cigarette smoking about lung disease. Which information will be most important for the nurse to include? a. Options for smoking cessation b. Reasons for annual sputum cytology testing c. Erlotinib (Tarceva) therapy to prevent tumor risk d. Computed tomography (CT) screening for lung cancer

Options for smoking cessation

Which assessment information is most important for the nurse to obtain to evaluate whether treatment of a patient with anaphylactic shock has been effective? a. Heart rate b. Orientation c. Blood pressure d. Oxygen saturation

Oxygen saturation Because the airway edema that is associated with anaphylaxis can affect airway and breathing, the oxygen saturation is the most critical assessment. Improvements in the other assessments will also be expected with effective treatment of anaphylactic shock.

The nurse cares for a patient who has just had a thoracentesis. Which assessment information obtained by the nurse is a priority to communicate to the health care provider?

Oxygen saturation is 88%. Oxygen saturation would be expected to improve after a thoracentesis. A saturation of 88% indicates that a complication such as pneumothorax may be occurring

An older adult patient who has colorectal cancer is receiving IV fluids at 175 mL/hour in conjunction with the prescribed chemotherapy. Which finding by the nurse is most important to report to the health care provider? a. Patient complains of severe fatigue. b. Patient needs to void every hour during the day. c. Patient takes only 50% of meals and refuses snacks. d. Patient has audible crackles to the midline posterior chest.

Patient has audible crackles to the midline posterior chest.

A patient is scheduled for a computed tomography (CT) of the chest with contrast media. Which assessment findings should the nurse immediately report to the health care provider (select all that apply)?

Patient is allergic to shellfish. d. Blood urea nitrogen (BUN) and serum creatinine levels are elevated. Because the contrast media is iodine-based and may cause dehydration and decreased renal blood flow, asking about iodine allergies (such as allergy to shellfish) and monitoring renal function before the CT scan are necessary.

Management of Cancer Pain

Patient report should always be believed and accepted as primary source for pain assessment data. Drug therapy should be used to control pain.

A patient with idiopathic pulmonary arterial hypertension (IPAH) is receiving nifedipine (Procardia). Which assessment would best indicate to the nurse that the patient's condition is improving? a. Blood pressure (BP) is less than 140/90 mm Hg. b. Patient reports decreased exertional dyspnea. c. Heart rate is between 60 and 100 beats/minute. d. Patient's chest x-ray indicates clear lung fields.

Patient reports decreased exertional dyspnea.

A patient with idiopathic pulmonary arterial hypertension (IPAH) is receiving nifedipine (Procardia). Which assessment would best indicate to the nurse that the patient's condition is improving?

Patient reports decreased exertional dyspnea. Because a major symptom of IPAH is exertional dyspnea, an improvement in this symptom would indicate that the medication was effective.

After change-of-shift report on the oncology unit, which patient should the nurse assess first? a. Patient who has a platelet count of 82,000/µL after chemotherapy b. Patient who has xerostomia after receiving head and neck radiation c. Patient who is neutropenic and has a temperature of 100.5° F (38.1° C) d. Patient who is worried about getting the prescribed long-acting opioid on time

Patient who is neutropenic and has a temperature of 100.5° F (38.1° C)

After receiving change-of-shift report, which patient should the nurse assess first? a. Patient with serum potassium level of 5.0 mEq/L who is complaining of abdominal cramping b. Patient with serum sodium level of 145 mEq/L who has a dry mouth and is asking for a glass of water c. Patient with serum magnesium level of 1.1 mEq/L who has tremors and hyperactive deep tendon reflexes d. Patient with serum phosphorus level of 4.5 mg/dL who has multiple soft tissue calcium-phosphate precipitates

Patient with serum magnesium level of 1.1 mEq/L who has tremors and hyperactive deep tendon reflexes

After change-of-shift report in the progressive care unit, who should the nurse care for first? a. Patient who had an inferior myocardial infarction 2 days ago and has crackles in the lung bases b. Patient with suspected urosepsis who has new orders for urine and blood cultures and antibiotics c. Patient who had a T5 spinal cord injury 1 week ago and currently has a heart rate of 54 beats/minute d. Patient admitted with anaphylaxis 3 hours ago who now has clear lung sounds and a blood pressure of 108/58 mm Hg

Patient with suspected urosepsis who has new orders for urine and blood cultures and antibiotics Antibiotics should be administered within the first hour for patients who have sepsis or suspected sepsis in order to prevent progression to systemic inflammatory response syndrome (SIRS) and septic shock. The data on the other patients indicate that they are more stable. Crackles heard only at the lung bases do not require immediate intervention in a patient who has had a myocardial infarction. Mild bradycardia does not usually require atropine in patients who have a spinal cord injury. The findings for the patient admitted with anaphylaxis indicate resolution of bronchospasm and hypotension.

The nurse develops a plan of care to prevent aspiration in a high-risk patient. Which nursing action will be most effective?

Place patients with altered consciousness in side-lying positions The risk for aspiration is decreased when patients with a decreased level of consciousness are placed in a side-lying or upright position

A patient with small cell lung carcinoma who is suspected to be at a high risk for metastases has received chemotherapy treatment. For which treatment should the nurse prepare the patient after chemotherapy?

Prophylactic cranial radiation

A patient who has been involved in a motor vehicle crash arrives in the emergency department (ED) with cool, clammy skin; tachycardia; and hypotension. Which intervention ordered by the health care provider should the nurse implement first? a. Insert two large-bore IV catheters. b. Initiate continuous electrocardiogram (ECG) monitoring. c. Provide oxygen at 100% per non-rebreather mask. d. Draw blood to type and crossmatch for transfusions.

Provide oxygen at 100% per non-rebreather mask. The first priority in the initial management of shock is maintenance of the airway and ventilation. ECG monitoring, insertion of IV catheters, and obtaining blood for transfusions should also be rapidly accomplished but only after actions to maximize oxygen delivery have been implemented.

A nurse who is caring for patient with a tracheostomy tube in place has just auscultated rhonchi bilaterally. If the patient is unsuccessful in coughing up secretions, what action should the nurse take?

Put on sterile gloves and use a sterile catheter to suction This patient needs suctioning now to secure a patent airway.

When caring for a patient with continuous bladder irrigation after having transurethral resection of the prostate, which action could the nurse delegate to unlicensed assistive personnel (UAP)? a. Teach the patient how to perform Kegel exercises. b. Report any complaints of pain or spasms to the nurse. c. Monitor for increases in bleeding or presence of clots. d. Increase the flow rate of the irrigation if clots are noted.

Report any complaints of pain or spasms to the nurse. UAP education and role includes reporting patient concerns to supervising nurses. Patient teaching, assessments for complications, and actions such as bladder irrigation require more education and should be done by licensed nursing staff.

A patient who was involved in a motor vehicle crash has had a tracheostomy placed to allow for continued mechanical ventilation. How should the nurse interpret the following arterial blood gas results: pH 7.48, PaO2 85 mm Hg, PaCO2 32 mm Hg, and HCO3 25 mEq/L? a. Metabolic acidosis b. Metabolic alkalosis c. Respiratory acidosis d. Respiratory alkalosis

Respiratory alkalosis

A patient receiving head and neck radiation for larynx cancer has ulcerations over the oral mucosa and tongue and thick, ropey saliva. Which instructions should the nurse give to this patient? a. Remove food debris from the teeth and oral mucosa with a stiff toothbrush. b. Use cotton-tipped applicators dipped in hydrogen peroxide to clean the teeth. c. Gargle and rinse the mouth several times a day with an antiseptic mouthwash. d. Rinse the mouth before and after each meal and at bedtime with a saline solution.

Rinse the mouth before and after each meal and at bedtime with a saline solution.

During the teaching session for a patient who has a new diagnosis of acute leukemia the patient is restless and is looking away, never making eye contact. After teaching about the complications associated with chemotherapy, the patient asks the nurse to repeat all of the information. Based on this assessment, which nursing diagnosis is most appropriate for the patient? a. Risk for ineffective adherence to treatment related to denial of need for chemotherapy b. Acute confusion related to infiltration of leukemia cells into the central nervous system c. Risk for ineffective health maintenance related to anxiety about new leukemia diagnosis d. Deficient knowledge: chemotherapy related to a lack of interest in learning about treatment

Risk for ineffective health maintenance related to anxiety about new leukemia diagnosis

A new 19-year-old male patient has familial adenomatous polyposis (FAP). Which action will the nurse in the gastrointestinal clinic include in the plan of care? a. Obtain blood samples for DNA analysis. b. Schedule the patient for yearly colonoscopy. c. Provide preoperative teaching about total colectomy. d. Discuss lifestyle modifications to decrease cancer risk.

Schedule the patient for yearly colonoscopy. Patients with FAP should have annual colonoscopy starting at age 16 and usually have total colectomy by age 25 to avoid developing colorectal cancer. DNA analysis is used to make the diagnosis, but is not needed now for this patient. Lifestyle modifications will not decrease cancer risk for this patient.

Which patient information is most important for the nurse to monitor when evaluating the effectiveness of deferoxamine (Desferal) for a patient with hemochromatosis? a. Skin color b. Hematocrit c. Liver function d. Serum iron level

Serum iron level Because iron chelating agents are used to lower serum iron levels, the most useful information will be the patient's iron level. The other parameters will also be monitored, but are not the most important to monitor when determining the effectiveness of deferoxamine.

A patient who has a small cell carcinoma of the lung develops syndrome of inappropriate antidiuretic hormone (SIADH). The nurse should notify the health care provider about which assessment finding? a. Reported weight gain b. Serum hematocrit of 42% c. Serum sodium level of 120 mg/dL d. Total urinary output of 280 mL during past 8 hours

Serum sodium level of 120 mg/dL

An hour after a thoracotomy, a patient complains of incisional pain at a level 7 (based on 0 to 10 scale) and has decreased left-sided breath sounds. The pleural drainage system has 100 mL of bloody drainage and a large air leak. Which action is best for the nurse to take next? a. Milk the chest tube gently to remove any clots. b. Clamp the chest tube momentarily to check for the origin of the air leak. c. Assist the patient to deep breathe, cough, and use the incentive spirometer. d. Set up the patient controlled analgesia (PCA) and administer the loading dose of morphine.

Set up the patient controlled analgesia (PCA) and administer the loading dose of morphine.

Obstructive emergencies: Third space syndrome

Shifting of fluid from vascular space to interstitial space Signs of hypovolemia including hypotension, tachycardia, low central venous pressure, and ↓ urine output Treatment: Replacement of fluids, electrolytes, and plasma protein

The nurse assesses a patient with non-Hodgkin's lymphoma who is receiving an infusion of rituximab (Rituxan). Which assessment finding would require the most rapid action by the nurse? a. Shortness of breath b. Temperature 100.2° F (37.9° C) c. Shivering and complaint of chills d. Generalized muscle aches and pains

Shortness of breath

The nurse prepares a patient with a left-sided pleural effusion for a thoracentesis. How should the nurse position the patient?

Sitting upright with the arms supported on an over bed table The upright position with the arms supported increases lung expansion, allows fluid to collect at the lung bases, and expands the intercostal space so that access to the pleural space is easier.

A patient is admitted to the emergency department complaining of sudden onset shortness of breath and is diagnosed with a possible pulmonary embolus. How should the nurse prepare the patient for diagnostic testing to confirm the diagnosis?

Start an IV so contrast media may be given. Spiral computed tomography (CT) scans are the most commonly used test to diagnose pulmonary emboli, and contrast media may be given IV.

The nurse administers an IV vesicant chemotherapeutic agent to a patient. Which action is most important for the nurse to take? a. Infuse the medication over a short period of time. b. Stop the infusion if swelling is observed at the site. c. Administer the chemotherapy through a small-bore catheter. d. Hold the medication unless a central venous line is available.

Stop the infusion if swelling is observed at the site.

Which nursing action could the registered nurse (RN) working in a skilled care hospital unit delegate to an experienced licensed practical/vocational nurse (LPN/LVN) caring for a patient with a permanent tracheostomy?

Suction the tracheostomy when needed. Suctioning of a stable patient can be delegated to LPNs/LVNs.

A patient is admitted to the emergency department with an open stab wound to the left chest. What is the first action that the nurse should take? a. Position the patient so that the left chest is dependent. b. Tape a nonporous dressing on three sides over the chest wound. c. Cover the sucking chest wound firmly with an occlusive dressing. d. Keep the head of the patient's bed at no more than 30 degrees elevation.

Tape a nonporous dressing on three sides over the chest wound.

Which problem reported by a patient with hemophilia is most important for the nurse to communicate to the physician? a. Leg bruises b. Tarry stools c. Skin abrasions d. Bleeding gums

Tarry stools Melena is a sign of gastrointestinal bleeding and requires collaborative actions such as checking hemoglobin and hematocrit and administration of coagulation factors. The other problems indicate a need for patient teaching about how to avoid injury, but are not indicators of possible serious blood loss.

Which intervention will the nurse include in the plan of care for a patient who is diagnosed with a lung abscess? a. Teach the patient to avoid the use of over-the-counter expectorants. b. Assist the patient with chest physiotherapy and postural drainage. c. Notify the health care provider immediately about any bloody or foul-smelling sputum. d. Teach about the need for prolonged antibiotic therapy after discharge from the hospital.

Teach about the need for prolonged antibiotic therapy after discharge from the hospital.

The health care provider writes an order for bacteriologic testing for a patient who has a positive tuberculosis skin test. Which action should the nurse take? a. Teach about the reason for the blood tests. b. Schedule an appointment for a chest x-ray. c. Teach about the need to get sputum specimens for 2 to 3 consecutive days. d. Instruct the patient to expectorate three specimens as soon as possible.

Teach about the need to get sputum specimens for 2 to 3 consecutive days.

The nurse is caring for a patient who has acute pharyngitis caused by Candida albicans. Which action is appropriate for the nurse to include in the plan of care?

Teach patient to "swish and swallow" prescribed oral nystatin (Mycostatin). Oral or pharyngeal fungal infections are treated with nystatin solution

Which action by the nurse will be most effective in decreasing the spread of pertussis in a community setting? a. Providing supportive care to patients diagnosed with pertussis b. Teaching family members about the need for careful hand washing c. Teaching patients about the need for adult pertussis immunizations d. Encouraging patients to complete the prescribed course of antibiotics

Teaching patients about the need for adult pertussis immunizations

Which action by the nurse will be most effective in decreasing the spread of pertussis in a community setting?

Teaching patients about the need for adult pertussis immunizations The increased rate of pertussis in adults is thought to be due to decreasing immunity after childhood immunization.

When assessing a pregnant patient with eclampsia who is receiving IV magnesium sulfate, which finding should the nurse report to the health care provider immediately? a. The bibasilar breath sounds are decreased. b. The patellar and triceps reflexes are absent. c. The patient has been sleeping most of the day. d. The patient reports feeling "sick to my stomach."

The patellar and triceps reflexes are absent.

A patient is diagnosed with both human immunodeficiency virus (HIV) and active tuberculosis (TB) disease. Which information obtained by the nurse is most important to communicate to the health care provider? a. The Mantoux test had an induration of 7 mm. b. The chest-x-ray showed infiltrates in the lower lobes. c. The patient is being treated with antiretrovirals for HIV infection. d. The patient has a cough that is productive of blood-tinged mucus.

The patient is being treated with antiretrovirals for HIV infection

Which finding about a patient who is receiving vasopressin (Pitressin) to treat septic shock is most important for the nurse to communicate to the health care provider? a. The patient's urine output is 18 mL/hr. b. The patient's heart rate is 110 beats/minute. c. The patient is complaining of chest pain. d. The patient's peripheral pulses are weak.

The patient is complaining of chest pain. Because vasopressin is a potent vasoconstrictor, it may decrease coronary artery perfusion. The other information is consistent with the patient's diagnosis and should be reported to the health care provider but does not indicate a need for a change in therapy.

Which information obtained by the nurse caring for a patient with thrombocytopenia should be immediately communicated to the health care provider? a. The platelet count is 52,000/µL. b. The patient is difficult to arouse. c. There are purpura on the oral mucosa. d. There are large bruises on the patient's back.

The patient is difficult to arouse. Difficulty in arousing the patient may indicate a cerebral hemorrhage, which is life threatening and requires immediate action. The other information should be documented and reported but would not be unusual in a patient with thrombocytopenia.

A patient has a serum calcium level of 7.0 mEq/L. Which assessment finding is most important for the nurse to report to the health care provider? a. The patient is experiencing laryngeal stridor. b. The patient complains of generalized fatigue. c. The patient's bowels have not moved for 4 days. d. The patient has numbness and tingling of the lips.

The patient is experiencing laryngeal stridor.

When admitting a patient with possible respiratory failure with a high PaCO2, which assessment information should be immediately reported to the health care provider? a. The patient is somnolent. b. The patient complains of weakness. c. The patient's blood pressure is 164/98. d. The patient's oxygen saturation is 90%.

The patient is somnolent.

The nurse is caring for a patient with colon cancer who is scheduled for external radiation therapy to the abdomen. Which information obtained by the nurse would indicate a need for patient teaching? a. The patient swims a mile 3 days a week. b. The patient snacks frequently during the day. c. The patient showers everyday with a mild soap. d. The patient has a history of dental caries with amalgam fillings.

The patient swims a mile 3 days a week.

A patient who has severe pain associated with terminal pancreatic cancer is being cared for at home by family members. Which finding by the nurse indicates that teaching regarding pain management has been effective? a. The patient uses the ordered opioid pain medication whenever the pain is greater than 5 (0 to 10 scale). b. The patient agrees to take the medications by the IV route in order to improve analgesic effectiveness. c. The patient takes opioids around the clock on a regular schedule and uses additional doses when breakthrough pain occurs. d. The patient states that nonopioid analgesics may be used when the maximal dose of the opioid is reached without adequate pain relief.

The patient takes opioids around the clock on a regular schedule and uses additional doses when breakthrough pain occurs.

A 22-year-old female patient with an exacerbation of ulcerative colitis is having 15 to 20 stools daily and has excoriated perianal skin. Which patient behavior indicates that teaching regarding maintenance of skin integrity has been effective? a. The patient uses incontinence briefs to contain loose stools. b. The patient asks for antidiarrheal medication after each stool. c. The patient uses witch hazel compresses to decrease irritation. d. The patient cleans the perianal area with soap after each stool.

The patient uses witch hazel compresses to decrease irritation. Witch hazel compresses are suggested to reduce anal irritation and discomfort. Incontinence briefs may trap diarrhea and increase the incidence of skin breakdown. Antidiarrheal medications are not given 15 to 20 times a day. The perianal area should be washed with plain water after each stool.

The nurse is providing preoperative teaching for a 61-year-old man scheduled for an abdominal-perineal resection. Which information will the nurse include? a. Another surgery in 8 to 12 weeks will be used to create an ileal-anal reservoir. b. The patient will begin sitting in a chair at the bedside on the first postoperative day. c. The patient will drink polyethylene glycol lavage solution (GoLYTELY) preoperatively. d. IV antibiotics will be started at least 24 hours before surgery to reduce the bowel bacteria.

The patient will drink polyethylene glycol lavage solution (GoLYTELY) preoperatively. A bowel-cleansing agent is used to empty the bowel before surgery to reduce the risk for infection. A permanent colostomy is created with this surgery. Sitting is contraindicated after an abdominal-perineal resection. Oral antibiotics (rather than IV antibiotics) are given to reduce colonic and rectal bacteria.

Which patient requires the most rapid assessment and care by the emergency department nurse? a. The patient with hemochromatosis who reports abdominal pain b. The patient with neutropenia who has a temperature of 101.8° F c. The patient with sickle cell anemia who has had nausea and diarrhea for 24 hours d. The patient with thrombocytopenia who has oozing after having a tooth extracted

The patient with neutropenia who has a temperature of 101.8° F A neutropenic patient with a fever is assumed to have an infection and is at risk for rapidly developing sepsis. Rapid assessment, cultures, and initiation of antibiotic therapy are needed. The other patients also require rapid assessment and care but not as urgently as the neutropenic patient.

The patient with neurogenic shock is receiving a phenylephrine (Neo-Synephrine) infusion through a right forearm IV. Which assessment finding obtained by the nurse indicates a need for immediate action? a. The patient's heart rate is 58 beats/minute. b. The patient's extremities are warm and dry. c. The patient's IV infusion site is cool and pale. d. The patient's urine output is 28 mL over the last hour.

The patient's IV infusion site is cool and pale. The coldness and pallor at the infusion site suggest extravasation of the phenylephrine. The nurse should discontinue the IV and, if possible, infuse the medication into a central line. An apical pulse of 58 is typical for neurogenic shock but does not indicate an immediate need for nursing intervention. A 28-mL urinary output over 1 hour would require the nurse to monitor the output over the next hour, but an immediate change in therapy is not indicated. Warm, dry skin is consistent with early neurogenic shock, but it does not indicate a need for a change in therapy or immediate action.

The nurse is caring for a 33-year-old patient who arrived in the emergency department with acute respiratory distress. Which assessment finding by the nurse requires the most rapid action? a. The patient's PaO2 is 45 mm Hg. b. The patient's PaCO2 is 33 mm Hg. c. The patient's respirations are shallow. d. The patient's respiratory rate is 32 breaths/minute.

The patient's PaO2 is 45 mm Hg.

A patient with acute respiratory distress syndrome (ARDS) is placed in the prone position. When prone positioning is used, which information obtained by the nurse indicates that the positioning is effective? a. The patient's PaO2 is 89 mm Hg, and the SaO2 is 91%. b. Endotracheal suctioning results in clear mucous return. c. Sputum and blood cultures show no growth after 48 hours. d. The skin on the patient's back is intact and without redness.

The patient's PaO2 is 89 mm Hg, and the SaO2 is 91%.

The nurse is caring for a patient with idiopathic pulmonary arterial hypertension (IPAH) who is receiving epoprostenol (Flolan). Which assessment information requires the most immediate action by the nurse? a. The oxygen saturation is 94%. b. The blood pressure is 98/56 mm Hg. c. The patient's central IV line is disconnected. d. The international normalized ratio (INR) is prolonged.

The patient's central IV line is disconnected.

Norepinephrine (Levophed) has been prescribed for a patient who was admitted with dehydration and hypotension. Which patient data indicate that the nurse should consult with the health care provider before starting the norepinephrine? a. The patient's central venous pressure is 3 mm Hg. b. The patient is in sinus tachycardia at 120 beats/min. c. The patient is receiving low dose dopamine (Intropin). d. The patient has had no urine output since being admitted.

The patient's central venous pressure is 3 mm Hg. Adequate fluid administration is essential before administration of vasopressors to patients with hypovolemic shock. The patient's low central venous pressure indicates a need for more volume replacement. The other patient data are not contraindications to norepinephrine administration.

A patient with chronic obstructive pulmonary disease (COPD) arrives in the emergency department complaining of shortness of breath and dyspnea on minimal exertion. Which assessment finding by the nurse is most important to report to the health care provider? a. The patient has bibasilar lung crackles. b. The patient is sitting in the tripod position. c. The patient's respirations have decreased from 30 to 10 breaths/minute. d. The patient's pulse oximetry indicates an O2 saturation of 91%.

The patient's pulse oximetry indicates an O2 saturation of 91%.

Which data collected by the nurse caring for a patient who has cardiogenic shock indicate that the patient may be developing multiple organ dysfunction syndrome (MODS)? a. The patient's serum creatinine level is elevated. b. The patient complains of intermittent chest pressure. c. The patient's extremities are cool and pulses are weak. d. The patient has bilateral crackles throughout lung fields.

The patient's serum creatinine level is elevated. The elevated serum creatinine level indicates that the patient has renal failure as well as heart failure. The crackles, chest pressure, and cool extremities are all consistent with the patient's diagnosis of cardiogenic shock.

A hospitalized patient who has received chemotherapy for leukemia develops neutropenia. Which observation by the nurse would indicate a need for further teaching? a. The patient ambulates several times a day in the room. b. The patient's visitors bring in some fresh peaches from home. c. The patient cleans with a warm washcloth after having a stool. d. The patient uses soap and shampoo to shower every other day.

The patient's visitors bring in some fresh peaches from home.

A patient with right lower-lobe pneumonia has been treated with IV antibiotics for 3 days. Which assessment data obtained by the nurse indicates that the treatment has been effective?

The patient's white blood cell (WBC) count is 9000/µL. The normal WBC count indicates that the antibiotics have been effective

A patient is receiving a 3% saline continuous IV infusion for hyponatremia. Which assessment data will require the most rapid response by the nurse? a. The patient's radial pulse is 105 beats/minute. b. There is sediment and blood in the patient's urine. c. The blood pressure increases from 120/80 to 142/94. d. There are crackles audible throughout both lung fields.

There are crackles audible throughout both lung fields.

A patient is admitted with active tuberculosis (TB). The nurse should question a health care provider's order to discontinue airborne precautions unless which assessment finding is documented? a. Chest x-ray shows no upper lobe infiltrates. b. TB medications have been taken for 6 months. c. Mantoux testing shows an induration of 10 mm. d. Three sputum smears for acid-fast bacilli are negative.

Three sputum smears for acid-fast bacilli are negative.

A 54-year-old critically ill patient with sepsis is frequently incontinent of watery stools. What action by the nurse will prevent complications associated with ongoing incontinence? a. Apply incontinence briefs. b. Use a fecal management system c. Insert a rectal tube with a drainage bag. d. Assist the patient to a commode frequently.

Use a fecal management system Fecal management systems are designed to contain loose stools and can be in place for as long as 4 weeks without causing damage to the rectum or anal sphincters. Although incontinence briefs may be helpful, unless they are changed frequently, they are likely to increase the risk for skin breakdown. Rectal tubes are avoided because of possible damage to the anal sphincter and ulceration of the rectal mucosa. A critically ill patient will not be able to tolerate getting up frequently to use the commode or bathroom.

When assessing a patient with a sore throat, the nurse notes anterior cervical lymph node swelling, a temperature of 101.6° F (38.7° C), and yellow patches on the tonsils. Which action will the nurse anticipate taking?

Use a swab to obtain a sample for a rapid strep antigen test. The patient's clinical manifestations are consistent with streptococcal pharyngitis and the

Employee health test results reveal a tuberculosis (TB) skin test of 16-mm induration and a negative chest x-ray for a staff nurse working on the pulmonary unit. The nurse has no symptoms of TB. Which information should the occupational health nurse plan to teach the staff nurse?

Use and side effects of isoniazid (INH) The nurse is considered to have a latent TB infection and should be treated with INH daily for 6 to 9 months

Following assessment of a patient with pneumonia, the nurse identifies a nursing diagnosis of ineffective airway clearance. Which assessment data best supports this diagnosis? a. Weak, nonproductive cough effort b. Large amounts of greenish sputum c. Respiratory rate of 28 breaths/minute d. Resting pulse oximetry (SpO2) of 85%

Weak, nonproductive cough effort

An older patient is receiving standard multidrug therapy for tuberculosis (TB). The nurse should notify the health care provider if the patient exhibits which finding? a. Yellow-tinged skin b. Orange-colored sputum c. Thickening of the fingernails d. Difficulty hearing high-pitched voices

Yellow-tinged skin

A 62-year old man with chronic anemia is experiencing increased fatigue and occasional palpitations at rest. The nurse would expect the patient's laboratory findings to include a. a hematocrit (Hct) of 38%. b. an RBC count of 4,500,000/L. c. normal red blood cell (RBC) indices. d. a hemoglobin (Hgb) of 8.6 g/dL (86 g/L).

a hemoglobin (Hgb) of 8.6 g/dL (86 g/L). The patient's clinical manifestations indicate moderate anemia, which is consistent with a Hgb of 6 to 10 g/dL. The other values are all within the range of normal.

The primary protective rold of the immune system related to malignant cells is a. surveillance for cells with tumor-associated antigens b. binding with free antigen released by malignant cells c. production of blocking factors that immobilize cancer cells d. responding to a new set of antigenic determinant of cancer cells

a.

The presence of carcinoembryonic antigens (CEA's) and a-fetaoprotein (AFP) on cell membranes is in an indication that cells have a. shifted to more immature metabolic pathways and functions b. spread from areas of original development to different body tissues c. become more differentiated as a result of repression of embryonic functions d. produced abnormal toxins or chemicals that indicate abnormal cellular function

a. Rationale: Cancer cells become more fetal and embryonic (undifferentiated) in appearance and function, and some produce new proteins, such as CEA and AFP, on cell membranes that reflect a return to more immature functioning

The most important method for identifying the presence of infection in a neutropenic patient is a. frequent temperature monitoring b. routine blood and sputum cultures c. assessing for redness and swelling d. monitoring WBC count

a. Rationale: Elevated temp is of most significance in recognizing the presence of an infection in the neutropenic patient because there is no leukocytic response.

The nurse is presenting to a community education program related to cancer prevention. Based on current cancer death rates, the nurse stresses that the most important preventive action for both men and women is a. smoking cessation b. routine colonoscopies c. protection from the ultraviolet light d. regular examination of reproductive organs

a. Rationale: Lung cancer is the leading cause of cancer death in the US for both men and women, and smoking cessation is one of the most important cancer-prevention behaviors.

During preoperative teaching for the patient scheduled for a total laryngectomy, the nurse includes information related to a. the postoperative use of nonverbal communication techniques b. techniques that will be used to alleviate a dry mouth and prevent stomatitis c. the need for frequent, vigorous coughing in the first 24 hours postoperatively d. self-help groups and community resources for patients with cancer of the larynx

a. Rationale: With removal of the larynx, the patient will not be able to communicate verbally, and it is important to arrange with the patient a method of communication before surgery so that postoperative communication can take place.

A 73-year-old patient with diverticulosis has a large bowel obstruction. The nurse will monitor for a. referred back pain. b. metabolic alkalosis. c. projectile vomiting. d. abdominal distention.

abdominal distention. Abdominal distention is seen in lower intestinal obstruction. Referred back pain is not a common clinical manifestation of intestinal obstruction. Metabolic alkalosis is common in high intestinal obstruction because of the loss of HCl acid from vomiting. Projectile vomiting is associated with higher intestinal obstruction.

An appropriate nursing intervention for a hospitalized patient with severe hemolytic anemia is to a. provide a diet high in vitamin K. b. alternate periods of rest and activity. c. teach the patient how to avoid injury. d. place the patient on protective isolation.

alternate periods of rest and activity. Nursing care for patients with anemia should alternate periods of rest and activity to encourage activity without causing undue fatigue. There is no indication that the patient has a bleeding disorder, so a diet high in vitamin K or teaching about how to avoid injury is not needed. Protective isolation might be used for a patient with aplastic anemia, but it is not indicated for hemolytic anemia.

A 54-year-old woman with acute myelogenous leukemia (AML) is considering treatment with a hematopoietic stem cell transplant (HSCT). The best approach for the nurse to assist the patient with a treatment decision is to a. emphasize the positive outcomes of a bone marrow transplant. b. discuss the need for adequate insurance to cover post-HSCT care. c. ask the patient whether there are any questions or concerns about HSCT. d. explain that a cure is not possible with any other treatment except HSCT.

ask the patient whether there are any questions or concerns about HSCT. Offering the patient an opportunity to ask questions or discuss concerns about HSCT will encourage the patient to voice concerns about this treatment and also will allow the nurse to assess whether the patient needs more information about the procedure. Treatment of AML using chemotherapy is another option for the patient. It is not appropriate for the nurse to ask the patient to consider insurance needs in making this decision.

The nurse plans to teach a patient how to manage allergic rhinitis. Which information should the nurse include in the teaching plan?

b. Identification and avoidance of environmental triggers are the best way to avoid symptoms The most important intervention is to assist the patient in identifying and avoiding potential allergens.

The nurse is caring for a patient who has been admitted to the hospital while receiving home hospice care. The nurse interprets that the patient has a general prognosis of which of the following? a. 3 Months or less to live b. 6 Months or less to live c.12 Months or less to live d.18 Months or less to live

b. Rationale: There are two criteria to be eligible for hospice care. The first is that the patient wishes to receive it, and the second is that the physician certifies that the patient has a prognosis of 6 months or less to live.

The nurses uses many precautions during IV administration of vesicant chemotherapeurtic agents primarily to prevent a. septicemia b. extravasation c. catheter occlusion d. anaphylactic shock

b. Rationale: one of the major concerns with IV administration of chemo agents is infiltration of drugs into tissue surrounding the infusion site. Many of these drugs are vesicants-drugs that, when infiltrated into the skin, cause severe local breakdown and necrosis. Specific measures to ensure adequate dilution, patency, and early detection of injury are important.

The clinic nurse is teaching a patient with acute sinusitis. Which interventions should the nurse plan to include in the teaching session (select all that apply)?

b. Saline nasal spray can be made at home and used to wash out secretions. c. Decongestants can be used to relieve swelling. d. You will be more comfortable if you keep your head in an upright position. e. Taking a hot shower will increase sinus drainage and decrease pain.

The nurse notes scleral jaundice in a patient being admitted with hemolytic anemia. The nurse will plan to check the laboratory results for the a. Schilling test. b. bilirubin level. c. stool occult blood test. d. gastric analysis testing.

bilirubin level. Jaundice is caused by the elevation of bilirubin level associated with red blood cell (RBC) hemolysis. The other tests would not be helpful in monitoring or treating a hemolytic anemia.

A routine complete blood count indicates that an active 80-year-old man may have myelodysplastic syndrome. The nurse will plan to teach the patient about a. blood transfusion b. bone marrow biopsy. c. filgrastim (Neupogen) administration. d. erythropoietin (Epogen) administration.

bone marrow biopsy. Bone marrow biopsy is needed to make the diagnosis and determine the specific type of myelodysplastic syndrome. The other treatments may be necessary if there is progression of the myelodysplastic syndrome, but the initial action for this asymptomatic patient will be a bone marrow biopsy.

The goals of cancer treatment are based on the principle that a surgery is the single most effective treatment for cancer b. initial treatment is always directed toward cure of the cancer c. a combination of treatment modalities is effective for controlling many cancers d. although cancer cure is rare, quality of life can be increased with treatment modification

c.

The nurse counsels the patient receiving radiation therapy or chemotherapy that A. effective birth control should be used for the rest of the patients life b. if N/V occur during treatment, the treatment plan will be modified c. following successful treatment, a return to the persons previous functional level can be expected d. the cycle of fatigue-depression-fatigue that may occur during treatment can be reduced by restricting activity

c.

a patient with pancreatic cancer is admitted to the hospital for evaluation of possible treatment options. The patient asks the nurse to explain the Whipple procedure that the surgeon has described. The explanation includes information that a Whipple procedure involves A. creating a bypass around the obstruction caused by the tumor by joining the gallbladder to the jejunum b. resection of the entire pancreas and the distal portion of the stomach, with anastomosis of the common bile duct and stomach into the duodenum c. removal of part of the pancreas, part of the stomach, the duodenum and the gallbladder, with joining of the pancreatic duct, common bile duct, and stomach into the jejunum d. radical removal of the pancreas, duodenum, and spleen, and attaching the stomach to the jejunum which requires oral supplementation of pancreatic digestive enzymes and insulin replacement therapy

c.

During admission of a patient diagnosed with metastatic lung cancer, the nurse would assess for which of the following as a key indicator of clinical depression related to terminal illness? a. Frustration with pain b. Anorexia and nausea c. Feelings of hopelessness d. Inability to carry out activities of daily living

c. Rationale: Feelings of hopelessness are likely to be present in a patient with a terminal illness who has clinical depression. This can be attributed to lack of control over the disease process or outcome. The nurse should assess for depression routinely when working with patients with a terminal illness.

Normal tissues that may manifest early, acute responses to radiation therapy include a. spleen and liver b. kidney and nerve tissue c. bone marrow and gastrointestinal mucosa d. hollow organs such as the stomach and bladder

c. Rationale: Tissue that is actively proliferating, such as GI mucosa, esophageal and oropharyngeal mucosa and bone marrow, exhibits early acute responses to radiation therapy.

An appropriate nursing intervention for a patient with non-Hodgkin's lymphoma whose platelet count drops to 18,000/µL during chemotherapy is to a. check all stools for occult blood. b. encourage fluids to 3000 mL/day. c. provide oral hygiene every 2 hours. d. check the temperature every 4 hours.

check all stools for occult blood. Because the patient is at risk for spontaneous bleeding, the nurse should check stools for occult blood. A low platelet count does not require an increased fluid intake. Oral hygiene is important, but it is not necessary to provide oral care every 2 hours. The low platelet count does not increase risk for infection, so frequent temperature monitoring is not indicated.

A 19-year-old female is brought to the emergency department with a knife handle protruding from the abdomen. During the initial assessment of the patient, the nurse should a. remove the knife and assess the wound. b. determine the presence of Rovsing sign. c. check for circulation and tissue perfusion. d. insert a urinary catheter and assess for hematuria.

check for circulation and tissue perfusion. The initial assessment is focused on determining whether the patient has hypovolemic shock. The knife should not be removed until the patient is in surgery, where bleeding can be controlled. Rovsing sign is assessed in the patient with suspected appendicitis. A patient with a knife in place will be taken to surgery and assessed for bladder trauma there.

The nurse will plan to teach a patient with Crohn's disease who has megaloblastic anemia about the need for a. oral ferrous sulfate tablets. b. regular blood transfusions. c. iron dextran (Imferon) infusions. d. cobalamin (B12) spray or injections.

cobalamin (B12) spray or injections. Crohn's disease frequently affects the ileum, where absorption of cobalamin occurs. Cobalamin must be administered regularly by nasal spray or IM to correct the anemia. Iron deficiency does not cause megaloblastic anemia. The patient may need occasional transfusions but not regularly scheduled transfusions.

A 50-year-old female patient calls the clinic to report a new onset of severe diarrhea. The nurse anticipates that the patient will need to a. collect a stool specimen. b. prepare for colonoscopy. c. schedule a barium enema. d. have blood cultures drawn.

collect a stool specimen. Acute diarrhea is usually caused by an infectious process, and stool specimens are obtained for culture and examined for parasites or white blood cells. There is no indication that the patient needs a colonoscopy, blood cultures, or a barium enema.

The nurse preparing for the annual physical exam of a 50-year-old man will plan to teach the patient about a. endoscopy. b. colonoscopy. c. computerized tomography screening. d. carcinoembryonic antigen (CEA) testing.

colonoscopy. At age 50, individuals with an average risk for colorectal cancer (CRC) should begin screening for CRC. Colonoscopy is the gold standard for CRC screening. The other diagnostic tests are not recommended as part of a routine annual physical exam at age 50.

A patient on chemotherapy for 10 weeks started at a weight of 121 pounds now weighs 118 pounds and has no sense of taste. Which intervention would be a priority? A. advise the patient to eat foods that are fatty, fried, or high in calories b. discuss with the physician the need for parenteral or enteral feedings c. advise the patient to drink nutritional supplement beverages at least three times a day d. advise the patient to experiment with spices and seasonings to enhance the flavor of food

d,

A characteristic of the stage of progression in the development of cancer is a. oncogenic viral transformation of target cells b. a reversible steady growth facilitated by carcinogens c. a period of latency before the clinical detection of cancer d. proliferation of cancer cells in spite of host control mechanisms

d.

The most effective method of administering a chemo agent that is a vesicant is to A. give it orally b. give it intraarterially c. use an Ommaya reservoir d. use a central venous access device

d.

The nurse is care for a 59 year old woman who had surgery one day ago for removal of a suspected malignant abdominal mass. The patient is awaiting the pathology report. She is tearful and say that she is scared to die. The most effective nursing intervention at this point is to use this opportunity to a. motivate change in unhealthy lifestyle b. educate her about the seven warning signs of cancer c. instruct her about healthy stress relief and coping practices d. allow her to communicate about the meaning of this experience

d.

The primary difference between benign and malignant neoplasms is the a. rate of cell proliferation b. site of malignant tumor c. requirements of cellular nutrients d. characteristic of tissue invasiveness

d.

The primary purpose of hospice care is to a. allow patients to die at home b. provide better quality of care than the family can c. coordinate care for dying patients and their families d. provide comfort and support for the dying patients and their families

d.

Trends in the incidence and death rates of cancer include the fact that a. lung cancer is most common type of cancer in men b. a higher percentage of women than men have lung cancer c. breast cancer is the leading cause of cancer deaths in women d. African Americans have a higher death rate from cancer than whites

d.

A patient with pneumonia has a fever of 101.4° F (38.6° C), a nonproductive cough, and an oxygen saturation of 88%. The patient complains of weakness, fatigue, and needs assistance to get out of bed. Which nursing diagnosis should the nurse assign as the highest priority?

d. Impaired gas exchange related to respiratory congestion All these nursing diagnoses are appropriate for the patient, but the patient's oxygen saturation indicates that all body tissues are at risk for hypoxia unless the gas exchange is improved

A patient has been receiving palliative care for the past several weeks in light of her worsening condition following a series of strokes. The caregiver has rung the call bell, stating that the patient now "stops breathing for a while, then breathes fast and hard, and then stops again." The nurse would recognize that the patient is experiencing a. Apnea. b.Bradypnea. c.Death rattle. d.Cheyne-Stokes respirations.

d. Rationale: Cheyne-Stokes respiration is a pattern of breathing characterized by alternating periods of apnea and deep, rapid breathing. This type of breathing is usually seen as a person nears death.

The major method of preventing infection in the patient with neutropenia is use of a. HEPA filtration rooms b. prophylactic antibiotics c. a diet that eliminates fresh fruits and vegetables d. strict hand washing by all person in contact with the patients

d. Rationale: Hand washing before, during and after care of the patient with neutropenia is the major method to prevent transmission of harmful pathogens to the patient.

A patient with AML is considering a hematopoietic stem cell transplant and asks the nurse what is involved. The best response by the nurse is, a. Your bone marrow is destroyed by radiation, and new bone marrow cells from a matched donor are injected into your bones b. a specimen of your bone marrow may be aspirated and treated to destroy any leukemic cells and then re-infused when your disease becomes worse c. during chemo and total body radiation to destroy all your blood cells, you are given transfusions of RBC and platelets to prevent complications d. leukemic cells and bone marrow cells are eliminated with chemo and radiation and new bone marrow cells from a donor are infused

d. Rationale: Whether the donor bone marrow is from a matched donor or taken from the patient during a remission for later use, hematopoietic stem cell transplant always involves the use of combinations of chemotherapy and total body radiation to eliminate leukemic cells and the patient's bone marrow stem cells totally before IV infusion of the donor cells

The defect in cellular proliferation that occurs in the development of cancer involves a. a rate of cell proliferation that is more rapid than that of normal body cells b. shortened phases of cell life cycles with occasional skipping of G1 or S phases c. rearrangement of stem cell RNA that causes abnormal cellular protein synthesis. d. indiscriminate and continuous proliferation of cells with loss of contact inhibition

d. Rationale:Malignant cells proliferate indiscriminately and continuously and also lose the characteristic of contact inhibition, growth on top of and in between other cells

A patient in the emergency department complains of back pain and difficulty breathing 15 minutes after a transfusion of packed red blood cells is started. The nurse's first action should be to a. administer oxygen therapy at a high flow rate. b. obtain a urine specimen to send to the laboratory. c. notify the health care provider about the symptoms. d. disconnect the transfusion and infuse normal saline.

disconnect the transfusion and infuse normal saline. The patient's symptoms indicate a possible acute hemolytic reaction caused by the transfusion. The first action should be to disconnect the transfusion and infuse normal saline. The other actions also are needed but are not the highest priority.

A 47-year-old female patient is transferred from the recovery room to a surgical unit after a transverse colostomy. The nurse observes the stoma to be deep pink with edema and a small amount of sanguineous drainage. The nurse should a. place ice packs around the stoma. b. notify the surgeon about the stoma. c. monitor the stoma every 30 minutes. d. document stoma assessment findings.

document stoma assessment findings. The stoma appearance indicates good circulation to the stoma. There is no indication that surgical intervention is needed or that frequent stoma monitoring is required. Swelling of the stoma is normal for 2 to 3 weeks after surgery, and an ice pack is not needed.

It is important for the nurse providing care for a patient with sickle cell crisis to a. limit the patient's intake of oral and IV fluids. b. evaluate the effectiveness of opioid analgesics. c. encourage the patient to ambulate as much as tolerated. d. teach the patient about high-protein, high-calorie foods.

evaluate the effectiveness of opioid analgesics. Pain is the most common clinical manifestation of a crisis and usually requires large doses of continuous opioids for control. Fluid intake should be increased to reduce blood viscosity and improve perfusion. Rest is usually ordered to decrease metabolic requirements. Patients are instructed about the need for dietary folic acid, but high-protein, high-calorie diets are not emphasized.

The nurse will inform a patient with cancer of the prostate that side effects of leuprolide (Lupron) may include a. flushing. b. dizziness. c. infection. d. incontinence.

flushing. Hot flashes may occur with decreased testosterone production. Dizziness may occur with the alpha-blockers used for benign prostatic hyperplasia (BPH). Urinary incontinence may occur after prostate surgery, but it is not an expected side effect of medication. Risk for infection is increased in patients receiving chemotherapy.

To determine the severity of the symptoms for a 68-year-old patient with benign prostatic hyperplasia (BPH) the nurse will ask the patient about a. blood in the urine. b. lower back or hip pain. c. erectile dysfunction (ED). d. force of the urinary stream.

force of the urinary stream. The American Urological Association (AUA) Symptom Index for a patient with BPH asks questions about the force and frequency of urination, nocturia, etc. Blood in the urine, ED, and back or hip pain are not typical symptoms of BPH.

The nurse will determine that teaching a 67-year-old man to irrigate his new colostomy has been effective if the patient a. inserts the irrigation tubing 4 to 6 inches into the stoma. b. hangs the irrigating container 18 inches above the stoma. c. stops the irrigation and removes the irrigating cone if cramping occurs. d. fills the irrigating container with 1000 to 2000 mL of lukewarm tap water.

hangs the irrigating container 18 inches above the stoma. The irrigating container should be hung 18 to 24 inches above the stoma. If cramping occurs, the irrigation should be temporarily stopped and the cone left in place. Five hundred to 1000 mL of water should be used for irrigation. An irrigation cone, rather than tubing, should be inserted into the stoma; 4 to 6 inches would be too far for safe insertion.

The health care provider prescribes finasteride (Proscar) for a 67-year-old patient who has benign prostatic hyperplasia (BPH). When teaching the patient about the drug, the nurse informs him that a. he should change position from lying to standing slowly to avoid dizziness. b. his interest in sexual activity may decrease while he is taking the medication. c. improvement in the obstructive symptoms should occur within about 2 weeks. d. he will need to monitor his blood pressure frequently to assess for hypertension.

his interest in sexual activity may decrease while he is taking the medication. A decrease in libido is a side effect of finasteride because of the androgen suppression that occurs with the drug. Although orthostatic hypotension may occur if the patient is also taking a medication for erectile dysfunction (ED), it should not occur with finasteride alone. Improvement in symptoms of obstruction takes about 6 months. The medication does not cause hypertension.

The nurse will plan to teach the patient scheduled for photovaporization of the prostate (PVP) a. that urine will appear bloody for several days. b. how to care for an indwelling urinary catheter. c. that symptom improvement takes 2 to 3 weeks. d. about complications associated with urethral stenting.

how to care for an indwelling urinary catheter. The patient will have an indwelling catheter for 24 to 48 hours and will need teaching about catheter care. There is minimal bleeding with this procedure. Symptom improvement is almost immediate after PVP. Stent placement is not included in the procedure.

A patient develops increasing dyspnea and hypoxemia 2 days after heart surgery. To determine whether the patient has acute respiratory distress syndrome (ARDS) or pulmonary edema caused by heart failure, the nurse will plan to assist with a. obtaining a ventilation-perfusion scan. b. drawing blood for arterial blood gases. c. insertion of a pulmonary artery catheter. d. positioning the patient for a chest x-ray.

insertion of a pulmonary artery catheter.

A 51-year-old male patient has a new diagnosis of Crohn's disease after having frequent diarrhea and a weight loss of 10 pounds (4.5 kg) over 2 months. The nurse will plan to teach about a. medication use. b. fluid restriction. c. enteral nutrition. d. activity restrictions.

medication use. Medications are used to induce and maintain remission in patients with inflammatory bowel disease (IBD). Decreased activity level is indicated only if the patient has severe fatigue and weakness. Fluids are needed to prevent dehydration. There is no advantage to enteral feedings.

After receiving 2 L of normal saline, the central venous pressure for a patient who has septic shock is 10 mm Hg, but the blood pressure is still 82/40 mm Hg. The nurse will anticipate an order for a. nitroglycerine (Tridil). b. norepinephrine (Levophed). c. sodium nitroprusside (Nipride). d. methylprednisolone (Solu-Medrol).

norepinephrine (Levophed). When fluid resuscitation is unsuccessful, vasopressor drugs are administered to increase the systemic vascular resistance (SVR) and blood pressure, and improve tissue perfusion. Nitroglycerin would decrease the preload and further drop cardiac output and BP. Methylprednisolone (Solu-Medrol) is considered if blood pressure does not respond first to fluids and vasopressors. Nitroprusside is an arterial vasodilator and would further decrease SVR.

The laboratory has just called with the arterial blood gas (ABG) results on four patients. Which result is most important for the nurse to report immediately to the health care provider?

pH 7.31, PaO2 91 mm Hg, PaCO2 50 mm Hg, and O2 sat 96% These ABGs indicate uncompensated respiratory acidosis and should be reported to the health care provider.

A patient returning from surgery for a perineal radical prostatectomy will have a nursing diagnosis of risk for infection related to a. urinary incontinence. b. prolonged urinary stasis. c. possible fecal wound contamination. d. placement of a suprapubic bladder catheter.

possible fecal wound contamination. The perineal approach increases the risk for infection because the incision is located close to the anus and contamination with feces is possible. Urinary stasis and incontinence do not occur because the patient has a retention catheter in place for 1 to 2 weeks. A urethral catheter is used after the surgery.

When performing discharge teaching for a patient after a vasectomy, the nurse instructs the patient that he a. should continue to use other methods of birth control for 6 weeks. b. should not have sexual intercourse until his 6-week follow-up visit. c. may have temporary erectile dysfunction (ED) because of swelling. d. will notice a decrease in the appearance and volume of his ejaculate.

should continue to use other methods of birth control for 6 weeks. Because it takes about 6 weeks to evacuate sperm that are distal to the vasectomy site, the patient should use contraception for 6 weeks. ED that occurs after vasectomy is psychologic in origin and not related to postoperative swelling. The patient does not need to abstain from intercourse. The appearance and volume of the ejaculate are not changed because sperm are a minor component of the ejaculate.

The nurse will anticipate that a 61-year-old patient who has an enlarged prostate detected by digital rectal examination (DRE) and an elevated prostate specific antigen (PSA) level will need teaching about a. cystourethroscopy. b. uroflowmetry studies. c. magnetic resonance imaging (MRI). d. transrectal ultrasonography (TRUS).

transrectal ultrasonography (TRUS). In a patient with an abnormal DRE and elevated PSA, transrectal ultrasound is used to visualize the prostate for biopsy. Uroflowmetry studies help determine the extent of urine blockage and treatment, but there is no indication that this is a problem for this patient. Cystoscopy may be used before prostatectomy but will not be done until after the TRUS and biopsy. MRI is used to determine whether prostatic cancer has metastasized but would not be ordered at this stage of the diagnostic process.

The nurse taking a focused health history for a patient with possible testicular cancer will ask the patient about a history of a. testicular torsion. b. testicular trauma. c. undescended testicles. d. sexually transmitted infection (STI).

undescended testicles. Cryptorchidism is a risk factor for testicular cancer if it is not corrected before puberty. STI, testicular torsion, and testicular trauma are risk factors for other testicular conditions but not for testicular cancer.

A widowed mother of four school-age children is hospitalized with metastatic ovarian cancer. The patient is crying and tells the nurse that she does not know what will happen to her children when she dies. Which response by the nurse is most appropriate? a. "Why don't we talk about the options you have for the care of your children?" b. "I'm sure you have friends that will take the children when you can't care for them." c. "For now you need to concentrate on getting well and not worrying about your children." d. "Many patients with cancer live for a long time, so there is still time to plan for your children."

"Why don't we talk about the options you have for the care of your children?"

Which actions should the nurse initiate to reduce the risk for ventilator-associated pneumonia (VAP) (select all that apply)? a. Obtain arterial blood gases daily. b. Provide a "sedation holiday" daily. c. Elevate the head of the bed to at least 30°. d. Give prescribed pantoprazole (Protonix). e. Provide oral care with chlorhexidine (0.12%) solution daily.

-Provide a "sedation holiday" daily. -Elevate the head of the bed to at least 30°. -Give prescribed pantoprazole (Protonix). -Provide oral care with chlorhexidine (0.12%) solution daily.

Indication for diagnostic bronchoscopy

-chronic cough - persistent, unresolved -Hemoptysis- long term (Best standard) -Diagnostic bronchoalveolar lavage: infections, disease -Intrathoracic lymphadenopathy -foreign body, strictures, smoke inhalation(chemical burns or thermal) - can be done ICU can be done to help intubate

Which assessment is most important for the nurse to make regarding a patient with myasthenia gravis? a. Pupil size b. Grip strength c. Respiratory effort d. Level of consciousness

ANS: C Because respiratory insufficiency may be life threatening, it will be most important to monitor respiratory function. The other data also will be assessed but are not as critical. DIF: Cognitive Level: Apply (application) REF: 1438-1439 OBJ: Special Questions: Prioritization TOP: Nursing Process: Assessment MSC: NCLEX: Physiological Integrity

While caring for a patient who has been admitted with a pulmonary embolism, the nurse notes a change in the patient's oxygen saturation (SpO2) from 94% to 88%. Which action should the nurse take next? a. Increase the oxygen flow rate. b. Suction the patient's oropharynx. c. Instruct the patient to cough and deep breathe. d. Help the patient to sit in a more upright position.

Increase the oxygen flow rate.

A 64-year-old patient who has amyotrophic lateral sclerosis (ALS) is hospitalized with pneumonia. Which nursing action will be included in the plan of care? a. Assist with active range of motion (ROM). b. Observe for agitation and paranoia. c. Give muscle relaxants as needed to reduce spasms. d. Use simple words and phrases to explain procedures.

ANS: A ALS causes progressive muscle weakness, but assisting the patient to perform active ROM will help maintain strength as long as possible. Psychotic manifestations such as agitation and paranoia are not associated with ALS. Cognitive function is not affected by ALS, and the patient's ability to understand procedures will not be impaired. Muscle relaxants will further increase muscle weakness and depress respirations. DIF: Cognitive Level: Apply (application) REF: 1439 TOP: Nursing Process: Planning MSC: NCLEX: Physiological Integrity

A patient undergoing external radiation has developed a dry desquamation of the skin in the treatment area. The nurse teaches the patient about management of the skin reaction. Which statement, if made by the patient, indicates the teaching was effective? a. "I can buy some aloe vera gel to use on the area." b. "I will expose the treatment area to a sun lamp daily." c. "I can use ice packs to relieve itching in the treatment area." d. "I will scrub the area with warm water to remove the scales."

ANS: A Aloe vera gel and cream may be used on the radiated skin area. Ice and sunlamps may injure the skin. Treatment areas should be cleaned gently to avoid further injury.

A 45-year-old patient has a dysfunction of the cerebellum. The nurse will plan interventions to a. prevent falls. b. stabilize mood. c. avoid aspiration. d. improve memory.

ANS: A Because functions of the cerebellum include coordination and balance, the patient with dysfunction is at risk for falls. The cerebellum does not affect memory, mood, or swallowing ability. DIF: Cognitive Level: Apply (application) REF: 1339-1340 TOP: Nursing Process: Planning MSC: NCLEX: Physiological Integrity

After change-of-shift report, which patient should the nurse assess first? a. Patient with myasthenia gravis who is reporting increased muscle weakness b. Patient with a bilateral headache described as "like a band around my head" c. Patient with seizures who is scheduled to receive a dose of phenytoin (Dilantin) d. Patient with Parkinson's disease who has developed cogwheel rigidity of the arms

ANS: A Because increased muscle weakness may indicate the onset of a myasthenic crisis, the nurse should assess this patient first. The other patients should also be assessed, but do not appear to need immediate nursing assessments or actions to prevent life-threatening complications. DIF: Cognitive Level: Analyze (analysis) REF: 1438-1439 OBJ: Special Questions: Prioritization; Multiple Patients TOP: Nursing Process: Planning MSC: NCLEX: Safe and Effective Care Environment

A postoperative patient receiving a transfusion of packed red blood cells develops chills, fever, headache, and anxiety 35 minutes after the transfusion is started. After stopping the transfusion, what action should the nurse take? a. Draw blood for a new crossmatch. b. Send a urine specimen to the laboratory. c. Administer PRN acetaminophen (Tylenol). d. Give the PRN diphenhydramine (Benadryl).

Administer PRN acetaminophen (Tylenol). The patient's clinical manifestations are consistent with a febrile, nonhemolytic transfusion reaction. The transfusion should be stopped and antipyretics administered for the fever as ordered. A urine specimen is needed if an acute hemolytic reaction is suspected. Diphenhydramine (Benadryl) is used for allergic reactions. This type of reaction does not indicate incorrect crossmatching.

The nurse is performing complete assessment of a patient who has lung cancer. Which question does the nurse ask in the interview while assessing the cognitive-perceptual pattern in the patient? 1 "Do you have a nauseous feeling?" 2 "Do you have a habit of smoking?" 3 "Do you have pain in the shoulder and arm?" 4 "Do you have persistent cough and fatigue?"

Answer:3

Which action will the nurse include in the plan of care for a patient who has thalassemia major? a. Teach the patient to use iron supplements. b. Avoid the use of intramuscular injections. c. Administer iron chelation therapy as needed. d. Notify health care provider of hemoglobin 11g/dL.

Administer iron chelation therapy as needed. The frequent transfusions used to treat thalassemia major lead to iron toxicity in patients unless iron chelation therapy is consistently used. Iron supplementation is avoided in patients with thalassemia. There is no need to avoid intramuscular injections. The goal for patients with thalassemia major is to maintain a hemoglobin of 10 g/dL or greater.

A patient with metastatic cancer of the colon experiences severe vomiting following each administration of chemotherapy. Which action, if taken by the nurse, is most appropriate? a. Have the patient eat large meals when nausea is not present. b. Offer dry crackers and carbonated fluids during chemotherapy. c. Administer prescribed antiemetics 1 hour before the treatments. d. Give the patient two ounces of a citrus fruit beverage during treatments.

Administer prescribed antiemetics 1 hour before the treatments.

While caring for a patient with respiratory disease, the nurse observes that the patient's SpO2 drops from 93% to 88% while the patient is ambulating in the hallway. What is the priority action of the nurse?

Administer the PRN supplemental O2. The drop in SpO2 to 85% indicates that the patient is hypoxemic and needs supplemental oxygen when exercising.

A patient with renal failure has been taking aluminum hydroxide/magnesium hydroxide suspension (Maalox) at home for indigestion. The patient arrives for outpatient hemodialysis and is unresponsive to questions and has decreased deep tendon reflexes. Which action should the dialysis nurse take first? a. Notify the patient's health care provider. b. Obtain an order to draw a potassium level. c. Review the magnesium level on the patient's chart. d. Teach the patient about the risk of magnesium-containing antacids

Notify the patient's health care provider.

A patient has had lung cancer and is suspected to have metastasis. On what system should the nurse focus the assessment to determine this complication? 1 Liver 2 Lungs 3 Spleen 4 Pancreas

Answer: 1

A patient is suspected to have mediastinal involvement and lymph node enlargement related to a diagnosis of lung cancer. For which diagnostic procedure does the nurse prepare the patient that is most appropriate in confirming this suspicion? 1 X-ray 2 Bronchoscopy 3 Sputum cytology 4 Computerized tomography (CT) scan

Answer: 1

The diagnostic studies of a patient with primary lung cancer indicate that the tumor is peripherally located and has a moderate growth rate. What does the nurse interpret from these findings? 1 The patient has adenocarcinoma. 2 The patient has small cell carcinoma. 3 The patient has squamous cell carcinoma. 4 The patient has undifferentiated carcinoma

Answer: 1

When teaching the patient about reducing the risks of lung cancer, what is the most important topic for the nurse to address? 1 Smoking cessation 2 Drinking more fluids 3 Eating more grains 4 Using steam inhalation

Answer: 1

A 19-year-old patient with massive trauma and possible spinal cord injury is admitted to the emergency department (ED). Which assessment finding by the nurse will help confirm a diagnosis of neurogenic shock? a. Inspiratory crackles. b. Cool, clammy extremities. c. Apical heart rate 45 beats/min. d. Temperature 101.2° F (38.4° C).

Apical heart rate 45 beats/min. Neurogenic shock is characterized by hypotension and bradycardia. The other findings would be more consistent with other types of shock.

A 27-year-old female patient is admitted to the hospital for evaluation of right lower quadrant abdominal pain with nausea and vomiting. Which action should the nurse take? a. Encourage the patient to sip clear liquids. b. Assess the abdomen for rebound tenderness. c. Assist the patient to cough and deep breathe. d. Apply an ice pack to the right lower quadrant.

Apply an ice pack to the right lower quadrant. The patient's clinical manifestations are consistent with appendicitis, and application of an ice pack will decrease inflammation at the area. Checking for rebound tenderness frequently is unnecessary and uncomfortable for the patient. The patient should be NPO in case immediate surgery is needed. The patient will need to know how to cough and deep breathe postoperatively, but coughing will increase pain at this time.

A patient has acute bronchitis with a nonproductive cough and wheezes. Which topic should the nurse plan to include in the teaching plan? a. Purpose of antibiotic therapy b. Ways to limit oral fluid intake c. Appropriate use of cough suppressants d. Safety concerns with home oxygen therapy

Appropriate use of cough suppressants

An alcoholic and homeless patient is diagnosed with active tuberculosis (TB). Which intervention by the nurse will be most effective in ensuring adherence with the treatment regimen? a. Arrange for a friend to administer the medication on schedule. b. Give the patient written instructions about how to take the medications. c. Teach the patient about the high risk for infecting others unless treatment is followed. d. Arrange for a daily noon meal at a community center where the drug will be administered.

Arrange for a daily noon meal at a community center where the drug will be administered.

To evaluate the effectiveness of ordered interventions for a patient with ventilatory failure, which diagnostic test will be most useful to the nurse? a. Chest x-ray b. Oxygen saturation c. Arterial blood gas analysis d. Central venous pressure monitoring

Arterial blood gas analysis

After 2 months of tuberculosis (TB) treatment with isoniazid (INH), rifampin (Rifadin), pyrazinamide (PZA), and ethambutol, a patient continues to have positive sputum smears for acid-fast bacilli (AFB). Which action should the nurse take next?

Ask the patient whether medications have been taken as directed. The first action should be to determine whether the patient has been compliant with drug therapy because negative sputum smears would be expected if the TB bacillus is susceptible to the medications and if the medications have been taken correctly.

To prevent the debilitating cycle of fatigue-depression-fatigue that can occur in patients with cancer, an appropriate nursing intervention is to: a. Have the patient rest after any major energy expenditure b. Encourage the patient to implement a daily walking program c. Teach the patient to ignore the fatigue to maintain normal daily activities d. Prevent the development of depression by informing the patient to expect fatigue during cancer treatment.

B. Rationale: Exercise and activity within tolerable limits are often helpful in managing fatigue. Walking programs are a way for most patients to stay active without being overtaxed.

Which information about continuous bladder irrigation will the nurse teach to a patient who is being admitted for a transurethral resection of the prostate (TURP)? a. Bladder irrigation decreases the risk of postoperative bleeding. b. Hydration and urine output are maintained by bladder irrigation. c. Antibiotics are infused continuously through the bladder irrigation. d. Bladder irrigation prevents obstruction of the catheter after surgery.

Bladder irrigation prevents obstruction of the catheter after surgery. The purpose of bladder irrigation is to remove clots from the bladder and to prevent obstruction of the catheter by clots. The irrigation does not decrease bleeding or improve hydration. Antibiotics are given by the IV route, not through the bladder irrigation

A patient has just been admitted with probable bacterial pneumonia and sepsis. Which order should the nurse implement first? a. Chest x-ray via stretcher b. Blood cultures from two sites c. Ciprofloxacin (Cipro) 400 mg IV d. Acetaminophen (Tylenol) rectal suppository

Blood cultures from two sites

A patient has just been admitted with probable bacterial pneumonia and sepsis. Which order should the nurse implement first?

Blood cultures from two sites Initiating antibiotic therapy rapidly is essential, but it is important that the cultures be obtained before antibiotic administration.

The nurse is caring for a patient with a massive burn injury and possible hypovolemia. Which assessment data will be of most concern to the nurse? a. Blood pressure is 90/40 mm Hg. b. Urine output is 30 mL over the last hour. c. Oral fluid intake is 100 mL for the last 8 hours. d. There is prolonged skin tenting over the sternum.

Blood pressure is 90/40 mm Hg.

Which cellular dysfunction in the process of cancer development allows defective cell proliferation? A. Proto-oncogenes B. Cell differentiation C. Dynamic equilibrium D. Activation of oncogenes

C. Dynamic equilibrium Dynamic equilibrium is the regulation of proliferation that usually only occurs to equal cell degeneration or death or when the body has a physiologic need for more cells. Cell differentiation is the orderly process that progresses a cell from a state of immaturity to a state of differentiated maturity. Mutations that alter the expression of proto-oncogenes can activate them to function as oncogenes, which are tumor-inducing genes and alter their differentiation.

A 33-year-old patient has recently been diagnosed with stage II cervical cancer. What should the nurse understand about the patient's cancer? A. It is in situ. B. It has metastasized. C. It has spread locally. D. It has spread extensively.

C. It has spread locally Stage II cancer is associated with limited local spread. Stage 0 denotes cancer in situ; stage I denotes tumor limited to the tissue of origin with localized tumor growth. Stage III denotes extensive local and regional spread. Stage IV denotes metastasis.

The patient is told that the adenoma tumor is not encapsulated but has normally differentiated cells and that surgery will be needed. The patient asks the nurse what this means. What should the nurse tell the patient? A. It will recur. B. It has metastasized. C. It is probably benign. Correct D. It is probably malignant.

C. It is probably benign. Benign tumors usually are encapsulated and have normally differentiated cells. They do not metastasize and rarely recur as malignant tumors do.

Which finding about a patient with polycythemia vera is most important for the nurse to report to the health care provider? a. Hematocrit 55% b. Presence of plethora c. Calf swelling and pain d. Platelet count 450,000/L

Calf swelling and pain The calf swelling and pain suggest that the patient may have developed a deep vein thrombosis, which will require diagnosis and treatment to avoid complications such as pulmonary embolus. The other findings will also be reported to the health care provider but are expected in a patient with this diagnosis.

After being hit by a baseball, a patient arrives in the emergency department with a possible nasal fracture. Which finding by the nurse is most important to report to the health care provider?

Clear nasal drainage Clear nasal drainage may indicate a meningeal tear with leakage of cerebrospinal fluid

Cancer Gerontologic Considerations

Clinical manifestations may be mistaken for age-related changes. More vulnerable to complications of cancer and cancer therapy Functional status should be considered when a treatment plan is selected. Survival rates have steadily increased over past 30 years. Goal is to maintain an optimal quality of life.

Which breakfast choice indicates a patient's good understanding of information about a diet for celiac disease? a. Oatmeal with nonfat milk b. Whole wheat toast with butter c. Bagel with low-fat cream cheese d. Corn tortilla with scrambled eggs

Corn tortilla with scrambled eggs Avoidance of gluten-containing foods is the only treatment for celiac disease. Corn does not contain gluten, while oatmeal and wheat do.

A 72-year-old male patient with dehydration caused by an exacerbation of ulcerative colitis is receiving 5% dextrose in normal saline at 125 mL/hour. Which assessment finding by the nurse is most important to report to the health care provider? a. Patient has not voided for the last 4 hours. b. Skin is dry with poor turgor on all extremities. c. Crackles are heard halfway up the posterior chest. d. Patient has had 5 loose stools over the last 6 hours.

Crackles are heard halfway up the posterior chest. The presence of crackles in an older patient receiving IV fluids at a high rate suggests volume overload and a need to reduce the rate of the IV infusion. The other data will also be reported, but are consistent with the patient's age and diagnosis and do not require a change in the prescribed treatment.

The nurse assesses a patient who is receiving interleukin-2. Which finding should the nurse report immediately to the health care provider? a. Generalized muscle aches b. Complaints of nausea and anorexia c. Oral temperature of 100.6° F (38.1° C) d. Crackles heard at the lower scapular border

Crackles heard at the lower scapular border

Which item would be most beneficial when providing oral care to a patient with metastatic cancer who is at risk for oral tissue injury secondary to chemotherapy? a. Firm-bristle toothbrush b. Hydrogen peroxide rinse c. Alcohol-based mouthwash d. 1 tsp salt in 1 L water mouth rinse

D. Rationale: A salt-water mouth rinse will not cause further irritation to oral tissue that is fragile because of mucositis, which is a side effect of chemotherapy. A soft-bristle toothbrush will be used. One teaspoon of sodium bicarbonate may be added to the salt-water solution to decrease odor, alleviate pain, and dissolve mucin. Hydrogen peroxide and alcohol-based mouthwash are not used because they would damage the oral tissue.

A patient is admitted for hypovolemia associated with multiple draining wounds. Which assessment would be the most accurate way for the nurse to evaluate fluid balance? a. Skin turgor b. Daily weight c. Presence of edema d. Hourly urine output

Daily weight

A 21-year-old woman presents to the ED complaining of lightheadedness. Her symptoms appeared 45 minutes ago. She has no other symptoms and is not on any medications. She has a medical history of mitral valve prolapse. Her HR is 170 beats per minute and BP is 105/55 mm Hg. Physical examination is unremarkable. After administering the appropriate medication, her HR slows down and her symptoms resolve. You repeat a 12-lead ECG that shows a rate of 89 beats per minute with a regular rhythm. The PR interval measures 100 msec and there is a slurred upstroke of the QRS complex. Based on this information, which of the following is the most likely diagnosis? a. Ventricular tachycardia b. Atrial flutter with 3:1 block c. Atrial fibrillation d. Lown-Ganong-Levine (LGL) syndrome e. Wolff-Parkinson-White (WPW) syndrome

E. WPW syndrome is caused by an accessory electrical pathway (ie, Bundle of Kent) between the atria and ventricles. The primary significance of WPW syndrome is that it predisposes the individual to the development of reentry tachycardias. The classic ECG findings include a short PR interval (< 120 msec), widened QRS interval (> 100 msec), and a delta wave (slurred upstroke at the beginning of the QRS).

An older adult patient who is malnourished presents to the emergency department with a serum protein level of 5.2 g/dL. The nurse would expect which clinical manifestation? a. Pallor b. Edema c. Confusion d. Restlessness

Edema

A nurse is caring for a patient who is orally intubated and receiving mechanical ventilation. To decrease the risk for ventilator-associated pneumonia, which action will the nurse include in the plan of care? a. Elevate head of bed to 30 to 45 degrees. b. Suction the endotracheal tube every 2 to 4 hours. c. Limit the use of positive end-expiratory pressure. d. Give enteral feedings at no more than 10 mL/hr.

Elevate head of bed to 30 to 45 degrees.

A patient with a possible pulmonary embolism complains of chest pain and difficulty breathing. The nurse finds a heart rate of 142 beats/minute, blood pressure of 100/60 mmHg, and respirations of 42 breaths/minute. Which action should the nurse take first?

Elevate the head of the bed to a semi-Fowler's position. The patient has symptoms consistent with a pulmonary embolism (PE). Elevating the head of the bed will improve ventilation and gas exchange. The other actions can be accomplished after the head is elevated (and oxygen is started).

Cancer Psychologic Support

Emphasis is placed on maintaining optimal quality of life. Positive attitude of patient, family, and health care providers can have a positive impact on the patient's quality of life. May also influence prognosis Common fears Disfigurement, emaciation Dependency Disruption of relationships Pain Financial depletion Abandonment Death Coping with cancer Coping with stress in the past Availability of significant others Ability to express feelings/concerns Age at time of diagnosis Extent of disease Disruption of body image Presence of symptoms Past experiences with cancer Attitude associated with cancer

A patient complains of gas pains and abdominal distention two days after a small bowel resection. Which nursing action is best to take? a. Encourage the patient to ambulate. b. Instill a mineral oil retention enema. c. Administer the ordered IV morphine sulfate. d. Offer the ordered promethazine (Phenergan) suppository.

Encourage the patient to ambulate. Ambulation will improve peristalsis and help the patient eliminate flatus and reduce gas pain. A mineral oil retention enema is helpful for constipation with hard stool. A return-flow enema might be used to relieve persistent gas pains. Morphine will further reduce peristalsis. Promethazine (Phenergan) is used as an antiemetic rather than to decrease gas pains or distention.

Which nursing action will be included in the plan of care for a 27-year-old male patient with bowel irregularity and a new diagnosis of irritable bowel syndrome (IBS)? a. Encourage the patient to express concerns and ask questions about IBS. b. Suggest that the patient increase the intake of milk and other dairy products. c. Educate the patient about the use of alosetron (Lotronex) to reduce symptoms. d. Teach the patient to avoid using nonsteroidal antiinflammatory drugs (NSAIDs).

Encourage the patient to express concerns and ask questions about IBS. Because psychologic and emotional factors can affect the symptoms for IBS, encouraging the patient to discuss emotions and ask questions is an important intervention. Alosetron has serious side effects, and is used only for female patients who have not responded to other therapies. Although yogurt may be beneficial, milk is avoided because lactose intolerance can contribute to symptoms in some patients. NSAIDs can be used by patients with IBS.

A 24-year-old woman with Crohn's disease develops a fever and symptoms of a urinary tract infection (UTI) with tan, fecal-smelling urine. What information will the nurse add to a general teaching plan about UTIs in order to individualize the teaching for this patient? a. Bacteria in the perianal area can enter the urethra. b. Fistulas can form between the bowel and bladder. c. Drink adequate fluids to maintain normal hydration. d. Empty the bladder before and after sexual intercourse.

Fistulas can form between the bowel and bladder. Fistulas between the bowel and bladder occur in Crohn's disease and can lead to UTI. Teaching for UTI prevention in general includes good hygiene, adequate fluid intake, and voiding before and after intercourse.

The following interventions are ordered by the health care provider for a patient who has respiratory distress and syncope after eating strawberries. Which will the nurse complete first? a. Start a normal saline infusion. b. Give epinephrine (Adrenalin). c. Start continuous ECG monitoring. d. Give diphenhydramine (Benadryl).

Give epinephrine (Adrenalin). Epinephrine rapidly causes peripheral vasoconstriction, dilates the bronchi, and blocks the effects of histamine and reverses the vasodilation, bronchoconstriction, and histamine release that cause the symptoms of anaphylaxis. The other interventions are also appropriate but would not be the first ones completed.

A patient with septic shock has a BP of 70/46 mm Hg, pulse 136, respirations 32, temperature 104° F, and blood glucose 246 mg/dL. Which intervention ordered by the health care provider should the nurse implement first? a. Give normal saline IV at 500 mL/hr. b. Give acetaminophen (Tylenol) 650 mg rectally. c. Start insulin drip to maintain blood glucose at 110 to 150 mg/dL. d. Start norepinephrine (Levophed) to keep systolic blood pressure >90 mm Hg.

Give normal saline IV at 500 mL/hr. Because of the low systemic vascular resistance (SVR) associated with septic shock, fluid resuscitation is the initial therapy. The other actions also are appropriate, and should be initiated quickly as well.

A patient who had a transverse colectomy for diverticulosis 18 hours ago has nasogastric suction and is complaining of anxiety and incisional pain. The patient's respiratory rate is 32 breaths/minute and the arterial blood gases (ABGs) indicate respiratory alkalosis. Which action should the nurse take first? a. Discontinue the nasogastric suction. b. Give the patient the PRN IV morphine sulfate 4 mg. c. Notify the health care provider about the ABG results. d. Teach the patient how to take slow, deep breaths when anxious.

Give the patient the PRN IV morphine sulfate 4 mg.

A patient with septicemia develops prolonged bleeding from venipuncture sites and blood in the stools. Which action is most important for the nurse to take? a. Avoid venipunctures. b. Notify the patient's physician. c. Apply sterile dressings to the sites. d. Give prescribed proton-pump inhibitors.

Notify the patient's physician The patient's new onset of bleeding and diagnosis of sepsis suggest that disseminated intravascular coagulation (DIC) may have developed, which will require collaborative actions such as diagnostic testing, blood product administration, and heparin administration. The other actions also are appropriate, but the most important action should be to notify the physician so that DIC treatment can be initiated rapidly.

When assessing a patient who has just arrived after an automobile accident, the emergency department nurse notes tachycardia and absent breath sounds over the right lung. For which intervention will the nurse prepare the patient? a. Emergency pericardiocentesis b. Stabilization of the chest wall with tape c. Administration of an inhaled bronchodilator d. Insertion of a chest tube with a chest drainage system

Insertion of a chest tube with a chest drainage system

A 76-year-old patient with obstipation has a fecal impaction and is incontinent of liquid stool. Which action should the nurse take first? a. Administer bulk-forming laxatives. b. Assist the patient to sit on the toilet. c. Manually remove the impacted stool. d. Increase the patient's oral fluid intake.

Manually remove the impacted stool. The initial action with a fecal impaction is manual disimpaction. The other actions will be used to prevent future constipation and impactions.

The nurse notes that a patient has incisional pain, a poor cough effort, and scattered rhonchi after a thoracotomy. Which action should the nurse take first?

Medicate the patient with prescribed morphine A major reason for atelectasis and poor airway clearance in patients after chest surgery is incisional pain (which increases with deep breathing and coughing).

A patient who is lethargic and exhibits deep, rapid respirations has the following arterial blood gas (ABG) results: pH 7.32, PaO2 88 mm Hg, PaCO2 37 mm Hg, and HCO3 16 mEq/L. How should the nurse interpret these results? a. Metabolic acidosis b. Metabolic alkalosis c. Respiratory acidosis d. Respiratory alkalosis

Metabolic acidosis

The nurse administers prescribed therapies for a patient with cor pulmonale and right-sided heart failure. Which assessment would best evaluate the effectiveness of the therapies?

Observe for distended neck veins Cor pulmonale is right ventricular failure caused by pulmonary hypertension, so clinical manifestations of right ventricular failure such as peripheral edema, jugular venous distention, and right upper-quadrant abdominal tenderness would be expected.

Which action will the nurse include in the plan of care for a 72-year-old woman admitted with multiple myeloma? a. Monitor fluid intake and output. b. Administer calcium supplements. c. Assess lymph nodes for enlargement. d. Limit weight bearing and ambulation.

Monitor fluid intake and output. A high fluid intake and urine output helps prevent the complications of kidney stones caused by hypercalcemia and renal failure caused by deposition of Bence-Jones protein in the renal tubules. Weight bearing and ambulation are encouraged to help bone retain calcium. Lymph nodes are not enlarged with multiple myeloma. Calcium supplements will further increase the patient's calcium level and are not used.

The nurse is planning to administer a transfusion of packed red blood cells (PRBCs) to a patient with blood loss from gastrointestinal hemorrhage. Which action can the nurse delegate to unlicensed assistive personnel (UAP)? a. Verify the patient identification (ID) according to hospital policy. b. Obtain the temperature, blood pressure, and pulse before the transfusion. c. Double-check the product numbers on the PRBCs with the patient ID band. d. Monitor the patient for shortness of breath or chest pain during the transfusion.

Obtain the temperature, blood pressure, and pulse before the transfusion. UAP education includes measurement of vital signs. UAP would report the vital signs to the registered nurse (RN). The other actions require more education and a larger scope of practice and should be done by licensed nursing staff members.

The clinic nurse teaches a patient with a 42 pack-year history of cigarette smoking about lung disease. Which information will be most important for the nurse to include?

Options for smoking cessation Because smoking is the major cause of lung cancer, the most important role for the nurse is teaching patients about the benefits of and means of smoking cessation

A patient who has been receiving diuretic therapy is admitted to the emergency department with a serum potassium level of 3.0 mEq/L. The nurse should alert the health care provider immediately that the patient is on which medication? a. Oral digoxin (Lanoxin) 0.25 mg daily b. Ibuprofen (Motrin) 400 mg every 6 hours c. Metoprolol (Lopressor) 12.5 mg orally daily d. Lantus insulin 24 U subcutaneously every evening

Oral digoxin (Lanoxin) 0.25 mg daily

A nurse is caring for a patient with acute respiratory distress syndrome (ARDS) who is receiving mechanical ventilation using synchronized intermittent mandatory ventilation (SIMV). The settings include fraction of inspired oxygen (FIO2) 80%, tidal volume 450, rate 16/minute, and positive end-expiratory pressure (PEEP) 5 cm. Which assessment finding is most important for the nurse to report to the health care provider? a. Oxygen saturation 99% b. Respiratory rate 22 breaths/minute c. Crackles audible at lung bases d. Heart rate 106 beats/minute

Oxygen saturation 99%

The nurse cares for a patient who has just had a thoracentesis. Which assessment information obtained by the nurse is a priority to communicate to the health care provider? a. Oxygen saturation is 88%. b. Blood pressure is 145/90 mm Hg. c. Respiratory rate is 22 breaths/minute when lying flat. d. Pain level is 5 (on 0 to 10 scale) with a deep breath.

Oxygen saturation is 88%.

A patient experiences a chest wall contusion as a result of being struck in the chest with a baseball bat. The emergency department nurse would be most concerned if which finding is observed during the initial assessment? a. Paradoxic chest movement b. Complaint of chest wall pain c. Heart rate of 110 beats/minute d. Large bruised area on the chest

Paradoxic chest movement

A patient experiences a chest wall contusion as a result of being struck in the chest with a baseball bat. The emergency department nurse would be most concerned if which finding is observed during the initial assessment?

Paradoxic chest movement Paradoxic chest movement indicates that the patient may have flail chest, which can severely compromise gas exchange and can rapidly lead to hypoxemia.

Hypercalcemia

Parathyroid hormone-like substance secreted from cancer cells in absence of bony metastasis Signs include Apathy, depression, fatigue, weak muscles Electrocardiogram changes, polyuria, nocturia, anorexia, nausea, vomiting Can be life-threatening Nursing Care: Treatment is aimed at primary disease. Acute hypercalcemia is treated by Hydration Diuretic administration Bisphosphonate

Which action should the nurse take first when a patient develops a nosebleed?

Pinch the lower portion of the nose for 10 minutes. The first nursing action for epistaxis is to apply direct pressure by pinching the nostrils.

The nurse develops a plan of care to prevent aspiration in a high-risk patient. Which nursing action will be most effective? a. Turn and reposition immobile patients at least every 2 hours. b. Place patients with altered consciousness in side-lying positions. c. Monitor for respiratory symptoms in patients who are immunosuppressed. d. Insert nasogastric tube for feedings for patients with swallowing problems.

Place patients with altered consciousness in side-lying positions.

Which action will the nurse include in the plan of care for a 42-year-old patient who is being admitted with Clostridium difficile? a. Educate the patient about proper food storage. b. Order a diet with no dairy products for the patient. c. Place the patient in a private room on contact isolation. d. Teach the patient about why antibiotics will not be used.

Place the patient in a private room on contact isolation. Because C. difficile is highly contagious, the patient should be placed in a private room and contact precautions should be used. There is no need to restrict dairy products for this type of diarrhea. Metronidazole (Flagyl) is frequently used to treat C. difficile. Improper food handling and storage do not cause C. difficile.

Following a thyroidectomy, a patient complains of "a tingling feeling around my mouth." Which assessment should the nurse complete immediately? a. Presence of the Chvostek's sign b. Abnormal serum potassium level c. Decreased thyroid hormone level d. Bleeding on the patient's dressing

Presence of the Chvostek's sign

A patient with right lower-lobe pneumonia has been treated with IV antibiotics for 3 days. Which assessment data obtained by the nurse indicates that the treatment has been effective? a. Bronchial breath sounds are heard at the right base. b. The patient coughs up small amounts of green mucus. c. The patient's white blood cell (WBC) count is 9000/µL. d. Increased tactile fremitus is palpable over the right chest.

The patient's white blood cell (WBC) count is 9000/µL

A patient scheduled for a total laryngectomy and radical neck dissection for cancer of the larynx asks the nurse, "Will I be able to talk normally after surgery?" What is the best response by the nurse?

You will have a permanent opening into your neck, and you will need to have rehabilitation for some type of voice restoration." Voice rehabilitation is planned after a total laryngectomy, and a variety of assistive devices are available to restore communication

A patient is admitted to the emergency department (ED) for shock of unknown etiology. The first action by the nurse should be to a. administer oxygen. b. obtain a 12-lead electrocardiogram (ECG). c. obtain the blood pressure. d. check the level of consciousness.

administer oxygen. The initial actions of the nurse are focused on the ABCs—airway, breathing, and circulation—and administration of oxygen should be done first. The other actions should be accomplished as rapidly as possible after oxygen administration.

An alcoholic and homeless patient is diagnosed with active tuberculosis (TB). Which intervention by the nurse will be most effective in ensuring adherence with the treatment regimen?

b. Arrange for a daily noon meal at a community center where the drug will be administered Directly observed therapy is the most effective means for ensuring compliance with the treatment regimen, and arranging a daily meal will help ensure that the patient is available to receive the medication.

A nurse is caring for a patient who has had a total laryngectomy and radical neck dissection. During the first 24 hours after surgery what is the priority nursing action?

c. Keep the patient in semi-Fowler's position The most important goals after a laryngectomy and radical neck dissection are to maintain the airway and ensure adequate oxygenation.

The nurse is caring for a patient who is intubated and receiving positive pressure ventilation to treat acute respiratory distress syndrome (ARDS). Which finding is most important to report to the health care provider? a. Blood urea nitrogen (BUN) level 32 mg/dL b. Red-brown drainage from orogastric tube c. Scattered coarse crackles heard throughout lungs d. Arterial blood gases: pH 7.31, PaCO2 50, PaO2 68

Red-brown drainage from orogastric tube

An occupational health nurse works at a manufacturing plant where there is potential exposure to inhaled dust. Which action, if recommended by the nurse, will be most helpful in reducing the incidence of lung disease? a. Treat workers with pulmonary fibrosis. b. Teach about symptoms of lung disease. c. Require the use of protective equipment. d. Monitor workers for coughing and wheezing.

Require the use of protective equipment.

The nurse should include which food choice when providing dietary teaching for a patient scheduled to receive external beam radiation for abdominal cancer? a. Fresh fruit salad b. Roasted chicken c. Whole wheat toast d. Cream of potato soup

Roasted chicken

The nurse is assessing a 31-year-old female patient with abdominal pain. Th nurse,who notes that there is ecchymosis around the area of umbilicus, will document this finding as a. Cullen sign. b. Rovsing sign. c. McBurney sign. d. Grey-Turner's signt.

Rovsing sign. Cullen sign is ecchymosis around the umbilicus. Rovsing sign occurs when palpation of the left lower quadrant causes pain in the right lower quadrant. Deep tenderness at McBurney's point (halfway between the umbilicus and the right iliac crest), known as McBurney's sign, is a sign of acute appendicitis

When a patient with splenomegaly is scheduled for splenectomy, which action will the nurse include in the preoperative plan of care? a. Discourage deep breathing to reduce risk for splenic rupture. b. Teach the patient to use ibuprofen (Advil) for left upper quadrant pain. c. Schedule immunization with the pneumococcal vaccine (Pneumovax). d. Avoid the use of acetaminophen (Tylenol) for 2 weeks prior to surgery.

Schedule immunization with the pneumococcal vaccine (Pneumovax). Asplenic patients are at high risk for infection with Pneumococcus and immunization reduces this risk. There is no need to avoid acetaminophen use before surgery, but nonsteroidal antiinflammatory drugs (NSAIDs) may increase bleeding risk and should be avoided. The enlarged spleen may decrease respiratory depth and the patient should be encouraged to take deep breaths.

Which prescribed intervention for a 61-year-old female patient with chronic short bowel syndrome will the nurse question? a. Ferrous sulfate (Feosol) 325 mg daily b. Senna (Senokot) 1 tablet every day c. Psyllium (Metamucil) 2.1 grams 3 times daily d. Diphenoxylate with atropine (Lomotil) prn loose stools

Senna (Senokot) 1 tablet every day Patients with short bowel syndrome have diarrhea because of decreased nutrient and fluid absorption and would not need stimulant laxatives. Iron supplements are used to prevent iron-deficiency anemia, bulk-forming laxatives help make stools less watery, and opioid antidiarrheal drugs are helpful in slowing intestinal transit time.

A patient is admitted to the emergency department with severe fatigue and confusion. Laboratory studies are done. Which laboratory value will require the most immediate action by the nurse? a. Arterial blood pH is 7.32. b. Serum calcium is 18 mg/dL. c. Serum potassium is 5.1 mEq/L. d. Arterial oxygen saturation is 91%.

Serum calcium is 18 mg/dL.

Which information obtained by the nurse assessing a patient admitted with multiple myeloma is most important to report to the health care provider? a. Serum calcium level is 15 mg/dL. b. Patient reports no stool for 5 days. c. Urine sample has Bence-Jones protein. d. Patient is complaining of severe back pain.

Serum calcium level is 15 mg/dL. Hypercalcemia may lead to complications such as dysrhythmias or seizures, and should be addressed quickly. The other patient findings will also be discussed with the health care provider, but are not life threatening

The nurse is caring for a patient who has septic shock. Which assessment finding is most important for the nurse to report to the health care provider? a. Blood pressure (BP) 92/56 mm Hg b. Skin cool and clammy c. Oxygen saturation 92% d. Heart rate 118 beats/minute

Skin cool and clammy Because patients in the early stage of septic shock have warm and dry skin, the patient's cool and clammy skin indicates that shock is progressing. The other information will also be reported, but does not indicate deterioration of the patient's status.

Which action should the nurse take when caring for a patient who is receiving chemotherapy and complains of problems with concentration? a. Teach the patient to rest the brain by avoiding new activities. b. Teach that "chemo-brain" is a short-term effect of chemotherapy. c. Report patient symptoms immediately to the health care provider. d. Suggest use of a daily planner and encourage adequate rest and sleep.

Suggest use of a daily planner and encourage adequate rest and sleep.

Metabolic emergencies caused by production of ectopic hormones

Syndrome of inappropriate antidiuretic hormone (ADH) secretion Hypercalcemia Tumor lysis syndrome Septic shock Disseminated intravascular coagulation (DIC)

A 30-year-old man with acute myelogenous leukemia develops an absolute neutrophil count of 850/µL while receiving outpatient chemotherapy. Which action by the outpatient clinic nurse is most appropriate? a. Discuss the need for hospital admission to treat the neutropenia. b. Teach the patient to administer filgrastim (Neupogen) injections. c. Plan to discontinue the chemotherapy until the neutropenia resolves. d. Order a high-efficiency particulate air (HEPA) filter for the patient's home.

Teach the patient to administer filgrastim (Neupogen) injections. The patient may be taught to self-administer filgrastim injections. Although chemotherapy may be stopped with severe neutropenia (neutrophil count less than 500/µL), administration of filgrastim usually allows the chemotherapy to continue. Patients with neutropenia are at higher risk for infection when exposed to other patients in the hospital. HEPA filters are expensive and are used in the hospital, where the number of pathogens is much higher than in the patient's home environment.

A patient who has a right-sided chest tube following a thoracotomy has continuous bubbling in the suction-control chamber of the collection device. Which action by the nurse is most appropriate? a. Document the presence of a large air leak. b. Notify the surgeon of a possible pneumothorax. c. Take no further action with the collection device. d. Adjust the dial on the wall regulator to decrease suction.

Take no further action with the collection device.

A 62-year-old patient has had a hemorrhoidectomy at an outpatient surgical center. Which instructions will the nurse include in discharge teaching? a. Maintain a low-residue diet until the surgical area is healed. b. Use ice packs on the perianal area to relieve pain and swelling. c. Take prescribed pain medications before a bowel movement is expected. d. Delay having a bowel movement for several days until healing has occurred.

Take prescribed pain medications before a bowel movement is expected. Bowel movements may be very painful, and patients may avoid defecation unless pain medication is taken before the bowel movement. A high-residue diet will increase stool bulk and prevent constipation. Delay of bowel movements is likely to lead to constipation. Warm sitz baths rather than ice packs are used to relieve pain and keep the surgical area clean.

Which information will the nurse plan to include when teaching a community health group about testicular self-examination? a. Testicular self-examination should be done in a warm room. b. The only structure normally felt in the scrotal sac is the testis. c. Testicular self-examination should be done at least every week. d. Call the health care provider if one testis is larger than the other.

Testicular self-examination should be done in a warm room. The testes will hang lower in the scrotum when the temperature is warm (e.g., during a shower), and it will be easier to palpate. The epididymis is also normally palpable in the scrotum. One testis is normally larger. The patient should perform testicular self-examination monthly.

When caring for a patient who is pancytopenic, which action by unlicensed assistive personnel (UAP) indicates a need for the nurse to intervene? a. The UAP assists the patient to use dental floss after eating. b. The UAP adds baking soda to the patient's saline oral rinses. c. The UAP puts fluoride toothpaste on the patient's toothbrush. d. The UAP has the patient rinse after meals with a saline solution.

The UAP assists the patient to use dental floss after eating.

The nurse supervises the care of a patient with a temporary radioactive cervical implant. Which action by unlicensed assistive personnel (UAP), if observed by the nurse, would require an intervention? a. The UAP flushes the toilet once after emptying the patient's bedpan. b. The UAP stands by the patient's bed for 30 minutes talking with the patient. c. The UAP places the patient's bedding in the laundry container in the hallway. d. The UAP gives the patient an alcohol-containing mouthwash to use for oral care.

The UAP stands by the patient's bed for 30 minutes talking with the patient.

The nurse is caring for a hospitalized older patient who has nasal packing in place to treat a nosebleed. Which assessment finding will require the most immediate action by the nurse?

The oxygen saturation is 89% Older patients with nasal packing are at risk of aspiration or airway obstruction. An O2 saturation of 89% should alert the nurse to further assess for these complications.

The nurse analyzes the results of a patient's arterial blood gases (ABGs). Which finding would require immediate action?

The partial pressure of oxygen in arterial blood (PaO2) is 59 mm Hg. All the values are abnormal, but the low PaO2 indicates that the patient is at the point on the oxyhemoglobin dissociation curve where a small change in the PaO2 will cause a large drop in the O2 saturation and a decrease in tissue oxygenation. The nurse should intervene immediately to improve the patient's oxygenation.

A patient who had a total laryngectomy has a nursing diagnosis of hopelessness related to loss of control of personal care. Which information obtained by the nurse is the best indicator that this identified problem is resolving?

The patient asks how to clean the tracheostomy stoma and tube. Independently caring for the laryngectomy tube indicates that the patient has regained control of personal care and hopelessness is at least partially resolved.

Which assessment information is most important for the nurse to report to the health care provider when a patient asks for a prescription for testosterone replacement therapy (TRT)? a. The patient has noticed a decrease in energy level for a few years. b. The patient's symptoms have increased steadily over the last few years. c. The patient has been using sildenafil (Viagra) several times every week. d. The patient has had a gradual decrease in the force of his urinary stream.

The patient has had a gradual decrease in the force of his urinary stream. The decrease in urinary stream may indicate benign prostatic hyperplasia (BPH) or prostate cancer, which are contraindications to the use of testosterone replacement therapy (TRT). The other patient data indicate that TRT may be a helpful therapy for the patient.

Which information obtained by the nurse interviewing a 30-year-old male patient is most important to communicate to the health care provider? a. The patient has a history of constipation. b. The patient has noticed blood in the stools. c. The patient had an appendectomy at age 27. d. The patient smokes a pack/day of cigarettes.

The patient has noticed blood in the stools. Blood in the stools is a possible clinical manifestation of colorectal cancer and requires further assessment by the health care provider. The other patient information will also be communicated to the health care provider, but does not indicate an urgent need for further testing or intervention.

A nurse is caring for a patient with ARDS who is being treated with mechanical ventilation and high levels of positive end-expiratory pressure (PEEP). Which assessment finding by the nurse indicates that the PEEP may need to be reduced? a. The patient's PaO2 is 50 mm Hg and the SaO2 is 88%. b. The patient has subcutaneous emphysema on the upper thorax. c. The patient has bronchial breath sounds in both the lung fields. d. The patient has a first-degree atrioventricular heart block with a rate of 58.

The patient has subcutaneous emphysema on the upper thorax.

A 19-year-old woman with immune thrombocytopenic purpura (ITP) has an order for a platelet transfusion. Which information indicates that the nurse should consult with the health care provider before obtaining and administering platelets? a. The platelet count is 42,000/L. b. Petechiae are present on the chest. c. Blood pressure (BP) is 94/56 mm Hg. d. Blood is oozing from the venipuncture site.

The platelet count is 42,000/L. Platelet transfusions are not usually indicated until the platelet count is below 10,000 to 20,000/L unless the patient is actively bleeding. Therefore the nurse should clarify the order with the health care provider before giving the transfusion. The other data all indicate that bleeding caused by ITP may be occurring and that the platelet transfusion is appropriate.

Tumor lysis syndrome

Triggered by chemotherapy's rapid destruction of large numbers of tumor cells ↑ Serum phosphate causes calcium to go ↓ resulting in hypocalcemia. Can cause biochemical changes resulting in renal failure Can be fatal Four hallmark signs Hyperuricemia Hyperphosphatemia Hyperkalemia Hypocalcemia Usually occurs within 24 to 48 hours after chemotherapy May persist for 5 to 7 days Nursing Care: Primary goal Preventing renal failure and severe electrolyte imbalance Primary treatment Increase urine production with hydration therapy Decreased uric acid concentrations

A patient with a large stomach tumor that is attached to the liver is scheduled to have a debulking procedure. Which information should the nurse teach the patient about the outcome of this procedure? a. Pain will be relieved by cutting sensory nerves in the stomach. b. Relief of pressure in the stomach will promote better nutrition. c. Tumor growth will be controlled by the removal of malignant tissue. d. Tumor size will decrease and this will improve the effects of other therapy.

Tumor size will decrease and this will improve the effects of other therapy.

The nurse supervises unlicensed assistive personnel (UAP) who are providing care for a patient with right lower lobe pneumonia. The nurse should intervene if which action by UAP is observed? a. UAP splint the patient's chest during coughing. b. UAP assist the patient to ambulate to the bathroom. c. UAP help the patient to a bedside chair for meals. d. UAP lower the head of the patient's bed to 15 degrees.

UAP lower the head of the patient's bed to 15 degrees.

Thoracentesis Contraindications

Uncooperative patient Bleeding disorder/anticoagulation Inadequate fluid volume Severe COPD-high risk for pneumothorax Overlying skin infection Loculated fluid collection-should be done with ultrasound localization

Which finding is the best indicator that the fluid resuscitation for a patient with hypovolemic shock has been effective? a. Hemoglobin is within normal limits. b. Urine output is 60 mL over the last hour. c. Central venous pressure (CVP) is normal. d. Mean arterial pressure (MAP) is 72 mm Hg.

Urine output is 60 mL over the last hour. Assessment of end organ perfusion, such as an adequate urine output, is the best indicator that fluid resuscitation has been successful. The hemoglobin level, CVP, and MAP are useful in determining the effects of fluid administration, but they are not as useful as data indicating good organ perfusion.

Which information will the nurse include in teaching a patient who had a proctocolectomy and ileostomy for ulcerative colitis? a. Restrict fluid intake to prevent constant liquid drainage from the stoma. b. Use care when eating high-fiber foods to avoid obstruction of the ileum. c. Irrigate the ileostomy daily to avoid having to wear a drainage appliance. d. Change the pouch every day to prevent leakage of contents onto the skin.

Use care when eating high-fiber foods to avoid obstruction of the ileum. High-fiber foods are introduced gradually and should be well chewed to avoid obstruction of the ileostomy. Patients with ileostomies lose the absorption of water in the colon and need to take in increased amounts of fluid. The pouch should be drained frequently but is changed every 5 to 7 days. The drainage from an ileostomy is liquid and continuous, so control by irrigation is not possible.

During change-of-shift report on a medical unit, the nurse learns that a patient with aspiration pneumonia who was admitted with respiratory distress has become increasingly agitated. Which action should the nurse take first? a. Give the prescribed PRN sedative drug. b. Offer reassurance and reorient the patient. c. Use pulse oximetry to check the oxygen saturation. d. Notify the health care provider about the patient's status.

Use pulse oximetry to check the oxygen saturation.

The nurse is caring for a patient who has a central venous access device (CVAD). Which action by the nurse is appropriate? a. Avoid using friction when cleaning around the CVAD insertion site. b. Use the push-pause method to flush the CVAD after giving medications. c. Obtain an order from the health care provider to change CVAD dressing. d. Position the patient's face toward the CVAD during injection cap changes.

Use the push-pause method to flush the CVAD after giving medications.

Following assessment of a patient with pneumonia, the nurse identifies a nursing diagnosis of ineffective airway clearance. Which assessment data best supports this diagnosis?

Weak, nonproductive cough effort The weak, nonproductive cough indicates that the patient is unable to clear the airway effectively.

Needle insertion for thoracentesis:

While withdrawing fluid always stabilize the needle/catheter at insertion site May use simple needle, needle-catheter system, or special thoracentesis needle Fluid withdrawn with syringe and vacuum bottle Be careful to avoid withdrawal of large volume from a chronic effusion *appropriate interspace with lidocaine

A patient with terminal cancer tells the nurse, "I know I am going to die pretty soon, perhaps in the next month." Which of the following is the most appropriate response by the nurse? a."What are your feelings about being so sick and thinking you may die soon?" b."None of us know when we are going to die. Is this a particularly difficult day?" c."Would you like for me to call your spiritual advisor so you can talk about your feelings?" d."Perhaps you are depressed about your illness; I will speak to the doctor about getting some medications for you."

a. Rationale: The most appropriate response to psychosocial questions is to acknowledge the patient's feelings and explore his or her concerns. This option does both and is a helpful response that encourages further communication between patient and nurse.

A side effect common to biologic therapies is a. flu like symptoms b. bone marrow suppression c. CNS deficits d. nausea, vomiting, anorexia, and diarrhea

a. Rationale: Virtually all biologic therapies may cause flu-like syndrome that include-HA, fever, chills, myalgias, fatigue, and anorexia.

The primary use of biologic therapy in cancer treatment is to a. prevent the fatigue associated with chemotherapy and high dose radiation b. enhance or supplement the effects of the host's immune responses to tumor cells c. depress the immune system and circulating lymphocytes, as well as increasing a sense of well-being d. protect normal rapidly reproducing cells of the GI system from damage during chmotherapy

b. Rationale: Biologic therapies are normal components of the immune system that have been identified and isolated and are used therapeutically to restore, augment, or modulate host immune system mechanisms to assist in immune activity against cancer cells.

The risk factor associated with cancer of the pancreas is a. alcohol intake b. cigarette smoking c. exposure to asbestos d. increased dietary intake of milk and milk products

b. Rationale: Major risk factors for pancreatic cancer are believed to be cigarette smoking, high-fat diet, diabetes, and exposure to benzidine and cocaine.

When obtaining a health history from a patient with possible cancer of the mouth, the nurse would expect the patient to report a. long-term denture use b.heavy tobacco and alcohol use c. persistent swelling of the neck and face d. chronic herpes simplex infections of the mouth and lips

b. Rationale: The primary risk factors for head and neck cancers are heavy tobacco and alcohol use and family history.

A patient is hospitalized with metastatic cancer of the liver. The nurse plans care for the patient based on the knowledge that a. chemotherapy is highly successful in the treatment of liver cancer b. the patient will undergo surgery to remove the involved portions of the liver c. supportive care that is appropriate for all patients with severe liver damage is indicated d. metastatic liver cancer is more responsive to treatment that primary carcinoma of the liver

c. Rationale: Because the prognosis for cancer of the liver is poor, and treatment is largely palliative, supportive nursing care is appropriate.

The major difference between benign and malignant tumors is that malignant tumors a. grow at a faster rate b. are often encapsulated c. invade and metastasize d. cause death whereas benign tumors do not

c. Rationale: The major difference between benign and malignant cells is the ability of malignant tumor cells to invade and metastasize.

Which of the following is a late symptom of head and neck cancer? A. hoarseness b. change in fit of dentures c. mouth ulcers that do not heal d. decreased mobility of the tongue

d,

A patient being admitted with an acute exacerbation of ulcerative colitis reports crampy abdominal pain and passing 15 or more bloody stools a day. The nurse will plan to a. administer IV metoclopramide (Reglan). b. discontinue the patient's oral food intake. c. administer cobalamin (vitamin B12) injections. d. teach the patient about total colectomy surgery.

discontinue the patient's oral food intake. An initial therapy for an acute exacerbation of inflammatory bowel disease (IBD) is to rest the bowel by making the patient NPO. Metoclopramide increases peristalsis and will worsen symptoms. Cobalamin (vitamin B12) is absorbed in the ileum, which is not affected by ulcerative colitis. Although total colectomy is needed for some patients, there is no indication that this patient is a candidate.

A 52-year-old man tells the nurse that he decided to seek treatment for erectile dysfunction (ED) because his wife "is losing patience with the situation." The most appropriate nursing diagnosis for the patient is a. situational low self-esteem related to effects of ED. b. ineffective role performance related to effects of ED. c. anxiety related to inability to have sexual intercourse. d. ineffective sexuality patterns related to infrequent intercourse.

ineffective role performance related to effects of ED. The patient's statement indicates that the relationship with his wife is his primary concern. Although anxiety, low self-esteem, and ineffective sexuality patterns may also be concerns, the patient information suggests that addressing the role performance problem will lead to the best outcome for this patient

A critical action by the nurse caring for a patient with an acute exacerbation of polycythemia vera is to a. place the patient on bed rest. b. administer iron supplements. c. avoid use of aspirin products. d. monitor fluid intake and output.

monitor fluid intake and output. Monitoring hydration status is important during an acute exacerbation because the patient is at risk for fluid overload or underhydration. Aspirin therapy is used to decrease risk for thrombosis. The patient should be encouraged to ambulate to prevent deep vein thrombosis (DVT). Iron is contraindicated in patients with polycythemia vera.

A 53-year-old man is scheduled for an annual physical exam. The nurse will plan to teach the patient about the purpose of a. urinalysis collection. b. uroflowmetry studies. c. prostate specific antigen (PSA) testing. d. transrectal ultrasound scanning (TRUS).

prostate specific antigen (PSA) testing. An annual digital rectal exam (DRE) and PSA are usually recommended starting at age 50 for men who have an average risk for prostate cancer. Urinalysis and uroflowmetry studies are done if patients have symptoms of urinary tract infection or changes in the urinary stream. TRUS may be ordered if the DRE or PSA is abnormal.

Four hours after a bowel resection, a 74-year-old male patient with a nasogastric tube to suction complains of nausea and abdominal distention. The first action by the nurse should be to a. auscultate for hypotonic bowel sounds. b. notify the patient's health care provider. c. reposition the tube and check for placement. d. remove the tube and replace it with a new one.

reposition the tube and check for placement. Repositioning the tube will frequently facilitate drainage. Because this is a common occurrence, it is not appropriate to notify the health care provider unless other interventions do not resolve the problem. Information about the presence or absence of bowel sounds will not be helpful in improving drainage. Removing the tube and replacing it are unnecessarily traumatic to the patient, so that would only be done if the tube was completely occluded.

The emergency department (ED) nurse receives report that a patient involved in a motor vehicle crash is being transported to the facility with an estimated arrival in 1 minute. In preparation for the patient's arrival, the nurse will obtain a. hypothermia blanket. b. lactated Ringer's solution. c. two 14-gauge IV catheters. d. dopamine (Intropin) infusion.

two 14-gauge IV catheters. A patient with multiple trauma may require fluid resuscitation to prevent or treat hypovolemic shock, so the nurse will anticipate the need for 2 large bore IV lines to administer normal saline. Lactated Ringer's solution should be used cautiously and will not be ordered until the patient has been assessed for possible liver abnormalities. Vasopressor infusion is not used as the initial therapy for hypovolemic shock. Patients in shock need to be kept warm not cool.

The nurse performing a focused examination to determine possible causes of infertility will assess for a. hydrocele. b. varicocele. c. epididymitis. d. paraphimosis.

varicocele. Persistent varicoceles are commonly associated with infertility. Hydrocele, epididymitis, and paraphimosis are not risk factors for infertility.


Ensembles d'études connexes

Brunner & Suddarth's Textbook of Medical-Surgical Nursing

View Set

Prueba Escrita Leccion 5 VHL Answers

View Set

SBE Refresher 1 - Privacy and Confidentiality

View Set